lançamentos e promoções. -...

Post on 18-Jan-2019

231 Views

Category:

Documents

0 Downloads

Preview:

Click to see full reader

TRANSCRIPT

Cadastre-seemwww.elsevier.com.brparaconhecer‐

nossocatálogocompleto,teracessoaserviços

exclusivosnositeereceberinformaçõessobrenossos

lançamentosepromoções.

©2013,ElsevierEditoraLtda.

TodososdireitosreservadoseprotegidospelaLeino9.610,de19/02/1998.

Nenhumapartedestelivro,semautorizaçãopréviaporescritodaeditora,poderáserreproduzidaou

transmitidasejamquaisforemosmeiosempregados:eletrônicos,mecânicos,fotográficos,gravaçãoou

quaisqueroutros.

RevisãoGráfica:ClaraDiament

EditoraçãoEletrônica:SBNigriArteseTextosLtda.

CoordenadordaSérie:SylvioMotta

ElsevierEditoraLtda.

ConhecimentosemFronteiras

RuaSetedeSetembro,111–16oandar

20050-006–Centro–RiodeJaneiro–RJ–Brasil

RuaQuintana,753–8oandar

04569-011–Brooklin–SãoPaulo–SP–Brasil

ServiçodeAtendimentoaoCliente

0800-0265340

atendimento1@elsevier.com

ISBN978-85-352-6999-4

ISBN:978-85-352-7000-6(recursoeletrônico)

Nota:Muitozeloetécnicaforamempregadosnaediçãodestaobra.Noentanto,podemocorrererrosde

digitação,impressãooudúvidaconceitual.Emqualquerdashipóteses,solicitamosacomunicaçãoaonosso

ServiçodeAtendimentoaoCliente,paraquepossamosesclarecerouencaminharaquestão.

Nemaeditoranemoautorassumemqualquerresponsabilidadeporeventuaisdanosouperdasapessoasou

bens,originadosdousodestapublicação.

CIP-BRASIL.CATALOGAÇÃO-NA-FONTE

SINDICATONACIONALDOSEDITORESDELIVROS,RJ

Rennó,Rodrigo

Administraçãogeralparaconcursos/RodrigoRennó.–RiodeJaneiro:Elsevier,

2013.

Incluibibliografia

ISBN978-85-352-6999-4

R331a

1.Administração–Brasil–Problemas,questões,exercícios.2.Serviçopúblico–

Brasil–Concurso.I.Título.II.Série.

13-

CDD:658

0894.

CDU:005

Dedicatórias

AosmeuspaisJoséLuiz(inmemoriam)eSuzana,

portodoocarinhoeatençãoquesempretiveram

comigo,alémdosexemplosquemederam

etentoseguir.

AomeufilhoRicardo,quemealegraocoraçãoe

medáforçasparaencararosdesafiosdavida.

ÀminhanamoradaNívia,portodooamor,carinhoe

suportequesempremedeu.

AomeutioWalter,portodososconselhose

portersempreacreditadoemmim.

Agradecimentos

AoscolaboradoresdaEditoraCampus/Elsevier,pelaatençãoeprossionalismo

quesemprepautaramnossarelação.AosprofessoresJoãoAntônioeSylvioMotta,

pelaconfiançanomeutrabalho.

AosmeusantigoscolegasdetrabalhonaSubsecretariadoTesourodoDistrito

Federal,peloapoioeatenção,principalmenteaosmeusamigosBrunodeSousa

SimõeseMarceloPachecoBastos,pelosincentivoseconselhosquandoeste

trabalhoaindaestavaemseunascedouro.

Eatodososprofessoresealunosque,dealgumaforma,contribuíramparaesta

obracomsugestõesecríticas.

OAutor

RodrigoRennótem40anoseéEspecialistaemPolíticasPúblicaseGestão

GovernamentaldoMinistériodoPlanejamento,OrçamentoeGestão.Formado

emAdministraçãopelaPUC-RJ,comMBAemGestãoAdministrativa,foi

aprovadoemdiversosconcursospúblicos.

Atuacomoprofessorespecializadoemconcursosemváriosestadose

atualmenteécolaboradordocursoDamásiodeJesusemSãoPauloedossites:

www.euvoupassar.com.br;

www.estrategiaconcursos.com.br;

www.tecconcursos.com.br/.

Prefácio

Acadaano,percebem-semudançassignicativasnavariedadeenoníveldas

matériasexigidasemconcursospúblicos.Nosúltimosanos,especialmente,vêmse

tornandoconcretas,eimportantíssimas,asmatériasrelacionadascoma

AdministraçãodeEmpresas.

Nãomencionoapenasuma,mas“várias”,porque,realmente,formamuma

miríadedeassuntosqueantesDESTAOBRAsóeramtragáveisparaosque

tiveramcontatocomelasnasFaculdadesdeAdministração.

OprofessorRodrigoRennó,aquemmedámuitahonrachamardeAMIGO,

consolidanumaúnicaobraescritaosprincipaispontosexigidosnosmaisvariados

certames:desdeTécnicoJudiciário(dequasetodososTRT,TRE,TJeTRFdo

país),passando,claro,peloscargosdeAnalistadosmesmostribunais,atéos

concursosemqueaexigênciachegaasersurreal,comosepodeexperimentarem

concursosparaAuditoresFiscaisEstaduaiseFederais.

Alémdeumconhecimentoquesepodeencontrarempouquíssimos

prossionais,RodrigoRennóédonodeumadidáticaprivilegiada,encontradaem

umnúmeromuitomenordeautores…Aquiestáasolução,amigoleitor,casoo

seuproblemaresidanasmatériasrelacionadasàAdministração.

AEditoraCampus/Elseviertraz,maisumavez,omelhordecadaáreaparaseus

leitoresdeConcursos.Semdúvidaalguma,oestudodaAdministraçãopara

Concursospassaráporumanovíssimafaseapartirdestaobraquevocêestátendo,

nestemomento,achancedeadquirir!Nãoaperca!Aproveite!

JoãoAntonio

FundadoreCoordenadordoEuVouPassar(www.euvoupassar.com.br)

AutordolivroInformáticaparaConcursos,daCampus/Elsevier

AutordolivroEuVouPassaremConcursos,daCampus/Elsevier

Apresentação

Estaobratemcomoobjetivoatenderaoscandidatosaconcursospúblicosno

queserefereaoconteúdodeAdministração,umadasmaiscobradasatualmente,

tendosidoinseridanoseditaisdaReceitaFederal,PolíciaFederal,MPU,tribunais

diversos,empresasestatais,etc.

Estelivroapresentaamatériadeformaclaraeobjetiva.Estávoltadoaostemas

realmentecobradospelasbancasenoníveldediculdadeatualmentesolicitado.

Alémdisso,englobadiversasquestõesdeconcursoscomentadasepropostaspara

quevocêspossamavaliarseudesempenhoeevoluçãonamatéria.

Sumário

Capa

FolhadeRosto

Cadastro

Créditos

Dedicatórias

Agradecimentos

OAutor

Prefácio

Apresentação

Capítulo1–EvoluçãodasTeoriasAdministrativas

1.1.AAdministraçãoCientífica

1.2.ATeoriaClássica

1.3.TeoriadasRelaçõesHumanas

1.4.Burocracia

1.5.TeoriaEstruturalista

1.6.TeoriadosSistemas

1.7.TeoriaContingencial

Capítulo2–CulturaOrganizacional

2.1.NíveisdaCultura

2.2.CulturaForteeCulturaFraca

2.3.ElementosdaCultura

2.3.1.ArtefatosVerbais

2.3.2.ArtefatosComportamentais

2.3.3.ArtefatosFísicos

2.4.CulturaMecanicistaeCulturaOrgânica

2.5.CulturaDominanteeSubculturas

2.6.DisfunçõesdaCultura

2.7.ClimaOrganizacional

Capítulo3–PlanejamentoeEstratégia

3.1.Planejamento

3.2.NíveisdoPlanejamento

3.3.MissãoeVisãoeNegócio

3.4.DiagnósticoEstratégico–AnáliseSWOT

3.5.Objetivos,MetasePlanos

3.6.EscolasdoPlanejamento

3.7.PlanejamentoporCenários

3.8.AnáliseCompetitivaeEstratégiasGenéricas–Estratégia

3.9.EstratégiacomoPosicionamento

3.10.ModelodeCincoForçasdePorter

3.11.EstratégiasGenéricasdePorter

3.12.MatrizdeAnsoff

3.13.EstratégiacomoMovimento

3.14.TiposdeEstratégia

3.14.1.EstratégiadeSobrevivência

3.14.2.EstratégiadeManutenção

3.14.3.EstratégiadeCrescimento

3.14.4.EstratégiadeDesenvolvimento

3.15.Redesealianças

3.16.AdministraçãoporObjetivos

3.17.ControleEstratégico

3.18.BalancedScorecard

3.19.MapaEstratégico

Capítulo4–EstruturaOrganizacional

4.1.EstruturasFormaiseInformais

4.2.EspecializaçãodoTrabalho

4.3.EnriquecimentodoTrabalho

4.4.CadeiadeComando

4.5.AmplitudedeControle

4.6.CentralizaçãoeDescentralização

4.7.PosiçõesdeLinhaeStaff

4.8.Departamentalização

4.9.DepartamentalizaçãoFuncional

4.9.1.DepartamentalizaçãoporProduto

4.9.2.DepartamentalizaçãoporCliente

4.9.3.DepartamentalizaçãoTerritorialouGeográfica

4.9.4.DepartamentalizaçãoporProcesso

4.10.DesenhoEstruturaldasOrganizações

4.11.EstruturaLinear

4.12.EstruturaFuncional

4.13.EstruturaDivisional

4.14.EstruturaMatricial

4.15.EstruturaemRede

4.16.FatoresqueInfluenciamaEstrutura

4.16.1.AEstruturaSegueaEstratégia

4.16.2.AEstruturaSegueoAmbiente

4.16.3.AEstruturaseAdaptaàTecnologia

4.16.4.AEstruturaSegueoTamanho

4.17.EstruturasdeMintzberg

Capítulo5–Motivação

5.1.TeoriasdeProcessoeTeoriasdeConteúdo

5.1.1.TeoriadasNecessidadesdeMaslow

5.1.2.TeoriaXeYdeMcGregor

5.1.3.TeoriadosDoisFatoresdeHerzberg

5.1.4.TeoriadoReforço

5.1.5.TeoriaERC(ouERG)–Alderfer

5.1.6.TeoriadaExpectânciadeVroom

5.1.7.TeoriadasNecessidadesAdquiridas,deMcCleland

5.1.8.TeoriadaEquidade

5.1.9.TeoriadoEstabelecimentodeObjetivos(Autoeficácia)

5.2.“Empoderamento”ouEmpowerment

Capítulo6–Liderança

6.1.DiferençasentreLidereseGerentes

6.2.AbordagensdeLiderança

6.3.TeoriadosTraçosdeLiderança

6.4.EstilosdeLiderança

6.5.ContinuumdaLiderança

6.6.GridGerencialdeBlakeeMouton

6.7.LiderançasContingenciaiseSituacionais

6.8.ModelodeFiedler

6.9.TeoriaSituacionaldeHerseyeBlanchard–Ociclodevidada

Liderança

6.10.LiderançaTransacionalxTransformacional

6.11.LiderançaCarismática

6.12.LiderançaVisionária

Capítulo7–ControleeAvaliação

7.1.MomentodoControle

7.2.ConceitosdeEficiência,EficáciaeEfetividade

7.3.UsodeControleseIndicadoresdeProdutividade

7.4.IndicadoresdeDesempenho

7.5.Indicadoresligadosàdimensãoresultado

7.6.IndicadoresLigadosàDimensãoEsforço

7.7.QualidadedosIndicadores

Capítulo8–Coordenação

8.1.EstratégiasdeCoordenação

8.2.AjusteMútuo

8.3.SupervisãoDireta

8.4.PadronizaçãodosProcessosdeTrabalho

8.5.PadronizaçãodosResultados

8.6.PadronizaçãodeHabilidades

8.7.ConflitosnaCoordenação

Capítulo9–Comunicação

9.1.ComunicaçãoEficienteeEfetiva

9.2.CanaisdeComunicação

9.3.ComunicaçãoVerbaleNãoVerbal

9.4.ComunicaçãoOrganizacional

9.5.BarreirasàComunicação

Capítulo10–GestãodasMudanças

10.1.Criatividadeeinovaçãoorganizacional

10.1.1.Criatividade

10.1.2.Inovação

10.1.3.InovaçãoIncrementaleRadical

10.2.MudançaOrganizacional

10.2.1.AgentesdeMudança

10.2.2.ModelosdeMudança

10.2.3.MudançadeCimaparaBaixo(Top-Down)

10.2.4.MudançadeBaixoparaCima(Bottom-Up)

10.2.5.MudançaIntegrada

10.2.6.ResistênciasàsMudanças

10.2.7.MudançasPlanejadas

Capítulo11–GestãodeConflitos

11.1.CausasdosConflitos

11.2.EstilosdeGestãodeConflitos

11.3.AbordagensnaResoluçãodeConflitos

11.3.1.AbordagemEstrutural

11.3.2.AbordagemdeProcesso

11.3.3.AbordagemMista

11.4.EfeitosdoConflito

Capítulo12–GerenciamentodeProcessos

12.1.CadeiadeValor

12.2.TiposdeProcessos

12.3.NíveisdeDetalhamentodosProcessos

12.4.MapeamentodeProcessos

12.5.TécnicasdeMapeamento,AnáliseeMelhoriadeProcessos

12.6.ProjetodeMapeamentoeModelagemdeProcessos

12.7.ProcessoseCertificaçãoISO9000:2000

12.8.NoçõesdeEstatísticaAplicadaaoControleeàMelhoriade

Processos

12.9.PopulaçãoeAmostra

12.10.MédiaeVariabilidade

12.11.MelhoriadosProcessos

12.12.Reengenharia

12.13.OqueaReengenhariaNãoÉ?

Capítulo13–GestãodaQualidade

13.1.ErasdaGestãodaQualidade

13.1.1.EradaInspeção

13.1.2.EradoControleEstatísticodaQualidade

13.1.3.EradaGarantiadaQualidade

13.1.4.EradaGestãoEstratégicadaQualidadeouGestãodaQualidade

Total

13.2.PrincipaisAutoresou“Gurus”daQualidade

13.2.1.Shewart

13.2.2.CicloDemingouPDCA

13.2.3.Deming

13.2.4.Juran

13.2.5.Feigenbaum

13.2.6.Crosby

13.2.7.Ishikawa

13.3.Ferramentasdegestãodaqualidade

13.4.DiagramadeCausaeEfeitooudeIshikawa

13.5.FolhadeVerificação

13.6.Histograma

13.7.GráficodePareto

13.8.DiagramadeCorrelaçãoouDispersão

13.9.Fluxograma

13.10.GráficodeControle

13.11.Programa5S

13.12.SeisSigma

13.13.Kaizen

13.14.ModelodaFundaçãoNacionaldaQualidade

13.15.QualidadeeProdutividade

Capítulo14–GestãodoConhecimento

14.1.ConceitosdeDados,InformaçãoeConhecimento

14.2.GestãodoConhecimento

14.3.ConhecimentoTácitoeExplícito

14.4.AprendizagemOrganizacional

14.5.EspiraldoConhecimentodeNonakaeTakeuchi

Capítulo15–ProcessoDecisório

15.1.Estruturadasdecisõesempresariais

15.1.1.Tiposdedecisões

15.1.1.1.DecisõesProgramadaseNãoprogramadas

15.2.RacionalidadeeRacionalidadeLimitada

15.3.Certeza,RiscoeIncerteza

15.4.ProcessoLineareSistêmico

15.5.TécnicasdeAnáliseeSoluçãodeProblemas

15.6.FerramentasdeDesenvolvimentodeAlternativas

15.6.1.BrainstormingeBrainwriting

15.6.2.AnálisedoCampodeForças

15.6.3.DiagramadeÁrvoredeDecisão

Capítulo16–GestãoporCompetências

16.1.CompetênciasHumanaseOrganizacionais

16.2.EtapasdaGestãoporCompetências

16.3.HabilidadesAdministrativasdeKatz

Capítulo17–Trabalhoemequipe

Capítulo18–GestãodePessoas

18.1.PanoramadaÁreadeRHnoSetorPúblico

18.2.Recrutamentoeseleçãodepessoas

18.2.18.Recrutamento

18.2.2.SeleçãodePessoas

18.2.3.RecrutamentoeSeleçãonosÓrgãosPúblicos

18.3.Cargos,CarreiraseRemuneração

18.4.AvaliaçãodoDesempenho

18.4.1.MétododasEscalasGráficas

18.4.2.ListasdeVerificação

18.4.3.MétododaEscolhaForçada

18.4.4.MétododosIncidentesCríticos

18.4.5.MétododaPesquisadeCampo

18.5.ProcessodeRecompensarPessoas

18.6.ProcessodeDesenvolverPessoas

18.6.1.TreinamentoeDesenvolvimento

18.6.1.1.MétodosdeTreinamento

18.7.ProcessodeManterPessoas

18.7.1.QualidadedeVida

18.7.2.Equilíbrioorganizacional

18.8.ProcessodeMonitorarPessoas

18.8.1.SistemasdeInformaçõesGerenciais

Capítulo19–GerênciadeProjetos

19.1.DiferençasentreProjetoseOperações

19.2.StakeholdersouPartesInteressadas

19.3.GruposdeProcessosdosProjetos

19.4.ÁreasdoConhecimentonoGerenciamentodeProjetos

19.5.OGerentedeProjetos

19.6.Portfólios,Programas,ProjetoseSubprojetos

19.7.CiclosdeVidadosProjetos

19.8.CiclodeVidadeumProjetoxCiclodeVidadeumProduto

19.9.CustoseRiscosnoCiclodeProjetos

19.10.RestriçõesaosProjetos

Capítulo20–AdministraçãodeMateriais

20.1.AdministraçãodeEstoques

20.2.PrevisãoparaosEstoques

20.2.1.MétododoÚltimoPeríodo

20.2.2.MétododaMédiaMóvel

20.2.3.MétododaMédiaMóvelPonderada

20.3.CustosdosEstoques

20.3.1.CustosdeArmazenamento

20.3.2.CustosdePedido

20.3.3.CustosdeFaltadeEstoque

20.4.NíveisdeEstoque

20.5.ClassificaçãoABC

20.6.AvaliaçãodeEstoques

20.7.Just-in-time

20.8.AdministraçãodeCompras

20.9.EstratégiasdoSetordeCompras

20.9.1.VerticalizaçãoouHorizontalização

20.9.2.CentralizaçãoouDescentralização

20.10.ComprasnoSetorPúblico–Licitações

20.11.ModalidadesdeLicitação

20.11.1.Concorrência

20.11.2.TomadadePreços

20.11.3.Convite

20.11.4.Pregão

20.11.5.Concurso

20.11.6.Leilão

20.11.7.Consulta

20.12.Almoxarifado

20.13.IdentificaçãodoMaterial

20.14.Picking

20.15.Inventários

20.16.FasesdoInventário

20.17.TiposdeInventário

20.17.1.InventárioAnual

20.17.2.InventáriosRotativos

20.18.AvaliaçãodosBensnoInventário

Notas

Capítulo1

EvoluçãodasTeoriasAdministrativas

1.1.AAdministraçãoCientífica

ComaRevoluçãoIndustrial,asrelaçõesdetrabalhoeascondiçõesemquea

produçãoocorriasetransformaramtremendamente.Amáquinaavapor

proporcionouumamelhorianostransportes(principalmenteosnaviosavapore

trens)quepermitiuqueumaempresa“entregasse”seusprodutosparaumpúblico

cadavezmaioremaisdistante.

Alémdisso,essenovomaquináriolevouaumnovotipodeprocessoprodutivo:

aproduçãoemmassa.Aprodutividadeeavelocidadedeproduçãoforam

ampliadasenormemente.Apequenaocinaaospoucosdeuespaçoagrandes

indústrias,emqueoambientedetrabalhoerainsalubreeperigoso,comjornadas

detrabalhodemaisdedozehorasdiárias.

Equemeramosoperários?Aindústrianaépocacontratava,emgrandeparte,

osmoradoresdocampo,queeramatraídospormelhoressalários.Assim,esses

trabalhadoreschegavamàsindústriassemqualicaçãoespecícaeefetuavamum

trabalhobasicamentemanual(ou“braçal”).

Comisso,existiaumambientedegrandedesperdícioebaixaeciêncianas

indústrias.1OprimeiroteóricoabuscarmudaressarealidadefoiFrederick

Taylor.

Taylorismo:ésinônimodeAdministraçãoCientíca.Muitoschamamessateoriapelo

nomedeseuprincipalautor:FrederickTaylor.

Nasfábricas,osfuncionáriosfaziamseutrabalhodeformaempírica,ouseja,na

basedatentativaeerro.Osgerentesnãoestudavamasmelhoresformasdese

trabalhar.

Osfuncionáriosnãosecomprometiamcomosobjetivos(deacordocomTaylor,

cavam“vadiando”),ecadaumfaziaotrabalhocomo“achavamelhor”–não

existiaassimumapadronizaçãodosprocessosdetrabalho.2

Tayloracreditavaqueotrabalhopoderiaserfeitodemodomuitomais

produtivo.3

AAdministraçãoCientícabuscou,então,amelhoriadaeciênciaeda

produtividade.4

Elepassouaestudarentãoa“melhormaneira”desefazerastarefas.Esse

trabalhofoichamadodeestudodetemposemovimentos.5Otrabalhodo

operárioeraanalisadoecronometrado,demodoqueosgerentespudessem

determinaramaneiramaiseciente–“theonebestway”ouamaneiracertadese

fazerumatarefa.

Apósadeniçãodomodomaisrápidoefácildeexecutarumatarefa(por

exemplo,amontagemdeumaroda),osfuncionárioseramtreinadosparaexecutá-

lasdessaforma–criandoassimumapadronizaçãodotrabalho.

Essapadronizaçãoevitariaaexecuçãodetarefasdesnecessáriasporpartedos

empregados.Tudoissoajudarianaeconomiadeesforçoseevitariaumarápida

fadigahumana.ParaTaylor,aAdministraçãoCientícadeveriaanalisaros

movimentosefetuadospelostrabalhadoresparaconseguirdesenharumprocesso

comummínimodeesforçoemcadatarefa.

Figura1.1–ContextodaAdministraçãoCientífica

Outroaspectoimportantefoiadivisãodotrabalho.6Deacordocomos

teóricosdaAdministraçãoCientíca,seriamuitomaisfáciltreinarecapacitarum

funcionárioaexecutarumatarefaespecíca(aparafusarumassento,porexemplo)

doquefazertodootrabalhosozinho(montarumabicicletainteira,porexemplo).

Esseconceitofoiabasedalinhademontagem–processoprodutivoemquea

peçaaserfeitavaipassandodefuncionárioafuncionário,atéquetodostenham

montado“suaparte”.

Chamamosissodeespecialização.Oempregadocavarestritoaumapequena

partedoprocessoprodutivo,demodoqueseutreinamentoeadaptaçãoà“melhor

maneira”(omodopadronizadodesetrabalhar)fossemfacilitados.

Taylortambémbuscouaumentaroincentivoaofuncionário.Eleacreditavaque

aremuneraçãoporhoranãotrazianenhumincentivoaofuncionário.Assim,

indicouopagamentoporprodutividade(pagamentoporpeça,porexemplo)

comoessencialparaqueofuncionáriobuscasseummaioresforço.7

Fordismo–seguidordeTaylor,HenryFordtornouoscarrosprodutosdeconsumo.Ele

conseguiureduzirospreçosatravésdaproduçãoemmassadeprodutospadronizados,

comfortemecanizaçãodotrabalho.

Portanto,Tayloracreditavaqueoincentivomateriallevavaaumamaior

motivaçãoparaotrabalho.Essafoiabasedoconceitodo“Homoeconomicus”.Ou

seja,aideiadequeaprincipalmotivaçãodeumapessoanotrabalhoseriaa

remuneração(oubenefíciosmateriais).8

Entretanto,aAdministraçãoCientícapecoupornãoanalisaraorganização

emtodoseucontexto.Ouseja,sóanalisavaseuambienteinternoeseusproblemas

easdemandasdeprodução(ouseja,osproblemasdo“chãodefábrica”).Assim,

nãocaptavatodaacomplexidadeemqueaadministraçãoestavaenvolvida.

Dessamaneira,dizemosqueéumateoriadesistemafechado(dentrodeuma

lógicaquenãovêasinter-relaçõeseinuênciasentreasdemaisorganizações).9

Seriacomoanalisarumaempresanovácuo,semimaginararespostadosseus

concorrentesaolançamentodeumproduto,porexemplo.

Figura1.2–CaracterísticasdaAdministraçãoCientífica

Dessemodo,entreasprincipaiscríticasàAdministraçãoCientífica,temos:10

ØOmecanicismo–aideiadequeaorganizaçãofuncionariacomouma

“máquina”eseusfuncionáriosseriam“engrenagens”quedeveriamfuncionar

nomáximodaeficiência;

ØAsuperespecializaçãodotrabalhador–seastarefasmaissimpleserammais

fáceisdeseremtreinadasepadronizadas,tambémtornavamotrabalho

extremamentechato!Empoucotempootrabalhadorjánãotinhamais

desafiosesuamotivaçãodiminuía;

ØVisãomicroscópicadohomem–aAdministraçãoCientícafocava

principalmentenotrabalhomanual(nãosepreocupandocomsua

criatividade)esebaseavanaideiadequeohomemsemotivava

principalmenteporinuênciadosincentivosmateriais(sematentarpara

outrosfatores,comoumambientedesafiador,porexemplo).

ØAabordagemdesistemafechado–Taylornãosepreocupoucomo

ambienteexterno–omercadodetrabalho,osconcorrentes,osfornecedores

etc.Suavisãoévoltadaparadentrodaempresasomente.

ØAexploraçãodosempregados–apesardeTaylorproporum

relacionamento“ganha-ganha”entrepatrõeseempregados,napráticaa

aplicaçãodospreceitosdaAdministraçãoCientícalevouaumamaior

exploraçãodosempregados.11

ØRecompensaslimitadas–paraTaylor,oserhumanoeramotivadoapenas

porincentivosmateriais.Atualmente,sabemosqueexistemdiversosoutros

fatoresqueservemdemotivadoresparaaspessoas.

Figura1.3–ProblemasdaAdministraçãoCientífica

1.2.ATeoriaClássica

EmumcontextosemelhanteaodaAdministraçãoCientíca(poisforam

geradasnamesmaépoca),aTeoriaClássicadaAdministração,desenvolvidapor

HenriFayol,buscouamelhoriadaeciênciaatravésdofoconasestruturas

organizacionais.12

Figura1.4–ContextodaTeoriaClássica

Dessaforma,ofococomFayolsaiudastarefasparaaestrutura.Eletinhauma

visãode“cimaparabaixo”dasempresas.Atravésdosestudosda

departamentalização,viaosdepartamentoscomopartesdaestruturada

organização.

Foi,portanto,umdospioneirosnoquesechamoudeteóricosfisiologistasda

administração.Assim,oescopodotrabalhodoadministradorfoibastante

ampliadodentrodavisãodeFayol.

Fayoléconsideradoo“paidateoriaadministrativa”,poisbuscouinstituir

princípiosgeraisdotrabalhodeumadministrador.13Seutrabalhoaindaé(após

umséculo)consideradorelevanteparaquepossamosentenderotrabalhodeum

gestoratual.

Oautordeniuseisfunçõesempresariaisqueasempresasemgeraldevem

conter.Asfunçõesseriam:atécnica,acomercial,afinanceira,adesegurança,a

contábileaadministrativa.Essaúltimaseriaaresponsávelpelacoordenaçãodas

outrasfunções.14

Figura1.5–FunçõesEmpresariaisporFayol

Alémdisso,Fayoldeniuotrabalhodeumadministradordentrodoqueele

chamoudeprocessoadministrativo–asfunçõesdoadministrador.Deacordo

comFayol,15elassão:

ØPrever:visualizarofuturoetraçaroprogramadeaçãoemmédioelongo

prazos;

ØOrganizar:constituiraestruturamaterialehumanapararealizaro

empreen-dimentodaempresa;

ØComandar:dirigireorientaropessoalparamantê-loativonaempresa;

ØCoordenar:ligareharmonizartodososatosetodososesforçoscoletivos;

ØControlar:cuidarparaquetudoserealizedeacordocomosplanosda

empresa.

Essesseriamelementosqueestariampresentesnotrabalhodecada

administrador,independentementedeseunívelhierárquico.Assim,desdeo

presidentedaempresaatéummerosupervisordeveriamdesempenharessas

funçõesemseudiaadia.

ATeoriaClássicatambémsebaseavanamesmapremissadoTaylorismo:ade

queohomemseriamotivadoporincentivosmateriais,ouseja,oconceitode

Homoeconomicus.

Alémdisso,tambémsepreocupavamaiscomosaspectosinternosdas

organizações,semanalisarasinter-relaçõesetrocasentreaorganizaçãoeseu

ambienteexterno.Assim,tambémeraumateoriadesistemafechado.

Cuidado–muitasbancastentamconfundiraTeoriaClássicadeFayolcomaabordagem

clássicadaAdministração.EstaenglobatambémaAdministraçãoCientícaeaTeoriada

Burocracia.

DeacordocomFayol,existem14princípiosgeraisdaadministração:16

1)Divisãodotrabalho:consistenaespecializaçãodastarefasedaspessoaspara

aumentaraeficiência;

2)Autoridadeeresponsabilidade:autoridadeéodireitodedarordenseo

poderdeesperarobediência.Aresponsabilidadeéumaconsequêncianaturalda

autoridadeesignificaodeverdeprestarcontas;

3)Disciplina:dependedeobediência,aplicação,energia,comportamentoe

respeitoaosacordosestabelecidos;

4)Unidadedecomando:cadaempregadodevereceberordensdeapenasum

superior;

5)Unidadededireção:umacabeçaeumplanoparacadaconjuntodeatividades

quetenhamomesmoobjetivo;

6)Subordinaçãodosinteressesindividuaisaosgerais;

7)Remuneraçãodopessoal:devehaverjustaegarantidasatisfaçãoparaos

empregadoseparaaorganizaçãoemtermosderetribuição;

8)Centralização:refere-seàconcentraçãodaautoridadenotopodahierarquia

daorganização;

9)Cadeiaescalar:linhadeautoridadequevaidoescalãomaisaltoaomaisbaixo

dahierarquia;

10)Ordem:umlugarparacadacoisa,ecadacoisaemseulugar;

11)Equidade:amabilidadeejustiçaparaalcançaralealdadedosempregados;

12)Estabilidadedopessoal:arotatividadedopessoaléprejudicialparaa

eficiênciadaorganização;

13)Iniciativa:acapacidadedevisualizarumplanoeassegurarpessoalmenteseu

sucesso;

14)Espíritodeequipe:aharmoniaeuniãoentreaspessoassãograndesforças

paraaorganização.

Figura1.6–CaracterísticasdaTeoriaClássica

ComoproblemasdaTeoriaClássica,podemoscitarafaltadepreocupaçãocom

aorganizaçãoinformaldasorganizações(sófocavanaorganizaçãoformal–

linhasdeautoridade,descriçãodecargos,hierarquiaetc.),alémdeumaênfase

exageradanacentralização,comooprincípiodaunidadedecomando

exemplifica.Aideiadeumaorganizaçãoflexívelaindanãoestavanaagenda.

Infelizmente,aTeoriaClássicatambémnãosepreocupoumuitocomos

aspectosligadosàspessoas.Temascomocomunicação,motivação,negociaçãoe

liderançaaindaerampoucorelevantesnestesestudos.17Esse“vácuo”seria

ocupadopelaTeoriadasRelaçõesHumanas.

1.3.TeoriadasRelaçõesHumanas

Ocrescimentodasciênciassociais,comoapsicologia,levouadiversosestudos

dentrodocontextodohomemnotrabalho.Alémdisso,noiníciodadécadade

1930,aeconomiapassouporumagrandedepressãoemtodoomundo.Coma

crise,odesempregocresceumuito.

Asmáscondiçõesdetrabalhopredominavamnaindústria,eosconitosentre

trabalhadoresepatrõesestavamaumentando.Nessaépoca,ocorrerammuitas

greveseconitosnasfábricasportodoomundo.Aideiadequeohomemdeveria

serumaengrenagemdeuma“máquina”passouanãosermaisaceita.18

OTaylorismocomeçouasercriticadopornãosepreocuparcomoaspecto

humano.Alémdisso,aprodutividadeprometidamuitasvezesnãoseconcretizou.

Nessecenário,aTeoriadasRelaçõesHumanascomeçouatomarforma.

Assim,aTeoriadasRelaçõesHumanasbuscouoaumentodaprodutividade

atravésdeumaatençãoespecialàspessoas.Deacordocomseusteóricos,seos

gestoresentendessemmelhorseusfuncionáriosese“adaptassem”aeles,assuas

organizaçõesteriammaiorsucesso.19

Dentreosestudosqueimpulsionaramessateoriasedestacouotrabalhodeum

pesquisadordeHarvard:EltonMayo.

Figura1.7–ContextodaAbordagemHumanística

Esseautordesenvolveuumapesquisadentrodeumaindústriadaempresa

WesternElectric,emHawthorne.Seuintuitoinicialfoientenderoefeitoda

iluminaçãonodesempenhohumano.

QualfoiasurpresadeMayoaodescobrirqueumamudançanailuminação–

sejaelaqualfor–aumentavaamotivaçãodosempregados.Assim,eleaospoucos

foicompreendendoquenaverdadeoqueestavamotivandoessesfuncionáriosera

aatençãodospesquisadores,nãoailuminaçãoemsi.20

Essestrabalhadorespassaramasesentirimportantes.Passaramaperceberque

seutrabalhoestavasendoobservadoemedidoporpesquisadores.Comisso,se

esforçavammais.Ailuminaçãoemsieraumaspectomenor.Jáosentimentode

orgulhoporfazerumtrabalhobem-feitoerafundamentalnoaumentoda

produtividade.

Comessasdescobertas,todooenfoquedaadministraçãofoialterado.Ofocode

umgestornãodeveriaservoltadoaosaspectossiológicosdotrabalhador,mas

aosaspectosemocionaisepsicológicos.

Comesseaparecimentodanoçãodequeaprodutividadeestáligadaao

relacionamentoentreaspessoaseaofuncionamentodosgruposdentrodeuma

empresa,nasceuessanovateoria.Oconceitoquesermouentãofoiodehomem

social.21

Figura1.8–ExperiênciadeHawthorne

DeacordocomSobral,22asconclusõesdapesquisadeHawthorneforam:

üAintegraçãosocialafetaaprodutividade–assim,nãoéacapacidade

individualdecadafuncionáriooquedenesuaprodutividade,esimasua

capacidadesocial,suaintegraçãonogrupo.

üOcomportamentoédeterminadopelasregrasdogrupo–osfuncionários

nãoagemisoladamenteounovácuo,mascomomembrosdeumgrupo.

üAsorganizaçõessãoformadasporgruposinformaiseformais–volta-seo

focoparaosgruposqueexistemdemodoinformalnaempresaequenãosão

relacionadosaoscargosefunções.

üAsupervisãomaiscooperativaaumentaprodutividade–osupervisormais

ecazéaquelequetemhabilidadeecapacidadedemotivareliderarseus

funcionáriosemtornodosobjetivosdaempresa.

üAautoridadedogerentedevesebasearemcompetênciassociais–O

gerentedevesercapazdeinteragir,motivarecomandarseusfuncionários.

Apenasterconhecimentotécnicodosmétodosdeproduçãonãoémaisvisto

comoobastante.

Figura1.9–CaracterísticasdaTeoriadasRelaçõesHumanas

Dessamaneira,aTeoriadasRelaçõesHumanastrouxeparaodebatea

necessidadedesecriarumambientedetrabalhomaisdesaadoredese

compreenderainuênciadamotivaçãoedosaspectosdeliderançana

produtividadedasorganizações.23

Alémdisso,asrecompensasnãopoderiam

carreduzidasaosaspectos

materiais.Oreconhecimentosocialéumaforçamotivadora,eumambientede

trabalhosaudáveltambéminfluencianaprodutividade.

Apesardisso,aTeoriadasRelaçõesHumanasrecebeumuitascríticas.24A

primeiradelaséadequepermaneceuaanálisedaorganizaçãocomoseela

existissenovácuo,semserelacionarcomo“mundoexterior”.Ouseja,a

abordagemdesistemafechadosemanteve.

Asegundaéadequenemsemprefuncionários“felizes”esatisfeitossão

produtivos.Ouseja,apenasosaspectospsicológicosesociaisnãoexplicamde

todoofatorprodutividade.

Outracríticaéadequeexistiuuma“negação”doconitoinerenteentreos

funcionárioseaempresa.25Osobjetivosindividuaissãomuitasvezesdiferentes

dosobjetivosorganizacionais.Esseconflitodeveseradministradoenão“negado”.

Assim,podemosdizerqueaTeoriadasRelaçõesHumanasse“esqueceu”dos

aspectostécnicosenvolvidosnaprodutividade.Oaspectohumanoéimportante,

masnãoéaúnicavariáveldaprodutividadeedosucessodeumaorganização.

Figura1.10–ProblemasdaTeoriadasRelaçõesHumanas

1.4.Burocracia

Otermo“burocracia”éderivadodotermofrancês“bureau”(signica

escritório)edotermogrego“kratia”,queserelacionaapoderouregra.Dessa

forma,aburocraciaseriaummodeloemqueo“escritório”ouosservidoresde

carreiraseriamosdetentoresdopoder.

Ateoriadaburocraciafoimaisumaquebuscouadaptarasorganizaçõesaos

novosdesaosdeumasociedademoderna,comsuaurbanizaçãoaceleradae

industrialização,quemudouocenárioeconômicoesocialemqueelasoperavam.

Comaindustrializaçãoeaintroduçãoderegimesdemocráticosnomdo

séculoXIX,associedadescaramcadavezmaiscomplexas.Aintroduçãoda

máquinaavaporacarretouumaevoluçãotremendadosmeiosdetransporte.

OmodeloburocráticodeWebertinhacomoobjetivoumamaiorprevisibilidadee

padronizaçãododesempenhodosseusfuncionários,atingindoassimumamaior

eficiência.

SeantesselevavamesesparaumaviagemdoBrasilàEuropa,porexemplo,uma

viagempormeiodenaviosavaporpassouaserfeitaempoucosdias.

Otremavaporfezamesmarevoluçãonotransporteinterno.Dessaforma,as

notíciaspassarama“correr”muitomaisrápido,eosprodutosdecadaregião

puderampassarasercomercializadosemcadavezmaismercadosconsumidores.

Essesfatoreslevaramaumaurbanizaçãoacelerada,poisasindústrias,agoracom

máquinas,necessitavamdecadavezmais“braços”parapoderproduziremlarga

escala.

Diantedoaumentodademandaportrabalhadoresnosetorindustrial,os

saláriosnaindústriaficarammelhoresdoqueosdocampo.

Dessaforma,oêxodorural(massadetrabalhadoressaídosdocampoese

dirigindoparaascidadesembuscadetrabalho)foimarcantenesseperíodo.

Essaspessoasencontravamnacidadegrandeumarealidadetotalmentediferente

daqualestavamacostumadas,poistinhamnecessidadesqueoEstado(quetinha

umalosoaliberal)aindanãoestavacapacitadoparaatender.Eraoiníciodo

queiríamosdenominar“sociedadedemassa”.

Asempresaseosgovernosnecessitavamdeumaadministraçãomaisracionale

quemaximizasseosrecursos,alémdeterumamaiorestabilidadeeprevisibilidade

emsuasoperaçõeseprocessosdetrabalho.

OEstado,porexemplo,queantessósepreocupavaemmanteraordeminterna

eexterna,passouaterdeseorganizarcadavezmaisparainduzirocrescimento

econômico,aumentarainfraestruturadopaíseprestarcadavezmaisserviçosà

população.

OPatrimonialismo(modelodegestãopúblicaemqueopatrimôniopúblicose

“mesclava”comoprivadoeasrelaçõessebaseavamnaconançaenãonomérito)

nãoconseguiamaisatenderaessenovoEstado,queconcentravacadavezmais

atividadesemsuamáquina.

Omodeloburocrático,inspiradoporMaxWeber,veioentãosuprira

necessidadedeimporumaadministraçãoadequadaaosnovosdesaosdoEstado

modernoedasgrandesempresas,comoobjetivodecombaterodesperdício,a

ineciênciaeacorrupção,ouseja,umaadministraçãomaisracionaleimpessoal.

Nocasoespecícodasgrandesempresas,omodelobuscavatambémoaumento

consistentedaprodução.

Figura1.11–Contextodaburocracianosetorpúblico

Dessaforma,omodeloburocráticosurgiucomoumanecessidadehistórica

baseadaemumasociedadecadavezmaiscomplexa,emqueasdemandassociais

cresceram,ehaviaumambientecomempresascadavezmaiores,comuma

populaçãoquebuscavaumamaiorparticipaçãonosdestinosdosgovernos.

Portanto,nãosepodiamais“depender”doarbítriodeumsóindivíduo.

UmacoisaquedevemosteremmenteéqueaBurocraciafoiumagrande

evoluçãodomodelopatrimonialistadegovernança.Weberconcebeua

Burocraciacomoomodelomaisracionalexistente,oqualseriamaisecientena

buscadosseusobjetivos.

Continuando,ascaracterísticasprincipaisdaBurocraciasão:

ØFormalidade–aautoridadederivadeumconjuntodenormaseleis,

expressamenteescritasedetalhadas.Opoderdochefeérestritoaosobjetivos

propostospelaorganizaçãoesomenteéexercidonoambientedetrabalho–

nãonavidaprivada.Ascomunicaçõesinternaseexternastambémsãotodas

padronizadaseformais.

ØImpessoalidade–Osdireitosedeveressãoestabelecidosemnormas.As

regrassãoaplicadasdeformaigualatodos,conformeseucargoemfunção

naorganização.SegundoWeber,aBurocraciadeveevitarlidarcom

elementoshumanos,comoaraiva,oódio,oamor,ouseja,asemoçõeseas

irracionalidades.Aspessoasdevemserpromovidaspormérito,enãopor

ligaçõesafetivas.Opoderéligadonãoàspessoas,masaoscargos–sósetem

opoderemdecorrênciadeestarocupandoumcargo.

ØProfissionalização–Asorganizaçõessãocomandadasporespecialistas,

remuneradosemdinheiro(enãoemhonrarias,títulosdenobreza,sinecuras,

prebendasetc.),contratadospeloseuméritoeseuconhecimento(enãopor

algumarelaçãoafetivaouemocional).

Omodeloburocráticosecaracterizoupelameritocracianaformade

contrataçãoepromoçãodosempregados.Nosetorpúblico,temosoexemplodos

concursospúblicos.

Essetipodeingressonascarreiraspúblicasbuscoueliminarohábitoarraigado

domodelopatrimonialistadeocuparespaçonoaparelhodoEstadoatravésde

trocasdecargospúblicosporfavorespessoaisaosoberano.

Antigamente,aspessoasseriamnomeadasporseusconhecimentose

habilidades,nãoporseuslaçosfamiliaresoudeamizade.Prebendasesinecuras,

característicasdomodelopatrimonialista,ouseja,aquelassituaçõesemque

pessoasocupamfunçõesnogovernoganhandoumaremuneraçãoemtrocade

poucoounenhumtrabalho,sãosubstituídaspeloconcursopúblicoepelanoção

decarreira.

Dessaforma,oquesebuscaéaprofissionalizaçãodofuncionário,sua

especialização.DeacordocomWeber,cadafuncionáriodeveserumespecialista

noseucargo.Assim,devesercontratadocombaseemsuacompetênciatécnicae

terumplanodecarreira,sendopromovidodevidoàsuacapacidade.

Cuidado–asbancasnormalmentetentamconfundiromodeloidealdaBurocraciade

Webercomseusproblemasoudisfunçõesnaprática!

Veja,emresumo,ascaracterísticasdaBurocracia:

Formalidade

Impessoalidade

Profissionalismo

Autoridadeéexpressa

Isonomianotratamento.

Comandoédosespecialistas.

emleis.

Meritocracia.

Remuneraçãoemdinheiro.

Comunicaçãoé

Racionalidade.

Administradoréespecialista–noção

padronizada.

Sistemalegaleeconômico

decarreira.

Controlede

previsível.

Hierarquia.

procedimentos.

Figura1.12–CaracterísticasdaBurocracia

Aimpessoalidadenotratamentofoipensadademodoaevitarasemoçõesnos

julgamentosedecisões.Seria,portanto,ummododealcançarumaisonomiano

tratamentodaspessoaseumamaiorracionalidadenatomadadedecisões.Semal

conhecemosnossosfuncionários,tenderemosanosconcentrarnosaspectosmais

“concretos”dosproblemas,nãoémesmo?

Acomunicaçãoformalajudarianisso,poisoscanaisdetransmissãode

informações(comoosofíciosememorandos)nãoabremespaçoparaumcontato

maisíntimoepessoal.Boatose“fofocas”nãosãousualmenteescritosemcartas,

nãoéverdade?

Alémdisso,outracaracterísticaimportantedaburocraciaéanoçãode

hierarquia.Todaaorganizaçãoéfeitademodohierarquizado,comaautoridade

sendobaseadanasnormaseleisinternasquedeterminamacompetênciadecada

cargo.Assim,seuchefetemopodereaautoridadeconcedidosaelepordeterum

cargoacimadoseu.Aobediênciaéaocargoenãoàpessoa.26

Portanto,asorganizaçõessãoestruturadasemváriosníveishierárquicos,em

queoníveldecimacontrolaodebaixo.Éoquechamamosdeestrutura

verticalizada,naqualasdecisõessãotomadasnacúpula(topodahierarquiaou

nívelestratégico).

Essasituaçãoacabagerandoumademoranatomadadedecisõesenouxode

informaçõesdentrodaorganização!

EntreasprincipaisvantagensqueaBurocraciatrouxe,podemoscitar:o

predomíniodeumalógicacientícasobreumalógicadaintuição,do

“achismo”;areduçãodosfavoritismosedaspráticasclientelistas;uma

mentalidademaisdemocrática,quepossibilitouigualdadedeoportunidadese

tratamentobaseadoemleiseregrasaplicáveisatodos.

Hojeemdia,otermoBurocraciavirousinônimodeineciênciaelentidão,

poisconhecemososdefeitosdomodelo(quechamamosdedisfunçõesda

Burocracia),maselefoiumpassoadiantenasuaépoca!

NaBurocracia,existeumadesconançaextremaemrelaçãoàspessoas,

portantosãodesenvolvidoscontrolesdosprocessosedosprocedimentos,de

formaaevitarosdesvios.

Ouseja,osfuncionáriostêmpoucadiscricionariedade,ouliberdadedeescolha

damelhorestratégiapararesolverumproblemaouatenderseusclientes!Tudoé

padronizado,émanualizado!Comisso,osservidorespassamasepreocuparmais

emseguirregulamentosdoqueematingirbonsresultados.

Olhecomoessetemajáfoicobrado:

1.(FCC/MP/SE/Administrador/2009)NÃOconstituicaracterísticadomodelodeAdministração

PúblicaBurocrática,quetementreseusprincipaisexpoentesMaxWeber:

a)ênfasenaideiadecarreiraeprofissionalizaçãodocorpofuncionalpúblico;

b)estruturahierárquicafortementeverticalizada,impessoalidadeeformalismo;

c)rigidezdocontroledosprocessos,compredominânciadocontroledalegalidadecomocritériode

avaliaçãodaaçãoadministrativa(dueprocess);

d)rotinaseprocedimentossegundoregrasdefinidasapriori,emdetrimentodaavaliaçãoporresultados;

e)utilizaçãodecritérioseminentementepolíticosparacontrataçãoepromoçãodefuncionários,em

detrimentodaavaliaçãopormérito.

Essaquestãopedeaalternativaincorreta.AalternativaAdescreveumdos

pilaresdaBurocracia,queéoprossionalismo,ouseja,anoçãodequeoservidor

doEstadodeveserumespecialista,eestácorreta.

Esseservidordeveentãoserremuneradoemdinheiro(enãoemfavoresou

honrarias,comoacontecianaépocadaMonarquia)eteriaseuingressoesua

progressãonacarreiraatravésdameritocracia,ouseja,pormeiodoresultadode

seutrabalho,enãodealgumfavoritismoouamizadedealguémpoderoso.

AalternativaBtambémestácorreta,eabordaprincípiosquesãoimportantes

naburocracia,comoaimpessoalidade(todossãotratadosdeacordocomasregras

enormasquevalematodos),oformalismo(oprópriosistemaderegrasenormas

queregulaasorganizações)eahierarquia.

AalternativaCabordaoutracaracterísticadaBurocracia,ocontroleapriori,

ouporprocedimentos.Ateoriadaburocraciasebaseiaemumadesconançano

papeldaspessoasnaorganização,portantotodososprocessossãoformalizados,

demodoadeixarpoucadiscricionariedadeaogestorpúblico,ouseja,oservidor

públicotemdiversasregrasaseguireseráavaliadoporelas.

EssacaracterísticadaBurocracia(controledeprocedimentos)criouentãouma

culturalegalista,emqueofuncionáriosepreocupamaisemcumprirregrase

regulamentos,emenosemprestarumserviçodequalidadeeematingirresultados

superiores.AalternativaDtocanessemesmoponto,etambémestácorreta.

OgabaritoémesmoaalternativaE,poiséameritocraciaqueserelaciona

comaTeoriadaBurocracia,enãoasnomeaçõesporcritériospolíticos.Jáaté

seioquevocêestápensando!Mas,professor,etodasessasnomeaçõesdepolíticos

paracargosnogoverno?Pessoal,essassãocaracterísticasaindapersistentesdo

patrimonialismoemnossosistemaadministrativo,enãocaracterísticasdomodelo

“puro”daBurocracia,ok?

Nonossocontextoatual,temosaindaaspectospresentesquesãoherançasdo

patrimonialismo(nomeaçõesemcargosdeconança)easpectosdaTeoriada

Burocracia(concursospúblicosenoçãodecarreira,entreoutros).

Alémdisso,éimportantenãoconfundiraTeoriadaBurocracia,ouseumodelo

“puro”,comosproblemasqueaBurocraciacausou–oquechamamosde

disfunçõesdaBurocracia.Normalmenteabancacitaráuma“disfunção”da

BurocraciaediráqueéumacaracterísticadaTeoriadaBurocracia.

Nocasodaquestãoanterior,asnomeaçõessembasenoméritorealmente

ocorrem.Sabemosqueéumdosproblemasdemuitasorganizaçõesnaprática,mas

nãofazpartedaTeoriadaBurocracia,ouseja,domodeloidealizadoporWeber!

Portanto,vocêsdevementenderquenenhummodeloouteoriaexistiu

isoladamente,masqueconviverameconvivemjuntos.Ouseja,nuncaaplicamos

omodelo“puro”daburocraciaweberiana.Presteatenção,poisasbancas

costumamcobrarmuitoisso.

AsprincipaisdisfunçõesdaBurocraciasão:

ØDiculdadederespostaàsmudançasnomeioexterno–visão

excessivamentevoltadaparaasquestõesinternas(sistemafechado,ouseja,

autorreferente,comapreocupaçãonãonasnecessidadesdosclientes,mas

nasnecessidadesinternasdaprópriaburocracia).

ØRigidezeapreçoextremoàsregras–ocontroleésobreprocedimentose

nãosobreresultados,levandoafaltadecriatividadeeineficiências.

ØPerdadavisãoglobaldaorganização–adivisãodetrabalhopodelevara

queosfuncionáriosnãotenhammaisacompreensãodaimportânciadeseu

trabalhonemquaissãoasnecessidadesdosclientes.

ØLentidãonoprocessodecisório–hierarquia,formalidadeefaltade

conançanosfuncionárioslevamaumademoranatomadadedecisões

importantes.

ØExcessivaformalização–emumambientedemudançasrápidasnãose

conseguepadronizareformalizartodososprocedimentosetarefas,gerando

umadiculdadedaorganizaçãodeseadaptaranovasdemandas.A

formalizaçãotambémdificultaofluxodeinformaçõesdentrodaempresa.

Podemosresumirasprincipaisdisfunçõesouproblemasdomodeloburocrático

noseguintequadro:

Figura1.13–DisfunçõesdaBurocracia

1.5.TeoriaEstruturalista

ATeoriaEstruturalistaveiocomoumacríticatantoàsteoriasclássicasquantoà

TeoriadasRelaçõesHumanas.Umdeseusmaisimportantesteóricos,Amitai

Etzione,27consideravaaorganizaçãocomo“umcomplexodegrupossociaiscujos

interessespodemounãoserconflitantes”.

Dessamaneira,essateoriabuscou“complementar”ousintetizarasteorias

anteriores(clássicasehumanas),poisacreditaqueaquelasfocavamapenasem

partesdotodo.Destaforma,aideiaprincipalfoiconsideraraorganizaçãoem

todososaspectoscomoumasóestrutura–integrandotodasas“visões”

anteriores.28

Assim,umaspectoimportantefoiabuscadeumaanálisetantodaorganização

formal(abordadanateoriaclássica)quantodainformal(abordadanateoriadas

relaçõeshumanas).Dessamaneira,deveriaexistirumequilíbriodessasduas

visões.29

Paraosestruturalistas,asociedademodernaseriaumasociedadede

organizações.Ohomemdependeriadessasorganizaçõesparatudoenestas

cumpririaumasériede“papéis”diferentes.30

Assim,apareceuoconceitodehomemorganizacional,31aqueleque

desempenhadiversospapéisnasdiversasorganizações.

OutroconceitofoitrazidoporGouldner:32asdiferentesconcepçõesdas

organizações.Paraesseautor,existiriamomodeloracionaleomodelonaturalde

organização.

Omodeloracionalseriabaseadonocontroleenoplanejamento.Aideiaeraa

deumsistemafechado,compoucaincertezaepreocupaçãoparacomo“mundo

externo”àorganização.

Ooutromodeloeraonatural.Neste,existeanoçãodequearealidadeéincerta

edequeaorganizaçãoéumconjuntodeórgãosinter-relacionadose

interdependentes.Assim,éummodeloquesepreocupacomas“trocas”como

ambienteexterno,ouseja,éummodelodesistemaaberto.

1.6.TeoriadosSistemas

ATeoriadosSistemasnaAdministração–TGS–foiderivadadotrabalhodo

biólogoLudwigvonBertalanffy.Elebuscouintegrartodasasáreasdo

conhecimento.33

Umsistema,deacordocomBertalanffy,éumconjuntodeunidades

reciprocamenterelacionadasparaalcançarumpropósitoouobjetivo.34

Assim,aTeoriadosSistemasacolheuoconceitonoqualasorganizaçõessão

sistemasabertos,ouseja,quetrocamcontinuamenteenergia(oumatéria-prima,

informaçõesetc.)comomeioambiente.

Portanto,nãopodemosentenderumaorganizaçãosemsaberocontextoemque

elaopera.Domesmomodo,umaorganizaçãoéasomadesuaspartes(gerênciade

marketing,gerênciadenançasetc.)eumaáreadependedaoutra–oconceitode

interdependência.

FiqueAtento:Atualmente,asorganizaçõessãovistascomosistemasabertos

Ouseja,nãoadiantanadaumaáreadaempresasesairmuitobem(áreade

vendas,porexemplo)seoutraáreaestátendodiculdades(produção,por

exemplo).Nocasocitado,aempresaperderiaosclientespornãoconseguir

cumprirasvendasefetuadas.

Dessaforma,oadministradordeveterumavisãodotodo!Decomoasáreasda

organizaçãointeragemequaissãoasinterdependências.

Figura1.14–CaracterísticasdaTeoriadosSistemas

OsprincipaisconceitosdaTeoriadosSistemassão:

ØEntrada–serelacionacomtudooqueosistemarecebedoambienteexterno

parapoderfuncionar.

ØSaída–éoqueosistemaproduz.Umasaídapodeser:energia,uma

informação,umprodutoetc.

ØFeedback–éoretornosobreoquefoiproduzido,demodoqueosistema

possasecorrigiroumodificar.

ØCaixa-preta–serelacionacomumsistemaemqueo“interior”nãoé

facilmenteacessível(comoocorpohumano,porexemplo).Assim,sótemos

acessoaoselementosdeentradaesaídaparasabermoscomoelefunciona.

Vejaoutrosconceitosimportantesnográficoaseguir:

Figura1.15–ConceitosdaTeoriadosSistemas

1.7.TeoriaContingencial

ParaaTeoriaContingencial,nãoexisteuma“fórmulamágica”paraseresolver

osproblemasdasorganizações.Cadasituaçãopedeumarespostadiferente.Assim,

tudoérelativo.Tudodepende.

Ouseja,antesqueumadministradorpossadeterminarqualéo“caminho”

corretoparaumaempresa,énecessáriaumaanáliseambiental.Assim,dependendo

dasituaçãodaempresa,suaestratégiaouatecnologiaenvolvida,o“caminho”ou

mododegerenciarserádefinidodeumamaneiraoudeoutra.

Osfatoresprincipais–comootamanhodaempresaeseuambiente–são

consideradoscontingências,quedevemseranalisadasantesdesedeterminarum

cursodeação.Portanto,nãoexistemaisa“melhormaneira”deseadministrar

umaorganização.

Existiriaumarelaçãofuncionalentreosistemaeoambiente.Teríamosas

variáveisdependenteseasvariáveisindependentes.Todasasaçõesdedentro

daempresaseriamvariáveisdependentesdoquevieraocorrernoambiente

externo–quesãoasvariáveisindependentes.

Alémdisso,essateoriapostulaqueexistemváriasmaneirasdesealcançarum

objetivo.Oqueumgestordevebuscaréumajusteconstanteentreaorganizaçãoe

seumeio,suascontingências.35

Figura1.16–CaracterísticasdaTeoriaContingencial

Dentreessascontingênciasimportantes,Sobral36cita:oambienteinternoe

externo,atecnologia,otamanhoeotipodetarefa.

Figura1.17–Algunsfatoresimportantes

Umaconsequênciapráticadessasideiasnomundoorganizacionalfoia

tendênciadeasorganizaçõessetornaremmaisexíveis,paraquepossamreagir

maisrápidoàsmudançasnoambiente.

Entreosnovosmodelosadotados,temosasorganizaçõesemrede.Estassão

muitomaisexíveisedependemdeumanovavisãodemundoporpartedeseus

gestores.

DeacordocomMotta,37oambienteéumaredeformadapordiversas

organizaçõesinterligadas.Porexemplo,omercadoautomobilísticoéformadopor

diversasmontadoras,oficinas,seguradoras,fábricasdepeçasetc.

Alémdisso,aprópriaorganizaçãoécompostapordiversasredessociais

internas.Osdiversosdepartamentoseáreassãodependentesunsdosoutros.

Comoessasáreasougruposestãosempreemcontato,seusmembrosrecebemuma

pressãoouinfluênciaqueéderivadadessecontato.

Portanto,asdiversasdecisõesdosatoressociaisdevemserentendidasdentro

dessaspressõesquerecebemedasrelaçõesdepoderinternaseexternas.38Ouseja,

comodizemosno“popular”:temosde“calçarosapato”dooutroparaentender

suasdecisões.

Nasorganizaçõesemrede,emvezdeaempresa“verticalizar”suaproduçãoe

“fazertudosozinha”–comocomprarumaindústriaecontratarfuncionários–,faz

umcontratocomumparceiroquepassaacumpriressafunção.

Dessaforma,aorganizaçãoemrede“troca”ocontrolehierárquicodaprodução

pelagestãodecontratosdeparceria.Seademandamudareoprodutonãofor

maisdesejadopelosclientes,amudançaestratégicaseriamaisfácil(émaisfácil

cancelarumcontratodoquedemitirmilharesdefuncionários,nãoéverdade?).

Entretanto,apesardeessateoriaacertaraoidenticararealidadeea

complexidadedaatuaçãodasorganizaçõesatualmente,acaba“caindo”emum

relativismoexagerado.

Ouseja,paraaTeoriaContingencialtudodepende!Dessamaneira,não

existemprescriçõesquepossamsergeneralizadas.Cadacasoserásempreumcaso

específicoequedeveseranalisadodentrodeseucontexto.

Alémdisso,ascontingênciasqueinuenciamasituaçãodeumaorganização

são,muitasvezes,inúmeras.Ouseja,adeniçãodo“caminho”aserseguidopor

umaempresapodeserumtrabalhobastantecomplexo.

Figura1.18–CríticasàTeoriaContingencial

Vamosveragoraumaquestão?

2.(Esaf/MPOG/APO/2010)Oestudodaevoluçãodopensamentoadministrativopermiteconcluir,

acertadamente,que:

a)asTeoriasCientíficaedasRelaçõesHumanassãoabordagensdesistemasabertos;

b)aTeoriadasRelaçõesHumanasdesprezaosobjetivosorganizacionais;

c)aTeoriadaContingênciaenfatizaaimportânciadatecnologiaedoambiente;

d)asTeoriasEstruturalistaedosSistemasrefletemumaabordagemprescritivaenormativa;

e)aTeoriaComportamentalconcebeofuncionáriocomoum“homemsocial”.

Aprimeiraalternativaestáerrada,poisessasteoriassãoabordagensdesistema

fechado,ouseja,nãosepreocupamcomoambienteexternoeseuimpactona

organização.

AletraBtambéméabsurda,poisaTeoriadasRelaçõesHumanasnãodespreza

osobjetivosorganizacionais.Elaapenastemum“novocaminho”paraqueesses

objetivossejamalcançados.

AletraCestáperfeitaeéonossogabarito.ATeoriadaContingênciarealmente

vêatecnologiaeoambientecomofatoresimportantes.Entretanto,aletraDestá

incorreta,poisessasteoriastêmumaabordagemdescritivaeexplicativa.

JáaletraEinverteosconceitos.Ateoriarelacionadacomohomemsocialéa

TeoriadasRelaçõesHumanas.ATeoriaComportamentalérelacionadacomo

homemadministrativo.Assim,onossogabaritoémesmoaletraC.

QuestõesComentadas

3.(Cespe/Unipampa/Administrador/2009)Ofatodeosempregadosdesenvolveremsuaprópria

maneiradeexecutarastarefasconstituiquadrosimilaraopreconizadopelaTeoriada

AdministraçãoCientíca,quedefendeaautonomiaeiniciativadoscolaboradoresnarealização

desuasatividades.

AAdministraçãoCientíca,lançadaporFrederickTaylor,secaracterizava

exatamentepordesenharuma“melhor”maneiradeexecutarumatarefa.Através

dosestudosdetemposemovimentos,sechegavaaumamaneira“ideal”defazer

algumtrabalho,queeraentãopadronizada.

Essaconcepçãoveioexatamentelidarcomoproblemadecadaumfazero

trabalhodamaneiraqueachavamelhor,levandoaumaperdadetempoeesforço

físico.Ogabaritoéquestãoerrada.

4.(Cesgranrio/BNDES/Administração/2008)Desdeosprimórdiosdaadministração,especificamente

daAdministraçãoCientícadeTaylor,asorganizaçõestiveramdeseadaptaramovimentosde

constantesutuaçõesemsuasgestões.Emseu“estudodetemposemovimentos”,Taylor

analisavaasunidadesbásicasdetrabalhodecadaindivíduo.Partindo-sedessapremissa,a

validadedosestudosdetemposemovimentossedápela:

a)observaçãodasaçõesedaformacomoelassãoexecutadasnaprodução;

b)complementaçãodetarefas,verificandoasquesãoadequadasàsnovasrealidadesdaempresa;

c)implementaçãodenovastécnicasdetrabalhonaprodução;

d)participaçãoativadecadafuncionárionoprocessoprodutivo;

e)combinaçãodevaloresindividuaisnoprocessoprodutivo.

Tayloranalisavacienticamentecadamovimentodosfuncionáriosnaexecução

deumatarefa.Issolhepossibilitava“construir”ummododetrabalhoque

poupassetempoeesforçodostrabalhadores.Assim,aletraAestácorretaeéo

nossogabarito.

5.(Funiversa/MTUR/Administrador/2010)Assinaleaalternativaqueapresentaumadasdiferenças

entreasTeoriasClássicaseasdasRelaçõesHumanas.

a)AestruturadasTeoriasdasRelaçõesHumanaseramecanicistaeimpessoal.

b)Ocomportamentonaorganização,segundoasteoriasclássicas,eraprodutodesentimentose

atitudes.

c)Ofocoestudadonasteoriasclássicasfoiotrabalhoeasnecessidadeseconômicasdostrabalhadores.

d)Aênfasenasegurançapessoalenasnecessidadessociaisdostrabalhadoresparaoalcancedasmetas

organizacionaisfoianalisadanasteoriasclássicasdaadministração.

e)Empregadosfelizes,quebuscavamproduzirmais,foramresultadosobtidosnasteoriasclássicasda

administração.

ATeoriadasRelaçõesHumanasnãoeraimpessoal,pelocontrário.Nessateoria

équesurgiuapreocupaçãocomaspessoasnocontextodotrabalho.

Jáasegundafraseinverteuoconceito,poisestaeraumapreocupaçãodaTeoria

dasRelaçõesHumanas,nãodasteoriasclássicas.AletraCestáperfeitaeéonosso

gabarito.

Entretanto,aletraDtambéminverteuoconceito,poisessaênfaseapareceuna

TeoriadasRelaçõesHumanas.OmesmoocorrenaletraE.Portanto,onosso

gabaritoémesmoaletraC.

6.(Cespe/MDS/Administrador/2006)Impessoalidade,hierarquia,exibilizaçãodeprocedimentos,

especializaçãoeênfasenoscontrolessãocaracterísticasdosmodelosdasorganizações

burocráticosdegestão.

Afraseestáquasetodacorreta,masnãoécaracterísticadaBurocraciaa

exibilizaçãodeprocedimentos!Muitopelocontrário.NaBurocracia,não

existeconançanosfuncionários.Portanto,osprocedimentossãodetalhados,de

formaqueosindivíduostenhamdeseguir“àrisca”seuspassos.Assim,afraseestá

errada.

7.(Esaf/STN/Desenv.Institucional/2008)Vistacomoumaformadeorganizaçãoquesebaseiana

racionalidade,naadequaçãodosmeiosaosobjetivospretendidoscomoformadesegarantira

máximaeciênciapossível,aBurocraciasecaracterizaporencamparosseguintesatributos,

exceto:

a)impessoalidadenasrelações;

b)competênciatécnicaemeritocracia;

c)informalidadedasnormaseregulamentos;

d)hierarquiadaautoridade;

e)completaprevisibilidadedocomportamento.

QuestãobemtranquiladaEsaf.Todasasalternativasestãocorretas,menosa

letraC.ABurocraciasecaracterizaporumagrandeformalidade(enão

informalidade)dasnormaseregulamentos.Esteé,portanto,onossogabarito.

8.(FCC/Alesp/SP/GestãoProjetos/2010)Comrelaçãoàadministraçãopúblicaburocrática,

considere.

I.SurgenasegundametadedoséculoXIX,naépocadoEstadoliberal,comoobjetivode

combateracorrupçãoeonepotismopatrimonialista.

II.Essemodelodegestãopossuicomoprincípiosorientadoresaprossionalização,ouseja,a

ideiadecarreiraehierarquiafuncional,aimpessoalidadeeoformalismo.

III.Ospressupostosdaadministraçãoburocráticasãoaconançaprévianosadministradores

públicosenoscidadãosqueaeles,administradorespúblicos,dirigemdemandas.

IV.Ocontrolepodetransformar-senaprópriarazãodeserdofuncionário;voltando-separasi

mesmo,perdendoanoçãodesuamissãobásica,queéserviràsociedade.

V.Aadministraçãoburocráticatemcomoprincipalqualidadeaefetividadenoalcancedos

resultados;seufococentraléaeficiênciadoEstado.

EstácorretooqueseafirmaAPENASem:

a)IeI;

b)I,I,IIeV;

c)I,IIeIV;

d)IeV;

e)II,IVeV.

Aprimeirafraseestácorreta,poisdescreveocontextoemqueaTeoriada

BurocraciafoicriadaporMaxWeber.Acriaçãodessemodeloburocrático

ocorreunonaldoséculoXIXebuscavaummeiomaisracionaleecientepara

gerirasorganizaçõesdoqueopatrimonialismo(quetinhacomodefeitoa

tendênciaàcorrupçãoeaonepotismo–nomeaçãodeparentesparacargos

públicos).

Asegundafraseestáigualmentecorreta,descrevendoosprincípiosda

Burocracia,quejávimosanteriormente.Entretanto,aterceirafraseestáincorreta,

poisnãoexisteessaconançaprévianosservidorespúblicos.Muitopelo

contrário!

Nahoraemqueseimaginaquesejanecessáriomanualizartodososprocessos,

deformaareduziraomáximoaliberdadequeoadministradorteráparafazerseu

trabalho,jáexisteumaideiadedesconfiançaemseutrabalho,nãoémesmo?

Aquartafraseestáincorretaporumúnicodetalhe.Abancasebaseouemum

textodeChiavenato,queemsuaobraAdministraçãoGeralePública39menciona:

“Naadministraçãopúblicaburocrática,ocontrolepodetransformar-senaprópria

razãodeserdoESTADO,voltando-separasimesmo,perdendoanoçãodesuamissão

básica,queéserviràsociedade.”

Assim,abancatrocouEstadoporfuncionárioeconsiderouafraseincorreta.

Achoessaumaquestãobastantemaldosa.Alémdisso,ocontroledevesempre

existir.OqueocorrenaBurocraciaéqueexisteumcontroleprévio,de

procedimentosouapriori,enãoumcontrolederesultados,ouaposteriori.

Finalmente,aquintafrasetambémestáincorreta.ABurocracianãosedistingue

porsuaefetividade,ouseja,acapacidadedeatingirosefeitoseimpactosdesejados

nasociedade.

Normalmente,aBurocracia,porsuaculturalegalista,acabasendomaisvoltada

àsnecessidadesdesuamáquinadoqueàsnecessidadesdeseusclientes.Éoque

chamamosdeorganizaçãoautorreferida,ouseja,quesepreocupamaiscomos

problemasinternosdoquecomsuamissãoeseusobjetivos.Dessaforma,asúnicas

frasescorretassãoaIeaII,eogabaritoéletraA.

9.(Cesgranrio/Funasa/Administrador/2009)Umadasprincipaiscaracterísticasdasorganizaçõesé

suacapacidadedeperseguirpermanentemente“umequilíbriodinâmico”,apartirda

capacidadedossistemasdemanteremcertasvariáveisdentrodelimites,aindaqueestímulos

externosasforcemaassumirvaloresqueultrapassemesteslimites.Nessaperspectiva,

ocorremaçõesdeautorregulaçãoouautocontrolequeconduzemàtendênciadossistemasà

adaptação,embuscadeequilíbriointerno,emfacedasmudançasexternasdomeioambiente.

Essacaracterísticaédenominada:

a)entropia;

b)feedback;

c)homeostase;

d)morfogênese;

e)entropianegativa.

OconceitodaTeoriadeSistemasqueserelacionacomacaracterísticadeum

sistemabuscaroseuequilíbrioéahomeostase.Comoexemplodehomeostase,

podemoscitarofatodesuarmosquandoestamoscomcalor,poisnossocorpo

buscareduziratemperatura.OgabaritoéaletraC.

10.(Esaf/STN/Desenv.Institucional/2005)“ATeoriadaContingênciaenfatizaquenãohánadade

absolutonasorganizaçõesounateoriaadministrativa.Tudoérelativo.”(Chiavenato,2000,p.

585)

Selecioneaopçãoqueexpressacorretamenteosignificadodaafirmativaanterior.

a)Contingênciasignicaalgoprevisívelquenãodependedecircunstânciassociopolíticas,portantoo

diagnósticodoambienteeaescolhadeumatecnologiadeterminamodesenhoorganizacional.

b)Estateoriasurgiuapartirdepesquisasfeitasnochãodefábricaparavericarosprocessosprodutivos

maisecazes,cujoresultadoconduziuàconcepçãodequeaestruturafuncionaléamelhorformade

organizarotrabalho.

c)Contingênciasignicaumarelaçãodecausaeefeitoquepermitepreverodesenhoorganizacionaleas

estratégiasdemercadoaseremimplantadas,demodoagarantirmaioreficiênciaorganizacional.

d)Essateoriasurgiuapartirdepesquisasfeitasemempresasparavericarosmodelosdeestruturas

organizacionaismaisecientes,cujoresultadoconduziuàconcepçãodeummodelodediagnóstico

organizacional.

e)Essateoriasurgiuapartirdepesquisasfeitasemempresasparavericarosmodelosdeestruturas

organizacionaismaisecazes,cujoresultadoconduziuàconcepçãodequeaestruturadaorganização

édependentedoambienteexterno.

AletraAestáincorreta.Umacontingêncianãoéprevisível.Contingênciase

relacionacomoincerto,nãocomoprevisível.AletraBtambémestáerrada,pois

seriamaispróximadaAdministraçãoCientificadeTaylor.

NaletraC,nãoexisteessarelaçãodecausaeefeitonaTeoriadaContingência.

Oqueexisteéumarelaçãode“se-então”entreasvariáveisexternaseinternas.

ComrelaçãoàletraD,aorigemdaTeoriadaContingênciarealmentebuscou

osmodelosdeestruturasorganizacionaismaisecientes.Entretanto,aconclusão

foiadequenãoexisteuma“soluçãoúnica”paratodasasempresas,pois

dependeriadasdiversasvariáveisexternas.JáaletraEestácorretaeéonosso

gabarito.

QuestõesPropostas

11.(Cespe/MTE/Administração/2008)NaTeoriadaContingência,sãoenfatizadosmodelos

organizacionaismaisflexíveiseorgânicos,comoaestruturaemredes.

12.(FGV/Sefaz/RJ/Fiscalderendas/2009)Levandoemconsideraçãoaevoluçãodoconhecimento

administrativo,noquedizrespeitoàsperspectivasdasorganizaçõesemrede,assinalea

afirmativaincorreta.

a)Todasasorganizaçõespodemserconsideradasredessociais.

b)Oambienteéumaredeformadapororganizaçõesinterconectadas.

c)Asdecisõesdosatoressociaisdevemsercompreendidascombasenaracionalidade.

d)Oscontatosentregruposorganizacionaisexercempressãosobreseusmembros.

e)Ascomparaçõesentreorganizaçõesdevemlevaremconsideraçãoascaracterísticasdasredesonde

elasestãoinseridas.

13.(FGV/TCM/RJ/Auditor/2008)ComrelaçãoaoModeloContingencialdeAdministração,écorreto

afirmarque:

a)quantomaisestáveloambienteemqueestiverinseridaaorganização,maisadequadoéousodeum

modeloorgânicodeestrutura;

b)aEscolaContingencialprevêaexistênciadeumaformaúnicaecorretadeadministrar.

c)quantoàgestãodepessoas,oModeloContingencialpreconizaqueagestãodescentralizada,

participativaeflexívelésempreaquetrazmelhoresresultados.

d)atecnologiaeoambientecondicionamaformadegeriraorganização.

e)aEscolaContingencialpreviaaorganizaçãodeestruturasrígidascombasenameritocracia.

14.(FGV/MINC/AnalistaAdm./2006)Omodelodeadministraçãopúblicaeconsequentementede

formulaçãodepolíticaspúblicaséfortementeinuenciadopelasdiferentescorrentesde

pensamentodentrodaadministração.Dentrodessavisão,analiseasalternativasaseguir:

I.Aabordagemclássicadaadministraçãotemcomoumdosseusmaioresexpoentesos

trabalhosdeTaylor.Apreocupaçãoeraaumentaraeciêncianoníveloperacionalcomum

modeloquedavaatençãoaométododetrabalho,aosmovimentosnecessáriosàexecução

deumatarefaaotempopadrãodeterminadoparatal.

II.AabordagemclássicarecebeonomedeTeoriadaMáquinaporconsideraraorganizaçãosob

oprismadocomportamentomecânico.Outroaspectoéoracionalismodessaabordagem,

quevisaàeficiênciadospontosdevistatécnicoeeconômico.

III.Aabordagemhumanísticadaadministraçãotemumaforteinuênciadasciênciassociais,

principalmentedapsicologia.Aideiaportrásdessaabordagemeraanalisaraadaptaçãodo

trabalhadoraotrabalhoeaomesmotempoatenderàsnecessidadesindividuaisesociaisdo

trabalhador.

Assinale:

a)sesomenteaafirmativaIestivercorreta;

b)sesomenteaafirmativaIestivercorreta;

c)sesomenteaafirmativaIIestivercorreta;

d)sesomenteasafirmativasIeIestiveremcorretas;

e)setodasasafirmativasestiveremcorretas.

15.(FGV/Badesc/Analistaadm./2010)ComrelaçãoàTeoriaContingencial,analiseasarmativasa

seguir.

I.Asorganizaçõespossuemnaturezasistêmica.

II.Existemprincípiosuniversaisdeadministração.

III.Ascaracterísticasambientaiscondicionamoambienteorganizacional.

Assinale:

a)sesomenteaafirmativaIestivercorreta;

b)sesomenteaafirmativaIestivercorreta;

c)sesomenteaafirmativaIIestivercorreta;

d)sesomenteasafirmativasIeIIestiveremcorretas;

e)setodasasafirmativasestiveremcorretas.

16.(FGV/Sefaz/RJ/Fiscalderendas/2010)Comrelaçãoàsabordagenscontingenciaisdaecácia

organizacional,analiseasafirmativasaseguir.

I.Aresponsabilidadesocialéumindicadordaabordagemdemetas.

II.Aforteculturacorporativaéumindicadordaabordagembaseadaemrecursos.

III.Ahabilidadedosgerentesnousoderecursostangíveiséumindicadordaabordagemdo

processointerno.

Assinale:

a)sesomenteaafirmativaIestivercorreta;

b)sesomenteaafirmativaIestivercorreta;

c)sesomenteaafirmativaIIestivercorreta;

d)sesomenteasafirmativasIeIIestiveremcorretas;

e)setodasasafirmativasestiveremcorretas.

17.(Esaf/STN/Desenv.Institucional/2005)Ateoriaestruturalistaéuma(um):

a)síntesedateoriadaburocraciaeumaaproximaçãodateoriaclássica;

b)síntesedateoriaclássicaedaTeoriadasRelaçõesHumanas;

c)desdobramentodaTeoriadeSistemas;

d)desdobramentodaTeoriadasRelaçõesHumanas;

e)síntesedaTeoriaComportamentalistaeumaaproximaçãodaTeoriaBurocrática.

18.(Esaf/STN/Desenv.institucional/2008)Assinaleaopçãoquepreenchecorretamentealacunaa

seguir.

EnfatizandoquenãohánadadeabsolutonasorganizaçõesounaTeoriaAdministrativa,sendo

tudorelativo,a_____________________explicaqueexisteumarelaçãofuncionalentreas

condiçõesdoambienteeastécnicasadministrativasapropriadasparaoalcanceecazdos

objetivosdaorganização.Nessecontexto,asvariáveisambientaissãovariáveis

independentes,enquantoastécnicasadministrativassãovariáveisdependentesdentrode

umarelaçãofuncional.Narealidade,nãoexisteumacausalidadediretaentreessasvariáveis

independentesedependentes,poisoambientenãocausaaocorrênciadetécnicas

administrativas.Emvezdeumarelaçãodecausaeefeitoentreasvariáveisdoambiente

(independentes)easvariáveisadministrativas(dependentes),existeumarelaçãofuncional

entreelas.Essarelaçãofuncional,dotipo“se-então”,podelevaraumalcanceecazdos

objetivosdaorganização.

a)TeoriadeSistemas.

b)TeoriadasRelaçõesHumanas.

c)TeoriadoDesenvolvimentoOrganizacional.

d)TeoriaClássica.

e)TeoriadaContingência.

19.(Esaf/MPOG/EPPGG/2009)Conheceraevoluçãodopensamentoadministrativo,bemcomode

suasteoriaseabordagens,permiteconcluir,acertadamente,que:

a)aresoluçãodeproblemasorganizacionaispodesertentadapelaaplicaçãoconjuntadeváriasTeorias

Administrativas,aindaquesuasabordagenssejam,àprimeiravista,antagônicas;

b)comociência,aAdministraçãoindependedeoutrasciências;

c)aoenfatizarastarefas,aabordagemburocráticaenrijeceaorganização,afastando-adeseus

objetivos;

d)aTeoriaClássicadaAdministraçãopossuiapenasvalorhistóricoereferencial,nãosendoaplicávelem

nossosdias;

e)aculturadeumaorganizaçãoédeterminadaporsuaestrutura,sendopoucoafetadapelosvalorese

crençasdaspessoasqueaintegram.

20.(FGV/Sefaz-RJ/Auditor/2006)Emumsistemafechado,oestadoinicialédeterminadopelas

condiçõesiniciais.Seascondiçõesiniciaisouoprocessoforemalterados,oestadonal

tambémoserá.Issonãoéoqueacontecenossistemasabertos.Nossistemasabertos,o

mesmoestadonalpodeseralcançadopartindodediferentescondiçõesiniciaisepor

diferentesmaneiras.Essacaracterísticadenomina-se

a)entropianegativa;

b)equifinalidade;

c)homeostase;

d)uniformidadedepráticas;

e)ciclodeeventos.

21.(Cespe/TCU/ACE/2008)Atualmente,nãohámaisespaçoparaautilizaçãodateoriapropostapor

Taylor,emnenhumdeseusaspectos.

22.(Cespe/TCU/ACE/2008)Deacordocomotextoemapreço,abuscapormaioreciênciae

produtividadenasorganizaçõeséumatônicaemdiversasteoriasdaadministração.Nesse

sentido,umadasvantagensdestacadasporMaxWebernaabordagemburocráticaéarapidez

nasdecisões.

23.(Cespe/TCU/ACE/2008)Aabordagemcontingencialabarcaascontribuiçõesdetodasasdemais

abordagensqueaantecederam,principalmentedaabordagemclássicanoquetangeà

constataçãodaexistênciadeprincípiosuniversaisquepodemseraplicadosnosdiversosníveis

daorganização.

24.(Cespe/TCU/ACE/2008)AliderançacentradanaspessoasfoiumapreocupaçãoteóricadeTaylor,

quedefendiaaideiadequeresultadossópodiamserobtidosporintermédiodaspessoas.

25.(Cespe/MDS/Administrador/2006)Prebendasesinecuras,formaspatrimonialistasdeocupação

deespaçosnoaparelhodoEstado,sãosubstituídasporcritériosmeritocráticosnomodelo

burocrático.

26.(FCC/BAHIAGAS/Administrador/2010)NaadministraçãodoEstadomoderno,reforma

administrativaburocráticatrata-se:

a)daorientaçãodatransiçãodoEstadoburocráticoparaoEstadogerencial;

b)doprocessodetransiçãodoEstadopatrimonialparaoEstadoburocráticoweberiano;

c)dagestãodoprocessodetransiçãodaAdministraçãoPúblicatradicionalistaparaoEstadogerencial

patrimonial;

d)doprocessodetransiçãodoEstadoburocráticoweberianoparaoEstadopatrimonial;

e)dareformadagestãopúblicaorientandooconjuntodeatividadesdestinadasàexecuçãodeobrase

serviços,comissionadosaogovernoparaointeressedasociedade.

27.(Esaf/ANA/Analistaadministrativo/2009)AEscoladasRelaçõesHumanasdiferencia-seporideias

centraisdasquaisseoriginaramcríticasimplacáveiscontraoHomoeconomicus,modelo

elaboradopelaEscolaClássica.Analiseasarmaçõeseselecioneaopçãoquerepresentao

resultadodesuaanálise.

()NaEscoladasRelaçõesHumanas,ohomeméapresentadocomoumserquenãopodeser

reduzidoaesquemassimplesemecanicistas.

()NaEscoladasRelaçõesHumanas,ohomemnãoécondicionadopelosistemasocial,esim

pelasdemandasbiológicas.

()OmodelodaEscoladasRelaçõesHumanaséoHomocomplexus,devidoàcomplexidadedas

suasnecessidades.

a)C,C,C

b)C,C,E

c)C,E,E

d)E,E,E

e)E,C,C

Gabaritos

1.E

10.E

19.A

2.C

11.C

20.B

3.E

12.C

21.E

4.A

13.D

22.C

5.C

14.E

23.E

6.E

15.D

24.E

7.C

16.A

25.C

8.A

17.B

26.B

9.C

18.E

27.C

Bibliografia

Andrade,RuiOtávioBernardesde,andNérioAmboni.TeoriaGeralda

Administração.2aed.RiodeJaneiro:Elsevier,2011.

Certo,SamuelC.,andS.TrevisCerto.ModernManagement.10aed.UpperSaddle

River:Pearson-Prentice-Hall,2006.

Chiavenato,Idalberto.AdministraçãoGeralePública.2aed.RiodeJaneiro:

Elsevier,2008.

_______.HistóriadaAdministração:entendendoaadministraçãoesuapoderosa

influêncianomundomoderno.SãoPaulo:Saraiva,2009.

_______.IntroduçãoàTeoriaGeraldaAdministração.8aed.RiodeJaneiro:

Elsevier,2011.

Daft,RichardL.Management.Mason:Thomson,2005.

Gonçalves,JoséErnestoLima.“Processo,queProcesso?”RevistadeAdministração

deEmpresas–RAE,Out/Dez.2000:8-19.

Lima,CarlosAlbertoNogueirade.AdministraçãoPúblicaparaconcursos.Riode

Janeiro:Elsevier,2005.

Motta,F.,eI.Vasconcelos.TeoriaGeraldaAdministração.SãoPaulo:Pioneira.

ThompsonLearning,2004.

Robbins,StephenP.,andMaryCoulter.Administração.5oed.RiodeJaneiro:

Prentice-Hall,1998.

SchermerhornJr.,JohnR.Management.9aed.Hoboken:Wiley&Sons,2008.

Sobral,Felipe,eAlketaPeci.Administração:teoriaepráticanocontextobrasileiro.

SãoPaulo:Pearson-Prentice-Hall,2008.

Capítulo2

CulturaOrganizacional

Damesmaformaquetodosnóstemosumapersonalidade,ummodo

razoavelmenteestáveldeagirperanteavida(sedissermosquealguémétenso,é

inovadorousimpático,estamoscitandotraçosdepersonalidade),uma

organização

também

tem

sua

“personalidade”.Essa

“personalidade

organizacionalӎchamadadecultura.

Aculturaorganizacionalrefere-seaossignicadoscompartilhadosque

moldamamaneiraqueseusmembrossecomportamdentrodaorganização.Sãoos

valoresqueamaioriadosintegrantesdaquelegruposocialcompartilha.

Emumbancocomercial,porexemplo,aspessoascostumamvalorizaro

conservadorismo(devemtercuidadoaoemprestarseudinheiro,nãoémesmo?).

Jáemumaagênciadepropagandaessemesmoaspectodacultura

(conservadorismo)seriamuitomalvisto,provavelmente.

Dessaforma,aculturacondicionaomodocomoosfuncionáriospercebema

empresaecomorespondemaseusproblemasedesaos,etemumainuência

diretanodesempenho.

Emtodasasorganizaçõesexistemvalores,símbolos,rituaisepráticasqueforam

sedesenvolvendocomotempo.DeacordocomSchein,1culturaé:

ummodelodepressupostosbásicos,quedeterminadogrupoteminventado,descobertoou

desenvolvidonoprocessodeaprendizagemparalidarcomproblemasdeadaptaçãoexternae

integraçãointerna.Umavezqueospressupostostenhamfuncionadobemosucienteparaserem

consideradosválidos,sãoensinadosaosdemaismembroscomomaneiracorretaparaseproceder,

sepensaresentir-seemrelaçãoàquelesproblemas.

Assim,aculturadaempresapodemudar,masnemsemprecomoconsequência

deuma“vontade”dosseusdiretores.Essamudançanormalmenteocorrede

acordocomumarelaçãoentrevariáveisinternaseexternas,quecriamespaçopara

essamudançadacultura.

Figura2.1–CaracterísticasdaCulturaOrganizacional

CulturaOrganizacional:condicionaocomportamentodaspessoasnoambientede

trabalho.

Aculturadeumaorganizaçãopodeserdescritaemrelaçãoadezcaracterísticas

principais,deacordocomRobbins:2

ØIdentidadedomembro–Ograucomqueosempregadosseidenticam

comaorganizaçãocomoumtodo,enãocomoseutipodetrabalhoou

especialidadeprofissional.

ØÊnfasenogrupo–Ograucomqueasatividadesdetrabalhosãoorganizadas

emtornodegruposemvezdeindivíduos.

ØFoconaspessoas–Ograucomqueasdecisõesdaadministraçãolevamem

consideraçãooefeitodosresultadosempessoasdaorganização.

ØIntegraçãodasunidades–Ograucomqueasunidadesdaorganizaçãosão

estimuladasaoperardeumamaneiracoordenadaouinterdependente.

ØControle–Ograucomqueregras,regulamentosesupervisãodiretasão

usadosparainspecionarecontrolarocomportamentodosempregados.

ØTolerânciaarisco–Ograucomqueosempregadossãoincentivadosa

seremagressivos,inovadoresepropensosaorisco.

ØCritériosderecompensa–Ograucomquerecompensascomoaumentosde

salárioepromoçõessãodistribuídosporcritériodedesempenhodo

empregadoemcontrastecomotempodecasa,favoritismoeoutrosfatores

nãoligadosaodesempenho.

ØTolerânciaaconito–Ograucomqueosempregadossãoestimuladosa

expressarconflitosecríticasabertamente.

ØOrientaçãomeios-ns–Ograucomqueaadministraçãoenfatiza

resultadosouefeitosemdetrimentodetécnicaseprocessosusadospara

alcançaraquelesresultados.

ØFocoemsistemasabertos–Ograucomqueaorganizaçãomonitorae

respondeamudançasnoambienteexterno.

Normalmente,aculturadeumaorganizaçãoreeteamaneiradepensardeseus

fundadores.Comoelesforamoscriadoresdaorganização,tiveramtambémas

primeirasideiasdecomoeladeveriaseportaremrelaçãoaosdesaos,

determinando,dessaforma,comoseusocupantesdeveriamsecomportardentro

dela.

Masnãosãosóosfundadoresqueimpactamaculturaorganizacional.Como

tempo,asexperiênciasvividasporseusfuncionáriostambémvãosesomandoà

visãodeseusfundadores,moldandoeconsolidandoessacultura.

2.1.NíveisdaCultura

AclassicaçãomaisconhecidadosníveisdaculturaéadeSchein.Deacordo

comele,aculturaexisteemtrêsníveis:

ØArtefatos–Éoprimeironível,omaissupercial.Basicamente,sãotodos

osaspectosquepercebemosassimquetemoscontatocomumaorganização.

Dentrodesseníveltemos:osprodutos,ospadrõescomportamentais,o

vestuário,oespaçofísico,ossímbolos,oslogotipos,alinguagemetc.

ØValores–Relacionadoscomacrençanoqueécertoouerradodentroda

organização.Existememumnívelconscienteesãoutilizadosparaexplicare

justicarocomportamentodosintegrantes.Podemserpercebidosnas

histórias,naslendas,nalinguagemenossímbolos.

ØPressupostosBásicos–Sãoasverdadesinquestionáveis.Sãoaquelesvalores

tãoarraigadosquenemsãomaisexplicitados.Sãoasfontesoriginaisdos

valores.Éonívelmaisprofundoedifícildesermudado.Comoosvalores,

podemserpercebidosnashistórias,naslendas,nalinguagemenossímbolos.

Figura2.2

2.2.CulturaForteeCulturaFraca

Todasasorganizaçõestêmcultura,masnemtodaorganizaçãotemumacultura

forte.Umaculturaforteéaquelanaqualosvaloresprincipaissãorespeitadose

compartilhadospelosseusmembros.Otamanhodaorganização,arotatividade

dosfuncionárioseotempodeexistênciasãoalgunsdosfatoresquetornama

culturaforteoufraca.3

Quantomaisosmembrosdeumaorganizaçãorespeitamecompartilham

seusvalores,maisforteésuacultura!Umaculturafortegeracoesão,lealdade,

comprometimentoorganizacional,ediminuianecessidadedaorganizaçãode

fixarregulamentoseregrasdecomportamento.

Estudosindicamqueaculturafortetemumpodermaiordeinuenciaro

comportamentodosseusfuncionáriosegeraumdesempenhosuperiorparaa

organização.

2.3.ElementosdaCultura

Existemváriasmaneirasdeumaculturapodersertransmitidadentrodeuma

organização.Podemosdestacarcomoimportantesosseguintestiposdeartefatos:4

artefatosverbais,comportamentaisefísicos.

2.3.1.ArtefatosVerbais

ØEstórias–narrativasqueexplicamaorigemdaorganização,suaimportância

nocontextoeconômicoesocial.Sãocontadasaosnovosfuncionáriospara

mantervivososvaloresdaorganização,assimcomoparaajudara

compreendersuanaturezaeseuspropósitos.

ØTabus–Proibiçõesdiversasquebuscammoldarocomportamento,ouseja,

indicaramaneiraaceitáveldesecomportardentrodoambientedaempresa.

ØHeróis–Sãopessoasinuentesdentrodavidaorganizacional,utilizadas

comoexemploaseremuladopelosnovosintegrantes.

ØMitos–Sãocontosctíciosqueajudamamoldaraculturadaorganização.

Comoexemplos,teríamosa“imagemdagrandefamília”,bastantecomumnas

empresasbrasileiras,quebuscareforçarosvaloresdaorganização.

Figura2.3–Artefatosverbais

2.3.2.ArtefatosComportamentais

ØRitos,RituaiseCerimônias–Referem-seàsatividadesrepetidasque

expressamereforçamosvalores-chavedaorganização.Osrituaisde

socializaçãocorrespondemàsestratégiasdeintegraçãodoindivíduona

organização,tornandopossívelatransmissãodevaloresecomportamentos

paranovosmembros.Osprogramasdetreinamentoeintegraçãodenovos

funcionáriossãoexemplosdeatividadesquedesempenhamessepapel.

Figura2.4

2.3.3.ArtefatosFísicos

ØSímbolos–Podemserobjetos,açõesoueventosquetransmitemsignicados

aosmembrosdaorganização.Dessamaneira,aformacomoaspessoasse

vestem,otipodecarroqueosadministradoresusam,olayoutdosescritórios

sãoalgunsdossímbolosorganizacionais.

ØNormas–Todaorganizaçãotemsuasnormasemanuaisdecondutaede

procedimentos,quedelimitamocampodeatuaçãodeseusmembrose

indicamquaissãoasaçõesesperadasdecadaagentedentrodaorganização.

Osmanuais,oscódigosdecondutaedeéticasãoexemplosdessasnormas.

Figura2.5

2.4.CulturaMecanicistaeCulturaOrgânica

EssaclassicaçãodeculturasseoriginoudotrabalhodeMorgansobremodelos

estruturais.Asculturasmecanicistassãocaracterizadasporrigidez,formalidade

econtrolesobreseusintegrantes.

Essaculturafoidominantenaépocadaindustrializaçãoeémaisadequadaa

organizaçõesestáveis,emqueasmetaseosobjetivosnãomudam

constantemente,comfuncionáriosmaispropensosaaceitarasubmissãoàsregras

eparticiparpoucodoprocessodecisório.

Aculturaorgânica(quederivadeorganismosvivos,dabiologia)émais

preocupadacomainteraçãocomomeioambiente(fornecedores,clientes,

governosetc.).Seusprincípiossão:exibilidade,adaptabilidadeeinformalidade.

Portanto,seusfuncionáriosparticipamativamentedoprocessodecisório.Assim

sendo,emambientesdinâmicosessaéaculturarecomendada.

2.5.CulturaDominanteeSubculturas

Oreconhecimentodequeaculturadeumaorganizaçãopossuicaracterísticas

comunsnãosignicaquesomenteexisteumaculturadentrodeumaorganização!

Podemexistirsubculturas.Grandepartedasgrandesorganizaçõespossuiuma

culturadominanteealgumassubculturas.

DeacordocomRobbins,5umaculturadominanteexpressaosvalores

principaisquesãocompartilhadospelamaioriadosseusmembros.Quando

falamosdaculturadeumaorganização,nosreferimosàculturadominante.

Assubculturassedesenvolvememgrandesorganizaçõesparalidarcom

problemasesituaçõesdiferentesqueseusmembrosenfrentam.Portanto,os

funcionáriosdaáreadecompraspodemterumasubculturadiferentedado

pessoaldevendas.Umasubculturaincluiosvaloresprincipaisdaculturasomados

aosvalorespresentesnasubcultura.

CulturaDominante

Subculturas

Refere-seaosvaloresessenciaisqueamaioria

Ocorrememgrandesempresas,refletindoproblemas,

deseusfuncionárioscompartilha

experiênciasedesafiosdecadasetor

Figura2.6–Culturadominanteesubcultura

Vamosvercomoessetemajáfoicobrado?

1.(FCC/MP/SE/Administrador/2009)Valores,ritos,mitosetabussãocomponentes:

a)dosobjetivosemetas;

b)doplanejamentoestratégicoorganizacional;

c)dagestãoporcompetências;

d)daculturaorganizacional;

e)dasvariáveisestratégicasnãocontroláveis.

Vejamcomomuitasdessasquestõessãobemtranquilasemrelaçãoacultura

organizacional.Nãoacreditoquevocêstenhamtidodiculdadedemarcaraletra

Dcomoarespostacerta.

2.6.DisfunçõesdaCultura

Atéagoranãoanalisamosseumaculturaéboaouruim.Muitasdesuas

característicaspodemservantajosas.Umaculturapodetrazerestabilidadeao

sistemasocialnaorganização,proporcionandoumsentidodeidentidadeaos

integrantesdaorganizaçãoeumcomprometimentocomalgomaiordoqueos

interessespessoais.6

Entretanto,aculturatambémpodesernegativaemcertascircunstâncias.

Algumascaracterísticasdessasdisfunçõessão:7

ØBarreirasàsmudanças–aculturapodesernegativaquandoháum

ambientedinâmico(commuitasmudançasacontecendo)eessacultura

impedequeaorganizaçãoseadapteaosnovosdesaos.Aconsistênciano

comportamentonãoépositivaemambientesdinâmicos,apenasnosestáveis.

ØBarreirasàsdiversidades–culturasfortespõempressãonosintegrantes

parase“encaixarem”emumperldecomportamentoconsiderado

adequado.Acontrataçãodepessoasdecor,raça,idade,gêneroououtras

diferençaspodecolidircomessa“visão”doqueéaceitávelnaorganização.

Essaculturapodeentãodificultaraassimilaçãodepessoasdiferentes,quesão

necessáriasparaqueaorganizaçãotenhaqualidadesecaracterísticasque

somentepessoasdiferentescostumamtrazerparaumaorganização.

ØBarreirasàsaquisiçõesefusões–amaiordiculdadequeduas

organizaçõescostumamencontraraoocorrerumafusãoouaquisiçãoécomo

conciliarduasculturasdiferentes(principalmentequandoessasorganizações

sãograndes!).Normalmente,sóeramconsideradososfatoresnanceirosou

alinhadeprodutosenvolvidos,masatualmenteosfatoresculturaispassaram

tambémaserconsiderados.

Figura2.7

Vamosveroutraquestão?

2.(FCC/TJ/AP/AnalistaAdm./2009)Emtemposdeturbulênciaeinstabilidade,acultura

organizacionaldeumaempresadeveser:

a)reestruturadaatravésdetécnicasdereengenhariaparatornaraorganizaçãomaiseficiente;

b)amplamenterespeitada,poissuamanutençãoémaisimportantedoqueaadaptaçãodaorganização

àsmudançasexternas;

c)deixadadelado,poisreeteanaturezaconservadoradetodaorganizaçãoconfrontadacomos

desafiosdaglobalização;

d)compreendidacomoumconjuntodevalores,crençasenormasdaorganizaçãoquepodemser

mobilizadosparaenfrentarosdesafiosdoambienteexterno;

e)constantementeatualizadaatravésdaintroduçãodetecnologiadainformação,associadaamétodos

gerenciaisvoltadosàavaliaçãodecompetências.

AalternativacorretaéaletraD,poisaculturadeveserumativoda

organização,ouseja,possibilitá-laaencararseusdesaoseatingirseusobjetivos,

desdequesejaadequadaparaisso.Assimsendo,nemaculturadevesermantidade

qualquermaneira,nemdescartadaconstantementeporqualquermotivo.O

gabaritoémesmoaletraD.

2.7.ClimaOrganizacional

Oclimaorganizacionaléapercepçãoqueosfuncionáriostêmdoambientede

trabalho.Decertomodo,podemosdizerqueéareaçãoàculturaorganizacional.

Poderíamos,portanto,conceituaroclimaorganizacionalcomooconjuntode

percepções,opiniõesesentimentosqueseexpressamnocomportamentode

umgrupoouumaorganização,emumdeterminadomomentoousituação.

Assimsendo,oclimaorganizacionaléimportante,poisinuenciao

desempenhodosindivíduosedasorganizações.Existemdiversoselementosda

ambienteorganizacionalqueinfluenciamnoclimaorganizacional.

Entreelestemos:aformacomoacomunicaçãofuncionanaempresa,poispode

sermaisformaloumaisinformal;aintegraçãoentreasdiferentesáreas

funcionais;oestilodegerênciaouliderançadaschefias;aspolíticasderecursos

humanos(principalmentesalários,benefíciosecarreira);omodelodegestão

adotadopelaorganização;anaturezadotrabalhoqueéexecutado;o

relacionamentointerpessoal;ograudeidenticaçãocomaorganizaçãoea

clarezadeobjetivos.

Clima:éapercepçãoqueosmembrostêmdoambientedetrabalho.

Portanto,oclimaorganizacionalreeteasatisfaçãomaterialeemocionaldas

pessoascomotrabalho.

Apesquisadeclimaorganizacionaléummétodoquebuscamapearoambiente

internodaempresa.Éaformapelaqualosgestorespoderãoconhecermelhoros

efeitosdesuaspolíticasnoambientedetrabalho.Tendoesseconhecimento,

poderíamostrabalharosprincipaisfatoresproblemáticose,assim,melhoraro

climaorganizacional.8

Portanto,apesquisadeclimaorganizacionalanalisaoambienteinterno

buscandoapurarseuspontosfortesefracoseasexpectativasdostrabalhadores.

Aotrabalharospontosnegativosemelhoraroclimadaorganização,busca-se

aumentaramotivaçãodotrabalhadoreaumentarsuaprodutividade.

Oclimaorganizacionaldeveserbaseadonoplanejamentoestratégicoda

organização,ouseja,nãoexisteumtipodepesquisaquesirvaparatodasas

organizações.Cadaorganizaçãodevemoldaroclimaàssuascaracterísticas.O

fundamentaléqueexistamumacredibilidadenoprocesso,sigiloeconançapor

partedosfuncionários.9

Vamosvercomoessetemapodesercobrado?

3.(FCC/Defensoria/SP/Administrador/2010)Oclimaorganizacionalmostra-sepositivoquando:

a)frustraasexpectativasdeganhoimediatoemtrocadeganhosdelongoprazo;

b)atendeàsexpectativasimediatasdeganhosalarialdosfuncionários;

c)incentivaconflitosdeexpectativas,gerandocomportamentoscompetitivos;

d)provocanosfuncionáriosdiferentestiposdemotivaçãoparaotrabalho;

e)produzexpectativasdeacomodaçãoaospadrõesdeeficiênciaestabelecidos.

Essaquestãonãoapresentamuitasdiculdades,poisasduasprimeirasnãose

relacionamcomoclimaorganizacional,easalternativasCeEnãoserelacionam

comaspectospositivos,esimnegativosdoclimaorganizacional.Nossogabarito,

portanto,éaletraD.

QuestõesComentadas

4.(FCC/TRF1aRegião/ANAL.ADM.–2011)Comrelaçãoàculturaorganizacionaldeumaempresa,

considereasseguintesafirmativas.

I.Artefatossãoosindíciosvisíveis,materiaisousimbólicos,daculturadaempresa.

II.Oterceironíveldaculturacorporativasãoascrençasinconscientesnasquaistodosos

membrosacreditam.

III.Premissasbásicassãocomoosmembrossecomportam,interagemetrabalhamno

cotidianodaorganização.

IV.Cerimôniaseeventossãorituais,decaráterpositivoounegativo,quereforçamasnormase

enfatizamoqueéimportanteparaaorganização.

V.Opodernumaorganizaçãodependeapenasdocontroledasestruturasformaisda

organização.

EstácorretooqueseafirmaSOMENTEem:

a)II,IVeV.

b)IeIV.

c)IeII.

d)I,I,IIeV.

e)I,IeIV.

Asduasprimeirasfrasesestãocorretas.Naterceira,ospressupostosbásicossão

asverdadesinquestionáveis,afontedosvaloresorganizacionais.Assim,essafrase

estáerrada.

Aquartafraseestácorreta.Entretanto,aquintafraseestáincorreta,poiso

podernãodependeapenasdosfatoresformaisdeumaorganização.Nosso

gabaritoéaletraE.

5.(Cespe/MIN.ESPORTE/Administrador/2008)Aspráticaserituais,porconstituíremacamadamais

profundadaculturaorganizacional,sãoaspectoscujamudançarequermaistempo,

planejamentoecuidados.

Acamadamaisprofundadaculturaorganizacionalsãoosseuspressupostos

básicos.Estassãoverdadesinconscientes,quenãosãonemdiscutidasmais.Esses

pressupostosbásicossãonaverdadeextremamentedifíceisdemudar,pois

envolvemvaloresmuitoarraigados.Ogabaritoéquestãoerrada.

6.(FCC/TRT/PR/AnalistaAdm./2010)Oconjuntodepercepções,opiniõesesentimentosquese

expressamnocomportamentodeumgrupoouumaorganização,emumdeterminado

momentoousituação,édenominado:

a)liderançasituacional;

b)padrãovalorativo;

c)culturaorganizacional;

d)climaorganizacional;

e)culturacorporativa.

Quandofalamosempercepçõeseopiniõesdosempregadosemrelaçãoàs

empresas,estamosnosreferindoaoclimaorganizacional.Nossogabaritoé,

portanto,aletraD.

7.(FCC/TRT/24a/MS/Anal.adm./2011)Oestadodeânimocoletivoqueoscolaboradoresdeuma

organizaçãodemonstramemdadomomentoédenominado:

a)brainstorming;

b)culturaorganizacional;

c)ciclostemporais;

d)climainterno;

e)benchmarking.

Maisumavezumaquestãobastantetranquilasobreessetema.Seestivermos

falandodeânimodeumfuncionário,estamosnosreferindoaoclima

organizacional(percepções,sentimentos,opiniões).NossogabaritoéaletraD.

(Cespe/TCU/ACE/GestãodePessoas/2008)Umaempresatemapresentado,nosúltimosmeses,

perdadequalidadedosresultados,conitosentreosmembrosdasequipeseaumentono

númerodeatestadosmédicos.Apóslevantamento,vericou-sequeoclimaorganizacional

nãoestavafavorávelequealgumasmedidasdeveriamsertomadasparamelhoraroquadro

descrito.Nasituaçãohipotéticaemapreço,aempresadeve:

8.Utilizarosdadosdolevantamentoparaconstruirdiagnósticodoclimaorganizacional,oqual

deveráserrepassadoàdiretoriaparaquepossaserencaminhadoaoarquivopermanentede

pesquisas.

Odiagnósticonãoservedenadasefor“engavetado”.Saberquaissãoos

problemasémeiocaminhoandado,massãonecessáriastambémasações.

Percebamqueleraquestãoalgumasvezeséfundamental,poisécomumoCespe

utilizarumtextogrande,comoseuiníciotodocorreto,masnofinalzinhocolocar

umdadoquetornaaassertivaincorreta.Ogabaritoéquestãoerrada.

9.Buscaraçõesefetivasqueaumentemamotivaçãodosempregados,poisoclimaorganizacionalé

fatorintimamenterelacionadoaoníveldemotivaçãodosmembrosdaorganização.

Estafrasedeneoquediscutimosnaquestãopassada.Odiagnósticoé

importanteparaqueaorganizaçãopossaescolherasaçõesefetivasquepossam

melhoraroclimaorganizacional.Ogabaritoéquestãocerta.

10.Realizardiagnósticosobreoclimaorganizacionale,combasenosresultadosencontrados,

elaborarumplanodeaçãoexequíveldemelhoriadoclimaorganizacional.

Perfeito.Apesquisaéoprimeiropasso,poissemsaberoqueestácausandoum

climaorganizacionalruimseriadifíciltomardecisõeseaçõesquepudessem

corrigiroproblema.Ogabaritoéquestãocerta.

11.Manteremsigiloosresultadosdodiagnósticodeclimaorganizacional,ouseja,nãodivulgá-los

aosparticipantesdapesquisa.

Osegredonãoénecessário,nemdesejável.Umerrocomumnessaspesquisasé

exatamente“varrerparabaixodotapete”osresultadosparaevitartocaremcertos

pontoscríticos.Osfuncionáriosdevemconarnaempresa,eacomunicaçãodeve

sermaisaberta.

Aimagemquedevesertransmitidaé:“queremosouvirvocêeestamos

preocupadoscomapercepçãoquevocêtemdoambientedetrabalho”.

Obviamente,osegredovaicontratudoisso.Ogabaritoéquestãoerrada.

12.Tomarmedidasquevisemàmelhoriadoclimaorganizacional,asquaisdeverãoser

fundamentadasexclusivamentenaexperiênciadaequipetécnicadaáreadegestãode

pessoasdaorganização.

Olhemapegadinha!Oerroestásomentenoexclusivamente!Claroquea

experiênciadaequipedegestãodepessoasdaorganizaçãoéimportante,masnada

impedeamesmadeobterideiasemoutraárea,outraorganização,outropaísetc.

Comojádevemterouvidodealgumprofessor,sempredesconemdaspalavras:

SEMPRE,NUNCA,EXCLUSIVAMENTEetc.Estassãoutilizadasemprovasde

concursopara“complicar”asuavida.Ogabaritoéquestãoerrada.

13.Denirindicadoresparaoacompanhamentodasaçõesdemelhoriadoclimaorganizacional,

visandoàchecagemdaefetividadedessasações.

Adeniçãodeindicadoresauxilianomonitoramentodasaçõesquevisam

melhoraroclimaorganizacional.Questãotranquila.Ogabaritoéquestãocerta.

QuestõesPropostas

14.(Cespe–Hemobras/Administrador–2008)Aculturaorganizacionaltratadoconjuntode

valores,hábitosecondutasadotadospelosmembrosdeumaorganizaçãoequenecessitam

estarescritosparaquesejamaceitosportodososenvolvidos.

15.(Cespe–Hemobras/Administrador/2008)Percebe-seaculturadeumaorganização

identicandocomoosmembrossevestemesetratammutuamente,comoosclientese

fornecedoressãotratadosecomosedãoasrelaçõesentrechefesesubordinados.

16.(Cespe/Hemobras/Administrador–2008)Umaculturaorganizacionalfortesecaracteriza

semprecomoumavantagemparaaorganização.

17.(Cespe/Anal.GestãoPessoas/Serpro/2010)Ostrêsníveisdaculturaorganizacionalsão:

artefatosobserváveis,valoresadotadosepressupostosbásicos.

18.(Cespe/Anal.GestãoPessoas/Serpro/2010)Quandoasorganizaçõesvisampromovermaior

efeitodaculturaorganizacionalsobreocomportamentodosfuncionários,éessencialquese

garantaqueosfuncionárioscompreendamosvalorespraticadosemdetrimentodosvalores

adotados.

19.(Cespe/Anal.GestãoPessoas/Serpro/2010)Ospressupostosbásicosdaculturaorganizacional

sãodedifícilmudança,poissãoimplícitos,arraigadoseorientadoresdocomportamento

organizacional.

20.(Cespe/Anatel/Analistaadministrativo/2009)Fazpartedoprocessodeentradadenovos

colaboradoresemumaorganizaçãoaaculturaçãoquesofrerãodogrupo,tendoemvistao

novoambientequelheséapresentado,nãosendosalutarquealteremoclimaorganizacional

jáexistente,sobpenadecausaremprejuízoàempresa.

21.(Cespe/Anatel/Analistaadministrativo/2009)Naclassicaçãodoclimaorganizacionalquantoà

relaçãocomofuturo,asorganizaçõessãoclassificadasemadaptativasenãoadaptativas.

22.(Cespe/Anatel/Analistaadministrativo/2009)Aosecompararaculturaorganizacional

predominanteentreosjaponeseseadosnorte-americanos,verica-sequeambasvalorizam

muitoahierarquia.

23.(Cespe/Anatel/Analistaadministrativo/2009)Entreasclassicaçõesmecanicistaeorgânica,as

repartiçõespúblicassãoenquadradasnestaúltima.

24.(Cespe/STM/Anal.Jud.Psi–2011)Aculturaorganizacionaléholística,compartilhada,construída

socialmente,aprendida,intangíveledeterminadahistoricamente.

25.(Cespe/STM/Analjud.Psi/2011)Osconteúdosexplícitosintrojetadosepartilhadospelos

indivíduos,como,porexemplo,crenças,comportamentos,regras,visõesdemundoe

significados,compõemadimensãomaissubjetivadoclimaorganizacional.

26.(Cespe/STM/Anal.jud.Psi/2011)Aculturaeoclimaorganizacionalenfatizamanaturezado

vínculodoindivíduocomotrabalhoecomaorganização.

27.(Cespe/STM/Anal.jud.Psi./2011)Aculturaorganizacionaléutilizadacomoumamedidade

determinadasvariáveis,taiscomorecompensas,condiçõesdetrabalho,mecanismosde

controle,normasevalorespraticadosemumaorganização.

28.(Cespe/STM/Analjud.Psi/2011)Oclimaorganizacionalevidenciacomoaslosoasgerenciaise

aspráticasadministrativasmoldamocomportamentodaspessoas.

29.(Cespe/MPU/Anal.Administrativo/2010)Aculturaorganizacionalnãocomportagerenciamento,

dadoodescompasso,nasorganizações,entreasituaçãorealeasituaçãoideal.

30.(Cespe/MPU/Anal.Administrativo/2010)Oclimaorganizacionalresultadapercepção

compartilhadaentreindivíduos,deênfaseafetivaecognitiva,acercadepráticas,políticase

procedimentosformaiseinformaisdeumaorganização.

31.(Cespe/Correios/Administrador/2011)Aanálisedeculturaorganizacionaldivide-seemníveis.O

primeirodessesníveisécompostoporsuposiçõescomuns,histórias,rituaiseverdades

implícitas.

32.(Cespe/Correios/Administrador/2011)Aculturaorganizacionaloucorporativaenvolveo

compartilhamentodevaloresentrecolaboradoresdeumamesmaorganização,podendo

orientarsuascondutas.

33.(Cespe/Correios/Administrador/2011)Devidoacaracterísticasmuitopeculiares,empresasdo

mesmoramodeatuaçãocostumamterculturasidênticas.

34.(Cespe/ABIN/Oficialtécnico/2010)Define-seculturaorganizacionalfracacomoaquelaemqueos

valorescompartilhadosexercemaltoimpactonocomportamentodosfuncionários.

35.(FCC/TRT-23/Analista/2011)Schein(1992)propõequeaculturaorganizacionalsejaanalisadasob

trêscamadas:artefatos,valorescompartilhadosepressuposiçõesbásicas.Osvalores

compartilhados:

a)sãopressuposiçõesimportantes,asquaisdevemserseriamenteconsideradaspelosgestoresdemodo

geral.Taiscontribuiçõespodemmodificarpositivamenteoalcancedosresultados;

b)indicamasverdadesimplícitasqueoconjuntodosmembrosdacorporaçãocompartilham.Tais

verdadessurgememdecorrênciadasexperiênciascompartilhadas;

c)exercemfunçõesessenciaisnoquadrofuncionaldaorganização.Talprocessopoderepresentarum

intensomecanismodemotivaçãoparaaspessoasqueseencontraminseridasnaquelacultura;

d)sãoospadrõesdecomportamentofacilmentedetectadosnotrabalhocotidianodaspessoasda

organização.Eleséqueexercemumaligaçãocomaideiadequetodosfazemospartedeumgrupode

trabalho;

e)modicamsobremaneiraamaneiradepensardoscolaboradores,quepassamjuntosadesenvolver

crençasidênticasequefavorecemaconstruçãodeumaidentidadecorporativa.Essaidentidadeéque

estabeleceráotipodeculturaqueaempresadesenvolverá.

Gabaritos

1.D

13.C

25.E

2.D

14.E

26.E

3.D

15.C

27.E

4.E

16.E

28.E

5.C

17.C

29.E

6.D

18.E

30.E

7.D

19.C

31.E

8.E

20.E

32.C

9.C

21.E

33.E

10.C

22.C

34.E

11.E

23.E

35.C

12.E

24.C

Bibliografia

Robbins,StephenP.OrganizationalBehavior.11aed.UpperSaddleRiver:Pearson-

Prentice-Hall,2004.

Robbins,StephenP.&MaryCoulter.Administração.5aed.RiodeJaneiro:

Prentice-Hall,1998.

Rocha,Irani,SabrinadoNascimento,AlexandreMatosPereira&DeniseDelPrá

NettoMachado.“ArtefatosConcretosdaCulturaOrganizacionalnasGrandes

EmpresasAéreasBrasileiras.”XIIISemead.2010.

Schein,EdgarH.OrganizationalCultureandLeadership.SanFrancisco:Jossey-Bass

Publishers,1993.

Sobral,Felipe&AlketaPeci.Administração:teoriaepráticanocontextobrasileiro.

SãoPaulo:Pearson-Prentice-Hall,2008.

Capítulo3

PlanejamentoeEstratégia

Aadministração(ougestão)estratégicaéumconceitoabrangenteenovo.De

acordocomPearceeRobinson,1

éumprocessoqueconsistenoconjuntodedecisõeseaçõesquevisamproporcionaruma

adequaçãocompetitivamentesuperiorentreaorganizaçãoeseuambiente,deformaapermitir

queaorganizaçãoalcanceseusobjetivos.

Dessemodo,aadministraçãoestratégicaéumconceitoqueémaisabrangente

doqueumsimplesplanejamentoestratégico.Existemalgumasdiferençasentrea

doutrinacomrelaçãoàordemdasfasesprincipais,masosprincipaismodelos

separamessagestãoemtrêsfases:oplanejamentoestratégico,aexecução(ou

implementação)estratégicaeocontroleestratégico.

Figura3.1–FasesdaGestãoEstratégica

JáSobralePecicitammaisfasesdoprocessodeAdministraçãoEstratégica.2

Elesfazemum“desdobramento”dasfasesdoplanejamentoestratégico:

diagnósticodasituaçãoatual,análiseestratégica,formulaçãoestratégica,

implementaçãoestratégicaecontroleestratégico.

Figura3.2–FasesdaAdministraçãoEstratégica.(AdaptadodeSobralePeci2008)

Decertaforma,afasedoplanejamentoestratégico(quenomodelodeSobral

englobaastrêsprimeirasfases)montaumdiagnósticodasituaçãodaorganização

edasoportunidadeseameaçasqueacercam.Comisso,podemosformularuma

estratégiaadequadaàspossibilidadesdaempresa.

Estandoaestratégia“montada”,aorganizaçãopoderiapartirparaaexecução,

ouimplementaçãoestratégica.Nessafase,osplanossãocumpridoseas

atividadeseprocessosdaempresaseguemoquefoidenidonaetapade

formulação.

Continuando,duranteeapósaexecuçãoestratégica,ogestordevemonitoraro

desempenhodaorganizaçãoeavaliarseoquefoiplanejadoestásendo

“cumprido”.Esseprocessoéchamadodecontroleestratégico.

Comaxaçãodasmetasqualitativasequantitativas,eosdadosfornecidospelos

instrumentosdecontrole,serãopossíveisacorreçãodoserroseadisseminação

daspráticasquederamcerto.Portanto,ofeedback(realimentaçãoou

retroalimentação)fornecidonessafaseauxiliaoaprendizadoorganizacional.

3.1.Planejamento

Todosnósconhecemoseutilizamosoplanejamentoemnossasvidas,nãoé

mesmo?Quandoqueremosfazerumaviagemdecarro,porexemplo,analisamos

qualseráadistânciaatéodestino,qualseráaestradamaisadequada,quantos

postosdegasolinaexistemnocaminho,quantotempolevaráaviagem,quanto

custará,entreoutrosfatores.

Fazendoessaanálise,aumentamosachancedequenãoaconteçaalgoderuim

emnossaviagem,comocarsemgasolinanomeiodocaminho,nãoéverdade?

Nasorganizações,tambémutilizamosoplanejamentocomoumaformade

aumentaraschancesdesucesso.

Oplanejamentoéoprocessoadministrativoqueinuenciatodososoutros

(direção,organização,controle).Eleéopassoinicial–quandodeniremosoque

aorganizaçãoquerfazer,quandoecomoelaofará.

Comoofuturoésempreincerto,temosdeprocurarreduziressaincerteza

quandoestamosadministrandoumaorganização.Assim,oplanejamentoéo

processoqueutilizamosparareduziraincertezaeaumentarachancedea

organizaçãotersucesso.3

Osprincipaisautoresdenemoplanejamentocomoumprocessodedenição

dosprincipaisobjetivosedaformacomoessesobjetivosserãoalcançados.Vejam

algumasdessasdefinições:

Lembre-se:nenhumplanejamentoiráafastartotalmenteaincerteza.

ØSegundoChiavenato4–“Planejamentoéumprocessodeestabelecer

objetivosedefiniramaneiracomoalcançá-los.”

ØSegundoDjalmadeOliveira5–“Planejamentoéumprocessodesenvolvido

paraoalcancedeumasituaçãofuturadesejada,deummodomaiseciente,

ecazeefetivo,comamelhorconcentraçãodeesforçoserecursospela

empresa.”

Oplanejamentopossibilitaaosgestoresfocarnãosomentenocurtoprazo,mas

tambémnofuturodaorganização.6Oprocessodeplanejamento,deacordocom

Schermerhorn,7écompostoporcincopassos:

1.Deniçãodosobjetivos:nestemomentodevemosidenticaroquequeremos,

ouseja,aondequeremoschegar.

2.Determinarsuasituaçãoatual:nestepontodevemossituaraorganização–o

quãolongeestamosdosnossosobjetivos?Dessaforma,teremosumaideiado

“caminho”queterádeserpercorrido.

3.Desenvolverpremissassobreofuturo:éaetapaemquebuscaremosantecipar

oseventosquepoderãoacontecer,ouseja,quaissãoos“cenários”ousituações

maisprováveisdeacontecer.Comisso,poderemosidenticaroportunidadese

ameaçasaosnossosobjetivos.

4.Analisareescolherentreasalternativas:aqui,iremosgerareanalisaras

alternativaspossíveisdeação.Apósisso,escolheremosaalternativaqueseja

maisindicadaparaquepossamosatingirnossosobjetivos.

5.Implementaroplanoeavaliarosresultados:Agora,devemosexecutaro

planejadoeavaliarquaissãoosresultadosatingidos,demodoquepossamos

corrigirosdesvioserevisarosplanos,senecessário.

Vejaumgráficocomessescincopassosdoprocessodeplanejamento:

Atenção:nãoháconsensoentreosautoressobreaordemdasfasesdoplanejamento.

Figura3.3–Passosdoplanejamento

Dentreoutrasvantagens,oplanejamentoajudaogestorafocarseuesforço,a

darumsentidodedireçãoaosmembrosdaorganização,areduziroimpactodas

mudançasdomeioexterno,amaximizaraeciência,adenirosparâmetrosde

controleenopróprioautoconhecimentodaempresaedasforçasqueacercam.

Asprincipaisvantagensdoplanejamentoestãonestegráfico:

Figura3.4–Vantagensdoplanejamento.(Fonte:SobralePeci,2008)

3.2.NíveisdoPlanejamento

Oplanejamentonãoocorredamesmaformaemtodososníveisdaorganização.

Dependendodonívelhierárquico,oplanejamentoterácaracterísticasdiferentes,

comrelaçãoaoprazo,àabrangênciaeaoseuconteúdo.8

Nonívelestratégico,oenfoquedeveserglobal,ouseja,iremosanalisaros

grandesproblemasqueafetamaempresacomoumtodo.Nessenível,a

organizaçãodeniráseusobjetivosgeraisegenéricos.Depois,essesobjetivosserão

detalhadosnosníveisabaixo(táticoeoperacional).

Imaginequevocêéoprincipalexecutivodeumaindústriadealimentos.Nesse

caso,você,comoocupantedonívelestratégico,terádeanalisaroambiente

externoedecidirasdiretrizesestratégicasparaqueaempresatenhasucessono

longoprazoesobreviva.

Quaisserãoosprodutosqueosconsumidoresdesejarãonospróximoscinco

anos?Quemercadoscrescerãomais?Serámelhorinvestiremdocesparacrianças

ouprodutosdietéticos?Essasserãoalgumasdasquestõesquevocêdeveráanalisar

nessenível.

Dessaforma,imaginequesuadecisãofoiinvestiremumanovalinhadebarras

decereaisparapessoasjovensequesejampreocupadascomaestética.Paraatingir

esseobjetivo,aempresaterádemontarumafábricanovaeinvestirR$200milhões.

Comoveremosaseguir,essadecisãoestratégica(deinvestirR$200milhõesem

umanovafábricaeentraremumnovomercado)iráorientarotrabalhodosníveis

inferiores.

Assimsendo,onívelabaixo–otáticoougerencial–terádetomardecisõese

planejarseuprópriotrabalhobaseadonasdecisõesqueonívelestratégicodefiniu.

UmgerentedeRecursosHumanosdessaempresa,porexemplo,teráagoraque

iniciarseuplanotático,deformaacapacitaraempresaaabriressafábricacom

pessoascontratadas,treinadasemotivadasparaotrabalho,nãoémesmo?

Portanto,essegerentevaisebasearnasdecisõesestratégicaspara,nasuaárea,

tomarasdecisõesquepossibilitemàempresaatingirseusobjetivoscomoumtodo.

Omesmoacontecerácomumocupantedoníveloperacional.Umsupervisorde

recrutamentoeseleçãoteráotrabalhodemapearasáreasquenecessitarãode

funcionárioseospersideaisdecadacargo,deverábuscareselecionaressas

pessoasnaprópriaempresaounomercadodetrabalho.

Portanto,eleiráplanejaraexecuçãodaquelatarefaespecífica,daquelaatividade

derecrutamento,quefoiderivadadasdecisõestomadasnonívelestratégicoeno

níveltático.

Dessaforma,oplanejamentoestratégicoserefereàorganizaçãodemodo

globaleéfocadonolongoprazo.9Essetipodeplanejamentotemumavisãoforte

noambienteexterno,ouseja,emcomoprepararaorganizaçãoparaosdesaosdo

meioambiente(economia,clientes,governos,concorrentes,fornecedoresetc.).Os

objetivosnesseníveldevemsermaisgerais,ouseja,poucodetalhados.

Figura3.5–Níveisdoplanejamento

Oplanejamentoestratégicotemcomoprincipaisfases:adeniçãodonegócio,

missão,visãoevaloresdaorganização,odiagnósticoestratégico(compostoda

análiseinternaeexternadaorganização),aformulaçãodaestratégia,a

implantaçãodelaeocontroledetodooprocesso.

Ouseja,aorganizaçãodeveprimeiroanalisarquaissuarazãodeexistir,seus

valoresecomoelaquerimpactarasociedade(missão,visão,negócio).Apósisso,a

organizaçãodevedecidiraondequerchegar,ouseja,qualéo“destino”desejado

(visão)equaléasituaçãoatual(diagnósticoestratégico).

Aestratégiaentãoéo“caminho”escolhidoparaqueaorganizaçãopossa

chegarnesse“destinodesejado”pelavisãoestratégica.10

DeacordocomMatoseChiavenato,11oplanejamentoestratégicoconsisteem

cincocaracterísticasprincipais:

1.Oplanejamentoestratégicoestárelacionadocomaadaptaçãoda

organizaçãoaumambientemutável–ouseja,devemosentenderqueestamos

lidandocomaincerteza.Portanto,todoplanejamentodeveserdinâmico–

sendoconstantementereavaliadoemonitorado.

2.Oplanejamentoestratégicoéorientadoparaofuturo–oplanejamentoé

voltadoaolongoprazo,ecomoasdecisõesatuaispoderãoimpactara

organizaçãonessefuturo.

3.Oplanejamentoestratégicoécompreensivo–dessaforma,envolvea

organizaçãocomoumtodo.Todososrecursosepessoasdevemserenvolvidos

nesseprocessoparaqueaorganizaçãotenhasucesso.

4.Oplanejamentoestratégicoéumprocessodeconstruçãodeconsenso–

naturalmenteexistempensamentosdiferenteseconitantesdentrodeuma

organização.Entretanto,oplanejamentodevebuscaromelhorresultadopara

todosdentrodaorganização.Umadascaracterísticasdeumplanejamentode

sucessoéoenvolvimentoeocomprometimentodetodasasáreasepessoaspara

queelesejabemexecutado.

5.Oplanejamentoestratégicoéumaformadeaprendizagemorganizacional–

comapráticadoplanejamento,aorganizaçãopassatantoaseconhecermelhor

comoaconhecermelhorseuambienteexternoeseusdesafios.

Nográficoaseguirpodemosverasprincipaisfasesdoplanejamentoestratégico:

Figura3.6–Fasesdoplanejamento

Oplanejamentotáticodeveobservarasdiretrizesgeraisestipuladasno

planejamentoestratégicoedeterminarosobjetivosespecícosdecadaunidadeou

departamento.

Dessaforma,osgerentesdevemdetalharosobjetivosdecadasetor(Gerênciade

Finanças,porexemplo)paraqueosobjetivosestratégicosdaempresaserealizem.

Umplanejamentotáticocostumaserfocadonomédioprazo.

Jáoplanejamentooperacionalirádeterminarasaçõesespecícasnecessárias

paracadaatividadeoutarefaimportantedaorganização.Seusobjetivossãobem

detalhadoseespecíficos.

Esseplanejamentotambéméumdesdobramentodoplanejamentotático,pois

iráestabelecercomoastarefasdevemsercumpridasparaque,somadas,os

objetivostáticos(eosestratégicos)sejamatingidos.Nessenível,oplanejamentoé

focadonocurtoprazoeéconstantementerevisto.

Vamosvercomoissojáfoicobrado?

1.(Cespe/Serpro/Gestãoempresarial/2008)Háconsensoentreasmetodologiasdequeo

planejamentoestratégicodeveteriníciocomoestabelecimentodosobjetivosgeraisda

organização.

Nãoexisteesseconsenso.Comojámencionamosanteriormente,Chiavenato12

acreditaqueoplanejamentoseiniciacomadeniçãodosobjetivos.JáOliveira13

acreditaqueesteseiniciacomodiagnósticoestratégico.

Dessaforma,nãoexisteumconsensosobreaordemdessasfases.Ogabaritoé

questãoerrada.

3.3.MissãoeVisãoeNegócio

Amissãoeavisãodeumaorganizaçãosãomuitoimportantesemumprocesso

deplanejamentoestratégico,poisdãoum“norte”paratodososatoresenvolvidos

nesseprocesso.

Amissãodeumaorganizaçãoéarazãodeexistirdesta.Éomotivopeloqual

elafoicriada,oqueelaveiocontribuirouresolvernasociedade.

Porexemplo,umauniversidadedevefazeroquê?Gerarconhecimento,nãoé

mesmo?Portanto,umamissãodeumauniversidadepoderiaseralgoassim:

Gerarconhecimentoecontribuirparaodesenvolvimentosocioculturaldacomunidade,gerando

oportunidadesevalorizandoseuscolaboradores.

Dessaforma,amissãoserveparadeixarclaroqualéafunçãomaiordaquela

organização,paradenirquaisserãoasnecessidadesquedevemseratendidase

buscarocomprometimentodoscolaboradores.

Figura3.7–Missãoorganizacional

Muitosautorescitamtambémadeniçãodonegóciocomodegrande

importância.Basicamente,adeniçãodonegóciodeterminao“raio”deatuação

daorganização–suasatividadesprincipaisemumdeterminadomomento.

DeacordocomVasconcelosePagnoncelli,14

osbenefíciosadvindosdadeniçãodonegócioestãorelacionadosàdeterminaçãodoseuâmbito

deatuação.Assim,aorganizaçãopodeajustarseufoconomercadoedesenvolverseudiferencial

competitivo,orientandooposicionamentoestratégicodaorganizaçãoeevitandoamiopiade

mercadoexpostaporLevitt.

Algunsautoresfazemumadivisãoentreumadeniçãodenegóciorestrita(mais

focadanasatividadesatuais)eumadeniçãomaisampla(relacionadacomos

benefíciosoferecidos).Assim,estaúltimaseriamaisabrangente.

UmexemplocitadoporLobatoseriaocasodaNokia.15Emumadenição

restritadonegóciodaempresa,estaseriafornecedoradetelefonescelulares.Jáem

umadefiniçãomaisampla,ela“conectariapessoas”.

Oproblemaéqueadeniçãomaisamplaébemsemelhanteàmissãoeacaba

“confundindo”acabeçademuitoscandidatos.Aprincipaldiferençaéquea

missãofuncionamaiscomoumadeclaraçãodeintenções,aopassoquea

deniçãodonegócioémaiscircunstancial,maisfocadanasituaçãopresentee

nosbenefíciosatuaisqueelageraaosclientes.

Jáavisãoéumdestinodesejado.Umaorganizaçãodevesaberaondequer

chegar,quaissãoseusobjetivosmaiores.Vamosverumcasoprático?Imagineuma

empresadecosméticosbrasileira,quetenhaoperaçõessomentenoBrasil.Uma

visãodefuturodessaempresapoderiaseralgoassim:

SerreconhecidacomoalíderdemercadonoBrasilem2020einiciarnossasoperaçõesnosmercados

americanoeeuropeu.

Assimsendo,essavisãoserviráparaquetodososfuncionáriosentendamqualé

o“grandeplano”,ouseja,aondeaempresaestáquerendochegar.Issofacilitará

otrabalhodetodos,poiscarámaisfácilentenderquaisserãoasprioridadese

comocadaumdeveseprepararparaqueessesobjetivossejamalcançados.

Aseguirvemosnográficoasprincipaiscaracterísticasdavisão:

Figura3.8–Visãodeumaorganização

Vamosvercomootópicojáfoicobrado?

2.(Cespe/Inca/Gestãopública/2010)Manter-sereconhecidoereferenciadocomolaboratóriode

excelênciadegestãodaproduçãoempesquisa,desenvolvimentoeextensão,cométicae

responsabilidadesocial,éumadeclaraçãoqueexemplificaumavisão.

Umavisãoéumpontoimaginárionofuturorelativoàposiçãoquea

organizaçãodesejaseencontrar.Assim,refere-seaumobjetivodesaadoreque

deverá“mostrarocaminho”aosseusmembros.Portanto,essafraseéumexemplo

aceitáveldevisão.Ogabaritoéquestãocorreta.

3.4.DiagnósticoEstratégico–AnáliseSWOT

Entreasfasesdoplanejamento,umadasmaisimportanteséafasedo

diagnósticoestratégico.Nessafase,analisa-seoambienteexternoeinternoda

empresaparaqueogestorsaibaqualarealsituaçãoemqueseencontraa

organizaçãoepossadenirquaistiposdeestratégiasãomaisadequadosno

momento.

UmadasferramentasmaisutilizadaséaanáliseSWOT(aerônimodostermos

em

inglês:Strengths

=forças;Weaknesses

=fraquezas;Opportunities=

oportunidades;Threats=ameaças).

Portanto,éumaanálisedasforçasefraquezasdaorganização(ambiente

interno)edasameaçaseoportunidadesqueelapodeterdeenfrentar(ambiente

externo).

Aprincipaldiferençaentreoambienteinternoeexternoparaogestoréqueno

primeirocaso(ambienteinterno)asvariáveissãocontroláveis,enosegundocaso

(ambienteexterno),não!

Pontosfracossãofatoresinternosque,dealgumaforma,deixamaempresa

emdesvantagememrelaçãoaosseusconcorrentes.Podemser,porexemplo,

umamarcadesconhecidanomercado,umaequipedesmotivada,produtos

desatualizados,altoendividamentodaempresaetc.

Jáospontosfortessãoocontrário,ouseja,fatoresinternosquedeixama

empresaemvantagemperanteseuspares!Podemser,porexemplo,clientes

éis,prossionaiscapacitados,umalogísticaeciente,produtosinovadores,

custosbaixosetc.

Quandodizemosqueospontosfracosefortessãovariáveiscontroláveis,é

porqueessesfatorespodemser“trabalhados”maisfacilmentepelosgestores.

Umamarcadesconhecida(pontofraco)podesermelhoradacomuma

campanhaecientedemarketing.Umaempresaendividada(pontofraco)pode

renegociarsuasdívidasouconseguirparceirosqueafinanciem.

Jáasameaçaseoportunidadessãovariáveisexternase,aprincípio,não

controláveis!Seexisteapossibilidadedequeogovernolanceumanovalegislação

quepossaprejudicarsuaempresa(ameaça),muitasvezesnãoestáaoseualcance

evitarquealegislaçãosejacriada,somenteadaptar-seaosseusefeitos,nãoé

verdade?

Portanto,asameaçassãofatoresexternos(umnovoconcorrentenomercado,

umaumentonosimpostos,umaaumentonocâmbioetc.)quepodemprejudicar

aempresa,easoportunidadessãofatoresexternosquepodembeneciara

empresa(crescimentoeconômicodopaís,aquebradeumaempresarivaletc.).

3.5.Objetivos,MetasePlanos

Umadasmaisfrequentesdúvidasqueescutodoscandidatosé:anal,qualéa

diferençaentreumobjetivoeumameta?

Umobjetivoéumestado(ousituação)desejado.Ouseja,éaondequeremos

“chegar”.Porexemplo,ogovernopodedenircomoobjetivoreduzironúmero

dejovenscommenosde15anosforadasescolas.

Esseobjetivo,porém,deveserquanticado!Assim,paraqueaadministração

possadistribuirseusrecursos,eladevesaberquantaspessoasdevemserincluídas

noprogramaequando(exemplo:150milalunosatédezembrode2013).Isso

agoravirouumameta!

Assim,umametaéumdesdobramentodeumobjetivo.Comela,podemos

controlareavaliarmelhoraexecuçãodeumplanejamento,jáqueumobjetivo

finalpodesermuito“distante”.

Dessamaneira,devemos“quebrar”oobjetivoemdiversasmetasintermediárias.

Osomatóriodessasmetasnoslevará,então,aatingironossoobjetivo.

Imagineocasodeumcandidatoqueconheci:eleiniciouseusestudosnoano

retrasado.Seuobjetivoerapassaremumbomconcurso,parapoderdaruma

melhorqualidadedevidaàsuafamília.

Entretanto,elesabiaqueaindaestavadistantedessesonho.Suaprimeirameta

foinãosereliminadonasprovas(atingiranotamínima)emalgumaprovanaquele

ano.Comotempo,essametafoiatingida.

Apósisso,construiuoutrameta:tersuasredaçõescorrigidasemalgum

concursode2010.Eleconseguiuatingiressatambém.

Suametaposteriorfoiserclassicado“fora”dasvagasdentrodesseano.Por

m,suametanalseráatingiraaprovaçãodentrodasvagas–atingindo,assim,

seuobjetivoinicial!

Paraisso,eleprecisadesenvolverumplanodeação–ouseja,asaçõesqueserão

necessáriasparaqueatinjaseusobjetivos.16Dessamaneira,eledetalhoutodasas

matériasquenecessitaaprendereoshoráriosparacadaumadelasdentroda

semana.

Vamosvercomoissojáfoicobrado?

3.(Cespe/Inca/Gestãopública/2010)UmsecretárioexecutivodoMinistériodaSaúde,cujas

atividadesrelativasaoplanejamentoenvolvamdecisõesaseremtomadasacercadeobjetivos

eestratégiascaracteristicamentedelongoprazo,nãopoderáformularouimplementar

pessoalmentetodooplano.

NessaquestãooCespeestásereferindoaoplanejamentoestratégico.Assim,não

érecomendávelqueapenasumapessoa,mesmoquesejaochefemaisgraduado,

formuleouimplementeoplanejamentosozinho.Assim,aquestãoestácorreta.

3.6.EscolasdoPlanejamento

DeacordocomMintzberg,existemdezescolasdoplanejamentoestratégico,ou

seja,maneirasdiferentesdedescreveroprocessodeformaçãodaestratégia.17

Essasescolasreetemopensamentodediversosautoresemostramaevolução

dessesconceitosdesdeosanos1960.Elaspodemserclassicadasemtrêsgrupos:

asdenaturezaprescritiva,asdenaturezadescritivaeadeconfiguração.

Asescolasdenaturezaprescritivasepreocupamemdetalharcomooprocesso

deformaçãodaestratégiadeveserfeito,emvezdesepreocuparemcomcomo

essasestratégiassãorealmenteformadas.

Dentrodessegrupoosautoresincluemaescoladodesign,aescolado

planejamentoeaescoladoposicionamento.Aescoladodesignéamais

tradicionalefoiabaseparaasoutrasduasescolas.

Dentrodessaescola,aformaçãodaestratégiaévistacomoumajusteentreas

forçasinternas(forçasefraquezas)eexternas(ameaçaseoportunidades).De

acordocomoautor,seriaumprocessodeconcepção.Assim,oplanejamentoseria

umajusteentreasforças.

Asprincipaispremissasdessaescolasão:18

ØAformaçãodaestratégiadeveserumprocessodepensamento

deliberado–ouseja,aestratégiadeveserderivadadeumpensamento

consciente,racional.

ØAresponsabilidadepelaestratégiaédoexecutivomaisgraduado–nessa

escola,oprocessodeplanejamentodeveserdeixadoacargodeapenasuma

pessoa,ochefe!Osoutrosfuncionáriossãovistoscomosubsidiáriosnesse

processo.

ØOmodelodevesersimpleseinformal–muitaelaboraçãoedetalhamento

seriamprejudiciais–aestratégiadevepoderser“montada”dentrodamente

doexecutivoprincipal.Oprocessoseriaentãoumpoucoformaleumpouco

intuitivo.

ØAsestratégiasdevemserúnicas–cadacasodeveseranalisado

individualmente.

Asegundaescolaéadoplanejamento.Oautormaisconhecidodessalinhaé

Ansoff.Essaescolasebaseounaescoladodesign,mascomalgumasdistinções.A

primeiraéemrelaçãoaoprocessoformal.

Aquioplanejamentoé“tocado”portodaumaequipedeplanejadores,

especializados,quebuscamestruturartodososdadospossíveisdeformaamontar

aestratégiadaorganização.

Assim,acabaasimplicidade.Oplanejamentosetornaumprocessoelaborado

edetalhado,comdiversospassosaseremcumpridosechecklists(listasde

vericação)aseremmarcadas.Aresponsabilidadepeloplanejamento–emteoria

–continuacomoexecutivomáximo.

Mas,comooprocessoéextremamentecomplexo,napráticaquemmontaa

estratégiaacabasendoaequipedeplanejadores–oexecutivoapenasaprovaou

não!

Jáaescoladoposicionamentoapareceucomforçaemdecorrênciada

publicaçãodolivrodeMichaelPorter–EstratégiaCompetitiva.Paraessaescola,o

importantenãoésomenteoprocessodeformulaçãoestratégica,masaestratégia

emsi.

Ouseja,existiriamsomentealgumasestratégiasválidasemummercado

competitivo(chamadasdeestratégiasgenéricas–custo,diferenciaçãoefoco).

Umaorganizaçãodeveriaescolherumaestratégiademodoqueocupasseum

posicionamentoquepudesseserdefendidodeseusconcorrentes.

Assim,cadaorganizaçãodeveriaescolheraestratégiaquemelhorseadapteàs

suascapacidades–utilizando-seaprópriaanáliseSWOTeomodelodascinco

forçascompetitivas.19

Decertaforma,essaescolanãorenegaaspremissasdasduasanteriores,masse

preocupamaisementenderaimportânciadecadaestratégia,emvezdefocarno

processodeformulaçãodasestratégias.Aseguir,podemosverumresumodastrês

escolas:

Design

Planejamento

Posicionamento

Processode

Processoformaledetalhado–

Processoanalíticodasforçasdo

concepçãopelochefe

cometapasechecklists.

mercadoeestratégiasválidas.

máximo.

ResponsabilidadedoCEO–na

Posiçõesdevempoderser

Simpleseinformal.

práticadosplanejadores.

defendidas.

Figura3.9–Escolasdoplanejamento–prescritivas

Aspróximasescolasqueveremosestãonogrupodasescolasdescritivas–que

sepreocupamcomoprocessodeformulaçãodaestratégiacomoelerealmente

ocorre.

Aprimeiraqueveremoséaescolaempreendedora.Aqui,oprocessode

planejamentoévistocomoresultadodeumavisãodelongoprazodoprincipal

executivo,ouseja,todaaestratégiaderivadeumprocessovisionáriodesse

indivíduo!

Dessaforma,éumprocessobaseadonasexperiênciasanteriores,vivênciase

pontosdevistadesseexecutivo.Assim,ébaseadonaintuiçãoeanálisepessoaldas

capacidadesedodestinoqueessaorganizaçãodevealmejar.

Comisso,oplanejamentodevesercontroladoecomunicadopelochefe

máximo,etodaaestratégiadeveserrevisadaconstantementeporesseexecutivo.

Jáaescolacognitivasebaseianosprocessosmentaisnecessáriosaoprocessode

formulaçãodaestratégia.Assim,nãobastaapreocupaçãocomoqueénecessário

“saber”paraqueumindivíduopossaserumestrategista–hádesepreocuparcom

omodocomoessesconhecimentossão“construídos”dentrodamentedos

planejadores.

Essaescolaébastantenovaeaindaestásedesenvolvendo,massemostra

bastantepromissora.Entretanto,aindanãotrouxemuitassoluções,apenasuma

melhorcompreensãodasdiculdadesedistorçõesquetemosaoanalisarosdados

einformaçõesemontarosconceitoseesquemasemnossamente.

Umdasescolasmaisinteressanteséadoaprendizado.Nessaescola,

pensamentoédequeoprocessodemontagemdaestratégianãodependedeumsó

indivíduo–ochefe.Issoocorrepoisseacreditaquearealidadeémuitocomplexa

paraquesomenteumapessoapossa“dominar”todososdadosnecessários.

Assim,amontagemdaestratégiaévistacomoumprocessodeaprendizagemao

longodotempo.Dessamaneira,ochefeaprende.Masomaiscomuméquemuitas

pessoasdentrodaorganização–acoletividade–aprendamjuntascoma

experiência.

Assim,existemmuitospotenciais“estrategistas”dentrodaempresa.Eesse

planejamentonãoaconteceemummomentoinicial.Elevaiseformandocomo

passardotempoedosfatos–oqueMintzbergchamadeprocessoemergencial.20

Ouseja,écomopassardotempoqueaestratégiavaiseformando.

Opapeldolíderserianãoodeconcentraroplanejamento,masdecriarum

ambientepropícioaoaprendizadoestratégico,demodoquenovasestratégias

“floresçam”.

Apróximaescolaéadopoder.Nessavisão,oprocessodeformulaçãoda

estratégiaévistocomoumabarganhaentreosdiversos“atores”dentroda

organização.Assim,aestratégiaéformadaaospoucos–atravésdanegociaçãoe

persuasãodosmembrosinfluentesdaempresa.

Alémdisso,Mintzbergdivideessepoderemdoistipos.Omicropoderéo

processodeformaçãointernodaestratégia.Nessecaso,oprocessodeformulação

tambéméemergente,ouseja,vaiseformandocomotempo.

Jáomacropoderéautilizaçãodainuênciadaorganizaçãopara,emconjunto

comoutrasorganizações,buscarosinteressesdaempresa.Assim,umaempresaem

diculdadespodenegociarumempréstimocomumbancodogoverno,pois

poderiademitirmilharesdepessoasdeumavezsó.

Emoutrasituação,porexemplo,poderiamontarumaparceriacomum

concorrenteparadominarummercadoespecíco.Assim,aestratégiaseriaa

buscadealiançasenegociaçõescomoutrosentes,deformaamaximizaras

possibilidadesdaempresa.

Temosaindaaescolacultural,quevêoprocessodeplanejamentocomoum

processocoletivointerativoequereeteascrençaseosvaloresdessesindivíduos.

Assim,asestratégiasseriamderivadasdamaneiradepensar,eosvalores,das

pessoasdentrodasorganizações.

Umbancodeinvestimento,porexemplo,teriaatendênciadebuscarriscos

maioreseestratégiasmaisarriscadasdoqueumbancodogoverno–reetindoas

diferençasculturaisdeseusmembrosesócios.

Finalmente,aescolaambientaldizqueoatorprincipalnoprocessodo

planejamentoéoambiente,ouseja,queaorganizaçãoépassiva.Seupapelé

apenasadaptar-seàsmudançasnoambiente–ou“morrer”.

Dessaforma,opapeldolíderé“perceber”asmudançasedesenvolveras

transformaçõesnecessáriasparaqueaorganizaçãosobreviva.

Vejaumresumodasprincipaiscaracterísticasdessasescolas:

Empreendedora

Cognitiva

Aprendizado

Processo

Processodeapredizadocoletivo

visionáriodo

Processomentaldecriação.

daestratégia.

Líder.

Estratégiaévisualizadapormapas,

Processoemergente–estratégia

Baseadona

conceitoseesquemas.

seformacomotempo.

intuição.

Poder

Cultural

Ambiental

Processode

negociação.

Processocoletivoquereflete

Processoreativoaoambiente.

Micropoder–disputas

ascrençasevaloresdo

Organizaçõessãopassivas–ouse

internas.

grupo.

adaptamaoambienteou

Macropoder–usoda

Estratégiarefleteacultura

“morrem”.

influênciapela

dominante.

organização.

Figura3.10–Escolasdoplanejamento–descritivas

Oúltimogruposócontacomumaescola–adaconfiguração.Paraela,as

organizaçõestendemamanterumaconguraçãoestáveldecertascaracterísticas:

tipodeestrutura,atuaçãoemumcontextoeestratégiascoerentes.

Entretanto,detemposemtemposelaspassariamporumprocessode

transformação(comoumaempresaregionalpassandoaatuarnacionalmente,com

umacompetitividadeeescalamaiores).

Assim,osgestoresdeveriammantercertaestabilidadenasestratégias,aomesmo

tempoemquedeveriammanteraempresacomumacapacidadedeadaptação–

paraosperíodosdetransformação.

Dessamaneira,aempresadeveriaadotarumaestruturadeformaçãode

estratégias,emfunçãodaalternânciaentreestabilidadeenecessidadede

transformação.DeacordocomMintzberg,essaescolaabrangetodasaspremissas

dasescolasanteriores.

Vamosvercomoissojáfoicobrado?

4.(Cespe/Unipampa/Administrador/2009)Umaorganizaçãoqueatribuiaoseuprincipalexecutivoa

responsabilidadedeelaborartodooprocessodeconcepçãodasestratégiasadotaalosoada

escoladoaprendizado.

Aescoladoaprendizadodizexatamenteocontrário–queoprocessode

formaçãodaestratégiaécoletivo,queexistemdiversospotenciais“estrategistas”

dentrodaorganização.

AsescolasqueatribuemaoCEO(executivomaisgraduado)aresponsabilidade

pelaconcepçãodaestratégiasão:aescoladodesigneaescolaempreendedora.O

gabaritoéquestãoerrada.

3.7.PlanejamentoporCenários

Oplanejamentoporcenárioséumaferramentaqueusamosnoprocessode

planejamentoparapodermosterumavisãomaisabrangenteemumcontextode

muitaincerteza.

Cenáriospodemserdescritoscomopossíveissituaçõesnofuturo.Essas

“estórias”construídaspodemnosajudar,poisnospossibilitamreconhecerenos

adaptaràsmudançasquandoestasocorrerem.

Dessaforma,oplanejamentoporcenáriosserelacionacomaconstruçãodas

“estórias”possíveis,quemostramalgunscaminhosqueaorganização(eaténós

mesmos,emnossasprópriasvidas!)podepercorrer.

Assimpodemosimaginarqualdevesernossocomportamentosealgumasdessas

“estórias”setornaremrealidade.21

Imaginequevocêtrabalheemumamontadoradeautomóveis.Seutrabalhoé

analisarquaisseriamoscenáriosdomercadobrasileironoanode2020.

Assimsendo,vocêprecisaentenderquaissãoasforçasetendênciasquepoderão

impactaromercadodeautomóveis,deformaapreverquaisseriamosmodelos

queseriammaisdemandados,quaisregiõescresceriammais,quefaixaetáriaseria

maispropensaacomprarcarrosnovosetc.

Provavelmente,vocêchegariaàconclusãodequeexistemcertosfatoresquesão

maisimportantesnessaanálise,como:ocrescimentodaeconomia,o

envelhecimentodapopulação,onúmerodepessoasqueestarãosaindodas

grandescidadesetc.

Cenários:Sãosituaçõesfuturasquepodemnosajudaracompreendernossosdesaos

potenciais.

Comisso,oseusetormontariacenáriospossíveisparaomercadonoanode

2020.Comessescenáriosoupossíveis“futuros”,cariamaisfácilparaosgestores

decidirememquemodelosinvestirequaisseriamasestratégiasquedeveriamser

seguidasparacadatipodecenário.

VocêselembradolmeDeVoltaparaoFuturo?Esselmemostravacomoa

viagempelotempopoderiaalteraravidadaspessoasnoseufuturo.Umafesta

“perdida”poderiacausarumamudançadrásticanavidadeumapessoa:

casamentosnãoaconteceriam,filhosnãonasceriametc.

Decertaforma,ogestorbusca“imaginar”comocertasdecisõesatuaispodem

“mudarofuturo”daempresa.Comoaeleiçãodeumgovernodeoposiçãoafetará

asregrasparanossosetor?ComoaentradadevintemilhõesdepessoasnaclasseC

podealterarademandapornossosprodutos?

Comessasquestões,vamosmontando“situações”oucenáriospossíveis,oque

nosfaráaprendersobrenossoambienteenosprepararáparaospróximosdesafios.

Portanto,oplanejamentoporcenáriosestáligadoàtomadadedecisõesatuais

eàcompreensãodecomoessasdecisõespodemafetarnossasituaçãofutura.

Oscenáriospodemserutilizadostantoemgrandesempresasquantoem

pequenasorganizações,e,comofalamos,atéemnossasdecisõespessoaisdodiaa

dia.Essaferramentapodeajudaraspessoasatomarmelhoresdecisões

(principalmenteemquestõesdifíceis).

Devemos,porém,tercuidado–cenáriosnãosãoprevisões!Nãoépossível

preverofuturocomalgumgraudecerteza.Portanto,oscenáriosdevemser

vistoscomoumauxílioparaajudaraspessoasemseuaprendizado.

Aocontráriodosmétodosdeprevisãotradicionais–relacionadosàanálisede

tendências–,oscenáriosfornecemalgumasvisõesalternativasdofuturoenão

extrapolamsimplesmenteastendênciasatuais.

Entretanto,deacordocomDjalmaOliveira,essasimplesextrapolaçãode

tendênciaspassadaspodeserconsideradaumtipodeconstruçãode

cenário.22

Paraesseautor,ametodologiadeplanejamentoporcenáriosdivide-seemduas

abordagens:aprojetivaeaprospectiva.Naabordagemprojetivaprocura-secriar

umúnicocenáriobaseadonosfatospassados.Ocenáriofuturoseriaentão

único!

Figura3.11–Abordagemprojetiva

Jáaabordagemprospectivabuscariacriardiversoscenáriosfuturospossíveis

tendocomobaseoestadoatual,ouseja,opresente.

Figura3.12–Abordagemprospectiva

Vamosvercomoissojáfoicobrado?

5.(Cespe/MPS/Administrador/2010)Sãoasabordagensprospectivaeprojetivaconsideradasas

principaisformasbásicasdedesenvolvimentodecenários.

Aquestãoestáobviamentecorreta,seguindoaclassicaçãodeDjalmade

Oliveiracitada!

Aabordagemprospectivaé,apesardisso,muitomaisinteressante.Geralmente,

ogestorprefereailusãodacertezadeumdeterminadofuturoprojetadoà

compreensãodosriscosedasrealidades.

Mas,nolongoprazo,essarecusaemveraincertezaolevaráatersurpresas

desagradáveis.Aconstruçãodecenáriosnãolheforneceumaúnicavisãode

futuro,maspossibilita-lhepreparar-separa“oquepossaacontecer”!

Portanto,parapoderlidarcomummundoemconstantemudançaeincerteza,

aspessoasdevemsercapazesdemudarsuaspercepções–dequestionarsuasvisões

demundoeseusentendimentosdecomoomundofuncionaparaquepossam

“enxergar”melhorasengrenagensquefazemomundomudar.

Oobjetivodousodecenáriosé,portanto,ajudarogestoramudarsua

percepçãoarraigadadarealidade–aquestioná-la–paraquepossaentender

melhorsuasituaçãoecomoelapodemudarnofuturo.

Oscenáriosdevemterasseguintescaracterísticas–ouatributos:serrelevantes

(quetratemdefatoresnovoseimportantesnofuturodaempresa),plausíveis(que

tenhampossibilidadesreaisdeocorrerem),claros(umcenárioextremamente

complexopodenãosercompreendidoeaceitopelachea)efocados(demodoa

analisarsomentefatoresemudançasrelevantes).

Figura3.13–Atributosdoscenários

3.8.AnáliseCompetitivaeEstratégiasGenéricas–Estratégia

Nãoexisteumadeniçãoúnicasobreoconceitodeestratégia,masosconceitos

geralmentereetemanecessidadedesedeniremosobjetivosaseremalcançados,

comoessesobjetivosdevemseralcançados(omelhor“caminho”)eaanálisedas

forçasambientais(tantointernasquantoexternas)queaorganizaçãoenfrentano

momento.

Aseguir,descrevemosalgumasdefiniçõesdeestratégiadeautoresconhecidos:

Porter23–Estratégiacompetitivasãoaçõesofensivasoudefensivasparacriarumaposiçãodefensávelnumaindústria,paraenfrentarcomsucessoasforçascompetitivaseassimobterum

retornomaiorsobreoinvestimento.

Chandler24–Estratégiaéadeterminaçãodeobjetivosemetasbásicasdelongoprazodeumaempresaeaadoçãodecursosdeaçãoealocaçãoderecursosnecessáriosparaalcançaresses

objetivos.

Osconceitosdeestratégiaevoluírambastantenosúltimos50anos,eatualmente

temosduascorrentesprincipais:acorrentequeconsideraaestratégiacomo

posicionamentoeacorrentequeconsideraaestratégiaummovimento.

3.9.EstratégiacomoPosicionamento

Essacorrente,daqualPorteréoprincipalautor,temcomobaseanoçãode

adequaçãoestratégica.Nesseenfoque,aestratégiaserelacionaaosconceitosde

adaptaçãoeposicionamento.Ouseja,acorrentesugerequeasorganizaçõesdevem

seadaptaraoambienteparapoderemconquistarvantagenscompetitivaseuma

posiçãodominanteemseusmercados.

Essacorrentedefendeaanálisedoambiente,tendocomoferramentaprincipala

análiseSWOT.Apósconhecersuasituaçãoestratégica,aorganizaçãoutilizaria

seusrecursosnabuscadevantagenscompetitivas,deacordocomosconceitosde

Porterqueveremosadiante.

3.10.ModelodeCincoForçasdePorter

MichaelPorternostrouxetambémoconceitodascincoforçascompetitivas

queafetamaestratégiadeumaempresa.25Essesfatoressãoosseguintes:

AmeaçadeNovosEntrantes–Altoinvestimentonecessárioeeconomiasde

escalasãoalgunsdosfatoresquepodemdicultaraentradadeumnovo

competidoremummercado.Naturalmente,émaisdifícilabrirumanova

indústriaaeronáuticadoqueumanovalojaderoupas.Dessaforma,asempresas

queestãoemsetorescomaltasbarreirasàentradasofremmenoscompetiçãodos

queasqueestãoemmercadoscombaixasbarreirasdeentrada.

PoderdeNegociaçãodosClientes–Quantomaisinformadosestãoosclientes,

maiselesnormalmentepodemexigirdasempresasqualidade,preçoeserviços.Os

clientessãopoderososquandosãopoucos,oucompramemgrandequantidade,

quandooscustosdetrocardefornecedorsãobaixos,quandoelesconhecemas

estruturasdecustosdasempresasequandopodemdeixardeconsumiros

produtosoufabricá-losinternamente.

PoderdeNegociaçãodosFornecedores–Muitosdosfatoresquepodem

deixarosclientesfortespodemdeixarosfornecedorespoderososseforem

invertidos!Osfornecedoressãofortesquando:sãopoucose/oudominamo

mercado,quandoocustodetrocardefornecedoréalto,quandoosclientessão

poucoimportantesequandopodemsetornarcompetidores,ouseja,passara

concorrernomercadodocliente.

AmeaçadeProdutosSubstitutos–Umprodutoésubstitutoquandosatisfaza

mesmanecessidadedosclientes(exemplo:manteigaemargarina).Seexistem

muitosprodutosquepodemsubstituiroprodutoquesuaempresafornece,a

posiçãoestratégicaédifícileosetorserámenosatraenteelucrativo.

RivalidadeentreosConcorrentes–Seexistemmuitosconcorrentesemum

mercadoeseaforçadelesésemelhante,podeocorrerumaguerradepreços,

levandoaumaquedanaatratividadedosetor.Outrosfatoresquelevamaissosão:

custosxoselevados,quepodemlevarasempresasabuscaroperarcom

capacidadetotal,eumagrandebarreiradesaída,comoinstalaçõescaras,dedifícil

venda,maquinárioespecícoealtasindenizaçõesquepodemlevarempresasa

continuarinvestindoeoperandoemmercadoscomlucratividadesbaixas.

Figura3.14–ModelodePorterdascincoforçascompetitivas.

(Fonte:Porter,HowCompetitiveForcesShapeStrategy,1979)

3.11.EstratégiasGenéricasdePorter

Entenderascincoforçascompetitivaspossibilitaaogestorescolheramelhor

estratégiaparasuaorganização.DeacordocomPorter,astrêsprincipais

estratégiasgenéricassão:liderançaemcustos,diferenciaçãoefoco(ouenfoque).26

Liderançaemcustos–Nessaestratégia,aempresabuscaseramaiseficientena

produçãodeprodutoseserviçosemseumercado,demodoquetenhavantagem

competitivaemrelaçãoaosseusconcorrentes.Pode-sealcançarissocom:

economiasdeescala,acessoamatérias-primasmaisbaratas,entreoutras.Essa

posiçãodecustomaisbaixoqueseusconcorrentespermiteumasériede

vantagens,comooperarcomlucratividadequandoseusconcorrentesestão

perdendodinheiro,porexemplo.

Diferenciação–Umaempresatambémpodetervantagenscompetitivastendo

produtoscomcaracterísticasúnicasnapercepçãodeseusclientes,quelhe

possibilitemcobrarumpreçomaisaltosemperdersuaclientela.Umexemplo

atualéaApple.Essaempresa,comseusprodutosinovadorescomooiPhoneeo

iPad,temconquistadoumamaiorlealdadedeseusclientesemaiorlucratividade.

Adiferenciaçãopodeocorrernaqualidadedoproduto,noatendimento,noestilo

doproduto,namarcaetc.

FocoouEnfoque–Tambéméchamadadeestratégiadenicho.Nessasituação,a

empresafocaseusesforçosemummercadopequeno(sejageográco,produtoou

clientela)demodoaconseguirumavantagemespecícanaquelemercado,que

nãotenhacomoconseguiremtodoomercado(aFerraribuscouessaestratégia

comofocoemcarrosdealtodesempenho,poiserapequenaparaconcorrerno

mercadodeautomóveispopulares,muitomaior,antesdesercompradapelaFiat).

VantagemEstratégica

EscopodeAtuação

PosiçãodeBaixoCusto

UnicidadeObservadapeloCliente

Competitiva

TodaaIndústria

LiderançaemCustos

Diferenciação

SegmentoEstreito

Foco

Figura3.15–EstratégiasgenéricasdePorter

3.12.MatrizdeAnsoff

AmatrizdeAnsofféumaferramentaqueauxiliaogestornaavaliaçãodas

oportunidadesdecrescimentodeumadeterminadaunidadedenegócioemum

mercadoespecífico.27

Essamatriztambéméchamadade“produtoversusmercado”.28Atravésdessa

ferramenta,comparamososdadosdemercado(novoseexistentes)eosdados

dosprodutos(novoseexistentes)epodemoscriarquatro“estratégiaspossíveis”.

Vejaafiguraaseguir:

Produtos

Existentes

Novos

Existentes

PenetraçãodeMercado

DesenvolvimentodeProdutos

Mercados

Novos

DesenvolvimentodeMercado

Diversificação

Figura3.16–MatrizdeAnsoff.

(Fonte:AdaptadodeKotler,2000)

Asestratégiaspossíveissãoasseguintes:29

ØPenetraçãodemercado–Nessescasosemqueaempresajádispõedos

produtoseosmercadosjáestãosendoatendidos,eladevebuscarampliara

suapresençaeganhar“espaço”(participaçãodemercado,oumarket-share)

diantedosseusconcorrentes.

ØDesenvolvimentodemercado–Essaestratégiaocorrequandotentamos

utilizarprodutosexistentesemnovosmercados.Dessemodo,buscamos

ganharmercadoemmercadosaindanãoexploradospelaempresa.

ØDesenvolvimentodeprodutos–Nessasituação,aempresabuscalançar

novosprodutosparavenderaosclientesatuaisemercadosemqueelajáatua.

Aideiaéinovareatenderademandasnovasdosclientesqueacompanhiajá

tem.

ØDiversificação–Essaéaestratégiamaisarriscada,poisaempresalançará

umprodutonovoemummercadoqueaindanãoestápresente.

Essaferramentaauxiliaogestornaformulaçãoestratégica,demodoafacilitara

visualizaçãodasalternativaspossíveisdeação.

3.13.EstratégiacomoMovimento

Apartirdasdiversasmudançasqueocorreramnoambientedenegóciosapartir

dosanos1980,comoaaceleraçãodaglobalização,ainternet,oaumentoda

concorrêncianosmercados,aintroduçãodotrabalhovirtual,entreoutros,

levaramàcriaçãodeumasegundacorrente:acorrentedomovimento.

Nessavisão,aideiapredominanteéadequeasposiçõesestratégicasestãoem

constanteataquedomercadoequeamelhoralternativaésemprese

“movimentar”,ouseja,nuncaseacomodar.Umambientedinâmicotornaadefesa

deumaposiçãoestratégicaalgoextremamentedifícil.

OsautoresmaisconhecidosdessacorrentesãoHamelePrahalad,30que

criaramumnovoconceito:odaintençãoestratégica.Paraeles,asempresas

devemperseguirumavisãodelongoprazoeterumconjuntodecompetências

centraisparatersucesso.

Oobjetivodaestratégiadeixadeseraadequaçãoestratégicaaoambiente

externoparasetornaraeternarenovaçãoeinovação,buscandosempreo

“movimento”estratégico.Podemosresumiressaideiacomo:o“ataqueéamelhor

defesa”comoaestratégiaparasemantercompetitivo.

3.14.TiposdeEstratégia

SegundoDjalmadeOliveira,31asestratégiaspodemserclassicadasemquatro

tiposprincipais,deacordocomsuasituaçãoestratégica:

3.14.1.EstratégiadeSobrevivência

Essaestratégiadeveriaseradotadaquandoaempresaenfrentaumnúmero

grandedeameaçasenãocontacomumasituaçãointernafavorável,ouseja,tem

muitasfraquezas.

Dessemodo,nãorestariammuitasalternativasaogestoranãoser“apertaro

cinto”,ouseja,cortartodososinvestimentosereduzirosgastos.Oliveira

subdivideessaestratégiaem:32

ØReduçãodecustos–Nessaalternativaoobjetivoéreduzirosgastosa

qualquercusto!Todosossupéruossãocortadosparaqueaorganização

consigasuportarafasenegativaevolteaterresultadospositivos.

ØDesinvestimento–Muitasvezesamelhoropçãoédescontinuaraprodução

dealgumproduto/serviçoquenãoestejasendolucrativo,demodoa

possibilitarqueaempresaseconcentreemseusprodutosprincipais.

3.14.2.EstratégiadeManutenção

Aorganizaçãodevebuscarumaestratégiademanutençãoquandoexistirem

muitasameaçasnoambiente,mastambémcasoexistamforçasinternasque

possibilitemàempresaenfrentaressasameaças.Oliveirasubdivideessaestratégia

em:33

ØEstratégiadeestabilidade–basicamente,ocorrequandoumaempresaque

estavalucrativapassapormomentosdifíceisnanceiramente,comouma

dívidaouprejuízomomentâneo.Aempresabuscaentãoumretornoà

situaçãoconfortávelquedetinha.

ØEstratégiadeespecialização–aempresafocasuaatençãoempoucos

produtosoumercados,deformaaganharcompetitividadenessasáreas.O

problemaéqueaorganizaçãocamaisvulnerávelaataques,poisca

dependentedeumsósetorouprodutoparasobreviver.

ØEstratégiadenicho–aempresabuscaumespaçoaindamaisrestritodoque

naestratégiadeespecialização.Aqui,aempresabuscaseramelhoremum

únicoprodutooumercado,geralmentepequenoobastanteparapoderser

defendidoporumaempresamenor.

3.14.3.EstratégiadeCrescimento

Quandoaempresavislumbramuitasoportunidadesemseuambiente,apesarde

contaraindacommuitospontosfracos,elapodeoptarporumaestratégiade

crescimento.Oliveirasubdivideessaestratégiaem:34

ØEstratégiadeinovação–aorganizaçãoprocuralançarnovosprodutos,com

tecnologiaavançadaouinédita,embuscadecriarumimpactonamentedos

consumidores.

ØEstratégiadeinternacionalização–aempresavaibuscaroperarempaíses

ondeaindanãotemnegócios.

ØEstratégiadejointventure–ocorrequandoexisteumaparceriadeduas

empresaspara“atacar”ummercadoespecífico.

ØEstratégiadeexpansão–aexpansãoéoaumentodeproduçãodosprodutos

e/ouserviçosqueaempresaforneceaomercado.Seaempresanão

acompanharaevoluçãodomercado,podeperderparticipaçãodemercado

pornãopoderatenderaseusconsumidores.

3.14.4.EstratégiadeDesenvolvimento

Quandoexistemmuitasoportunidadeseforçasinternasdentrodaorganização,

aestratégiaaserbuscadaéadedesenvolvimento.Podemserbuscadostantonovos

mercados,produtosetecnologias,bemcomoumadiversicaçãodosatuais

produtosemercados.Oliveirasubdivideessaestratégiaem:35

ØDesenvolvimento–aempresapodeconquistarnovosmercadosparaseus

produtos,desenvolverprodutosmelhoresemaisavançados,buscara

associaçãovisandomelhoriasnospanoramasnanceiros,decapacidadesede

mercado(desenvolvimentodemanutenção).

ØDiversificação–naestratégiaconsistenabuscadenovasoportunidadese

mercadosquandoosprodutoseosmercadosatuaisjáestãosaturados,não

apresentamboalucratividadeouestãoemdeclínio.

Fazendoum“cruzamento”entreaanáliseSWOTeostiposdeestratégia,temos

aseguintematriz:

AmbienteExterno

Oportunidades

Ameaças

Forças

Desenvolvimento

Manutenção

AmbienteInterno

Fraquezas

Crescimento

Sobrevivência

Figura3.17–MatrizderelaçãoentreaanáliseSWOTeostiposdeestratégia

Vamosvercomoissojáfoicobrado?

6.(Esaf/CGU/AFC/2012)ComoensinaaanáliseSWOT,noscasosemque,dacombinaçãoentre

ambientesevariáveisresultarapredominânciasimultâneadepontosfortesedeameaças,

espera-sequeaorganizaçãosedecidapela(o):

a)sobrevivência;

b)desenvolvimento;

c)manutenção;

d)crescimento;

e)confrontação.

EssaquestãodaEsafdemandouumconhecimentosobreostiposdeestratégiase

seurelacionamentocomaanáliseSWOT.Aquestãoébem“decoreba”,mas

acreditoquemuitospoderiamteracertadopela“lógica”.

Setemosmuitospontosfortes,porexemplo,poderíamoslogo“eliminar”aletra

A,poisnãoseriaadequadaumaestratégiadesobrevivência.AletraEnão

apresentaumtipodeestratégia.Assim,tambémseriadescartada.

Aestratégiadecrescimentodependedetermosoportunidades.Assim,comoa

questãonosapresentaumcenáriodeameaças,poderíamosdescartartambéma

letraD.Aestratégiadedesenvolvimentonecessitadeforçaseoportunidades.Com

isso,aopçãocorretaéaestratégiademanutenção.OgabaritoémesmoaletraC.

3.15.Redesealianças

Emummundocadavezmaisglobalizadoecomempresascadavezmaiorese

atuantesemescalatransnacional,nãoémaisadequadopensarmosuma

organizaçãocomoumenteisolado,ouseja,semlevarmosemconsideraçãosuas

redesealianças.

DeacordocomPitassieMacedo-Soares,36

Parasustentarsuavantagemcompetitiva,umnúmerocrescentedeempresasnoBrasil,comono

restodomundo,estáestabelecendomúltiplasaliançasdediferentestipos,constituindo-seem

redes,inclusivevirtuais.

Atualmente,grandepartedasempresasbrasileirasemundiaistembuscado

formarredesealiançasestratégicasdemodoapodercompartilharrecursose

competências,alémdereduzirseuscustos.37

Sejaparaatenderaumnovomercadoouparamelhoratenderaosatuais

clientes,asredespossibilitamumfortalecimentoestratégicodaempresaperante

seusconcorrentessemumaumentograndedoscustos.

DeacordocomGulati,38

Permiteperceberasoportunidadeserestriçõescriadaspelasredesestratégicas,quepodem

proporcionaràsempresasacessoainformações,recursos,mercadosetecnologiasquecontribuem

paraavantagemcompetitiva,bemcomocompartilhamentoderiscos,outsourcingdecadeiasde

valorefunçõesorganizacionais,aomesmotempoquepodemencapsularasempresasemrelações

improdutivasouprivá-lasdeoutrasassociaçõesmaisvantajosas.

Portanto,asaliançaspermitemàsempresasdar“saltos”maioresdoque

seriapossívelsemelas.AEmbraer,porexemplo,rmounosanos1990diversas

parceriascomempresasestrangeirasnodesenhoenafabricaçãodomodeloERJ-

145,quandoenfrentavamuitasdificuldadesfinanceiras.

Cadaempresacouresponsávelpordesenharuma“parte”doaviãoeseria

contratadaparafabricá-laquandoalgumaviãofossevendido.Osmotoresforam

projetadosporumacompanhia,osequipamentoseletrônicos,poroutraetc.

Comisso,aempresaconseguiudesenvolveroprojetodoaviãocomum

investimentomenordoqueseriaantesimaginadoeconquistou“mercado”como

ganhodeimagempossibilitadopelaassociaçãocomosseusparceiros–todos

conhecidosevalorizadosnomercadoaeronáutico.

DeacordocomTauhataeMacedoSoares,asaliançaseredespodemserdos

seguintestipos:39

Øjointventures;

Øcontratosdefornecimentodelongoprazo;

Øinvestimentoacionáriominoritário;

Øcontratosdefornecimentodeinsumos/serviços;

Øpesquisaedesenvolvimentoemconjunto;

Øfusõeseaquisições.

Entreosefeitospositivosdessasredesealianças,podemoscitaroganhona

posiçãodebarganha–oudenegociação–queocorrequandoessasredespassama

negociarcomseusfornecedoresemconjunto,eoaumentodo“custodeentrada”

dospotenciaisconcorrentesemummercado(aumentodabarreiradeentrada).

3.16.AdministraçãoporObjetivos

Antigamente,asempresastinhamumprocessodeplanejamentocentralizado.

Nesseprocesso,aorganizaçãomantinhaumdepartamentodeplanejamentoe

pesquisasquefaziaodiagnósticointernoeexternoedeterminavaosobjetivosda

companhiacomoumtodoedecadadepartamento.

Naturalmente,essapráticademandavaumagrandecapacidadedeanálisedessa

equipe,poisdeveriaterconhecimentodediversosaspectosdecadamercadoem

queaempresaestivesseenvolvidaedascapacidadesinternasdecada

departamento.

Dessemodo,oprocessoeralentoemuitasvezesnãolevavaemconsideração

aspectosedetalhesqueseriamdesconhecidosdessesintegrantes.

Paramelhoraroprocessodeplanejamento,PeterDruckerpassouaproporum

novomodelo–aAdministraçãoporObjetivos.40Deacordocomele,aideiaéo

envolvimentodetodososmembrosorganizacionaisnoprocessodedeniçãodos

objetivos.

NaAdministraçãoporObjetivos–ouAPO–oplanejamentonãoteriaum

caráterde“cimaparabaixo”,comosdiretoresdecúpulaimpondometase

objetivosaossubordinados,massimumprocessode“baixoparacima”.

Nessemodelo,osobjetivossãoestabelecidosdecomumacordoentreoschefese

ossubordinadosdecadaórgão.Issoseriamaisadequadoaumambientemutávele

dinâmicoenoqualosenvolvidosdiretamentecomosproblemasteriamuma

melhorcondiçãodetomarasdecisõeseteracessoadadoseinformações.

Oconceitopartedapremissadeque,seossubordinados“comprarem”aideia

enegociaremosobjetivos,estarãomaiscomprometidoscomoatingimento

dosmesmos.

Ouseja,osfuncionáriosdecadadepartamentodiriamseasmetaseosobjetivos

poderiamounãoseratingidoscomosrecursosdisponíveis.Existiriaefetivamente

umanegociaçãoentrechefesdasáreaseosfuncionáriosemrelaçãoaosobjetivose

osrecursosnecessáriosparaaequipe.

DeacordocomSobral,aAdministraçãoporObjetivoséumprocessocomposto

dequatrofases:41

ØEspecicaçãodosobjetivos–nessafase,cadaórgãodeterminaria,demodo

participativo,osobjetivosemetasdeseusetor.

ØDesenvolvimentodeplanosdeação–apósadeniçãodosobjetivos,seriam

detalhadososplanosdeaçãoqueindicariamcomoosobjetivosseriam

alcançados.

ØMonitoramentodoprogresso–naturalmente,todoodesempenhodeveser

monitoradoparaquesaibamosseosobjetivosestãosendoatingidosese

serãonecessáriasalgumascorreçõesderumo.

ØAvaliaçãodosresultados–nessafase,osresultadossãocomparadoscomo

planejamentoeservemdesubsídioaoprocessodeplanejamentofuturo.

Assimcomoqualquerferramentagerencial,aAPOapresentaalgumascríticas.

Entreelas,teríamosumaexcessivapreocupaçãocomocurtoprazo,aperdada

noçãodo“todo”comodesmembramentodosobjetivoseumaexcessiva

preocupaçãocomosobjetivosindividuaisedepartamentaisemcontrapontoaos

objetivosestratégicosgerais.

3.17.ControleEstratégico

Apósadeniçãodaestratégiaorganizacionaledoiníciodaexecução,será

necessáriaautilizaçãodeferramentasdecontroleestratégico.Atualmente,a

principalferramentaparaocontroleestratégicoéoBalancedScorecard.De

acordocomLobatoetal.,42

Nagestãoestratégicacompetitiva,oalinhamentoecontroleestratégicossãosuportados

pelaferramentaBalancedScorecard(BSC),pelaqualseelaborammedidasnanceirasenão

financeirasquepossibilitamodesdobramentodasestratégiasaseremimplementadas.

Dessamaneira,osgestorespoderãoacompanharosresultadosdaorganização

deacordocomaestratégiatraçada,demodoacorrigirosdesvioseaprender,

gerandoconhecimentocomtodooprocesso.Vamosconheceraferramenta?

3.18.BalancedScorecard

Imaginedirigirumveículoesóteràdisposiçãoovelocímetro.Vocêconsegue

verqualavelocidadeemqueestádirigindo,masnãosabeseseutanqueestácheio,

quantosquilômetrosjárodou,ouseaáguadoradiadorestádentrodoslimitesou

não.Dessaforma,teráumagrandechancedenãochegaraoseudestino,nãoé

mesmo?

Portanto,vocêprecisateràdisposição,emseuveículo,umasériede

indicadoresquelhepossibilitementendercomoseucarroestásecomportando.O

mesmoocorrecomasorganizações.

Atépoucotempoatrás,osúnicosindicadoresutilizadoseramosnanceiros.

Dessaforma,osgestoresmediamasituaçãodeumaorganizaçãodeacordocom

umsópontodevista:suacapacidadenanceira.Entretanto,indicadorescomoo

retornosobreoinvestimento,liquidezemargemlíquidasobrevendassão

importantes,massócontamumladodahistória.

DeacordocomKaplaneNorton,43paraqueasorganizaçõesmodernaspossam

“navegar”emumfuturomaiscompetitivo,detecnologiaavançadaecompessoas

cadavezmaiscapacitadas,seránecessáriomaisdoqueapenasmonitorardados

financeirosdodesempenhopassado.

Paraosautores,osindicadoresnanceiroscontamosfatospassados,massão

inadequadosparaavaliarodesempenhodeempresasquebuscamcriarvalor

atravésdoinvestimentoemclientes,fornecedores,empregados,processos,

tecnologiaeinovação.

Umdosprincipaisproblemaséqueosdadoscontábeisenanceirossó

captamosativostangíveisdaorganização,ouseja,seusativosfísicos(imóveis,

bens,dinheiroetc.).Osativosintangíveis–valordamarca,percepçãode

qualidadedosprodutos,capacidadedosfuncionários,capacidadedeinovação–

normalmentenãosãocontabilizados.

Oproblemaéqueavaliaremediressesativosintangíveissetornamaisdifícil.44

Émaisfácilcontarodinheironobancodoqueavaliarquantovaleo

conhecimentodostrabalhadoresdeumaorganização,nãoéverdade?Entretanto,

essesativosintangíveis,antesrelegadosaumsegundoplano,serãocadavezmais

importantesnaeradoconhecimento.

Dessaforma,KaplaneNortonconstruíramummodeloquecomplementaos

dadosnanceirosdopassadocomindicadoresquebuscammedirosfatores

quelevarãoaempresaatersucessonofuturo.45

Dessaforma,nessemodeloexistemquatroperspectivas:nanceira,clientes,

processosinternoseaprendizadoeconhecimento.Osindicadoreseos

desempenhosavaliadosserãoderivadosdavisãoedaestratégiadaorganização.A

seguirpodemosverumdiagramaquerepresentaasquatroperspectivasdo

BalancedScorecard–BSC.

Figura3.18–BalancedScorecard

(Fonte:KaplaneNorton1996)

Nessecontexto,asperspectivaspodemserdescritasassim:

ØPerspectivananceira–analisaonegóciodopontodevistananceiro.

Relaciona-senormalmentecomindicadoresdelucratividade,46comoreceita

líquida,margemlíquida,retornosobreoinvestimento,entreoutros.Indica

seaestratégiadaempresaestásetraduzindoemresultadosfinanceiros.

ØPerspectivadosclientes–nessepontodevista,busca-seidenticaros

segmentos(declientesedemercados)emqueaempresaatuaráeasmedidas

dedesempenhoqueserãoaceitas.Geralmenteenvolveindicadorescomo:

satisfaçãodosclientes,retençãodeclientes,lucroporclienteeparticipação

demercado.Essaperspectivapossibilitaaogestorasestratégiasdemercado

quepermitirãoatingirresultadossuperioresnofuturo.

ØPerspectivadeprocessosinternos–identicaosprocessoscríticosquea

empresadevefocarparatersucesso.Ouseja,mapeiaosprocessosquecausam

omaiorimpactonasatisfaçãodosconsumidoresenaobtençãodosobjetivos

financeirosdaorganização.47Devemsermelhoradososprocessosexistentese

desenvolvidososqueserãoimportantesnofuturo.

ØPerspectivadoaprendizadoedocrescimento–identicaasmedidasquea

empresadevetomardemodoasecapacitarparaosdesaosfuturos.As

principaisvariáveissãoaspessoas,ossistemaseosprocedimentos

organizacionais.Dessaforma,asempresasdevemtreinaredesenvolverseu

pessoal,desenvolversistemasmelhoreseprocedimentosquealinhemos

incentivosaosobjetivoscorretos.

Vamosvercomoissojáfoicobrado?

7.(Cespe/TRE-ES/TECADM./2011)OusodeferramentascomooBalancedScorecard(BSC)éinviável

nomodelodegestãoestratégica,dadooníveldeatuaçãodessemodelo.

AutilizaçãodoBalancedScorecardnagestãoestratégicanãoéinviável,muito

pelocontrário.Essaferramentaéutilizadadentrodagestãoestratégicadeuma

organização.Ogabaritoéquestãoerrada

Continuando,navisãodosautores,oBSCdeveriasermaisdoqueumacoleção

deindicadorescríticos.Dessaforma,oBSCdeveincorporarumasériede

relacionamentosdecausaeefeitoeumamisturadeindicadoresdedesempenhoe

osvetoresquelevamaessesdesempenhos.

Atenção:OBSCnãoestábaseadosomenteemindicadoresfinanceirosecontábeis.

Ouseja,osistemademediçãodevefazerumarelaçãodiretaentreosobjetivos

nasváriasperspectivas,deformaqueelessejamgerenciadosevalidados.48

Vamosimaginarumcasoprático?

VocêtrabalhaemumaempresaderefrigerantesedevemontarumBSCparasua

empresa.Oprimeiroindicadorquevocêanalisaé:retornosobreocapital

investido.Dessamaneira,esseéumindicadordaperspectivafinanceira.

Masoquelevariaaumbomretornosobreocapitalinvestido?Pensandobem,

vocêconcluiqueaempresaprecisavendermaisaosseusconsumidores,demodoa

aumentarasreceitas.Paravendermaisaosseusconsumidores,aempresaprecisa

atendê-losbem,nãoéverdade?Elesprecisamestarsatisfeitoscomoserviço

existente.

Dessamaneira,oíndicedesatisfaçãodosclientespodeserumindicadorda

perspectivaclientes,poisvocêimaginaquequantomaiselesestiveremsatisfeitos

maiscomprarão!Mas,pensanovamentevocê,oquefazcomqueelesquem

satisfeitos?Imaginequeotempodeproduçãoedistribuiçãosejaoaspectomais

importanteparaseusclientes,poisquantomaisrápidoacervejachegaraoponto

deconsumo,mais“gostosa”elafica!

Portanto,otempoentreoprocessodeproduçãoeadistribuiçãoseriaum

indicadordaperspectivadosprocessosinternos,poisafetaaperspectiva

clienteseaperspectivananceira.Masoquedeveocorrerparaqueosprocessos

sejammaisbem-feitos?Talvezsejanecessáriotreinarsuaequipedafábricaemuma

novagestãodaqualidade.Portanto,oinvestimentoemqualicação

(horas/funcionário)seriaoindicadordeaprendizadoecrescimento.

Vejaumafiguraquesintetizaocaso:

Figura3.19–Relaçãocausaeefeitodosindicadores

Dessaforma,umBalancedScorecardbem-construídodevemostraraestratégia

daempresaoudodepartamentoenvolvido.49CadaindicadornoBSCdeveserum

passodaestratégia.DeacordocomKaplaneNorton,

EmpresasinovadorasestãousandooBalancedScorecardcomoumsistemadegestãoestratégica,

paragerenciarsuaestratégianolongoprazo.ElesestãoutilizandooScorecardpararealizarestes

processoscríticos:

1.Clarificaretraduziravisãoeaestratégia;

2.Comunicareligarosobjetivosestratégicosàsmedidas;

3.Planejar,estipularmetasealinharasiniciativasestratégicas;

4.Aumentarofeedbackestratégicoeoaprendizado.

3.19.MapaEstratégico

Osmapasestratégicossãoferramentasqueauxiliamnacomunicaçãoe

visualizaçãodasestratégiasgerenciadaspeloBalancedScorecard.50Atravésdo

mapaestratégico,conseguimos“resumir”gracamenteamissão,avisão,os

principaisobjetivosestratégicoseosindicadoresenvolvidos.

Muitasvezesaspessoasnãoentendemasestratégias.Senãoentendemosalgo,

provavelmentenãoiremosexecutarbem,nãoémesmo?Portanto,precisamosdos

mapasestratégicosparaqueessasestratégiasquemclarasparatodos.Deacordo

comHerrero,51

omapaestratégicoéarepresentaçãovisualdahistóriadaestratégiadeumaorganização.

Avisualizaçãoeacomunicaçãodasprincipais“diretrizes”daestratégiada

instituiçãosãofundamentaisparaqueaspessoascompreendamqualéo

“caminho”aseguireconsigam“alinhar”suasaçõeseatividadesparaqueos

objetivossejamatingidos.

Assim,omapaestratégicoassumeessepapeldefacilitaracomunicaçãoda

estratégia.Semele,cariamais“árida”etextualatarefadecomunicar.Aseguir,

podemosverumexemplodemapaestratégico(daAgênciaNacionaldeAviação

Civil–Anac).

Vejamqueainstituiçãoapresentasuamissãoesuavisãonodocumentoe

apontaseusfatores(ouperspectivas)estratégicos.Comoainstituiçãoédosetor

público,eleszeramadaptaçõesnomodelooriginaldeKaplaneNorton.Vejam

nodocumentoquesubstituíramos“clientes”e“aspectos

nanceiros”por

“regulados”e“sociedade”.

Dessamaneira,emapenasumapáginapodemos,mesmosemconhecerafundoa

Anac,perceberquaissãoosprocessoscríticoseobjetivosestratégicosemcada

perspectiva.Esseéoefeitodesejadodeummapaestratégicobenfeito!

DeacordocomKaplaneNorton,52osucessodaexecuçãodaestratégiadependede

suacompreensãopelosempregadosdaorganização,que,porsuavez,dependedeuma

nítidadescrição.

Paraosautores,53osdiferentessentidosdomapaestratégicosão:mostraro

destinoestratégico,destacarovalordocapitalintelectual,representar

visualmenteaestratégia,ligarotrabalhoindividualàestratégia,demonstraro

fluxodevalorereforçaraimportânciadoconhecimento.

VejaaseguiromapaestratégicodaAnac:

Figura3.20–MapaestratégicodaAnac54

Vamosvercomoissojáfoicobradoemconcursos?

8.(Esaf/CGU/AFC–2012)Consideradoumaimportanteferramentadegestãoestratégica,

oBalancedScorecardbuscaamaximizaçãodosresultadoscombasenasseguintes

perspectivas,exceto:

a)concorrênciaetecnologia;

b)financeira;

c)clientes;

d)processosinternos;

e)aprendizadoecrescimento.

QuestãobemsimplóriadaEsaf.Ocandidatodeveriaapenaslembrar-sedas

quatroperspectivasoriginaisdomodelodeKaplaneNorton:nanceira,clientes,

processosinternoseaprendizadoecrescimento.

AúnicaalternativaquenãoapresentaumaperspectivadoBalancedScorecardéa

letraA–concorrênciaetecnologia–,queé,portanto,ogabaritodabanca.

QuestõesComentadas

9.(Cespe/MPS/Administrador/2010)Emfunçãodasconstantesmudançasnosambientesde

negócios,oplanejamentoestratégicopossuicaráterdecurtoprazo.Umclaroexemplodissoé

aconstanterevisãoqueaaltagerênciaexecutasemestralmenteemalgumasempresas.

Oplanejamentoestratégicoéfeitotendoemvistaolongoprazo!Mesmo

quandoeleérevisto,sejasemestralmenteouporcausadeumeventocrítico(o

ataquede11desetembrofoiumdeles,porexemplo),seucaráterdelongoprazo

nãomuda.

Lembre-sedequeoplanejamentoéumprocessodinâmico,ouseja,podeedeve

serajustadoquandoascondiçõesdoambientesealteram.Assim,ogabaritoé

questãoerrada.

10.(Cespe/FUB/Administrador/2009)Oplanejamentonasorganizaçõeséfeitoemnível

institucionaleemníveisgerenciaisinferiores,comcadagerenteconstruindooplanejamento

paraasuaunidade.

Ogerenteénormalmenteoresponsávelpeloplanejamentotáticodesuadivisão

ougerência.Essesplanejamentostáticosdevemdenircomosuagerênciaapoiará

aorganizaçãoemseuplanejamentoestratégicoouinstitucional.Gabarito:Certo.

11.(Cespe/Abin/Ocialtécnico/2010)Aprimeiratarefadoplanejadorédenirumplano,partemais

importantedoprocessodeplanejamento.Emseguida,eledevecoletareprocessardados

relevantesparaaimplementaçãodesseplano,apartirdenovasinformaçõesedecisões.

Essaarmativaestátodaequivocada!DeacordocomChiavenato,55o

planejamentoseiniciacomadeniçãodosobjetivos;jáOliveira56acreditaque

esteseiniciacomodiagnósticoestratégico.

Dequalquerforma,adeniçãodoplanonãoserianuncaoprimeiropasso.

Tampoucoasinformaçõesseriamcoletadasdepoisdoplanojáestabelecido,pois

necessitamosdasinformaçõesexatamenteparadeniroquedeveráserfeito,nãoé

mesmo?Ogabaritoéquestãoincorreta.

12.(Esaf/CGU/AFC/2012)EntreasdiversasEscolasdoPensamentoEstratégico,umadelaspossui

caráterabrangenteeeclético,segundooqual,paracadaperíodoousituaçãodecontexto,a

organizaçãodeveadotarumadeterminadaestruturadeformaçãodeestratégias,emfunção

daalternânciaentreestabilidadeenecessidadedetransformação.Trata-seda:

a)EscoladoDesign;

b)EscolaEmpreendedora;

c)EscolaAmbiental;

d)EscoladaConfiguração;

e)EscoladoPlanejamento.

AEscoladoPlanejamentoquedeterminaqueosgestoresdeveriam

proporcionarumacertaestabilidadenasestratégiaseaomesmotempomantera

empresacomumacapacidadedeadaptação(paraosperíodosdetransformação)é

aEscoladaConfiguração.Dessamaneira,ogabaritoémesmoaletraD.

13.(Cespe/Unipampa/Administrador/2009)Umaorganizaçãoqueatribuiaoseuprincipalexecutivo

aresponsabilidadedeelaborartodooprocessodeconcepçãodasestratégiasadotaalosoa

daEscoladoAprendizado.

AEscoladoAprendizadodizexatamenteocontrário–queoprocessode

formaçãodaestratégiaécoletivo,queexistemdiversospotenciais“estrategistas”

dentrodaorganização.

AsescolasqueatribuemaoCEO(executivomaisgraduado)aresponsabilidade

daconcepçãodaestratégiasãoaEscoladoDesigneaEscolaEmpreendedora.O

gabaritoéquestãoerrada.

14.(Cespe/Unipampa/Administrador/2009)DeacordocomospressupostosdaEscolaCultural,a

formaçãodaestratégiaconstituiumprocessocoletivo.

Exato!AEscolaCulturaldizqueoprocessodeplanejamentoécoletivoereete

ascrençasevalorescomunsdosmembrosdaorganização.Dessamaneira,a

questãoestácorreta.

15.(FGV/Sefaz-RJ/Auditor/2011)Existemtrêsestratégiasgenéricasparatornarumaempresamais

competitiva:adiferenciação,aliderançadecustoeafocalização.ADiferenciaçãoéuma

estratégiaqueprocuratornarumaorganizaçãomaiscompetitivapormeiodo

desenvolvimentodeumprodutoqueoclientepercebacomodiferentedosdemaisprodutos

oferecidospelosconcorrentes.ALiderançadeCustoéumaestratégiaquefocalizatornaruma

organizaçãomaiscompetitivapormeiodeprodutosmaisbaratosdoqueosdosconcorrentes.

AFocalização(ouenfoque)éumaestratégiaqueprocuratornarumaorganizaçãomais

competitivaporconcentrar-seemumparticulareespecíficoconsumidor.

EssaalternativadaFGVtrouxedeniçõesperfeitasdasestratégiasgenéricasde

Porter.Ogabaritoéquestãocorreta.Nessaprova,aFGVcolocouenunciados

enormesebuscou“cansar”oaluno.

Aarmativaeraapenasumadasalternativasdeumamúltiplaescolha.Emuma

provadessas,calmaetranquilidadesãofundamentaisparasefazerumaboaprova.

Nãoseapresse!Leiacomcuidadoantesdemarcar.

QuestõesPropostas

16.(Cespe/Inca/GestãoPública/2010)Constituiumexemplodemissãodaorganizaçãoadeclaração:

aempresaXatuaránoBrasilenoexteriordeformaintegradanonegóciodeenergia,comum

crescimentomédioanualde11%naproduçãodeóleoegás,durantetrêsanos.

17.(Cespe/Inca/GestãoPública/2010)Éumexemplodemetaadeclaração:tiraremmatemática

notamínima7,emtodasasprovasdoperíodosemestral.

18.(Cespe/Inca/GestãoPública/2010)Considerandoqueumaempresafarmacêutica,cujas

pesquisasparanovosmedicamentosnamédiademandemcincoanosparaseremconcluídas,

estejacomumarotatividadedepesquisadoresnaordemde75%aoano,então,pelos

conceitosdeanáliseSWOT,tem-seumaameaçaparaaempresa.

19.(Cespe/MPU/Anal.Administrativo/2010)Oprocessodeplanejamentocontemplaadeniçãodos

meiosdeacompanhamentoeavaliaçãodosresultados.

20.(Cespe/BASA/Administração/2010)Aconcepçãodeorganizaçãocomoentidadepressupõeque

elaconstituiumsistemafrouxamenteunido,oqueimpossibilitaplanejamentos,namedida

emqueplanejaréapalavraapropriadaparaseprojetarumconjuntodeaçõesparaatingirum

resultadoclaramentedenido,quandosetemplenacertezadasituaçãoemqueasações

acontecerãoecontrolequaseabsolutodosfatoresqueasseguramosucessonoalcancedos

resultados.

21.(Cespe/Basa/Administração/2010)Enquantoprojetarofuturoéumprocessoqueenvolve

decidircomoagircombasenoqueestáocorrendonoambienteimediatoenofuturopróximo,

planejaréatraduçãodessadecisãoemaçõesgerenciáveis.Aprojeçãofuturaeoplanejamento

devemencontrar-senopontoemquesejapossíveldivisarumaestratégiadeaçãoetraduzi-la

emumametaoualvo.

22.(Cespe/Aneel/Anal.adm/2010)ConsiderandoametodologiaconhecidacomoavaliaçãoSWOT

(strengths:forças;weaknesses:fraquezas;opportunities:oportunidadesethreats:ameaças),que

podeservirdebaseaosprocessosdeplanejamentoestratégico,julgueoitemaseguir.

Aidentificaçãodasfraquezasrefere-seaoambienteexternodasempresasedasorganizações.

23.(FGV/Badesc/AnalistaAdm./2010)Aestratégiadeenfoqueapresentaoriscode:

a)asmudançastecnológicasanularemosinvestimentosanteriores;

b)aempresaserincapazdeenxergarasmudançasnecessáriasnosprodutos,emvirtudedeviés

financeiro;

c)aempresaencontrarcompradoresmaissofisticadosnomercado;

d)osconcorrentesencontraremsubmercadosdentrodoalvoestratégicodaempresa;

e)osconcorrentesinovaremcombaseemimitações.

24.(FCC/ALESP/SP/Gestãoprojetos/2010)EmpresasinovadorasutilizamoBalancedScorecardcomo:

a)umsistemadeindicadoresdoprópriodesempenhofinanceiro;

b)umprocessocontínuodemediçãodeprodutos,serviçosouprocessoscomrelaçãoaosconcorrentes

maisfortesoulíderesdosetor;

c)umrecursodeTIespecializadoemplanejamentoestratégicosituacional;

d)ummétododeimplementaçãodeestratégiadereduçãodosníveishierárquicos;

e)aestruturaorganizacionalbásicadosseusprocessosgerenciais.

25.(Esaf/MPOG/EPPGG/2009)Talcomonoesforçodeplanejamentoestratégico,uma

organização

quebusqueestabelecerummodelodedesenvolvimentoinstitucionaldevepercorreras

seguintesetapas:

I.Definiçãodamissão,visãoenegócio;

II.Análisedoambienteinterno–pontosfortesefracos;

III.Análisedoambienteexterno–oportunidadeseameaças;

IV.Definiçãodosobjetivos,metaseplanosdeação.

Estãocorretas:

a)todasestãocorretas;

b)apenasIeII;

c)apenasI,IIeIV;

d)apenasI,IeII;

e)apenasIeI.

26.(Esaf/ANA/Analistaadministrativo/2009)SegundoHenryMintzberg,sãodezasescolasde

planejamentoestratégico:design,planejamento,posicionamento,empreendedora,cognitiva,

aprendizado,poder,cultural,ambientaleconguração.Relacioneascolunasnatabelaaseguir

eselecioneaopçãoquerepresentaasequênciaencontrada.

1.Design

[]Formaçãodaestratégiacomoumprocessoemergenteemqueosistemacoletivoéqueaprende.

[]Formaçãodaestratégiacomoumprocessoreativoaomeioambiente,queseapresentaparaa

2.Planejamentoorganizaçãocomoumgrupodeforçascontraasquaiselaprecisareagir.

3.

[]Formaçãodaestratégiacomoumprocessodeconcepção,emqueocontrolepermanecenasmãosdo

PosicionamentoCEO(chiefexecutiveofficer),quemantémoprocessodemaneirainformalesimples.

4.

[]Formaçãodaestratégiacomoumprocessoanalíticoparalidarcomposiçõesdemercadonogeralede

Empreendedoraformareconhecível.

[]Formaçãodaestratégiacomoumprocessodetransformaçãoqueinterrompeemperíodode

5.Cognitiva

estabilidade.

[]Formaçãodaestratégiacomoumprocessoformaldivididoemetapas,quesãoapresentadasnaforma

6.Aprendizadodelistasdeverificação.

[]FormaçãodaestratégiacomoumprocessovisionáriodefinidopeloCEOcomoumapercepçãodadireção

7.Poder

queaempresadeveseguirnolongoprazo:aíéqueresideoseupontofracodevidoàconcentraçãono

comportamentodeumúnicoindivíduo.

[]Formaçãodaestratégiacomoumprocessomentalcomoformasdeenxergarconceitos,mapas,

8.Cultural

esquemaseestruturas.

[]Formaçãodaestratégiacomoumprocessocoletivobaseadonascrençaseinterpretaçõescomunsa

9.Ambiental

todososmembrosdaorganização.

[]Formaçãodaestratégiacomoumprocessodenegociaçãopormeiodoexercíciodainuênciapara

10.

negociarestratégiasfavoráveisacertosinteresses.Seupontofracoédesconsideraraspectoscomoacultura

Configuração

daempresaeoexercíciodaliderança.

TABELA1

a)5,10,4,3,7,6,8,2,9,1

b)6,9,1,3,10,2,4,5,8,7

c)2,4,3,5,10,6,8,7,9,1

d)10,9,8,7,6,5,2,3,4,1

e)5,9,2,3,8,1,4,6,10,7

27.(Esaf/CGU/AFC/2012)Acercadosconceitosdegestãoestratégicaeplanejamentoestratégico,é

corretoafirmarque:

a)oconceitodegestãoestratégicaémaisamploqueodeplanejamentoestratégico;

b)problemasrotineiroseprevisíveisconstituemobjetoprincipaldoplanejamentoestratégico;

c)oconceitodeplanejamentoestratégicoémaisamploqueodegestãoestratégica;

d)problemasrotineiroseprevisíveisconstituemobjetoprincipaldagestãoestratégica;

e)ambososconceitosseequivalem,podendosertidoscomosinônimos.

28.(Esaf/Susep/Adm.Financeira/2010)Concebe-sequeumprocessodeadministraçãoestratégica

temcomofasesprincipaisaFormulaçãodaEstratégia,aOperacionalizaçãodaEstratégiaeo

AcompanhamentoeControle.Naúltimafase(acompanhamentoecontrole),podemosarmar

que:

a)deveidentificarosculpadosporpossíveisfalhas;

b)devefixarmetasquantitativasemensuráveis;

c)devepermitirarealimentaçãonecessáriaàcorreçãoderumos;

d)deveteraimagemdeumprocessodeauditoria,realizadodetemposemtempos;

e)devecontrolarparaqueaexecuçãosejarigorosamentedeacordocomoplanejado

29.(FCC/TRT24a/MS/Téc.Adm./2011)SobreoPlanejamentoEstratégico,analise:

I.Éomesmoqueplanejamento,mascomênfasenoaspectodelongoprazodosobjetivos.

II.Éomesmoqueplanejamento,porémcomênfasenoaspectodecurtoprazodosobjetivos.

III.Éomesmoqueplanejamento,mascomênfasenaanáliseglobaldocenário.

EstácorretooqueconstaAPENASem

a)II;

b)IeII;

c)I;

d)IeII;

e)IeI.

30.(FCC/ARCE/Analistareg./2006)Aexperiênciaacumuladanomundocontemporâneotem

demonstradoqueaecáciadagerênciadepende,emgrandeparte,dacapacidadedodirigente

dedesenvolverfuturosalternativosparaasuaorganização,estabelecendotransações

ambientaisquelevemaoalcancedamissãoorganizacional.Essaéacapacidadedepensarde

forma

a)operacional.

b)tática.

c)estratégica.

d)holística.

e)racional.

31.(Esaf/Susep/Adm.Financeira/2010)Umplanejamentoéestratégicoquandosedáênfaseao

aspecto:

a)delongoprazodosobjetivoseàanáliseglobaldocenário;

b)deprazoemergencialdosobjetivoseàanáliseglobaldocenário;

c)delongoprazodosobjetivoseàanálisedasituaçãopassada;

d)demédioprazodosobjetivoseàanálisedasituaçãoatual;

e)deurgênciadosobjetivoseàanálisedasituaçãofutura.

32.(FCC/Alesp/SP/Gestãoprojetos/2010)Comreferênciaaonívelfuncional,oplanejamento

estratégicotemcomoobjetivo:

a)determinaramissãodaempresa,emtermosdesegmentodemercado;

b)definirasunidadesdenegóciosgeridascomocentrosdelucro;

c)alocarosrecursossegundoalucratividadedasunidadesdenegócio;

d)alinharasaçõessetoriaiscomasestratégiasdenegócioseamissãodaorganização;

e)influirnatomadadedecisõesdelongoprazoqueaempresadevatomar.

33.(FCC/TRT/PR/Analistaadm./2010)Aanáliseestratégicadaorganizaçãoenvolve:

I.aanálisedoambienteexterno,queampliaasensibilidadedoconjuntodepessoas,tornando-

asaptasaimplementarestratégiasantecipatóriasalinhadascomasprincipaistendênciase

demandas;

II.aanálisedoambienteexterno,quepossibilitaaalavancagemdeoportunidades,pontos

fortesefracoseaprevençãocontraasameaçasemergentes;

III.aanálisedoambienteinterno,baseadonaavaliaçãododesempenhodaorganização

perantesuamissãoeobjetivos,forçasefraquezas,políticasgovernamentaise

concorrência;

IV.aanálisedascaracterísticasinternasdaorganização,identicandoehierarquizandoosseus

pontosfortesefracos;

V.aanálisedoambienteinternosegundoumaavaliaçãodecaráterorganizacional,baseando-

senosconceitosdedesempenhodainstituiçãoemrelaçãoaocumprimentodamissão,

efetividade,eficácia,eficiênciaehumanização.

ÉcorretooqueconstaAPENASem:

a)IeV;

b)I,IeII;

c)II,IVeV;

d)IIeIV;

e)I,IVeV.

34.(Esaf/MPOG/Economista/2006)Escolhaaopçãoquecompletacorretamentealacunadafrasea

seguir.

“................refere-seàmaneirapelaqualumaorganizaçãopretendeaplicarumadeterminada

estratégia,geralmenteglobaledelongoprazo,criandoumconsensoemtornodeuma

determinadavisãodefuturo.”

a)FlexibilizaçãoOrganizacional

b)ProgramadeQualidadeTotal

c)Benchmarking

d)PlanejamentoEstratégico

e)AprendizagemOrganizacional

35.(Esaf/ANA/AnalistaAdministrativo/2009)Consideradoumaferramentademudança

organizacional,oplanejamentoestratégicopodesercaracterizadopelasseguintes

afirmações,exceto:

a)estárelacionadocomaadaptaçãodaorganizaçãoaumambientemutável,sujeitoàincertezaa

respeitodoseventosambientais;

b)éorientadoparaofuturo.Seuhorizontedetemposãoocurtoemédioprazos;

c)écompreensivo,envolveaorganizaçãocomoumtodo,nosentidodeobterefeitossinergísticosde

todasascapacidadesepotencialidadesdaorganização;

d)éumprocessodeconstruçãodeconsenso,poisofereceummeiodeatenderatodosnadireção

futuraquemelhorconvenhaàorganização;

e)éumaformadeaprendizagemorganizacional,poisconstituiumatentativaconstantedeaprendera

ajustar-seaumambientecomplexo,competitivoesuscetívelamudanças.

Gabaritos

1.E

13.E

25.A

2.C

14.C

26.B

3.C

15.C

27.A

4.E

16.E

28.C

5.C

17.C

29.D

6.C

18.E

30.C

7.E

19.C

31.A

8.A

20.E

32.D

9.E

21.C

33.E

10.C

22.E

34.D

11.E

23.D

35.B

12.D

24.E

Bibliografia

Chiavenato,Idalberto.Administraçãonosnovostempos.2aed.RiodeJaneiro:

Elsevier,2010.

Decourt,Felipe,HamiltondaRochaNeves,ePauloRobertoBaldner.

PlanejamentoeGestãoEstratégica.RiodeJaneiro:FGV,2012.

Hamel,Gary,eC.K.Prahalad.“StrategicIntent.”HarvardBusinessReview,May-

Jun1989.

Herrero,Emílio.BalancedScorecardeagestãoestratégica:umaabordagemprática.

RiodeJaneiro:Elsevier,2005.

Kaplan,RobertS.,eDavidP.Norton.eBalancedScorecard:translatingstrategy

intoaction.1aed.Boston:HarvardBusinessSchoolPress,1996.

Lobato,DavidMenezes,JamilMoisésFilho,MariaCândidaSoterlinoTorres,e

MuriloRamosAlambertRodrigues.Estratégiadeempresas.RiodeJaneiro:FGV,

2009.

Oliveira,DjalmadePinhoRebouçasde.PlanejamentoEstratégico–conceitos,

metodologiasepráticas.24aed.SãoPaulo:Atlas,2007.

Paludo,AugustinhoVicente.Administraçãopública:teoriaequestões.1aed.Riode

Janeiro:Elsevier,2010.

Porter,MichaelE.EstratégiaCompetitiva:técnicasparaanálisedeindústriaseda

concorrência.7aed.RiodeJaneiro:Campus,1986.

Porter,MichaelE.“HowCompetitiveForcesShapeStrategy.”HarvardBusiness

Review,Mar-Apr1979.

Porter,MichaelE.“WhatisStrategy?”HarvardBusinessReview,Nov-Dec1996.

Sobral,Felipe,eAlketaPeci.Administração:teoriaepráticanocontextobrasileiro.

SãoPaulo:Pearson-Prentice-Hall,2008.

Tauhata,TatianaL.,eT.DianaL.v.A.deMacedoSoares.“RedeseAlianças

EstratégicasnoBrasil:casoCVRD.”RaeEletrônicaV.3,n.1(Jan-Jun2004).

Capítulo4

EstruturaOrganizacional

Organizaréoprocessodealocarpessoaseoutrosrecursosparaatingirum

objetivo.Envolvetantoadivisãodetrabalhocomoacoordenaçãodesses

trabalhosparaalcançarumresultadodesejado.

Oprocessodeorganizaçãodenequemdevefazeroquê,queméresponsável

porquemecomoasdiferentespartesdaorganizaçãodevemserelacionare

trabalharjuntas.

Assim,poderíamosdizerqueaestruturaorganizacionaléoconjunto

ordenadoderesponsabilidades,autoridades,comunicaçõesedecisõesdeuma

empresaouorganização.

Figura4.1–Definiçãodeorganização

Vamosvercomoessetópicojáfoicobradoemconcursos?

1.(FCC/Arce/AnalistaReg./2006)Asomatotaldasmaneiraspelasquaisotrabalhoédivididoem

tarefasdistintase,posteriormente,comoacoordenaçãoérealizadaentreessastarefasé

denominada:

a)estruturadeumaorganização;

b)planejamentoestratégico;

c)missãodaorganização;

d)reengenhariadaorganização;

e)redesenhodosprocessos.

Comovimos,oprocessoadministrativoqueserelacionacomadivisãode

trabalhoeacoordenaçãodessetrabalhoéaorganização.Aestrutura

organizacionaléoresultadodasdecisõesdoprocessodeorganização.Dessa

forma,aletraAestácorretaeénossogabarito.

AsletrasBeCserelacionamcomoplanejamento.Assimsendo,estão

incorretas.Domesmomodo,asletrasDeEestãoerradas,poisserelacionamcom

agestãodeprocessos.

Continuando,oselementosprincipaisdaestruturaorganizacionaldeque

trataremosnestecapítuloestãodescritosnafiguraaseguir.

Figura4.2–Elementosdaorganização

4.1.EstruturasFormaiseInformais

Aestruturaformaléaestruturaocialdeumainstituição.Assimsendo,éa

estruturarepresentadanoorganograma,emqueexisteaidenticaçãodos

diversoscargosedaslinhasdeautoridadeecomunicaçãoentreeles.Deacordo

comSchermerhorn,1lendoumorganograma,vocêpodeentenderváriosaspectos

deumaorganização,como:

ØDivisãodetrabalho–cargosetítulosmostramquemfazoquedentroda

empresa.

ØRelaçãodesupervisão–linhasmostramquemsereportaaquem.

ØCanaisdecomunicação–linhasmostramoscanaisdecomunicação.

ØSubdivisõesprincipais–divisõesquesereportamaummesmogerentesão

mostradas.

ØNíveishierárquicos–osdiversosníveishierárquicossãoevidenciados.

Aestruturainformalestápresenteemtodasasorganizações.Éumaredede

relacionamentosnãoociais,masmuitoimportantes,quemoldamos

relacionamentosentreosocupantesdaorganização.

Portanto,seaestruturainformalfossedesenhada,mostrariaquemfalacom

quem,quemtemamizadeouinimizadecomquemetc.Assimsendo,indicaria

entãotodososrelacionamentosespontâneosdentrodaorganização,não

dependentesdocargooficial.

Atenção:nãopodemoseliminaraestruturainformal,nemissoseriadesejável.

Éimpossívelcompreendertotalmenteumaorganizaçãosemconhecersua

estruturainformal.Elapodesermuitobenécaparaqueumaorganizaçãopossa

atingirseusobjetivos,principalmenteemépocasdemudançasrápidas.

Comoédemoradoecomplicadooprocessodemudançadaestruturaformal,a

estruturainformalpossibilitaàempresaseadaptarmaisfacilmenteaosnovos

desaos.Outravantagemépossibilitaraosfuncionáriosconseguiralguma

informaçãoqueseriadifícilpeloscanaisformais,alémdepossibilitaro

aprendizadodemodoinformal,queocorrequandoaprendemosalgumatarefa

comumcolegadetrabalho,porexemplo.

Entretanto,aestruturainformaltambémtrazdesvantagens,comoboatos

infundados,fofocas,desentendimentospessoaisqueatrapalhamoandamentodo

trabalho,aperdadeenergiaemconflitosetc.

4.2.EspecializaçãodoTrabalho

Adivisãodetrabalhoéumaspectocentraldecadaorganização.Comadivisão

detrabalho,umfuncionáriopodeseespecializaremumadeterminadatarefa,

aumentandosuaeficiência.

DeacordocomSobral,2aespecializaçãodotrabalhoéograuemqueastarefassão

divididasepadronizadasparaquepossamseraprendidaserealizadasdeforma

relativamenterápidaporumúnicoindivíduoemdadaorganização.

Aespecialização,apesardeseusefeitosbenécos,podetambémsernegativa.

Quandoaespecializaçãoéexcessiva,podelevaraumainsatisfaçãodofuncionário

eàperdadeeciênciaedeprodutividade(otrabalhocamuitochatoe

repetitivo).

Figura4.3–Especialização

Dessaforma,quandoaumentamosaespecialização,tendemosaaumentara

nossaprodutividade.Entretanto,essamelhoriaocorreatécertoponto.Após

determinadolimite,aespecializaçãoémaléca,poisotrabalhotorna-se

extremamentecansativoepoucodesafiador.

4.3.EnriquecimentodoTrabalho

Fazersempreamesmacoisaémuitochato,nãoémesmo?Aespecializaçãopode

criarumagrandedesmotivaçãonostrabalhadores,poisempoucotempoestesse

veemsemdesafios.

Muitasorganizaçõessabemdissoebuscamaumentaramotivaçãodos

funcionárioscomoenriquecimentodoscargos.Issopodeserfeitodediversas

maneiras:comumrodíziodetarefasedecargos,comtreinamentosecomo

aumentoderesponsabilidadesedesafios,entreoutrasformas.

Dessamaneira,apessoapodeestarcontinuamenteaprendendoese

desenvolvendo.Aomesmotempo,aorganização“ganha”pessoasmaismotivadase

versáteis(poispassamacontarcomdiversashabilidadesnovas).

Seumtrabalhadorcadoente,porexemplo,outroprossionaléfacilmente

deslocadoparasuafunção,poisjátevecontatocomotrabalhodesenvolvido.

Esseenriquecimentopodeacontecerdeduasformas:oenriquecimentolaterale

oenriquecimentovertical.

Oenriquecimentolateralacontecequandoaspessoaspassamaexecutartarefas

comamesmadiculdadeeresponsabilidadedasanteriores.Ouseja,apessoa

continuanomesmonívelhierárquico,maspassaaexecutarumatarefadiferente.

Jánoenriquecimentoverticalapessoarecebetarefasmaisdifíceisecomum

nívelderesponsabilidademaior.Assim,deixadeexecutartarefasmaissimples

paraexecutartarefasmaiscondizentescomseunovoníveldematuridade.

4.4.CadeiadeComando

Adivisãodetrabalhotambémtrazanecessidadedehaverumadivisãodo

comandodentrodaorganização.Essadivisãotemcomoobjetivodirigiraspessoas

paraqueosobjetivossejamatingidos.

Issogeraoquesechamadeestruturahierárquicadentrodainstituição,

separandooscargosdeacordocomaautoridadedecadaum.Essahierarquiaé

chamadadecadeiadecomando.3

Acadeiadecomandoseriaentãoumalinhaquepassadotopodahierarquiaaté

ocargomaisbaixo,mostrandoquemmandaemquemdentrodaorganizaçãoe

quemdeveresponderaquem.

Figura4.4–Cadeiadecomando

Portanto,asdecisõesreferentesàcadeiadecomandodemonstramcomoa

autoridadeédivididaentreoscargoseasdivisõesnaorganização.Nagura

anterior,vemosasequênciadeumacadeiadecomando:ogerentedecobrança

reporta-seaogerentefinanceiro,quesereportaaopresidentedaempresa.

Oconceitodecadeiadecomandoestárelacionadoaoutroconceito:oda

unidadedecomando,oqualdizquecadafuncionáriosereportaaapenasum

superiorhierárquico.

Entretanto,atualmenteesseconceitoclássiconaadministraçãoestásendocada

vezmenosutilizado.Essaquestãoserámaisbemabordadaadiante,quando

estudarmosasestruturasmaismodernas.

Figura4.5–Característicasdacadeiadecomando

4.5.AmplitudedeControle

Aamplitudedecontrolerelaciona-seaonúmerodesubordinadosqueos

superiorescomandamemumaorganização.4Dessaforma,quantomaioro

númerodesubordinadosporchefe,maioréaamplitudedecontrole.Assim

sendo,seexistempoucossubordinadosporchefe,aamplitudedecontroleé

menor.

Figura4.6–Estruturaachatada

Essadivisãodeautoridadeeresponsabilidadevaidenirquantosníveis

hierárquicosequantosadministradoresdeveremosteremumadadaorganização.

Sedecidirmosterpoucossubordinadosporchefe,teremosumaestrutura

aguda.

Entretanto,hojeemdia,asempresasestãobuscandoestruturasachatadas,com

muitossubordinadosporcadachefeeumaamplitudedecontrolemaior.

Figura4.7–Estruturaaguda

Asestruturasachatadassãovistascomofacilitadorasdeumprocessode

delegação(delegaréconfiarresponsabilidadeeautoridadeaumsubordinadopara

queexecutealgumatarefaepossaatingirumobjetivo),aumentandoaagilidade

noprocessodecisórioeaadaptabilidadeàsmudançasnomeioexterno.5

Alémdisso,essasestruturasacarretamumcustoadministrativomenor.

Entretanto,ochefenãoconseguedaromesmosuporteaosseussubordinados,

poistemmuitosfuncionáriosparasupervisionar.

Figura4.8–Característicasdaamplitudedecontrole

4.6.CentralizaçãoeDescentralização

Acentralizaçãoocorrequando,emumaorganização,amaioriadasdecisõesé

tomadapelosocupantesdoscargosnotopodamesma,ouseja,peloschefes.Dessa

maneira,seemumaempresasomenteopresidentepudertomardecisões,dizemos

queexistecentralização.

Jáadescentralizaçãoocorrequandoaconteceocontrário,ouseja,quandoa

autoridadeparatomardecisõesestádispersapelaempresa,atravésdosseus

diversossetores.

Asvantagensdacentralização,deacordocomChiavenato,6são:

ØControle–facilitaocontroledeproblemascomplexosouquepodemter

consequênciasparatodaaorganização.

ØCustos–devidoaumaescalamaior,algumasdecisõessãomaisecientes

quandosãocentralizadasemumsetor.Issopodeocorreremcentraisde

compras,porexemplo,quepodemconseguirpreçosmelhoresdoque

diversossetorescomprandoomesmoprodutoseparadamente.

ØNovastecnologias–comasnovastecnologiasdeTIacúpuladaempresa

podeterinformaçõescadavezmaisrapidamente,podendoreterumcontrole

maiorsemperderaeficiência,arapidezeaflexibilidade.

Asprincipaisvantagensdadescentralizaçãosão:

ØAgilidade–Seadecisãoestáacargodapessoaenvolvidadiretamenteno

problema,elarespondecommaisrapidez;

ØIndependência–adescentralizaçãogeraumamaiorindependêncianos

funcionáriosquetrabalhamnosescalõesmaisbaixos,permitindotambém

queacriatividadedelespossaaflorar.

ØNovastecnologias–damesmaformaquepermitemumacentralizaçãomais

eciente,asnovastecnologiasdeTIpossibilitamumadescentralizaçãodas

decisõesparaascamadasmaisbaixasnahierarquiasemqueacúpulapercao

controledosresultadosdessasdecisões.

Figura4.9–Centralizaçãoedescentralização

Vamosvercomoissopodesercobrado?

2.(Cespe/Basa/Administração/2010)Aoadotaradescentralização,ogerentedeumaagência

bancáriatemcomovantagemextinguiroriscodesuperposiçãodeesforçospararealizaçãode

umatarefa.

Adescentralizaçãoocorrequandoopoderdedecidirestádistribuídoportoda

aorganização.Dessaforma,aspessoasenvolvidasdiretamente(na“linhade

frente”comosecostumadizer)podemtomarasdecisõesnecessáriaspararesolver

umproblemaoumelhoratenderaocliente.

Portanto,acentralizaçãoseriaentãoumasituaçãoemqueaautoridadeestá

localizadanacúpulaoutopodahierarquia,ouseja,centralizadanachea.

Quandodizemosquenossochefeé“centralizador”queremosexatamentedizerque

elequerdecidirtudo,nãoémesmo?

Adescentralizaçãotrazváriasvantagens,comoarapideznatomadadedecisões,

aflexibilidadeeapossibilidadeparaquemestálidandodiretamentecomaquestão

(etemmaisinformações)detomarasdecisõesparasolucionarseuproblema.

Apesardisso,adescentralização,pordistribuirasdecisõespordiversaspessoas

esetores,acarretasituaçõesemquepessoasdecidirãodeformadiferentelidando

comproblemassemelhantes(cadapessoatemumapersonalidadeeumamaneira

depensardiferente,nãoémesmo?).Etambémpodeacontecerdemaisdeuma

pessoabuscarsolucionaromesmoproblemaaomesmotempo(oquechamamos

desuperposiçãodeesforços).

Assim,adescentralizaçãoaumenta,sim,aschancesdeocorrerasuperposiçãode

esforços,aocontráriodoquedisseaquestão.Ogabaritoéquestãoincorreta.

4.7.PosiçõesdeLinhaeStaff

Outraclassicaçãodoscargosouatividadesdentrodeumaorganização

relaciona-secomaautoridade.Sãoasposiçõesdelinhaouasposiçõesdestaff(ou

assessoria).

Umamaneiradeidenticarediferenciaressasposiçõesrefere-seaopoderde

comando.Asposiçõesdelinhateriamocomandosobreseussubordinados,eas

posiçõesdestaffapenaspoderiamsugerir,aconselharourecomendaraos

funcionários.7

Outraformaqueexisteparaessadiferenciaçãoéconsiderarasfunçõesdelinha

ligadasàsatividades-mdaorganizaçãoeasfunçõesdestaffligadasàssuas

atividades-meio.

Naguraaseguir,odiretordelogísticaestariaemumaposiçãodelinha,pois

coordenaotrabalhodelogísticadosprodutos,enquantooconsultorjurídico

estariaemumaposiçãodestaff,tendoautoridadedestaff,oufuncional,de

advertir,aconselharourecomendaraosfuncionáriosquandootemafordesua

áreadeatuação.

Figura4.10–Posiçõesdelinhaestaff.

4.8.Departamentalização

Adepartamentalizaçãoéaformacomoumaorganizaçãopodejuntareintegrar

pessoas,tarefaseatividadesemunidades,tendocomoobjetivofacilitara

coordenaçãodaempresa.8

Odepartamentoé,portanto,umaunidadedetrabalhoqueagregaum

somatóriodeatividadessemelhantesoucoerentesentresi.Deacordocom

Chiavenato,9adepartamentalizaçãoéummeiodeseobterhomogeneidadedetarefas

emcadaórgão.

Adepartamentalizaçãodevefacilitarotrabalhodosadministradores,pois

possibilitaumaproveitamentomaisracionaldosrecursosdaorganização.

Assimsendo,oscritériosdevemserosmaisracionais,visandoatingirosobjetivos

daorganização.

Osprincipaistiposdedepartamentalizaçãosão:

4.9.DepartamentalizaçãoFuncional

Éocritériomaisusualnaadministração.Osdepartamentossãoagrupadosde

acordocomafunçãoorganizacional,ouseja,sãoformadosdepartamentosde

Marketing,deFinanças,deRecursosHumanosetc.

Nessetipodedepartamentalização,oagrupamentoéfeitotendoemvistaa

semelhançadastarefas,habilidadeserecursosparaaexecuçãodecadafunção

específica.

Figura4.11–Departamentalizaçãofuncional

Suasprincipaisvantagenssão:juntatodasaspessoasqueexecutamasmesmas

tarefasemumsódepartamento,facilitandoautilizaçãodascapacidadestécnicas

dasmesmas;facilitaotreinamentoeéindicadaparaorganizaçõesemsetoresmais

estáveisequenãomudemmuitoseusprodutoseserviços.

Jáasdesvantagensserelacionamcomumafaltadecoordenaçãoentreos

departamentos,adiculdadedeseadaptaràsmudançasexternaseumavisãomais

“estreita”dosmembrosdosdepartamentos,poissóanalisamseuspróprios

problemas.

4.9.1.DepartamentalizaçãoporProduto

Nessetipodedepartamentalização,asdivisõessãoagrupadasportipode

produtoouserviço.Dessaforma,umagrandeempresapoderiaserdivididaem:

setoresdealimentosinfantis,debebidas,delimpezadomésticaetc.

Essemodeloémaisadequadoquandoastecnologiasenvolvidasnafabricação,

comercializaçãoouatendimentoaoclientedecadaprodutoouserviçosãomuito

diferentes.

UmcasopráticoéodaempresaGE(GeneralElectric),quefabricadesde

turbinasdeaviãoatélâmpadaseequipamentosmédicos.Comocadaprodutoé

muitodiferente,esuascadeiasdesuprimentoededistribuiçãosãodiversas,faz

maissentidodividirosdepartamentosportipodeproduto.10

Assim,cadaadministradorestámaiscapacitadoafocarnosaspectos

importantesdecadasetor,facilitando,portanto,oprocessodegestãoede

inovaçãonaslinhasdeprodutoseserviços.Dessamaneira,camaisfácilatender

osclientesdeumamaneiramaisadequada.

Figura4.12–Departamentalizaçãoporprodutos

Essadepartamentalizaçãoaumentaacoordenaçãoentreosdiversos

departamentosdecadadivisão.Alémdisso,amaiorcapacidadedeinovaçãoede

flexibilidadeéoutravantagemdessemodelo.11

Entretanto,nemtudo“sãoores”.Nessetipodedepartamentalização,os

“especialistas”camespalhadospelaorganização,deixando,muitasvezes,o

treinamentoeodesenvolvimentotécnicoparasegundoplano.

4.9.2.DepartamentalizaçãoporCliente

Essetipodedepartamentalizaçãoocorrequandoaempresatemclientescom

necessidadesmuitodiferentes,oquejusticariaumatendimentopersonalizadoa

cadatipodeclientela.Emumbanco,podehaverumagerênciaparaclientes

corporativoseumagerênciaparaclientespessoafísica,porexemplo.

Dessaforma,oadministradoralocarárecursos,pessoaseatividadesmais

condizentescomasnecessidadesedemandasdecadaclientela.12Issoéimportante

emalgunssetoresemqueasdemandassejammuitodiferentesentreasdiversas

“clientelas”.

Assim,nessetipodedepartamentalização,oclienteéofocoprincipal,nãoas

necessidadesinternasdasorganizações.

Figura4.13–Departamentalizaçãoporcliente

4.9.3.DepartamentalizaçãoTerritorialouGeográfica

Adepartamentalizaçãogeográcaéinteressantequandoaempresaestá

distribuídaemumterritóriomuitogrande.Nessecaso,éimportanteaempresa

descentralizarrecursosespecíficosparacadaregião.

Outroaspectoquepodeapontaranecessidadedeumtipodessesde

departamentalizaçãoéquandoexistemumadiferençaeespecicidadegrandede

acessoamercadosconsumidoreseapessoasqualicadas,bemcomoamatérias-

primas.

Nessescasos,osadministradorespodemcarespecializadosnasdemandase

especicidadesdecadaregião.Comoexemplo,umaempresapoderiatergerências

nasregiõesSul,Sudeste,Nordesteetc.

Assim,quandoaempresaestiver“cobrindo”áreasmuitograndesediferentes,a

departamentalizaçãopodeserindicada.Cadasetorpoderá,atravésdamelhor

compreensãodecadaregião,atingirmaisfacilmenteosobjetivosdaorganização.

Figura4.14–Departamentalizaçãogeográfica

4.9.4.DepartamentalizaçãoporProcesso

Nessetipo,asatividadeseosrecursossãoagrupadosemtornode

processoschaveespecícosdaempresa.Emumamontadoradeautomóveis,por

exemplo,poderíamosterossetoresdepinturaemontagem.

Dessamaneira,dividimosotrabalhodeacordocomasatividadesprincipaisque

ocorremdentrodeumaorganização.Assimsendo,osprossionaisencarregados

decadaprocessoestariamlocalizadosnossetoresresponsáveisporeles.

Essetipodedepartamentalizaçãoémaisutilizadonosníveismaisbaixosda

hierarquia,relacionadoscomasáreasdeproduçãoelogística.Assim,atecnologia

passaaserofocodessemodelo.13

Omaiorproblemadessetipodedepartamentalizaçãoéapoucaexibilidadeem

umcontextodemudançatecnológicagrandeounecessidadedemudançados

processos.

Figura4.15–Departamentalizaçãoporprocessos

Vamosvercomoessetópicopodesercobrado?

3.(Cespe/Abin/Ocialtécnico/2010)Umdepartamento,emumaorganização,éumagregadode

cargos,nãopodendo,emnenhumacircunstância,corresponderaumúnicocargo.

Aquestãoestáerradaporqueumdepartamentopode,sim,emdeterminadas

circunstâncias,sercompostoporapenasumcargo.Issoérelativamentecomum

emempresaspequenas,comumapessoaapenas(umcargo)tomandocontadeum

departamento.

Umaempresaquetenhaapenasumvendedorestánessasituação,nãoéverdade?

Ogabaritoéquestãoincorreta.

4.10.DesenhoEstruturaldasOrganizações

Odesenhoestruturaléoresultadodeváriasdecisõesarespeitodosaspectosde

autoridade,dacomunicaçãointernaedadepartamentalizaçãonaorganização.14

Portanto,umgestordevelevaremconsideraçãoosprincipaiselementosdo

processoorganizacional,jáanalisados.Dessaforma,asestruturasorganizacionais

identicamosgruposdepessoasdeacordocomaposiçãodecadaumnaempresa,

ascadeiasdecomandoecomunicaçãoeasligaçõesverticaisentreacheaeo

níveloperacional,deformaacoordenarostrabalhos.DeacordocomSobrale

Alketa,15

Alémdosdepartamentos,nasestruturasorganizacionaissãodenidososcanaisporondeuema

autoridadeeacomunicação.

Existemváriostiposdeestruturaorganizacional,comoaestruturafuncional,a

divisional,amatricialeaestruturaemrede.Aseguirveremoscadaumaem

detalhes.

4.11.EstruturaLinear

Naestruturalinear,aautoridadeestácentralizadaemapenasumcargo.Dessa

forma,osuperiortemautoridadeúnicasobreseusfuncionáriosenãoareparte.

Essasestruturasadaptam-semelhoraambientesmaisestáveis.

Vamosvercomoissojáfoicobradoemconcursos?

4.(FCC/TRT24a/MS/Anal.adm./2011)Tipodeestruturaorganizacionalemquecadasuperiortem

autoridadeúnicaeabsolutasobreseussubordinadosequenãoarepartecomninguém:

a)gerencial;

b)funcional;

c)staff;

d)linear;

e)operacional.

Essaquestãotambémestáquasedegraça.Entreasopções,teríamosdeescolher

apenasentreaestruturafuncionalealinear,poisasoutrasnãosãotiposde

estruturaorganizacional.

Dessaforma,comovimos,otipodeestruturaemqueochefetemautoridade

únicasobreseuscomandadoséaestruturalinear.OgabaritoéaletraD.

4.12.EstruturaFuncional

Aestruturafuncionaléamaiscomumnaadministração.Quasetodasas

organizaçõesiniciamsuavidatendoumaestruturafuncional.Étambéma

estruturamaisintuitiva,poisédivididadeacordocomastarefasefunções

organizacionais,comofinanças,marketing,recursoshumanosetc.16

Essaestruturaémaisadequadaaempresasmenores,organizaçõesemambientes

estáveis,quetenhampoucosprodutose/ouserviçosouqueoperememumaregião

pequena.Umexemplodeestruturafuncionalpodeservistonoexemplodagura

aseguir.

Figura4.16–Estruturafuncional.

(Fonte:SobralePeci,2008)

Entreasprincipaisvantagensedesvantagensdessaestruturaestão:

Vantagens

Desvantagens

Estimulaumavisãolimitadadosobjetivos

Facilitaoaperfeiçoamentodosfuncionáriosnas

organizacionais

suasespecialidades

Dificultaacoordenaçãoeacomunicaçãoentreos

Facilitaacomunicaçãodentrodasáreasfuncionais

departamentos

Permiteeconomiasdeescalaeoaproveitamento

Diminuiarapidezderespostaaosdesafios

dosrecursos

externos

Facilitaacentralizaçãodasdecisões

Dificultaaresponsabilizaçãopelosproblemas

organizacionais

Figura4.17–Vantagensedesvantagensdaestruturafuncional.

(Baseadoem:SobralePeci,2008)

4.13.EstruturaDivisional

Quandoasempresascammaiores,comprodutoseserviçosdiversicadose

presençaemváriosmercados,setorese/oupaísesdiferentes,acabammigrando

paraumaestruturadivisional.

Nessaestrutura,aorganizaçãoagregaastarefaserecursosemdivisões,de

acordocomosprodutos,clientese/oumercadosimportantesparaela.

Dessaforma,cadadivisãocaentãoquaseautônoma,tendoseuprópriosetor

demarketing,deproduçãoelogística,deformaapodertomarquasetodasas

decisõesrelativasaoseuprodutoemercadoeatenderseusrespectivosclientes.17

Portanto,alógicadessetipodeestruturaéumadescentralizaçãodeautonomia

operacionalàsdivisões,mascomumplanejamento,coordenaçãoecontroleainda

centralizadosnacúpula.

Umexemplodeestruturadivisionalpodeservistonafiguraaseguir:

Figura4.18–Estruturadivisional

Portanto,cadaadministradordedivisãoécobradopelosresultadose

desempenhodecadalinhadeprodutosquerepresentaoumercadoqueatende.

Entretanto,asatividadesdeapoio(comonançaserecursoshumanos)continuam

operandonamatriz.18

Asprincipaisvantagensedesvantagensdessetipodeestruturasãomostradasa

seguir:

Vantagens

Desvantagens

Podegerarumamaiorpreocupaçãocom

Permiteumamelhordistribuiçãodosriscos,

osresultadosdadivisãodoquedaempresacomo

pelarelativadescentralizaçãoqueproporciona

umtodo

Geraumaumentonavelocidadederesposta

Criaumaredundânciaderecursos,poiscadadivisão

aosdesafiosexternos

criasuasestruturas

Facilitaaavaliaçãododesempenhodecada

Podegerarumadisputaentreasdivisõespor

unidade

recursosdaempresa

Possibilitamaiorproximidadecomoclientee

Geraummenordesenvolvimentotécnico,poisos

suasdemandas

departamentosfuncionaisdasdivisõessãomenores

Figura4.19–Vantagensedesvantagensdaestruturadivisional.

(Baseadoem:SobralePeci,2008)

4.14.EstruturaMatricial

Aestruturamatricialéummodelomisto,quecomportaaomesmotempouma

estruturafuncionalcomumaestruturahorizontal,quenormalmentesereferea

umprojeto,umadivisãoespecíficaouumproduto.

Nessemodelo,tenta-sejuntarasvantagensdasduasestruturas(funcionalepor

projeto)deformaajuntarosespecialistasemcadaáreanosprojetosouprodutos

quesejamnecessáriosnomomento.

Alógicadessemodeloéoquesechamaautoridadedual,ouseja,ofuncionário

respondeadoischefesaomesmotempo!Lembrem-sesempredestainformação,

queasbancasadoramcobrar:nãoháunidadedecomandonaestrutura

matricial!

Vamosimaginarquevocêtrabalheemumaempresa,comooexemploaseguir.

VocêpodeestarsubordinadoaodiretordalialdoBrasil,porexemplo,mas

tambémresponderaogerentedoproduto“seguros”daempresa.Estepodeestar

lançandoumprodutonomercadoenecessitardeumaequipemultidisciplinar,

compostadefuncionáriosdeváriasfiliaisaoredordomundo,porexemplo.

Figura4.20–Estruturamatricial

Essaestruturaéchamadadematricial,poisseuaspectoésemelhanteauma

matriz,comovocêspodemver.Dessaforma,aestruturamatricialfoiuma

tentativadeconciliarumaestruturarígidaehierárquicaaumamaior

flexibilidade,cadavezmaisnecessárianestestemposdemuitasmudançasno

contextodasorganizações.

Portanto,aestruturamatricialéutilizadanormalmenteparaatenderaprojetos

ounegóciosquedemandemequipescompessoasquetenhamváriospers

diferentes,funcionandotemporariamentenocasodosprojetosouquepossam

trabalharemváriasáreasdaempresaaomesmotempo.

Comovocêspodemimaginar,responderadoischefesaomesmotemponãoé

fácil!Damesmaforma,controlaralguémquenãorespondesóavocêtambémtraz

seusinconvenientes(comosaberseofuncionárioXestárealmentefazendoum

trabalhoparao“outrochefe”?).Assim,essacoordenaçãopoderealmentegerar

conflitosinternoseconfusão.

Entretanto,amaiormobilidadedosfuncionáriospodegerarumamaiortrocae

interaçãoentrepessoasdediferentesáreasdaempresa.Outrofatorimportanteé

aproveitarmelhorosrecursos(principalmentedepessoalqualicado)na

organização,evitandoquepessoasquempoucoaproveitadasemumsetor

quandohámuitotrabalhoemoutraáreaouprojeto.

Entreasprincipaisvantagensedesvantagensdaestruturamatricial,Sobral

aponta:

Vantagens

Desvantagens

Podegerarumamaiorpreocupaçãocom

Permiteumamelhordistribuiçãodosriscos,

osresultadosdadivisãodoquedaempresacomo

pelarelativadescentralizaçãoqueproporciona.

umtodo.

Geraumaumentonavelocidadederesposta

Criaumaredundânciaderecursos,poiscadadivisão

aosdesafiosexternos.

criasuasestruturas.

Facilitaaavaliaçãododesempenhodecada

Podegerarumadisputaentreasdivisõespor

unidade.

recursosdaempresa.

Possibilitamaiorproximidadecomoclientee

Geraummenordesenvolvimentotécnico,poisos

suasdemandas.

departamentosfuncionaisdasdivisõessãomenores.

Figura4.21–Vantagensedesvantagensdaestruturamatricial.

(Baseadoem:SobralePeci2008)

Vamosvercomoessetópicojáfoicobrado?

5.(Cespe/MPS/Administrador/2010)Aestruturamatricialdeumaorganização,soluçãopara

coordenarrecursosmultidisciplinaresdesignadosparadeterminadoprojeto,comprometeos

princípiosbásicosdaunidadedecomandoedaequivalênciaentreresponsabilidadee

autoridade.

Aestruturamatricialcaracteriza-seporconteroquechamamosdeautoridade

dual19(ouseja,vocêrespondeadoischefes,umfuncionaleoutrodoprojeto).

Dessaforma,nãoexisteunidadedecomando!

Essasituaçãopodeacarretaràsvezesalgunsproblemas.Imaginequevocêse

reportaaduaspessoaseaprimeirapedequevocêváaumareuniãonazonasulda

cidadeeooutrochefeordenaquevocêváaumareuniãocomumclientenazona

nortedacidade–nomesmohorário.Ficoucomplicadasuasituação,nãoé

mesmo?

Apesardessesproblemas,aestruturamatricialgeraumamaiorexibilidadena

organizaçãodoqueépossívelcomumaestruturatradicional.Ogabaritoéquestão

correta.

4.15.EstruturaemRede

Umaestruturaemredeatrairecursos,emqualquerlugardomundo,para

alcançarumaqualidadeeumpreçoquepossibilitemàempresavenderseus

produtoseserviçosemseumercado.Amaiorvantagemdessetipodeorganização

ésuaflexibilidadeecompetitividadeemescalaglobal.

Portanto,aexibilidadeadvémdapossibilidadedeaorganizaçãocontratar

qualquerserviçoouprodutonecessárionomomento,epoucotempodepois

podertrocardefornecedor,semteroscustosdepossuirmaquinário,instalações

fabrisouterdedemitirpessoal.

Dessaforma,aorganizaçãopodeseadaptaramudançasnoambienteexterno

muitomaisrapidamenteeaproveitarasoportunidadesdemercadoqueapareçam.

Figura4.22–Estruturaemrede

Outragrandevantageméqueessaestruturaéumadasmaisenxutas,poisnão

sãonecessáriosmuitosníveishierárquicos.Poucasupervisãoénecessária,sendo

entãoeliminadosmuitosníveisdegerência,deixandoaestruturamaisachatada.

Asmaioresdesvantagensdizemrespeitoàfaltadecontrolediretosobreos

processosdetrabalho.Comoaempresacontrataprodutoseserviços,a

administraçãotemdeconaremcontratos,coordenaçãoentreparceirose

comunicaçõescomoutrasempresasparaassegurarqueosprazoseaqualidade

sejamatendidos.

Vantagens

Desvantagens

Permiteumamaiorflexibilidadee

Aumentaadificuldadedeapurararesponsabilidade

adaptabilidadedaorganizaçãoaoambiente

poralgumproblema.

complexoemutável.

Dificuldadenacobrançaecontrole,poisdependedo

Estimulaodesenvolvimentoda

gerenciamentodecontratosenegociaçõesentreos

competitividadeglobal.

parceiros.

Promoveumambientedesafiadoremotivador

Possibilidademaiordeperdadeumparceiro

paratrabalhar.

estratégico,comimpactograndenosresultados.

Reduzosgastosdesupervisão.

Dificuldadedecriarumaculturaorganizacionalforte.

Reduzosníveishierárquicos.

Figura4.23–Vantagensedesvantagensdaestruturaemrede.

(Baseadoem:SobralePeci,2008)

Vamosvercomootemajáfoicobradoemconcursos?

6.(Cespe/MPS/Administrador/2010)Aorganizaçãoemredetemsidouminstrumentofacilitador

naformaçãodemonopóliossobretecnologiasemeiosdeprodução,assimcomonaexclusãode

diversasempresasemdiferentesmercados,sendomarcadapelaindividualidadedas

organizações.

Aorganizaçãoemredeveioaoencontrodanecessidadedeasempresas

enfrentaremosdesaosdomundocontemporâneo.Nessaestrutura,atendênciaé

terumamaiordescentralizaçãoemaiorbuscadeparceriasentreempresas

distintasparaatenderàsexigênciascadavezmaioresdomercado.20Aassertivadiz

exatamenteocontrário.

Umaestruturaemredepodeocorrerquandoumaempresacontacomdiversos

parceiros(quepodemestaremqualquerlugardomundo)paragerirseus

negócios.

Oinstrumentodecontroledeixadeserahierarquiaparaagestãodeparceriase

contratoscomfornecedoresexternos.Emvezdeverticalizarseuprocesso

produtivo(construindofábricas,contratandopessoasetc.),aorganizaçãocontrata

essesserviçosnomercado.

Quandoumaempresadecalçadosutilizaumafábricadeterceiros,localizadana

China,paraproduzirseuproduto,umarmadedistribuiçãoparaentregarseus

pedidosnaEuropa,tendoodesignsidoencomendadoemumescritórioitaliano,

estáutilizandoumaestruturaemrede.Assim,ogabaritoéquestãoincorreta.

4.16.FatoresqueInfluenciamaEstrutura

Adeniçãodaestruturacorretaparaumadeterminadaorganizaçãodependede

váriosfatores.Nãoexisteummodeloquesirvaparatodasasorganizações!Os

fatoresprincipaissão:aestratégia,oambiente,atecnologiaeotamanho.

4.16.1.AEstruturaSegueaEstratégia

DeacordocomChandler,asmudançasnaestratégiaprecedemesãoacausade

mudançasnaestruturadasorganizações.

UtilizandoasestratégiaspropostasporPortercomoexemplo,comuma

estratégiadecusto,aempresabuscaeciênciainterna.Essaestratégiaseriamais

adequadaaumaestruturafuncional,poiselautilizaosrecursosdemaneiramais

eficazcomsuahierarquiaecadeiadecomando.

Jácomumaestratégiadediferenciação,quenecessitadeinovaçãoede

exibilidadeorganizacional,aempresadeveriaadotarumaestruturahorizontal,

comoumaestruturaemrede,virtualouporequipes.

4.16.2.AEstruturaSegueoAmbiente

Aestruturatambéméafetadapelograudeincertezaambiental.Quantomais

incertooambienteexterno,maisdifícilsetornaoprocessodetomadadedecisões

dosgestores,levando-osadescentralizaradecisão.

Quantomaissimples,oumaisestáveloambienteexterno,maisfácilsetornao

trabalhode“entender”esseambiente,tornandomenosexistenteanecessidadede

descentralizar.

Damesmaforma,ambientescomplexosdicultamotrabalhodepadronizaçãoe

formalizaçãodostrabalhos.Portanto,estruturasburocráticasnãoseadaptam

muitobemasetoresdinâmicos,comváriosfatoresenvolvidosemudançasrápidas

nopanorama,comoprodutoseconcorrentesnovos,mudançasnastecnologias

envolvidasenoscostumesenecessidadesdosclientes.

Paraambientescomplexosedinâmicos,asestruturasemredesãomais

indicadas,poispossibilitammaiorflexibilidadeecapacidadedeinovação.

4.16.3.AEstruturaseAdaptaàTecnologia

Tecnologiaincluioconhecimento,astécnicas,asferramentaseasatividades

utilizadaspelaorganizaçãoparaentregarprodutoseserviçosaomercado.A

tecnologiatambémafetaotipodeestruturamaisadequadoaumaorganização.

Deacordocompesquisas,emorganizaçõesquetêmaproduçãoemmassa

(tecnologiadeproduçãoemgrandequantidadeeespecicaçõessemelhantes),o

modelomecanicistatemsidoomaisadequado.

Jáasempresasquecontamcomaproduçãoporunidade(tecnologiade

produçãoempequenasquantidadesparaatenderumaclientelaespecíca)ecoma

produçãoporprocessocontínuo(tecnologiadeproduçãoemquetodooprocesso

éautomatizadoenuncaéparado)adaptam-semelhoraestruturasorgânicas.

4.16.4.AEstruturaSegueoTamanho

Otamanhodaorganizaçãoafeta,naturalmente,asuaestrutura.Oaumentodo

tamanhodeumaempresanormalmentelevaaumamaiordelegaçãoe

descentralização,poiséimpossíveladministrarumagrandeempresasomentepela

cúpula.

Outrafacetadocrescimentoéoaumentodaformalização(usodenormase

regulamentos),poisasorganizaçõesnecessitamcontrolarosprocessosqueforam

descentralizadosdealgumaforma.

Outrosaspectosquenormalmentesãoinuenciadosquandoumaorganização

crescesão:aumentodonúmerodeníveishierárquicos,aumentodonúmerode

departamentos(diferenciaçãohorizontal),aumentodocontroleeampliaçãoda

importânciadoplanejamentonaadministraçãodaempresa.

4.17.EstruturasdeMintzberg

Umadasmaisimportantesclassicaçõesdedesignsestruturaisfoicriadapor

Mintzberg.21Oautordeniuascincopartesbásicasdecadaorganizaçãoeas

principaisconfigurações.Vejamosaspartesbásicassegundooautor:

ØONúcleoOperacional(OperatingCore)–écompostopelosfuncionários

queexecutamotrabalhodeproduzirosprodutoseprestarosserviços.

ØACúpulaEstratégica(StrategicApex)–sãoosadministradoresdotopo

hierárquico,responsáveisporsupervisionartodootrabalhonaorganização.

ØALinhaIntermediária(MiddleLine)–sãoosgerentes,quefazemaligação

entreacúpulaestratégicaeonúcleooperacional.

ØATecnoestrutura(Technostructure)–sãoanalistasqueexecutamtarefas

administrativas,comooplanejamentoecontroledotrabalhodosoutros.

ØOStaffdeSuporte(SupportStaff)–sãoosprossionaisqueprestamserviços

indiretosparaaorganização,comoentregasdemalotes,alimentação,

relações-públicasetc.

Figura4.24–EstruturasdeMintzberg.

(Fonte:MintzbergeQuinn2001)

AsprincipaisconfiguraçõesdeMintzbergsãoasseguintes:22

EstruturaSimples–Tambémchamadadeorganizaçãoempreendedora.É

compostadeumacúpulaestratégicapequenaedeumnúcleooperacional.Ocaso

maisusualéodapequenaempresa.

Nessaconguração,existempoucoscontroleseformalização,poisoexecutivo

principalconseguemuitasvezessozinhoadministraraempresaeconhecetodosos

funcionários.

Dessaforma,poucoexistetambémdeplanejamentoepadronizaçãodo

trabalho,candoaestruturamaisenxutaparapoderconcorrercomasgrandes

burocracias.

Comotempoeocrescimento,essasempresassetransformarãoemburocracias,

poisédifíciladministrarumaempresagrandenessaconguraçãopormuito

tempo.

Comopodemvernográcoaseguir,essaestruturanãocontanemcomostaff

desuporteetampoucocomumatecnoestrutura.

Figura4.25–Estruturasimples.

(Fonte:MintzbergeQuinn.2001)

BurocraciaMecanizada–Essaconguraçãoémuitocomumemgrandes

indústriasquetêmtecnologiadeproduçãoemmassa.Existeumagrande

padronizaçãodosprocessosdetrabalho,queérepetitivoeespecializadopara

possibilitarautilizaçãoeotreinamentodeumamassadetrabalhadores

desqualificada.

Portanto,essaconguraçãoébaseadanaburocraciaweberiana,comsua

formalidade,padronizaçãoeespecializaçãodotrabalho.Comoexisteumagrande

padronização,atecnoestruturaébastanteimportantenessaconfiguração.

Figura4.26–Burocraciamecanizada.

(Fonte:MintzbergeQuinn.2001)

BurocraciaProssional–Essetipodeburocraciabaseia-senapadronização

dehabilidades,enãodosprocessosdetrabalho.Oscasosmaislembradossãoas

universidades(emqueosprofessoresdasdiversasdisciplinasformamonúcleo

operacional)eoshospitais(mesmasituaçãoocorrecomosmédicose

enfermeiros).

Dessaforma,asorganizações,porsuapeculiaridade,dãomuitomaisliberdade

aosseustrabalhadores,poisestessãoaltamenteespecializados.

Portanto,nessaconguração,onúcleooperacionaltemmuitaimportância,

assimcomoostaffdesuporte(queserveaonúcleooperacional),masnãoháa

necessidadedeumatecnoestruturagrandenemdeumalinhaintermediária

importante,poisoexecutortemmaiorautonomiaparaexecutarseutrabalho,e

quempadronizasuaatividadesãoasassociaçõesprofissionais(Crea,porexemplo).

Figura4.27–Burocraciaprofissional.

(Fonte:MintzbergeQuinn.2001)

EstruturaDivisionalizada–Essaestruturanaverdadeéumconjuntode

organizaçõesdentrodeoutra,ouseja,adescriçãodeumconglomeradode

empresas.

Issoacontecequandoaempresacamuitograndeeseusprodutose/ou

mercadostornam-semuitodiversos,incentivandoacriaçãodeestruturas

própriasparacadasetorouprodutodistinto,possibilitandoumamaior

autonomiaeflexibilidadeparaosgestoresatingiremseusresultados.

Nessaconguração,ocontroleérealizadoporresultados(outputs),portanto

diz-sequeapadronizaçãoésobreessesoutputs,enãosobrecomoseráfeitoo

trabalhoemsi.Dessaforma,existeumapequenatecnoestruturaparacontrolaros

resultados.

Figura4.28–Estruturadivisionalizada.

(Fonte:MintzbergeQuinn.2001)

Adhocracia–Essaconguraçãoéamaisadequadaparaasempresasenvolvidas

emsetoresdinâmicos,comooaeroespacial,odeconsultoriaseocinematográfico.

Nessasorganizações,nãoexisteumaestruturaxa,masdiversosprossionais

altamentetreinados,quesãoagrupadosdeacordocomumanecessidadeespecíca

(porissootermoadhoc),portantoexisteumaestruturaporprojetos.

Imagineumfilmeaserproduzido.Aequipequevaitrabalharnofilmevaisendo

montadaporseuprodutordentrodeumnúmerograndedeprossionais

gabaritadosexistentenaempresaounomercadodetrabalho.

Aoterminarolme,essesprossionaisserãoescolhidosparaoutrosprojetos,

comequipesdiferentesdepessoas.Portanto,otrabalhonãoéformalizado,nem

existempadrõesdecomportamento.Aestruturalinha-staffperdeentãoosentido,

comaorganizaçãosefundindoeopoderseespalhandoportodaaempresa.

Dessaforma,comoosprossionaisdonúcleooperacionaltêmgrande

conhecimentoeperícia,confundem-secomalinhaintermediária,pois

influenciamnadefiniçãodosprojetosenoandamentodosmesmos.

Essasorganizaçõessãoorgânicas,exíveisedinâmicas,levandoaumambiente

inovadorecriativo.Portanto,onúcleooperacionalfunde-secomalinha

intermediária,comopodemveraseguir.

Figura4.29–Adhocracia.

(Fonte:MintzbergeQuinn.2001)

Vamosvercomoessetópicojáfoicobradoemconcursos?

7.(Cespe/MS/Administrador/2010)Asrecompensasnaadhocraciasãobaseadasnoresultadodo

funcionárionoexercíciodoseucargo.

EssaéumacaracterísticadaBurocracia,enãodaAdhocracia.NaAdhocraciaas

recompensasnãosãobaseadasapenasnoresultadodofuncionárionoexercício

doseucargo!Essaéumadúvidacomum!

OquevocêdeveteremmenteéqueaAdhocracianãoseprestamuitoa

atividadespadronizadas(quesãoasmaisfáceisdesemedir!).AAdhocraciase

prestamelhoraatividadesemqueestãoenvolvidosointelectoouhabilidades

complexas,emtrabalhosinovadorese/ouquenecessitemdealtacriatividade.

Nessetipodeatividade,nemsempreépossíveloudesejávelparametrizarum

sistemaderesultadosnormal.Comopoderíamosavaliarumtrabalhodeum

pesquisadordeumavacinaparaocâncer,quepodelevar15anosparaser

desenvolvida,porexemplo?

Podeserinteressantemontarumpacotedeincentivosbaseadotambémem

outrosfatores,comocapacidadeintelectual,númerodepropostas,projetos

envolvidosetc.Portanto,ogabaritoéquestãoincorreta.

QuestõesComentadas

8.(FCC/TRT22a/PI/AnalistaAdm./2010)Sobreascaracterísticasbásicasdasorganizações,a

estruturaorganizacional:

a)determinaasrelaçõesinformaisdesubordinação,entreasquaisosníveisdehierarquiaeaamplitudede

controledegerentesesupervisores;

b)nãoidenticaoagrupamentodeindivíduosemdepartamentosededepartamentosnaorganização

comoumtodo;

c)incluioprojetodesistemadeinformaçãoparaassegurarcomunicaçãoecaz,coordenaçãoe

integraçãodeesforçossomenteentreosdepartamentosoperacionaiseosstakeholders;

d)reete-senoorganogramaquemostraapenasaspartesoperacionaisdeumaorganizaçãoecomo

estasestãointer-relacionadas;

e)possuiligaçõesverticaisutilizadasparacoordenaratividadesentreacúpulaeabasedaorganização.

Aprimeiraalternativaestáincorreta,poisaestruturaorganizacionalnão

determinaasrelaçõesinformais,esimasrelaçõesformaisentreosindivíduose

cargos.Damesmaforma,aletraBestáequivocada,poisaestruturaidenticao

agrupamentodeindivíduosemdepartamentos(departamentalização).

AletraCestátambémerrada,poisosistemadecomunicaçãonãodeveser

apenasentreosdepartamentosoperacionaiseosstakeholders.Damesmamaneira,

aletraDtemtambémuma“pegadinha”,poisapalavra“apenas”invalidaaquestão.

NossogabaritoémesmoaletraE.

9.(Esaf/ANA–AnalistaAdm./2009)Comoinstrumentogerencialcontemporâneo,écorretoarmar

sobreosmecanismosderede:

a)seupressupostobásicoéodaarticulaçãoconjuntaentreasorganizações,visandoao

compartilhamentoderecursos,excetooacessoaoknow-how,quedevesermantidoemsigilo;

b)asredespodemsercompreendidascomoaformaçãoderelaçõesinterorganizacionaissegundouma

perspectivaeconômicaemercadológica;

c)asredessãovistascomoumaformarígidaecentralizadadegovernança;

d)areduçãodoscustosdetransaçãoéaúnicacausadaemergênciadasredesorganizacionais;

e)emborasejaumespaçoplural,ondecoexistemdiferentesagentes,aredeorganizacionalse

caracterizapelaunicidadedecapitaledeinteressescorporativos.

AletraAdeixa“defora”oacessoaoknow-how(“sabercomo”),tornandoa

assertivaincorreta.JáaletraBestácerta.

AletraCestáerrada,poisasredessãoexatamenteocontráriodoqueabanca

armou.OmesmoocorrecomaletraD,poisasredesnãoexistemapenasparaa

reduçãodecustos,mas,principalmente,paraganhosdeflexibilidade.

AletraEtambémnãofazsentido,poisnasestruturasemredenãoexisteessa

unicidadedecapitaloudeinteresses.OnossogabaritoémesmoaletraB.

10.(Cesgranrio/Termoaçu/Administradorjr./2008)Emrelaçãoàsestruturasorganizacionais,tem-

sequeaestrutura:

a)físicaéidênticaemqualquerempresa;

b)informalémaisimportantedoqueaestruturaformal;

c)informaléreconhecidajuridicamente,defatoededireito;

d)formalnãoestásujeitaaossentimentoserelacionamentos;

e)formalnãopodeserrepresentadapeloorganogramadaempresa.

Aprimeiraalternativaéabsurda,poisumaempresagrandeequeopereem

diversospaísesnãoteráamesmaestruturaqueumapequenaempresa,nãoé

mesmo?

NaletraB,nãoexisteessamaiorimportânciadaestruturainformalemrelação

àestruturaformal.AletraCtambémestáequivocada,poisaestruturainformalé

derivadadosrelacionamentosinformais,enãoéreconhecidalegalmente.

JáaletraDestáperfeita,poisaestruturaformaléderivadadoscargose

relacionamentosociaisdentrodeumaorganização.Entretanto,aletraEtrocaos

conceitosdeestruturainformalcomaformal.OnossogabaritoémesmoaletraD.

11.(Cespe/Abin/Ocialtécnico/2010)Entreoscritériosdedepartamentalização,inclui-seo

funcional,organizadoportarefasindependenteseorientadasparaumobjetivogeral.

Nadepartamentalizaçãofuncional,osórgãosnãosãoorganizadosportarefas

independentes,massiminterdependentes,ouseja,quedependemumasdas

outras.

Imagineumarmaquetenhaseusdepartamentosdevendasedeprodução.Seo

departamentodevendasnãozerumbomtrabalho,irácausarasuspensãoda

produção,nãoémesmo?Ouseja,seaempresanãovenderbem,terádereduzirsua

produção.

Poroutrolado,seaproduçãodaempresativerproblemasdeatrasoede

qualidade,issoirátornarotrabalhodevendercadavezmaisdifícil.Dessaforma,

osdepartamentosdevendasedeproduçãosãointerdependentes.Ogabaritoé

questãoincorreta.

QuestõesPropostas

12.(Esaf/CGU/AFC/2006)Indiqueaquecritériodedepartamentalizaçãosereferecadafrase.

I.Privilegiaaespecializaçãoeomáximodeutilizaçãodashabilidadestécnicasdaspessoas.

II.Facilitaoempregodetecnologia,dasmáquinaseequipamentos,doconhecimentoedamão

deobra.

III.Adapta-seàspeculiaridadesdecadamercadoconsumidor.

Escolhaaopçãocorreta.

a)IeII–departamentalizaçãoporcliente.

b)II–departamentalizaçãoporprocesso.

c)I–departamentalizaçãofuncional.

d)IeII–departamentalizaçãoporproduto.

e)I–departamentalizaçãoporperíodo.

13.

(Esaf/CGU/AFC/2004)

Uma

organização

pode

adotar

diferentes

critérios

de

departamentalizaçãoparasuaestruturaorganizacional,dependendodeseuporte,estratégia,

dispersãogeográca.Dadososcritériosdedepartamentalizaçãofuncional,porlocalidadee

porcliente,façaacorrespondênciacomasestratégiasindicadasaseguiremarqueaopçãoque

expressacorretamentearelaçãoestratégia/critériodedepartamentalização.

I.Aorganizaçãotemcomometaabrirseisfiliais,umaemcadacapitaldosestadosdoNordeste.

II.Aorganizaçãotemcomometaconsolidaraespecializaçãodaprodução,comercializaçãoe

informática.

III.Aorganizaçãotemcomometaconsolidarsuaposiçãojuntoaosconsumidores,prestando-

lhesserviçosdiferenciados.

IV.Aorganizaçãotemcomometaconsolidarsuaposiçãonomercadonascidadesondeatua.

V.Aorganizaçãotemcomometalançarumprodutodirecionadoajovensdaclassemédia.

a)I.porclienteI.funcionalII.porlocalidadeIV.porclienteV.porlocalidade.

b)I.funcionalI.porclienteII.porlocalidadeIV.funcionalV.porlocalidade.

c)I.porlocalidadeI.funcionalII.porclienteIV.porlocalidadeV.porcliente.

d)I.porclienteI.porlocalidadeII.funcionalIV.porclienteV.porlocalidade.

e)I.porclienteI.funcionalII.porclienteIV.porlocalidadeV.porcliente.

14.(FCC–TJ/AP–AnalistaAdm./2009)Comrelaçãoàsvantagensdadescentralizaçãosobrea

centralização,assinaleaafirmativaINCORRETA.

a)Visãomaisfocadanadefesadosucessodosdepartamentosemcomplementaçãoaosinteressesda

organizaçãocomoumtodo.

b)Hámaiorenvolvimentodosfuncionáriosoperacionaisnasdecisõestomadasemaiormotivaçãoentre

osgerentese,portanto,maiorcriaçãodevalormoralnaorganização.

c)Asdecisõesdosadministradoressãotomadascombasenumavisãoglobaldasmetaseobjetivosda

organização.

d)Asdecisõessãotomadasporadministradorescommaisinformaçãosobreocontextoeassuas

implicaçõessobretodaaorganização.

e)Melhoraoníveldecapacitaçãoeeciênciadasgerênciasintermediáriasedasuacomunicaçãocomo

níveloperacional.

15.(Cespe/Min.esporte/Administrador/2008)Adescentralizaçãoadministrativatemcomo

vantagembásicaamaioruniformidadedosprocedimentosdaorganização.

16.(FCC/TRF5aRegião/Anal.adm./2008)OPrincípiodeAdministração“Cadasubordinadorecebe

ordenseprestacontassomenteaumsuperior”refere-seaoprincípio:

a)cadeiaescalar;

b)unidadededireção;

c)unidadedecomando;

d)estabilidadedopessoal;

e)manteradisciplina.

17.(FCC/TRT/PA/Analistaadm./2010)Aodeniraestruturaorganizacionaldeumaempresa,seus

administradoresdevemconsiderarseiselementosbásicos:

a)tecnologia;produto/serviço;análisedotrabalho;agrupamentodefunções;custosenúmerodeníveis

decomando;

b)divisãodotrabalho;produtividade;segmentodenegócio;tecnologia;distribuiçãodasequipese

localizaçãogeográfica;

c)mãodeobranecessária;característicasdosclientes;amplitudedonegócio;concentraçãodotrabalho;

funçõesessenciaiseagrupamentodastarefas;

d)especializaçãodotrabalho;departamentalização;cadeiadecomando;amplitudedecontrole;

centralização/descentralizaçãoeformalização;

e)focodonegócio;público-alvo;mãodeobraespecializada;amplitudedecontrole;divisãodepodere

distribuiçãogeográfica;

18.(FCC/PGE/RJ/Auditor/2009)EmrelaçãoàAdhocracia,expressãodaautoriadeAlvinTofflere

popularizadaporRobertWatermancomolivroAdhocracy−ThePowertoChange:

I.OobjetivodaAdhocraciaéaidenticaçãodenovasoportunidadespormeiodoincentivoà

criatividadeindividualenquantocaminhoparaarenovaçãoorganizacional.

II.Éaestruturaquemaisestimulaainovação,poiséaquerespeitamenososprincípios

clássicosdegestão,eespecialmenteaunidadedecomando.

III.Podeserdenidacomoqualquerformadeorganizaçãoquecontacomtodosos

procedimentostípicosdasorganizaçõesburocráticasrelacionadoscomaresoluçãode

problemaseaobtençãoderesultados.

IV.Essaestruturaexigeumaespecializaçãohorizontalreduzida,impedindoaformaçãode

gruposdeespecialistasemunidadesfuncionaisautônomas.

V.Écaracterizadapeloelevadograudedescentralização,alémdeumaprecisadeniçãode

papéis,emboracomalgumadificuldadenofluxodecomunicaçãoinformal.

a)EstãocorretasAPENASasafirmativasIeI.

b)EstãocorretasAPENASasafirmativasI,IeV.

c)EstãocorretasAPENASasafirmativasI,IIeIV.

d)EstãocorretasAPENASasafirmativasIIeIV.

e)EstãocorretasAPENASasafirmativasII,IVeV.

19.(FCC/TJ/AP/AnalistaAdm./2009)Oformatoorganizacionalqueasseguramaiorflexibilidadeé:

a)aestruturaemredesdeequipes;

b)aestruturafuncionallinear;

c)osistemaorgânicohomeostático;

d)aestruturamatricial;

e)aestruturaburocrática.

20.(FCC/Defensoria/SP/Administrador/2010)Organizaçõesqueplanejamdesenvolveruma

estruturamaisexível,associadaamaioreciênciaemotivação,devemoptarporuma

estrutura:

a)departamentaldetipofuncional;

b)deredesintegradasdeequipes;

c)departamentalporprocessos;

d)detipolinha-staff;

e)detipolinear.

21.(Cespe/Ceturb-ES/Administrador/2010)Osgruposinformais,aliançasnãoestruturadas

formalmentepelaorganização,agememinteressepróprioe,porvezes,sãomaisfortesqueos

gruposformais.

22.(FCC/TRT22a/PI/Analistaadm./2010)Emumaestruturafuncional:

a)asatividadessãoagrupadasporprocessos;

b)nãoháodesenvolvimentoemprofundidadedashabilidadesdosfuncionários;

c)otempoderespostaàsmudançasambientaisérápido;

d)ograudeinovaçãoébaixoeháumavisãorestritadasmetasorganizacionais;

e)acoordenaçãohorizontalentreosdepartamentoséeficazeefetiva.

23.(FCC/Metrô/Administração/2008)Agruparnummesmoórgãoasatividadesansoudemesma

naturezaouespecialidadeéoprocessodedepartamentalização:

a)porprodutoouserviço;

b)porprocesso;

c)poráreageográfica;

d)funcional;

e)matricial.

24.(Esaf/MPOG/EPPGG/2009)Aolidarcomotema“departamentalização”,écorretopressupor

que:

a)ogerenciamentodeprojetoséinviávelemumambientematricialmenteestruturado;

b)cadanívelhierárquicocomportanãomaisqueumtipodedepartamentalização;

c)formastradicionaisdedepartamentalizaçãoestimulamacomunicaçãohorizontal;

d)noâmbitodaadministraçãopública,éinviávelaadoçãodeestruturasinovativas;

e)estruturasmatriciaisnãosecaracterizampeloprestígioàunicidadedecomando.

25.(FCC/TRT22a/PI–Analistaadm./2010)Estruturaorganizacionalquealcançaacoordenação

necessáriaparaatenderademandasduaisdeclientes;permiteocompartilhamentoflexívelde

recursoshumanosentreosprodutos;fazcomqueosparticipantesexperimentemautoridade

dual.Trata-sedaestrutura:

a)divisional;

b)matricial;

c)funcional;

d)geográfica;

e)adhoc.

26.(Esaf/MPOG/APO–2010)Sobreotema“departamentalizaçãoedivisãodotrabalho”,écorreto

afirmar:

a)porsercontemporânea,aabordagemmatricialépreferívelàfuncional;

b)emambientesqueprivilegiamresultados,inexisteespaçoparaaadoçãodeestruturascolegiadasou

deassessoramento;

c)aabordagemdeorganizaçãovirtualémaisaplicávelàestruturaçãodeindústriasnavais;

d)emorganizaçõesmatricialmenteestruturadas,apreocupaçãocomahierarquiaéabolida;

e)emgrandesorganizações,épossívelacoexistênciadedoisoumaistiposdedepartamentalização.

27.(FCC/MP/RS/Administrador/2008)Oquecaracterizaumaestruturaorganizacionaldetipo

matricialé:

a)aconstituiçãodeumaequipeheterogêneadeespecialistasemdiferentesáreas,quesãodeslocados

desuasfunçõeshabituaisparasededicaremaumatarefaespecífica;

b)acombinaçãodeformasdedepartamentalizaçãofuncionaledeprodutoouprojetonamesma

estruturaorganizacional;

c)adelegaçãodeautoridadeeresponsabilidadeàsequipes,quesetornamautônomaseautossuficientes

paradecidirsobreseutrabalho;

d)areuniãodeespecialistasemtornodeatividadessimilares,agrupadaseidenticadasdeacordocomos

seusfins;

e)acombinaçãodeórgãosdelinha,diretamenterelacionadosaosobjetivosvitaisdaempresaede

órgãosdestaffqueapoiamosprimeiros.

27.(Esaf/STN/Desenv.Institucional/2008)Acercadaestruturamatricial,éincorretoafirmar:

a)aessênciadaestruturamatricialécombinarascaracterísticasdasestruturasverticais(funções)edas

horizontais(processos),resolvendoemparteoproblemadacentralização/descentralização;

b)essetipodeestruturafoiprojetadoparaproporcionaràsorganizaçõesascondiçõesdeexibilidadee

defuncionalidade,possibilitando-as,assim,dereagiremrapidamenteàsmudançasambientaisaque

normalmenteestãosubmetidas;

c)oarranjomatricial,apesardesermaiscomplexoqueotradicional,apresenta-secomoumaopçãode

estruturaquepossibilitaàspessoasobteremumavisãoglobaldasoperaçõesedonegócioda

organização.Nessecontexto,oorganogramahierárquicoverticaldevesersubstituídoporalgoque

representeummapaorganizacional,detalformaqueonegóciodaempresaestejaclaroparatodasas

pessoasqueintegramaorganização;

d)considerandoqueacaracterísticamorfológicadessetipodeestruturaimplicaumamudançaradical

dasresponsabilidades,asinterfacesentreouxohorizontaleverticaldevemestarmuitobem-

definidaseclarasapenasparaonívelestratégico/gerencialdaorganização;

e)emboraomodelomatricialbusqueaapropriaçãodasvantagensdecadaumadasestruturas,verticale

horizontal,existeumacomplexidadeinerenteàsuaprópriaconstituiçãoquedicultasua

operacionalização.Aprincipalrestriçãoqueéfeitaaessetipodeestruturaéque,dependendode

comoéconcebida,podehaverconflitoderesponsabilidadesouduplocomando.

29.(FCC/Bahiagas/Administrador/2010)Otipohíbridodedepartamentalizaçãonoqualexistemos

órgãosprincipaisdetrabalho,quetêmvidalimitadaàduraçãodoprojetoeosórgãosdeapoio

funcional,permanentes,queapoiamosprojetoseosorientamemassuntosespecializadosé

denominado:

a)matricial;

b)funcional;

c)divisional;

d)porprodutos;

e)staff.

30.(Cespe/MS/Administrador/2010)Aadhocraciaprevêocompromissodosfuncionárioscoma

qualidade.

31.(FCC/Arce/Analistareg./2006)Umaestruturaaltamenteorgânica,compoucaformalizaçãodo

comportamento,especializaçãodotrabalhoaltamentehorizontalizada,baseadaem

treinamentoformal,tendênciaaagruparespecialistasdasunidadesfuncionaisparapropósitos

deorganizaçãointernadivididosempequenasequipesdeprojetosbaseadasnomercado,que

visamàexecuçãodeumtrabalhoespecíco,sãoparâmetrosdedesignorganizacional,

identificadosporH.Mintzberg,paraodesenhodeumaestruturadenominada:

a)burocraciaprofissional;

b)divisionalizada;

c)burocraciamecanizada;

d)estruturasimples;

e)adhocracia.

Gabaritos

1.A

12.C

22.D

2.E

13.C

23.D

3.E

14.C

24.E

4.D

15.E

25.B

5.C

16.C

26.E

6.E

17.D

27.B

7.E

18.A

28.D

8.E

19.A

29.A

9.B

20.B

30.C

10.D

21.C

31.E

11.E

Bibliografia

Chiavenato,Idalberto.AdministraçãoGeralePública.2aed.SãoPaulo:Elsevier,

2008.

—.Administraçãonosnovostempos.2aed.RiodeJaneiro:Elsevier,2010.

—.IntroduçãoàTeoriaGeraldaAdministração.8aed.RiodeJaneiro:Elsevier,2011.

Daft,RichardL.Management.Mason:Thomson,2005.

Maximiniano,AntonioCesarAmaru.IntroduçãoàAdministração.4aed.SãoPaulo:

Atlas,1995.

Mintzberg,Henry,eJamesBrianQuinn.OProcessodaEstratégia.3aed.Porto

Alegre:Bookman,2001.

Robbins,StephenP.,eMaryCoulter.Administração.5aed.RiodeJaneiro:

Prentice-Hall,1998.

SchermerhornJr.,JohnR.Management.9aed.Hoboken:Wiley&Sons,2008.

Sobral,Felipe,eAlketaPeci.Administração:teoriaepráticanocontextobrasileiro.

SãoPaulo:Pearson-Prentice-Hall,2008.

Capítulo5

Motivação

DeacordocomDa,1amotivaçãoérelativaàsforçasinternasouexternasque

fazemumapessoaseentusiasmarepersistirnabuscadeumobjetivo!Comoa

motivaçãoafetaaprodutividade,aorganizaçãodevecanalizaramotivaçãoparaos

objetivosorganizacionais.

Dessaforma,Robbins2dizque,noambienteorganizacional,amotivaçãoéa

vontadedeexerceraltosníveisdeesforçoparaalcançarosobjetivos

organizacionais.

Entretanto,adiculdadeexistenofatodequeaspessoassãomuitodiferentes

umasdasoutras.Oquepodemotivarumapessoapodenãomotivaroutras.Existe

umadiversidadedeteoriasmotivacionais,3masemgeralosautoresabordamdois

tiposdefatores,osextrínsecos(deforadapessoa)eosintrínsecos(dedentroda

pessoa).

DeacordocomBergamini,

Noprimeirocaso,pressupõe-sequeaforçaqueconduzocomportamentomotivadoestáforada

pessoa,querdizer,nascedefatoresextrínsecosquesão,decertaforma,soberanosealheiosàsua

vontade.Nosegundocaso,subjazacrençadequeasaçõeshumanassãoespontâneasegratuitas,

umavezquetêmsuasorigensnasimpulsõesinteriores;assimsendo,opróprioserhumanotrazem

siseupotencialeafontedeorigemdoseucomportamentomotivacional.

Existem,portanto,diversasteoriasquesebaseiamnosfatoresexternose/ou

internosdoindivíduo.Quandoumaempresaofereceumprêmioparao

funcionárioquebaterumametaestásebaseandonosfatoresexternos(vocênão

teriavontadedebaterameta,mascomoexisteumestímuloexterno–oprêmio–

vocêseesforçaparaisso).

Jáquandovocêdesejafazerumtrabalhobem-feito,poisachaseutrabalho

interessanteeimportante,estásebaseandoemfatoresinternos(suapreocupação

comosoutrosouseuorgulhodefazerumtrabalhobem-feito).

Aseguirpodemosverumresumodostiposdemotivação:

Figura5.1–Tiposdemotivação

5.1.TeoriasdeProcessoeTeoriasdeConteúdo

Asteoriasdemotivaçãopodemserdivididasemteoriasdeconteúdoeteorias

deprocesso.Asteoriasdeconteúdoseconcentramnasrazõesquelevamuma

pessoaaficarmotivada(seriaoQUEmotivaalguém).

Naturalmente,agrandemaioriadaspessoasdesejaounecessitadecoisasque

nãotêm.Quandoqueremosalgo(sejaumcarromelhor,umaviagemaoexterior,

ouquenossafamíliacomatodososdias),temosuma“motivação”paraalcançar

esseobjetivo,nãoémesmo?Asteoriasquesepreocupamcomasnecessidadesque

nos“impulsionam”sãoasteoriasdeconteúdo.4

Jáasteoriasdeprocessoseconcentramnomodoemqueocomportamentoé

motivado(seriaoCOMOessamotivaçãoocorre).

Osteóricosquetrabalhamcomessasteoriasacreditamqueaspessoassãomais

propensasasecomportarcom“motivação”setrêscondiçõesacontecem:as

pessoascreemqueasaçõesatingirãoodesempenhodesejado;queesse

desempenholevaráaoresultadobuscadoeoresultadoérealmentedesejadopor

elas.5

Aseguirpodemosverasprincipaisteoriasmotivacionais.

Figura5.2–TeoriasdeConteúdoedeProcesso

5.1.1.TeoriadasNecessidadesdeMaslow

Ateoriadahierarquiadasnecessidades,tambémconhecidacomoapirâmidede

Maslow(nomedeseucriador,opsicólogoAbrahamMaslow),éumadasmais

intuitivasteoriasdemotivação.

Deacordocomoautor,ocomportamentodoserhumanoémotivadopor

diversosestímulosinternosoupornecessidades.

Essasnecessidadesseriamdiversas,eeleasclassicouemumahierarquia,desde

asmaisbásicas,comoassiológicasedesegurança,àsdenívelsuperior(estimae

autorrealização),conformeoseguintegráfico.

Figura5.3–HierarquiadeMaslow

Oponto-chavedessateoriaéexatamenteahierarquiadessasnecessidades.A

teoriadizqueasnecessidadesdevemsersatisfeitasemumaordem,ouseja,seria

necessáriosuprirasnecessidadessiológicas(asmaisbásicas)antesquedeva

existirumtrabalhodeatendimentodasnecessidadesdeestima.

Umexemploseriaumapessoaqueestáfaminta.Essapessoabuscaráatodocusto

supriracarência(alimento)antesdesepreocuparcomasuarealização

profissional,statusereconhecimento,nãoémesmo?

Comisso,ogestorquebuscaimplantarumprogramademotivaçãodeve

analisaremqueestadoseencontraseufuncionárioepassaratrabalharsempreo

nívelhierárquicodenecessidadesatual,ouoprimeironíveldenecessidadesacima

doatual,ouseja,doqueseencontranomomentosendoatendido!

Assimqueumanecessidadeéatendida,elaperdeimportância,sendo“ativada”

entãoanecessidadedenívelsuperior.

Umadúvidaconstantedosalunosseconcentranadiferençaentreas

necessidadessociaisedeestima.Bom,vamosimaginarumasituaçãoprática.As

necessidadessociaisseriamligadasaoamor,asamizadesetc.

Umapessoapodedesejarumavidasimples,semluxos,semnotoriedade,nãoé

verdade?Masachoquequasetodosdesejamteramigos,terum(a)namorado(a)

etc.

Assim,umapessoaquebuscasseserumpescadoremumapequenavilanão

buscanotoriedade,status,prestígio,nãoémesmo?Masseelegostadeteramigos,

denamoraretc.temumanecessidadesocial.

Vamosveragoraumaquestãosobreessetema?

1.(FCC/MP/SE/Administrador/2009)NaTeoriaMotivacionaldeMaslow,anecessidadedaspessoas

desesentiremvalorizadaspelosqueasrodeiamrepresentaotipodenecessidade:

a)fisiológica;

b)deestima;

c)desegurança;

d)social;

e)deautorrealização.

Anecessidadesiológicadizrespeitoaosaspectosmaisbásicoscomo:comida,

águaetc.Portanto,aalternativaAestáincorreta.NossogabaritoéaalternativaB,

poisanecessidadedeaspessoassesentiremvalorizadaspelooutrosérelacionada

àestima.

Anecessidadedesegurançaserelacionacomasensaçãodenãoestarmos

passandoperigo(comoriscodeacidentes,assaltosetc.).Jáanecessidadesocialse

refereànossanecessidadedetermosamigos,colegasdetrabalho.

E,porúltimo,anecessidadedeautorrealizaçãoérelacionadacomnossas

necessidadesdecrescimentopessoaleprofissional.OgabaritoéaalternativaB.

5.1.2.TeoriaXeYdeMcGregor

AteoriaXeYdeMcGregor,umadasteoriasmaissimpleseintuitivasnoestudo

daAdministração,étambémumadasmaiscobradasemprovasdeconcurso!

McGregortrouxeaideiadequeexistemduas“maneiras”deseveraspessoas.

Essasvisõesseriamcontrárias,ouseja,antagônicas.Umaseriamaispositivae

moderna(ateoriaY,baseadanaconançanosindivíduos),eaoutraserianegativa

eantiquada(teoriaX,baseadanadesconfiança).6

PelateoriaX,aspessoasseriamnaturalmentepreguiçosas,poucoambiciosase

seminiciativaprópria.Aideiaéadequeoserhumanonãogostadetrabalhare

iráfazeromenoresforçopossível!

Seogestortemessavisãonegativadaspessoas,eletendeasermaisscalizador

econtrolador,poisacreditaquequando“virarascostas”otrabalhadorpararáde

trabalhar.Eleirátratarossubordinadosdemodomaisrígidoeseráautocrático

(decidirátudosozinho),poisachaqueoempregadonãogostadeassumir

responsabilidadesenãoconseguetomardecisões!Nãodelegaráresponsabilidades,

porqueachaqueosfuncionáriossãodependentes.

Fiqueatento:AteoriaXéumavisãonegativaeantiquadadotrabalhador.

Essavisãoseriamais“antiga”eadequadaaumaorganizaçãoemumambiente

estávelecomummodeloburocráticodegestão.

JáateoriaYseriamaisembasadanaconançanaspessoas!Nessasituação,o

gestoracreditaqueseusfuncionáriossãoambiciosos,gostamdetrabalhar,têm

capacidadededecidireiniciativa.

Aideiaéadequeaspessoasbuscamassumirresponsabilidadesedesaos!Desse

modo,opróprioempregadosecontrola,nãohavendoanecessidadedecoerção!

Comooadministradoracreditanopotencialdeseussubordinados,elebuscaa

participaçãodeles,delegapoderesparaqueelesassumamresponsabilidadesecria

umambientemaisdemocráticoeempreendedornotrabalho!

Essateoriaseriamaisadequadaaumaorganizaçãoqueestejaemumambiente

instáveledinâmicoequebusqueainovaçãonosprocessosdetrabalho.Hojeem

dia,essaéavisãomaisrecomendada!

TeoriaX

TeoriaY

Maioriadaspessoasnão

desgostadotrabalho.

Pessoasnãogostamdetrabalhar.

Pessoassãocapazesdeseautocontrolare

Pessoasdevemserameaçadaseforçadasaatingiros

dirigirquandoestãocomprometidas.

resultados.

Pessoasbuscamdesafioseassumir

Pessoaspreferemnãoassumirresponsabilidades,têm

responsabilidades.

poucaambiçãoebuscamsegurança.

Pessoassão,emsuamaioria,criativase

ambiciosas.

Figura5.4–TeoriaXeY

VamosanalisaragoraquestõessobreateoriaXeYdeMcGregor?

2.(Cespe/BASA/Administração/2010)Aadoçãodeumcontrolerigorosoporpartedeumgerente

sobreseuscolaboradores,porentenderqueestessãodesmotivadosedependemdeuma

liderançaforte,identifica-secomospressupostosdateoriaXpropostaporDouglasMcGregor.

Comopodemver,asquestõessobreateoriaXeYdeMcGregorsãobem

simples!Sevocêjáconheceoassunto,nãodeveráterproblemas.Comojá

estudamos,anecessidadedeumcontrolerígidoerigorososobreosfuncionários

seidentificacomateoriaXdeMcGregor.Ogabaritoéquestãocorreta.

3.(Cespe/AGU/Anal.adm./2010)Aoassumiragerênciadequalidadedeumaorganização,Maria

pretendecriarumaauditoriadeprocessosgerenciais,queavaliaráaconformidadedas

atividadesdesenvolvidaspelosdiversossetorescomponentesdaorganização.Paracompor

suaequipe,Mariadecidiurecrutarpessoalmentecadacolaborador,pormeiodeseleção

externa,poisacreditaque,demodogeral,aspessoasnãosãoresponsáveisenãogostamde

trabalhar.Visandoatraircolaboradoresparaocargo,elaofereceráasmelhoresgraticações

nanceirasdaorganização.Osfuturosauditoresdeprocessosterãoumarotinamuitobem

delineadaeserãoavaliadossemestralmentepelascheasimediatas.Osaspectosdaavaliação

julgadosdecientesserãoobjetodecapacitações.Comreferênciaaessasituaçãohipotéticae

aotemaporelaevocado,julgueospróximositens.

Maria,emtermosdeliderança,sepautanateoriaXdeMcGregor.

Comovimosnaquestão,Maria“acreditaque,demodogeral,aspessoasnãosão

responsáveisenãogostamdetrabalhar”.Seelatemessavisãosobreos

funcionários,elaestásepautandonateoriaXdeMcGregor.Tambémestácertaa

frase.

5.1.3.TeoriadosDoisFatoresdeHerzberg

AteoriadosdoisfatoresdeFrederickHerzbergéumadasmaisimportantesno

estudodaAdministraçãoeumadasquemaisseprestaa“pegadinhas”debancasde

concurso.Ateoriabasicamentedizqueosfatoresquelevamàsatisfaçãosão

diferentesdosquelevamàinsatisfação,daíonomedateoria:doisfatores!

Fiqueatento:ParaHerzberg,nemdinheironempessoasmotivamostrabalhadores.

Osfatoresmotivacionaisseriamosrelacionadosanecessidadesdomaisalto

nível,comooreconhecimentodaspessoas,oconteúdodotrabalho,a

possibilidadedecrescimentoprossionaledeaprendizagemeoexercícioda

responsabilidade.

Herzbergdissequequandoessesfatoresmotivacionaisnãoexistem,aspessoas

sãoneutrasemrelaçãoàmotivação(nãocammotivadasnemdesmotivadas).Mas

quandoessesfatoresestãopresentesgeramumaltoníveldemotivaçãono

profissional.

Jáosfatoreshigiênicosinuenciamainsatisfação,ouseja,podemgerar

insatisfaçãoseforemnegativos,masnãogeramsatisfaçãoseforempositivos!Entre

essesfatoresestãorelacionadas:condiçõesdetrabalho,remuneração,segurança,

relaçõespessoais,políticasdaempresaesupervisão.

Nocasodaorganizaçãoqueforneçabonssalários,boascondiçõesdetrabalhoe

segurança,elapodegerarumambienteharmônico,masnãonecessariamenteum

ambientealtamentemotivador.Prestembematençãonisto:osalárionãoéfator

motivacional!Arelaçãointerpessoaltambémnãoémotivadora!Ouseja,nem

dinheironempessoasmotivamosfuncionários.

Influenciamasatisfação

Influenciamainsatisfação

Salários

Crescimentopessoal

Relacionamentospessoais

Conteúdodotrabalho

Condiçõesdetrabalho

Exercíciodaresponsabilidade

Supervisores

Reconhecimento

Segurança

Realização

Políticasdaempresa

Figura5.5

Vamosimaginarumcasohipotético:Vocêpassaemumconcursopúblicoeé

nomeadoparaumcargoemumórgãofederal.Sesentemuitofelizdepassara

ganharumótimosalário,comestabilidadee,alémdisso,achaoambientede

trabalhoótimo!Todososseuscompanheirosdetrabalhosãopessoassimpáticase

atenciosas.Avidaestálinda,pensavocê!

Masotrabalhoquelhederamépassarfaxetirarcópias.Penseagora,vocênão

vaicarmuitomotivadonotrabalho,nãoéverdade?IssofoioqueHerzberg

notouemsuapesquisa.

DeacordocomHerzberg,otrabalhodogestorseriaremoverosfatores

higiênicosqueestejamcriandoinsatisfaçãoecriarosfatoresmotivadoresparaque

seusfuncionáriosatinjamumaltograudedesempenho.

Essesfatorestambémpodemserchamadosdeintrínsecos(motivacionais,ou

relacionadoscomoconteúdodocargoeaofuncionárioemsi)eextrínsecos

(higiênicos,ourelacionadosaoambientedaempresa).

Vamosvercomopodesercobradoestetópico?

4.(Esaf/STN/Desenv.institucional/2008)SãocaracterísticasdasteoriasmotivacionaisdeMaslowe

Herzberg,respectivamente:

a)asnecessidadeshumanaspodemsercolocadasemumahierarquiadeimportância;eháumaalta

correlaçãoentreanecessidadederealizaçãoeodesenvolvimentoeficazdaorganização;

b)asnecessidadesmaisbásicasdohomemsãoassiológicaseasdesegurança;eosadministradores

comgrandepoderusamessespoderesmaisembenefíciosdesiprópriosquedaorganização;

c)asnecessidadesobedecemaumahierarquiaquepodeserinuenciadapelacultura;osfatores

motivadoresrelacionam-secomocontextodotrabalho;

d)umanecessidadesatisfeitanãomotivamaisocomportamento;eexistemfatoresquenãomotivam,

masquecontribuemparaainsatisfaçãosenãoestãopresentes;

e)amotivaçãogeragrandenecessidadedefeedbackparaaorganização;eumhomemmotivadodeixade

seimportarcomaspequenasdificuldadesdodiaadia,aindaquesejamperenes.

AalternativaquecorretamenteserefereàsduasteoriaséaletraD.Deacordo

comateoriadeMaslow,umanecessidadequejáfoisatisfeitanãoémotivadora.E,

deacordocomHerzberg,existemfatoresquenãosãomotivadores(higiênicos),

masquepodemtrazerinsatisfaçãosenãoestiverempresentes(bomsalário,por

exemplo).OgabaritoémesmoaletraB.

5.(FCC/TRF5aRegião/Anal.adm./2008)PelaTeoriadaMotivaçãodeHerzberg,estilodesupervisão,

políticasempresariais,condiçõesambientais,relaçõesinterpessoais,status,remuneraçãoe

vidapessoalsãochamadosfatores:

a)deautoestima;

b)motivacionais;

c)fisiológicos;

d)higiênicos;

e)depoder.

Comoacabamosdever,osfatoresligadosaomeioexternocomooambientede

trabalho,osalário,asrelaçõespessoais,entreoutros,fazempartedosfatores

higiênicos.

Osfatoresmotivacionaissãorelacionados,paraHerzberg,comosfatores

internosdapessoa,comosuarealização,seuinteressepeloconteúdodotrabalho,

seureconhecimentoetc.OgabaritoéaalternativaD.

6.(Cespe/Unipampa/Administrador/2009)Segundoateoriadosdoisfatores,propostapor

Herzberg,aoconcederaumentonaremuneraçãodeseuscolaboradores,oadministrador

atuaráemumfatormotivacional.

EstaéapegadinhamaisrecorrenteemconcursosdeAdministração!Prestem

atenção:osalárionãoéfatormotivadorsegundoHerzberg,esimfator

higiênico.

Seseusalárioformuitobaixo,lhetraráinsatisfação,masseseusalárioforbom

nãoomotivaráporsisó!Fatoresmotivacionaisseriam:oconteúdodotrabalho,a

possibilidadedecrescimentoeaprendizado,oreconhecimentoetc.Ogabaritoé

questãoincorreta.

5.1.4.TeoriadoReforço

Essateoriaarmaqueoreforçocondicionaocomportamento.Ouseja,os

indivíduospodemsermanipuladosasecomportaremdecertamaneira,deacordo

comosestímulosaplicadosaeles.7

Assim,sequeremosqueosfuncionárioscheguemsemprecedo,deveremos

instituirumprêmiodeassiduidade.Sequisermosqueosvendedoresatinjamas

metas,deveremospagarumbônusaosqueatingirem.Essateoria,comojádevem

estarpensando,éumadasmaisutilizadasnomeioorganizacional.

Ateoriadoreforçonãotomaconhecimentodoquesepassano“interior”da

pessoa(comoasemoções,expectativas,atitudesetc.),apenasdoqueacontececom

oindivíduoquandoeleage.8

Assim,essateoriaémuitocriticadapeloseuaspecto“manipulador”daspessoas.

Apesardisso,seuimpactonãoérenegado.Apenasnãoseacreditamaisqueo

reforçosejaoúnicofatorquegeraumamaiormotivaçãoe,porconsequência,um

melhordesempenho.

CaracterísticasdaTeoriadoReforço

Buscaentendercomoasconsequências

Arelaçãoentreocomportamentoesuas

Oreforçoéumatentativade

doscomportamentosanteriores

consequênciassegueumaideiade

causararepetiçãoouainibiçãode

influenciamasaçõesfuturas

aprendizagemcíclica

umcomportamento

Figura5.6–CaracterísticasdaTeoriadoReforço

Deacordocomateoria,asferramentasqueumgestordepessoaspoderia

utilizaririamdesdeumestímulopositivo(umprêmio,porexemplo)atéum

estímulonegativo(umapunição).

Estímulospositivostenderiamareforçarocomportamento,aopassoqueos

estímulosnegativosbuscariamanularumcomportamentoindesejado.As

estratégiasseriamquatro:reforçopositivo,reforçonegativo,puniçãoeextinção.

Figura5.7–Estratégiasdemodificaçãodocomportamento

5.1.5.TeoriaERC(ouERG)–Alderfer

Basicamente,essaéumaadaptaçãodateoriadahierarquiadasnecessidadesde

Maslow.AlderferprocurouadequarosestudosdeMaslowparaqueateoria

pudesserefletirosdadosempíricos–osdadosdaspesquisas.9

AprimeiradiferençaentreasduasteoriaséofatodequeAlderfer“condensou”

osníveishierárquicos.Paraele,sãoapenastrêsosníveisdenecessidades:de

existência,derelacionamentoedecrescimento.

Figura5.8–HierarquiadeAlderfer

Oprimeironível–existência–englobaosprimeirosníveisdeMaslow

(siológicoesegurança).Jáosegundonível–relacionamento–englobaonível

socialdeMaslowealgunsfatoresexternosdoníveldeestima.Finalmente,onível

decrescimentoenglobaoscomponentesinternosdeestimaeonívelde

autorrealizaçãodeMaslow.

Asegundadiferençaentreasteoriasestánofatodeque,paraAlderfer,não

existeumahierarquiatãorígidaentreosníveisdenecessidadesquantoMaslow

acreditava.

NateoriaERC,váriosníveisdenecessidadespodemestarsendoestimuladosao

mesmotempo.Ouseja,asatisfaçãodeumnívelanteriornãoseriaum“pré-

requisito”paraqueonívelseguintesemanifestasse.

Alémdisso,nateoriaERCexisteanoçãodeque,seumníveldenecessidade

superiornãoforatendido,issopodelevarapessoaaaumentaranecessidadede

nívelinferior.

Assim,umapessoaquenãoconseguissestatusereconhecimentoemseu

trabalhopoderiaaumentarademandapormelhoressalários,porexemplo.

5.1.6.TeoriadaExpectânciadeVroom

AteoriademotivaçãoconsideradamaiscompletaatéomomentoéaTeoriada

ExpectânciadeVictorVroom.10ATeoriadaExpectânciadizqueamotivaçãoé

umprodutodasexpectativasdaspessoasemrelaçãoasuashabilidadesdeatingir

osresultadoseovalorqueelasdãoàsrecompensas.

Ouseja,oindivíduoprimeiroanalisaseametaqueaempresaestipuloué

possíveldeseratingidacomseusprópriosesforços.Seametaforinalcançável,ou

nãodependerdoseutrabalho,nãoirágerarmotivação!

Apósisso,eleanalisasearecompensaédesejável,poispodeserqueo“prêmio”

nãosejaatrativoparatodasaspessoasdamesmaforma.

Portanto,Vroomlevouemconsideraçãoasdiferençaspessoais,poiscada

indivíduotemumahabilidadeespecíficaedesejosenecessidadesdiferentes.

Naguraaseguirpodemosverumesquemaresumidodosfatoresenvolvidos.O

indivíduonaturalmentesefazestesquestionamentos:trabalharmaistardeànoite

faráametaserbatida?Seametaforalcançada,ganhareiumprêmiointeressante?

Figura5.9–TeoriadaExpectância

Vamosveralgumasquestõessobreessetema?

7.(FCC/Metrô/Administração/2008)Noprocessodegestãodasorganizações,aabordagemque

partedoprincípiodequeaspessoassãomotivadasparaarealizaçãodesuasatividades,

esperandoquecertasaçõesauxiliarãoaalcançarosresultadosdesejados,refere-seàteoria:

a)dahierarquiadenecessidades;

b)XeY;

c)daexpectância;

d)comportamental;

e)daadministraçãoporobjetivos.

Comovimos,ateoriamotivacionalqueabordaofatodeaspessoasanalisarema

expectativadequecertasações(ouesforços)possamounãolevararesultadose

premiaçõeséaTeoriadaExpectânciadeVictorVroom.

AteoriaXeYnãoabordaesseaspecto,muitomenosaTeoriadaHierarquiadas

NecessidadesdeMaslow.AalternativaDtambémestáincorreta,poisnãoaborda

asexpectativas,eaalternativaEnãoéumateoriamotivacional.Ogabaritoé

mesmoaletraC.

8.(Cespe/MTE/Administração/2008)SegundoomodelodeVroom,amotivaçãoparaproduzirem

umaentidadeestácalcadaestritamentenasrecompensasofertadaspelaorganização.

NomodelocontingencialdeVroom,amotivaçãoéfunçãodetrêsfatores

determinantes:asexpectativas,asrecompensasearelaçãoentreasexpectativase

asrecompensas.Ogabaritoéquestãoerrada.

5.1.7.TeoriadasNecessidadesAdquiridas,deMcClelland

DeacordocomMcClelland,amotivaçãoérelacionadacomasatisfaçãode

certasnecessidadesadquiridasdosindivíduos.Paraele,essasnecessidadesseriam

três:11

ØNecessidadedeafiliação–relativasaodesejodeterbonsrelacionamentose

amizades.

ØNecessidadedepoder–ligadasaocontroleeàinuênciadeoutraspessoas

eemrelaçãoaosdestinosdaorganização.

ØNecessidadederealização–ligadaaosdesejosdesucesso,defazerbem

algumtrabalho,desediferenciardosoutros.

Essasnecessidadesseriamgeradasatravésdaprópriaexperiênciadaspessoas,de

suavivência.Deacordocomoautor,pessoascomumaaltanecessidadede

realizaçãodeveriamtrabalharcomtarefasemquenãonecessitassemdotrabalho

dosoutros(ouseja,emtarefasemquepudessem“mostrarserviço”sozinhas).

Alémdisso,essaspessoassãomaisbemaproveitadasemáreasemqueastarefas

sãodifíceisobastanteparamotivá-las,masnãotantoqueasfaçamperceberqueo

sucessodependeda“sorte”oudaajudadeoutros.Assim,essaspessoasnão

costumamserbonsgerentes.

Jáaspessoascomumaaltanecessidadedepoderseadaptammelhoràsposições

degerência.DeacordocomMcClelland,pesquisascomprovamqueagrande

maioriadosocupantesdecargosaltostemaltanecessidadedepoderebaixa

necessidadedeafiliação.

Vamosveragoraumaquestãodessetema?

9.(Cespe/ABIN/Ocialtécnico/2010)Asorganizaçõesmodernasdevemreduzirosobjetivosiniciais

parasatisfazerasnecessidadesadquiridas,enãoajustaroatendimentodasnecessidades

adquiridasaosseusobjetivosiniciais.

PelaTeoriadasNecessidadesAdquiridas,deMcClelland,amotivaçãoé

relacionadacomasatisfaçãodesuasnecessidadesadquiridas.Essasnecessidades

seriamgeradasatravésdaprópriaexperiênciadaspessoas,desuavivência.

Paraoautor,asnecessidadesseriamtrês:necessidadedealiação(serelacionar

bemcomosoutros),depoder(controlareinuenciarosdestinosdaorganização)

ederealização(atingirdeterminadosobjetivos).

Nãofazsentidodizerqueosobjetivosdevemserreduzidosparase“encaixar”

nasnecessidadesadquiridas,poisamotivaçãoéquedevebuscarharmonizaros

objetivosindividuaiscomosorganizacionais.Ogabaritoéquestãoerrada.

5.1.8.TeoriadaEquidade

Essateoriaarmaqueaequidade,ouseja,apercepçãodequeoqueganhamos

estáemlinhacomoqueoferecemosemtroca(eemrelaçãoaosoutros),éum

aspectomotivador.Assim,anoçãodequeessarelaçãoéjustateriaumimpacto

significativonamotivação.

DeacordocomStacyAdams,todosnósfazemosumacomparaçãoentreoque

“entregamos”eoque“recebemos”emtrocadaempresa.Sepensarmosque

estamossendofavorecidos,nossentiremosculpados.Sesentirmosqueestamos

sendodesfavorecidos(recebendomenosdoqueentregamos),teremosraiva.12

Alémdisso,comparamostambémessa“relação”detrocacomosnossoscolegas

detrabalhoeatédepessoasquetrabalhamemoutrasempresase/ouprossões.

Dessamaneira,sesentimosquearelaçãodetrocanãotemequidade,tomaremos

alguma“providência”pararesolveressainequidade.

DeacordocomAdams,existemseispossibilidadesdeaçãodiantedeuma

inequidade:

1)Mudançanas“entregas”,ouseja,passaratrabalharmenos.

2)Mudançasnosresultados–ocorrequandopessoasqueganhamporprodução

começamaproduzirmaiscommenosqualidade(ouseja,na“pressa”).

3)Distorçãonasuapercepção–oindivíduopodecomeçaramudarsuaideia

sobresimesmo(“achavaquetrabalhavapouco,masvendoSicranotrabalhando

jácomeçoaacharquesoumuitotrabalhador”).

4)Distorçãonapercepçãodosoutros–oindivíduopassaaacharqueaposição

dosoutroséquenãoésatisfatória.

5)Mudançanoreferente–seapessoaaquemnoscomparamosestáemsituação

melhor,podemospassaranoscompararcomalguémqueestápiordoquenós

mesmos.

6)Sairdo“jogo”–porexemplo:sairdoempregoatual.

Vamosverumaquestãodessetema?

10.(Cespe/PolíciaFederal/Agente/2009)AoconsideraraTeoriadaEquidadecomoformade

maximizarodesempenhodoscolaboradoresquelheestãosubordinados,ogestorpúblico

develevaremcontaquequalquerinjustiçasentidaemrelaçãoàsrecompensaspodemotivar

essaspessoasaagiremdeformaarestaurarosensodeequidadepercebida.

Beleza.ATeoriadaEquidadeseconcentraexatamentenapercepçãoqueas

pessoastêmdaequidadenarelaçãoesforço/recompensa.Seessarelaçãoforvista

comoinjusta,ofuncionáriopoderáagirpara“restaurar”aequidade,comopassar

atrabalharmenos,porexemplo.Ogabaritoémesmoquestãocorreta.

5.1.9.TeoriadoEstabelecimentodeObjetivos(Autoeficácia)

Seráquefuncionamosmelhorquandoprocuramos“fazeronossomelhor”ou

quandotemosumametaespecíca?DeacordocomLocke,13aintençãodeatingir

umobjetivoéumgrandefatormotivador.

Ouseja,quandotemosumametaemmente,eaceitamosessameta,tendemosa

conseguirresultadosmelhoresdoquequandoapenas“tentamosonossomelhor”.

Alémdisso,quantomaisdifícilameta,melhorseráonossodesempenho(desde

que,obviamente,aceitemosameta,ouseja,realmentetentemosatingi-la).

Outrofatorimportanteéaretroação.Sesoubermoscomoestamosnossaindo,

dizateoria,tenderemosaobtermelhoresresultados.Assim,aretroaçãoafetariao

desempenho.Esseéumfatobastanteintuitivo,nãoémesmo?

Locketambémcitacomoumfatormotivadoraautoecácia.Deacordocomo

autor,essacaracterísticaseriaahabilidadequeaspessoaspodemterdeacreditar

queserãocapazesdeatingirosresultadosdeumaatividade.

Serealmenteacreditamosemnossahabilidadederealizarumatarefa,ateoria

arma,tenderemosanosmotivareconseguirmelhoresresultados.Indivíduos

comumaltoníveldeautoecáciatenderãoaterresultadosmelhoresdoque

pessoascombaixoníveldeautoeficácia.

DeacordocomYassuda,etal.,14

ATeoriadaAutoecáciaprevêqueoníveldeconançadoindivíduoemsuashabilidadeséumforte

motivadorereguladordeseuscomportamentos.Banduradefendequeoindivíduoquesepercebe

capazderealizarumadeterminadatarefafazmaioresforçopararealizá-la,temmaiormotivação

paraconcluí-laeperseveramaistemponasuarealizaçãodoqueoindivíduocombaixaautoeficácia.

5.2.“Empoderamento”ouEmpowerment

Oempowerment,ou“empoderamento”,éoprocessodefortalecimentodopapel

daspessoasedasequipesdentrodeumaorganização.Dessemodo,busca-se

ampliarasresponsabilidadeseautoridadesparaqueosprossionaispossamtomar

suasdecisõeseagir.15

Assimsendo,oempowermentestádiretamenterelacionadocomadelegaçãoea

descentralizaçãodepoderdentrodaempresa.16Aideiaportrásdesseconceitoéa

dequeofuncionárioquelidacomoproblemadiretamenteterámelhores

condiçõesdetomarumadecisãoacertadae,principalmente,notemponecessário.

Portanto,aempresaquedecideimplantaroempowermentdevedelegarpoder

aosfuncionárioseservidoresquetrabalhamdiretamentenochãodefábrica,ou

seja,queestãonabasedapirâmide.

Dessaforma,elesterãomaisautoridadeeresponsabilidadenosassuntos

operacionaiscomquelidamdiariamente.

Atualmente,oempowermentvemsendomaisutilizadonasempresaspordois

motivosprincipais:oprimeiroéanecessidade,noambientedinâmicoe

competitivoemquevivemos,detomarasdecisõesdemodomaisrápido.

Osegundoéqueoprocessodedownsizingdosanos1980e1990deixouas

empresascomumaestruturamaisachatada,portantoogerentenãoteve

alternativaanãoserdelegarpoderaosseussubordinados.17

Entretanto,oempowermentnãofuncionaemtodasasorganizações,nemem

todosossetores.Paraqueosfuncionáriospossamreceberessasautoridadese

responsabilidadesmaiores,énecessárioumesforçodecapacitaçãoetreinamento.

Portanto,umprocessodeempowermentdeveseriniciadocomuma

preocupaçãodeprepararessesfuncionáriosparatomarmaisdecisõeseagir

commaisliberdade.

ParaChiavenato,18oempowermentaumentaocontroledentrodaorganização,

incrementandoaautoestimadaspessoaseaumentandoaqualidadedentroda

organização.

Assim,oempowermentbuscacaptaraenergia,oesforçoeacriatividadede

todososmembrosdaempresaecanalizá-lasparaosobjetivosorganizacionais.De

acordocomKanter,19existemnoveprincípiosparaoempowermentfuncionar:

1)Daràspessoasumtrabalhoemqueelassesintamimportantes.

2)Daràspessoasplenaautoridadeeresponsabilidade,independênciaeautonomia

emsuastarefaserecursos.

3)Permitirqueaspessoastomemdecisõesarespeitodeseutrabalho.

4)Darvisibilidadeàspessoaseproporcionarreconhecimentopelosseusesforçose

resultados.

5)Construirrelacionamentosentreaspessoas,ligando-ascompessoasmais

importanteseapoiando-asatravésdelídereseimpulsionadores.

6)Moverainformaçãoemtodososníveis.Informaçãoépoderehabilitaas

pessoasapensareaagirmelhor.

7)Pediraopiniãodaspessoasarespeitodosassuntosdetrabalho.Fazercomque

elassesintamdonasdosprocessosdetrabalho.Fazercomqueelastenham

orgulhodepertenceràorganização.

8)Acentuaracolaboraçãoeoespíritodeequipe.Empoderarpessoaséempoderar

equipes.

9)Ajudaraspessoasempoderadasaempoderarasdemais.

Vamosverumaquestãosobreessetema?

11.(Cespe/Hemobras/Administrador/2008)Motivaçãoconsisteemforçasqueimpulsioname

sustentamosesforçosdeumapessoaàrealizaçãodealgo.Nessesentido,oempowermentse

caracterizacomoumapráticamotivacional.

Correto.Oempowerment(ouempoderamento)éconsiderado,sim,umfator

motivadordosindivíduosnotrabalho.Estáligadoaoprocessodefortalecimento

dopapeldaspessoaseequipesdentrodaempresa.Ogabaritoéquestãocorreta.

QuestõesPropostas

12.(FCC–MP/RSAdministrador/2008)SegundoaTeoriadaHierarquiadasNecessidadesdeMaslow

éINCORRETOdizer:

a)todapessoaorientaseucomportamentoapartirdemaisqueumúnicotipodemotivação;

b)apenasalgumaspessoasalcançamasatisfaçãodasnecessidadeslocalizadasnotopodapirâmide;

c)asatisfaçãodeumnívelinferiordenecessidadesnãoéobrigatóriaparaquesurjaimediatamenteum

nívelmaiselevadonocomportamento;

d)asnecessidadesfundamentaispodemserexpressaspordiferentestiposdecomportamento;

e)todanecessidadeprimárianãoatendidapassaaserconsideradaumaameaçapsicológica.

13.(Cespe/Unipampa/Administrador/2009)Casooadministradorpretendamotivarsuaequipe

baseando-senateoriademotivaçãochamadahierarquiadasnecessidades,aprimeira

providênciaasertomadadeveráseraadoçãodeumprogramaquevisepreservare

desenvolverasrelaçõessociaisnogrupodeempregados.

14.(FCC/MP/SE/Administrador/2009)SegundoHerzberg,queressaltouaimportânciadamotivação

notrabalho,apossibilidadedeaumentodestatusoumesmodeposiçãosocialéuma

determinantemotivacionalassociada:

a)aodesenvolvimentopessoal;

b)àrealização;

c)àpossibilidadedecrescimento;

d)aotrabalhoemsi;

e)àresponsabilidade.

15.(Esaf/MPOG/EPPGG/2009)Sobreotema“motivação,climaecultura”,podemosarmar

corretamenteque:

a)umfuncionáriosatisfeitotambémestá,necessariamente,motivado;

b)análisesdeclimanãoseprestamparaorientarpolíticasderecursoshumanos;

c)culturaeclimaorganizacionalsãovariáveisindependentesentresi,quenãosecomunicam;

d)deumaformageral,oselementosdaculturapodemseralteradosnocurtoprazo;

e)osalário,isoladamente,nãoseconstituiemumfatormotivacional.

16.(FCC/ARCE/Analistareg./2006)ParaHerzberg,aspessoastêmcategoriasdiferentesde

necessidades.Políticaeadministração,relaçõesinterpessoaisesegurançasãodenominados

fatores:

a)sociais;

b)motivadores;

c)fisiológicos;

d)dehigiene;

e)deestima.

17.(Cespe/MTE/Administração/2008)SegundoateoriapropostaporHezberg,oaumentosalarial

dadoaumacategoriadeservidorespúblicos,porexemplo,nãoseriaumfatormotivacional.

18.(Cespe/MPS–Administrador/2010)AteoriadeMaslowcitaasnecessidadeshumanascomouma

pirâmide,fazendoumparalelocomumahierarquia.Nabasedessapirâmide,encontram-seas

necessidadesbásicasousiológicaseopressupostoé:umanecessidade,emqualquerponto

dahierarquia,precisaseratendidaantesqueanecessidadedonívelseguintesemanifeste.

19.(Cespe/Ceturb-ES/Administrador/2010)Aoportunidadedeliderarumaequiperepresenta,

muitasvezes,umfatordemotivaçãoparaocolaborador,queconsideraatarefacomoumvoto

deconfiançadosseussuperiores.

20.(Cespe/Basa/Administração/2010)Seogerentedeumaagênciabancáriacom35funcionários

doquadro,10terceirizadose5estagiáriosadotarumaposturaúnicacomoformademotivar

seuscolaboradores,essacondutaestarádeacordocomospressupostosdaTeoriadeMaslow.

21.(Cespe/Basa/Administração/2010)Apreocupaçãodogerentedeumaagênciabancáriacoma

salubridadedoambientedetrabalhoidentica-secomosaspectosmotivacionaispropostos

naTeoriadeHerzberg.

22.(Cespe/Basa/Administração/2010)Aimplantaçãodeumnovoplanoderemuneraçãoque

contempleumaumentosubstancialnosaláriodoscolaboradoresdeumaorganizaçãovaiao

encontrodosfatoresextrínsecospropostosnaTeoriadeHerzberg.

23.(Cespe/TCU/ACEGestãodePessoas–2008)ConformeaTeoriadaHierarquiadasNecessidades

deMaslow,porseremasnecessidadesdabasedapirâmidedehierarquiasasúnicasque

realmenteproduzemmotivaçãoparaotrabalho,asrecompensasoferecidasaosempregados

devemtersidodenaturezamonetária.

24.(Cespe/STM/Anal.jud.esp.adm.–2011)ATeoriadaEquidadeeaTeoriadaExpectativapodem

serconsideradasteoriasdeconteúdodamotivaçãonotrabalho.

25.(Cespe/Inca/GestãoRH/2010)Ateoriamotivacionaldaequidadereconhecequeosindivíduos

julgamaquantidadeabsolutadesuasrecompensasorganizacionaisnãosópelosseusesforços,

mastambémpelarecompensaobtidapelosoutrosemfacedosinsumosporelesempregados.

26.(Cespe/Inca/GestãoRH/2010)Considerequeumgestordepesquisasdedeterminadoórgão

governamentaldaáreadesaúde,emvezdeseesforçarparasolucionarproblemasedenir

novasmetasmaisdesaadoras,idealizaserinuenteecontrolarosoutrosintegrantesdasua

unidade.Asuapreferênciaéatuaremsituaçõescompetitivasvoltadasparaostatus,eelese

preocupamaiscomoprestígiodecorrentedosresultadosedainuênciasobreosoutros.Com

basenaTeoriadasNecessidadesdeMcClelland,esseindivíduopossuianecessidadede

afiliaçãocomoamaispreponderante.

27.(Cespe/Inca/GestãoRH/2010)Amotivaçãodeumindivíduoinseridoemumaorganizaçãotem

relaçãodiretacomaintensidadedeesforçosqueeleemprega,porissoindivíduosmotivados

geramresultadosfavoráveisparaaorganização.

28.(Cespe/STM/Anal.adm./2011)Épossívelarmarquequantomaiorforamotivaçãodeum

funcionárioparaotrabalho,tantomelhorseráoseudesempenhoemdeterminadocontexto

laboral.

29.(Cesgranrio/Petrobras/AdministradorJR/2010)Umadiretoraexecutivadeumamultinacional

brasileiratrabalhamaisde12horasdiárias.Constantemente,visitaassubsidiáriasemdiversos

paísesparaacompanharodesenvolvimentodasestratégias.Aaltadireçãoestá

desenvolvendoumplanodereestruturaçãodasuadiretoriaparaqueelapossaviajarmenos,

trabalharumnúmeroinferiordehorasesededicarmaisaseusprojetospessoaisefamiliares.

Aexecutivanãoaceitouaproposta,sugerindoqueoutraspessoasnãoconseguiriamrealizar

suasfunçõesexecutivasadequadamente.Adicionalmente,armouquenãopoderiaabrirmão

desuaposiçãopara,emtroca,sededicaraosseusprojetospessoaisefamiliares.Conclui-se

quesuamotivaçãoestánasatisfaçãodenecessidades:

a)básicas;

b)desegurança;

c)departicipação;

d)deestima;

e)deautorrealização.

30.(Cesgranrio/Funasa/Administrador//Administrador/2009)Umadasteoriassobremotivaçãomais

conhecidaséadesenvolvidaporMaslow.Paraele,épossívelordenarasnecessidadeshumanas

emumahierarquiadeinfluenciaçãoeimportância,quecontémascaracterísticasaseguir.

I.Necessidadessociais.

II.Necessidadesfisiológicas.

III.Necessidadesdeestima.

IV.Necessidadesdesegurança.

V.Necessidadesdeautorrealização.

Asequênciacorretadessahierarquiaé:

a)I,I,V,IVeII;

b)I,II,IV,VeI;

c)I,IV,I,IIeV;

d)IV,V,I,IeII;

e)V,I,II,IeIV.

31.(Cesgranrio/Termomacaé/Administradorjr/2009)Ateoriamotivacionalmaisconhecidaéade

Maslow,baseadanahierarquiadasnecessidadeshumanas.Paraele,asnecessidadesestão

arranjadasemumapirâmidedeimportânciaeinuenciaçãodocomportamentohumano,na

qualsedestacaadeestima,queenvolvenecessidadesque:

a)orientamavidahumanadesdeomomentodonascimento,comoaalimentação,osono,orepouso,o

abrigoetc.;

b)levamapessoaaseprotegerdequalquerperigorealouimaginário,físicoouabstrato;

c)levamcadapessoaarealizaroseuprópriopotencialebuscarsedesenvolvercontinuamenteaolongo

davida;

d)serelacionamàassociaçãodoindivíduoaoutraspessoas,vinculadasàparticipação,àaceitaçãopor

partedoscolegas,àtrocadeamizadeeamor;

e)serelacionamàmaneiracomoapessoasevêeseavalia,compreendendoaautoapreciação,a

autoconfiança,anecessidadedereconhecimento,deprestígioedeconsideração.

32.(FGV/Senado/Administrador/2008)AteoriamotivacionalmaisconhecidaéadeAbrahamHarold

Maslow.Elasebaseianahierarquiadenecessidadeshumanas.Entreessas,segundooautor,há

asnecessidades:

a)patrimoniais;

b)financeiras;

c)fisiológicas;

d)psicológicas;

e)morais.

33.(Cesgranrio/Termoaçu/Administradorjr/2008)Asteoriassobremotivaçãotratamdediferentes

variáveisderesultados.Enquantoalgumassãovoltadasàexplicaçãodarotatividade,outras

enfatizamaprodutividade.Ateoriaque,emboranãoofereçamuitaexplicaçãosobrea

satisfaçãodosfuncionários,nemsobresuadecisãodedeixaraempresa,éútilnopapelde

previsoradefatorestaiscomoqualidadeequantidadedetrabalho,persistênciadeesforço,

absenteísmo,impontualidadeetaxasdeacidenteséa:

a)doreforço;

b)daequidade;

c)daexpectativa;

d)dasnecessidades;

e)dafixaçãodosobjetivos.

34.(FGV/Sefaz/RJ–Fiscalderendas/2007)Nateoriaepráticadaadministração,amotivaçãoparao

trabalhoéumaáreadegrandepreocupação,umavezquepodeafetarentusiasmo,dedicação,

cooperaçãoeprodutividade.Considerandoasteoriasclássicasecontemporâneassobre

motivação,assinaleaalternativacorreta.

a)Níveismínimosdefatoreshigiênicos,comostatusesalário,sãoimportantes,mas,quandopresentes,

nãocausamsatisfação,apenasimpedeminsatisfação.

b)SegundoateoriaX,existempessoasquesãopredispostasaotrabalhoepessoasquesãoindolentes,

devendoaorganizaçãobuscaraspessoasinteressadaspelotrabalho.

c)Navisãocontemporânea,astarefasdevemserdenidaspormeiodaespecializaçãorígida,compouca

amplitudeeflexibilidade.

d)Paraqueumindivíduoadquiraumcomportamentomotivado,oobjetivotemdeserporelevalorado,

emdetrimentodarecompensa.

e)Aautonomiadepensareagirdoindivíduonãopodeserutilizadacomofontedemotivação,poisas

tarefasprecisamterseusprocedimentospadronizados.

35.(Esaf/AFC/CGUDesen.inst./2008)Amotivaçãoéalgopresenteemnossavidadesdeomomento

emquenascemos,eseuentendimentotem-seconstituídoumdesaoparaaspessoasque

ocupamfunçõesgerenciais.Assinaleseasfrasesaseguirsobremotivaçãosãoverdadeirasou

falsas.

I.Pormotivaçãoseentendeumaforçaintrínsecaqueimpulsionanadireçãodealgumacoisa.

II.Estímulossãoenergiasinternasquedeterminamocomportamentodosindivíduos.

III.Ateoriademotivaçãoconhecidacomoteoriadaexpectativarelacionadesempenhocom

recompensa.

IV.ATeoriadeMotivaçãodeMaslowéconhecidacomoteoriadasnecessidadeshumanas.

V.Asmotivaçõesparaotrabalhosãoiguaisparaindivíduosqueexercemcargosdomesmo

grupoocupacional.

Escolhaaopçãocorreta.

a)I,IeIIsãoverdadeiras.

b)II,IVeVsãofalsas.

c)IeVsãoverdadeiras.

d)I,IIeIVsãoverdadeiras.

e)I,IIeIVsãofalsas.

36.(FGV/Fiocruz/Analistadegestão/2010)Comrelaçãoaosfatoresmotivacionais(intrínsecosao

cargo),apartirdateoriadosdoisfatorespropostaporFrederickHerzberg,pode-searmar

comosendoseuscomponentes:

I.Condiçõesgeraisdoambientelaboralcomoiluminação,limpeza,ruído,remuneraçãoe

relaçõescomsuperioresecolegas.

II.Nívelderesponsabilidade,conteúdoeatribuiçõesdocargo.

III.Nívelderesponsabilidadedocargo,níveldereconhecimentodotrabalhoexecutado.

Assinale:

a)sesomenteaafirmativaIestivercorreta.

b)sesomenteaafirmativaIestivercorreta.

c)sesomenteaafirmativaIIestivercorreta.

d)sesomenteasafirmativasIeIIestiveremcorretas.

e)sesomenteasafirmativasIeIestiveremcorretas.

37.(Funrio/MPOG/Analistaadm./2009)Amotivaçãopodeserdenidacomoodesejodeexercer

altosníveisdeesforçoemdireçãoadeterminadosobjetivos.Ateoriadamotivaçãoproposta

porHerzbergéconhecidacomoateoriadosfatoressatisfacienteseinsatisfacientes.NÃO

consisteemumfatorsatisfaciente:

a)arealizaçãopessoal;

b)oprogressoprofissional;

c)ascondiçõesdetrabalho;

d)otrabalhoemsi;

e)aresponsabilidade.

38.(FCC/MPE-PE/Técnico/2012)ATeoriadaHierarquiadasNecessidadespartedoprincípiodeque

aspessoassãomotivadascontinuamentepelasatisfaçãodesuasnecessidades,queobedecem

aumahierarquia.Asnecessidadesquetêmrelaçãocomaspossibilidadesdedesenvolvimento

dascapacidadesetalentosdaspessoassãoconhecidaspor:

a)autorrealização;

b)estimaeprestígio;

c)sociais;

d)segurança;

e)fisiológicasedesobrevivência.

39.(FCC/TRF2a/Analista/2012)Dentreasteoriasdamotivação,aquelaque,numaprimeiravisão,

sugerequeosgerentesdevemcoagir,controlareameaçarosfuncionáriosamdemotivá-los

e,numasegundavisão,acreditaqueaspessoassãocapazesdeserresponsáveis,nãoprecisam

sercoagidasoucontroladasparaterumbomdesempenho,éateoria:

a)damotivaçãoehigiene;

b)dahierarquiadasnecessidades;

c)XeY;

d)dosmotivoshumanos;

e)doreforçopositivoedeaversão.

40.(FGV/TRE-PA/Técnico/2012)SegundoaHierarquiadasNecessidadesdeMaslow,aamizadedos

colegasnotrabalhoéumanecessidadedotipo:

a)fisiológica;

b)desegurança;

c)social;

d)deestima;

e)deautorrealização.

Gabaritos

1.B

15.E

29.D

2.C

16.D

30.C

3.C

17.C

31.E

4.B

18.C

32.C

5.D

19.C

33.A

6.E

20.E

34.A

7.C

21.E

35.D

8.E

22.C

36.D

9.E

23.E

37.C

10.C

24.E

38.A

11.C

25.C

39.C

12.C

26.E

40.C

13.E

27.E

14.A

28.E

Bibliografia

Bergamini,CecíliaW.Motivação:mitos,crençasemal-entendidos.Revistade

AdministraçãodeEmpresas,Abr./Jun.1990:23-34.

Chiavenato,Idalberto.AdministraçãoGeralePública.2aed.RiodeJaneiro:

Elsevier,2008.

_______.Administraçãonosnovostempos.2aed.RiodeJaneiro:Elsevier,2010.

_______.GestãodePessoas:eonovopapeldosrecursoshumanosnasorganizações.2a

ed.RiodeJaneiro:Elsevier,2004.

Clegg,Stewart,MartinKornbergereTyronePitsis.Managing&Organizations:An

introductiontotheoryandpractice.ThousandOaks:Sage,2008.

Daft,RichardL.Management.Mason:Thomson,2005.

Robbins,StephenP.OrganizationalBehavior.11aed.UpperSaddleRiver:Pearson-

Prentice-Hall,2004.

Robbins,StephenP.,eMaryCoulter.Administração.5aed.RiodeJaneiro:

Prentice-Hall,1998.

Yassuda,MonicaSanches,ValeriaBelliniLasca,eAnitaLiberalessoNeri.Meta-

memóriaeautoeficácia:umestudodevalidaçãodeinstrumentosdepesquisasobre

memóriaeenvelhecimento.Psicologia:reflexãoecríticaV.18,n.1(Jan/Abr2005).

Capítulo6

Liderança

Oconceitodeliderançaérelacionadocomautilizaçãodopoderpara

inuenciarocomportamentodeoutraspessoas.1Obomlíderdevebuscarse

comunicarcomseussubordinadoseguiá-losemdireçãoaosobjetivosda

organização.

Dessaforma,umdosaspectosmaisimportantesparaumgestoréacapacidade

deliderarseuscolaboradores,poisaliderançaenvolveahabilidadede

influenciarpessoasparaquesejamalcançadososobjetivosdesejados.

Lideraré,basicamente,mostrarocaminhoaserseguido.Éincentivaraequipe

emdireçãoàsmetasquedevemseratingidas.Masliderarpessoasnãoéfácil.Cada

indivíduotemumamaneiradepensardiferente,experiênciasdiversas,traumas

etc.

Dessamaneira,acapacidadedeliderançademandadiversashabilidades

diferentes,comosabersecomunicar,terpaciência,manteroequilíbrioemocional

etc.

Portanto,aliderançaédinâmica.Éumaspectoqueenvolveorelacionamento

humanoeabordaousodopoderparaqueasmetassejamatingidas.ParaDa,2a

liderançapodeserdefinidacomo:

“Liderançaéahabilidadedeinfluenciarpessoasemdireçãoaoalcancedasmetasorganizacionais.”

JáChiavenato3defineliderançacomo:

umfenômenotipicamentesocialqueocorreexclusivamenteemgrupossociaisenasorganizações.

Aliderançaéexercidacomoumainuênciainterpessoalemumadadasituaçãoedirigidaatravésdo

processodecomunicaçãohumanaparaaconsecuçãodeumoumaisobjetivosespecíficos.

Umaspectomuitocobradoemconcursoséarelaçãoentrealiderançaeos

cargosdecheaougerência.Émuitocomumqueaspessoasliguemaliderançade

certaspessoasaocargodechefe(comoumgerente,umgeneral,umgovernante

etc.).

Entretanto,nãoénecessáriaaocupaçãodeumcargoparaqueumapessoa

tenhaumpapeldeliderançaemumgrupo.Eocontráriotambéméverdade.

Nãoésemprequeumchefefazessepapel(liderar)nasorganizações.

Muitaspessoasocupamcargosdechea,masnãotêmperlparaliderar

pessoas,nãosabemsecomunicar,nãosabemincentivaraspessoasetc.Dessa

forma,sãogerentes,masnãosãolíderes.

Figura6.1

Vamosvercomoessetemajáfoicobrado?

1.(Cespe/Hemobras/Administrador/2008)Oexercícioplenodaliderançadependedaocupaçãode

cargodechefiapelolíder.

Comovimos,nemsempreolíderéoocupantedocargodechea.Muitasvezes

quemacabaliderandoéumcolegamaisexperiente,maiscomunicadore

respeitado.Portanto,ogabaritoéquestãoerrada.

6.1.DiferençasentreLidereseGerentes

Umaempresanecessitatantodegerentescomodelíderes.DeacordocomDaft,4

osgerentescuidamdo“lugaremquevocêestáhoje”,enquantooslídereso

“levam”aumnovolugar.Assim,umgerenteteriaumavisãodecurtoprazoeuma

preocupaçãocomaeficiência(fazerascoisasbem-feitas,domodocorreto).

Jáumlíderteriaumavisãodelongoprazoeumapreocupaçãoemfazeracoisa

certa.Olíderseria,então,importanteparaqueaorganizaçãocontinueevoluindo

econsigaatingirosucessonofuturo.Umapessoapodeterumperlmaisligado

aosgerentesouaoslíderes,masidealmentedeveterqualidadesbalanceadas.

ParaZaleznik,5oslíderes,comoosartistas,toleramocaoseafaltadeestrutura.

Osgerentesbuscamaordem,ocontroleearápidaresoluçãodosproblemas.Para

oautor,asprincipaisdiferençasentreoslídereseosgerentessão:

Gerentes

Líderes

Tomamumaatitudeativaepessoalem

relaçãoaosobjetivos.

Tomamumaatitudeimpessoalepassivaemrelaçãoaos

Buscamnovasabordagensparaos

objetivos.

problemas.

Negociametomamatitudescoercitivas.

Procuramoriscoquandoas

Evitamorisco.

oportunidadessãointeressantes.

Preferemtrabalharcompessoas,mascompouco

Serelacionamcomaspessoasdemodo

envolvimentoemocional.

diretoecomempatia.

Focamemprocessos,no"como"asdecisõessãotomadas,

Focamnospontosimportantesdos

emvezde"quais"decisõesdevemsertomadas.

eventosedecisões.

Moldamasideiasemvezdereagira

elas.Alterampercepçõeseexpectativas.

Figura6.2

Vamosvercomoessepontojáfoicobrado?

2.(FCC/Defensoria/SP–Administrador/2010)Comrelaçãoàsdistinçõesentreliderançae

capacidadegerencial,considereasafirmativasabaixo.

I.Liderarenvolveumprocessodeinuêncianãocoercitivadecorrentedaautoridadeformaldo

cargoe,nestesentido,édistintodegerenciar.

II.Osgerentestendemaadotaratitudesimpessoais,àsvezesatépassivas,emrelaçãoàs

metas,enquantooslíderestêmumaatitudepessoaleativaemrelaçãoàsmetas.

III.Hojeolíderéocomponentemaisestratégicodaorganização,éoresponsávelpelo

desenvolvimentodeoutraspessoasepelosucessodaempresa.

IV.Obomgerenciamentotrazordemeconsistênciaaoseprojetarplanoseacompanharos

seusresultados;liderança,emcomparação,élidarcomamudança.

V.Ogerentetemumavisãodelongoprazo;olíder,perspectivadecurtoprazo.Ogerentefaz

ascoisascertas;olíderfazascoisasdireito.

EstácorretooqueseafirmaAPENASem:

a)IeII;

b)II,IVeV;

c)I,IeIV;

d)IeV;

e)I,I,IIeIV.

Aprimeirafraseestácorreta,poisquemutilizaacoerçãoéogerente,nãoo

líder.Asegundafrasetambémestácorreta,pois,paraZaleznik,osgerentessão

passivoseimpessoaisemrelaçãoàsmetas,aocontráriodosgerentes.

Aterceirafraseestácorreta,poisolíderrealmenteéfundamentalparaquea

empresaevoluaenodesenvolvimentodeoutraspessoas.Aquartafrasetambém

reetecorretamenteapreocupaçãodosgerentescomaordemeocurtoprazo.

Entretanto,aquintafraseestáincorreta,poisinverteascaracterísticasdoslíderes

egerentes.Sãooslíderesquesepreocupamemfazerascoisascertasenolongo

prazo.OgabaritoéaalternativaE.

6.2.AbordagensdeLiderança

Continuando,asabordagenstradicionaisarespeitodaliderançasão:ateoria

dostraçosdepersonalidade,dosestilosdeliderançaecontingenciaisou

situacionais.

Antigamente,osautoresimaginavamserpossívelaidenticaçãodecertos

“traços”depersonalidadenoslíderes,deformaapoderencontrarmais

facilmentelíderesempotencial.6

JáasTeoriasdeEstilodeLiderançaanalisavamoefeitodediversosestilosdos

líderesnodesempenhodasorganizaçõesenasatisfaçãodaspessoas.

Porm,asTeoriasContingenciaisouSituacionaislevamemcontadiversos

fatoresambientaisparadeterminarqualseriaomelhorestilodeliderançaemcada

situação.Portanto,sãoteoriasquemostramumaevoluçãoemrelaçãoàsteorias

baseadassomentenosestilosdeliderança.

6.3.TeoriadosTraçosdeLiderança

Comovimosantes,asprimeirasteoriasdeliderançaimaginavamserpossívela

identicaçãodecertos“traços”depersonalidadenoslíderes,deformaase

poderencontrarmaisfacilmentelíderesempotencial.Masoqueseriamesses

traços?Ostraçossãoosaspectospessoaisquedistinguemaspessoas,como

valores,inteligência,confiançaeaparência.

Dessaforma,seriamanalisadasascaracterísticas(pessoais,intelectuais,

emocionaisefísicas)delíderesconhecidos(Napoleão,ChurchillouJuscelino

Kubitschek,porexemplo)edepoissebuscariampessoasquetivessemasmesmas

característicasdesseslíderes.

Decertaforma,aideiaéadequepoderíamosanalisaropotencialdeuma

pessoacomolíderapenasanalisandosuascaracterísticaspessoais,semvê-laem

ação.

Oproblemaéqueessascaracterísticassãodedifícilmedição.Imagine-setendo

deavaliarquepessoadesuaconvivênciaémaisexível.Nãoseriamuitofácil,não

émesmo?A“exibilidade”nãoéumacaracterísticaquepodesermedida

objetivamente.Dependedaopiniãodecadaum.

Alémdisso,essateorianãolevavaemconsideraçãoocontextoemquecada

lídertinhasesobressaído.Ouseja,nãoanalisavaofatodequecaracterísticas

diferentesseriamimportantesemcadasituação.Umlíder“durão”poderiater

sucessonoExército,porexemplo,masenfrentarmuitasdiculdadesemuma

reitoriadeuniversidadepública.

Assimsendo,cadasituaçãoanalisada“pediria”umlíderdiferente.Citando

outrapossibilidade,umlídermaisenérgicopoderiasermuitobomemuma

situaçãodecrise,maspéssimoemumasituaçãodebonança.

6.4.EstilosdeLiderança

Umdostrabalhospioneirosnatentativademapearosestilosdeliderançafoio

deKurtLewinedeseusassistentesnaUniversidadedeIowa.7Osestilosmapeados

peloautorforam:autocrático,democráticoeliberal(oulaissez-faire,deixarfazer

emfrancês).

Noestiloautocrático,olídercentralizatodasasdecisões,sempedira

participaçãodosseussubordinados.Esselíderdeterminacomootrabalhoserá

feito,quemofará,quandoofaráetc.Aparticipaçãodosfuncionáriosnoprocesso

detomadadedecisãoémuitolimitada!

Jáoestilodemocráticocaracteriza-sepeladelegaçãodeautoridadeaos

subordinados,quesãoincluídosnoprocessodetomadadedecisãopelolíder.A

decisãoentãoétomadaemconjuntopelolídereseusliderados.

Esseestilopodeserclassicadocomoconsultivo,noqualolíderpedea

opiniãodossubordinadosantesdetomaradecisão,ouparticipativo,emqueos

funcionáriosparticipamnãosócomaopinião,masdecidemconjuntamentecom

olíder.

Finalmente,liberaloulaissez-faireéoestiloemqueolíderdátotalliberdade

aossubordinadosparadecidiremcomoacharemmelhor!Olídertemsomentea

funçãoderesponderasperguntasoudúvidasdosfuncionárioseforneceros

recursosnecessáriosparaotrabalho.

OobjetivodeLewineradeterminarqualdosestilosseriaomaisecaz.Sua

primeiraconstataçãofoiadequeoestiloliberaleraomenosecaz,ouseja,não

geravaumdesempenhomaior,umasatisfaçãonossubordinadoseumamaior

qualidadedotrabalho!

Entretanto,Lewinnãoconseguiudefinirseoestilodemocráticoerasuperiorao

autocrático.Aparentemente,oestilodemocráticoindicavasersuperiorem

relaçãoàsatisfaçãonotrabalhoeàmaiorqualidade,maserasimilarna

quantidadedetrabalho.

Pesquisasposterioresmostraramresultadosdiferentes,mudandodeacordocom

ascaracterísticasdecadacaso.Dessaforma,osresultadosinvalidaramatesede

queoestilodemocráticoseriasempresuperior!

Figura6.3–Estilosdeliderança

Vamospensaremumcasoprático.Vocêfoicontratadoparaadministraruma

fazendadecafé.Emdoismesescomeçaráacolheitadocafé,evocêprecisa

contratarquinhentaspessoasparafazeresseserviço.

Otrabalhoébasicamentebraçal,easpessoasquesecandidatamàsvagassão

humildes,nãoconhecemafazendaemdetalhes,nãoestãoacostumadasadecidire

atomariniciativanotrabalho.Alémdisso,vocêsabequeotrabalhoacabaráem

trêsmeses,portantonãotemmuitotempoparatreinaressepessoal.

Provavelmente,vocêdecidiráquemvaitrabalharemcadaárea,qualseráaárea

queserácolhidaprimeiroequandoseráiniciadootrabalho,emvezdeteruma

reuniãocomcentenasdepessoaseperguntaroqueelespensam,nãoéverdade?

Portanto,essasituaçãoseprestamelhoraoestiloautocráticodeliderança.

Outroextremopoderiaserumaagênciadepropaganda,emquevocêestá

cheandoequipesmultidisciplinaresequeaplicamacriatividadeparaatenderàs

necessidadesdosclientes.

Essaspessoassãoacostumadasatomardecisões,têmaltonívelintelectual,têm

habilidadescomplexasdeanálisedoambienteexternoeestãonormalmente

envolvidasnoprocessodecisóriodaempresa.

Nessecaso,jáseriamaisapropriadoutilizaroestilodemocrático,nãoémesmo?

Oestiloautocráticoprovavelmentealienariaessaspessoaseasdesmotivaria.

Vamosvercomopodesercobradoessetópico?

3.(FCC/TRT/PA/Analistaadm./2010)Umlíderqueconduzeorientasuaequipe,incentivandoa

participaçãodaspessoasedesenvolvendocomunicaçãoespontânea,francaecordial,é

classificadocomoumlídercomestilodeliderança:

a)liberal;

b)autocrática;

c)democrática;

d)situacional;

e)centralizadora.

Oestilodemocráticoéumestilomais“equilibrado”,poisnãotema

característicacentralizadoradoestiloautocráticoetampoucoacaracterísticade

liderançaextremamente“frouxa”doestiloliberal.

Umlíderqueconduzeorientasuaequipe,incentivandoaparticipaçãodetodos

osfuncionários,adotaoestilodemocrático.OgabaritoéaalternativaC.

6.5.ContinuumdaLiderança

TannenbaumeSchimidtdesenvolveramentãoumaescaladecomportamentos

doslíderes,desdeaquelescentradosnolíder(autocrático)atéaquelescentrados

nossubordinados.

EssateoriafoichamadadeContinuumdaLiderança,poistinhaumavisão

antagônica–oualiderançaerafocadaempessoasouseriafocadanastarefas,e

nãonasduas!8

Figura6.4–ContinuumdaLiderança.

(AdaptadodeDaft,2005)

Paraqueosgestorespudessemescolherqualestiloseguir,elesrecomendaram

analisarfatorescomooníveldeconfortodolídercomoestilodeliderança

(muitoschefesnãotêmoperfilparaatuarnoestilodemocrático!),características

dosliderados(sãopessoasqueestãoacostumadasaassumirresponsabilidade?

Estãoacostumadasadecidir?)eapressãodotempo(quantotempoteríamospara

tomarumadecisão?).

6.6.GridGerencialdeBlakeeMouton

Essaéumadasteoriascomportamentaisdaliderançamaisconhecidas.Agrade

gerencialdeBlakeeMoutonfoiumaevoluçãodateoriadeTannenbaume

Schmidt,quepostulavaquealiderançaeraumcontinuumentrealiderança

orientadaparapessoaseaorientadaparatarefas,poisquestionouessavisão

antagônica(ouerafocadaempessoasoutarefas,enãonasduas!).9

ParaBlakeeMouton,apreocupaçãotantocomaspessoasquantocoma

produçãoéfundamentalparasealcançarumbomresultado.10Elesmontarama

gradegerencialbaseadanasduasdimensõescomportamentais:preocupaçãocom

aspessoas

epreocupaçãocomaprodução(porissoéchamadavisão

bidimensionaldoestilodeliderança).11

Aideiaportrásdateoriaéadequeolídernãodeveriasertotalmentefocado

naspessoas,poisprovavelmentetenderiaanãoatingirosresultadosdaempresa

(oferecendobenefíciosemexcessoecobrandopoucoosresultados).

Poroutrolado,tambémnãopoderiasertotalmentevoltadoparaosresultados

(ouparaaprodução),poispoderiaalienaraspessoasecriarumambiente

desmotivadoreafastarosmelhoresempregados.12

Aseguirpodemosvermaisfacilmenteogridgerencial:

Figura6.5–Gradegerencial.

(AdaptadodeSobralePeci2008)

Cadaumadasdimensõeséavaliadapormeiodeumaescalade1a9,contendo

81posiçõespossíveisparaoestilodeliderança.Ascincoposições-chavequeBlake

eMoutonidentificaramforam:13

Lídernegligenteouliderançaempobrecida(1.1):olíderseesforça

minimamenteparafazerotrabalhoejusticarsuapermanêncianaposição.O

líderébasicamenteausente.

Líder-tarefa(9.1):olídersepreocupacomaeciêncianaprodução,mastem

poucapreocupaçãocomamotivaçãoeodesenvolvimentodosfuncionários.Todo

ofocoseconcentranosresultados.Éaliderançanabasedo“chicote”!

Líder-pessoaouclubedecampo(1.9):Todaapreocupaçãoestáfocadanas

necessidadesdosfuncionárioseemproporcionarumambienteagradável,sem

preocupar-semuitocomaeciênciaeecáciadaprodução.Nessetipode

liderança,existepoucapressãoporresultado!

Lídermeio-termo(5.5):olídermantémumaeciênciadaproduçãorazoávele

ummoralsatisfatório.

Líder-equipe(9.9):SegundoBlakeeMouton,esseéoestilomaisecazpara

umaorganização!Olíderseesforçariatantoparaobtereciêncianaprodução

comoemmanterseusfuncionáriosmotivadoseemconstantedesenvolvimento.

AteoriadeBlakeeMoutonfoimuitoimportanteporsintetizarasteorias

comportamentaisdeliderança,masosestudosposterioresnãoconrmaramque

oestilolíder-equipesejarealmenteomaiseficazemtodososcasos.

Afalhadelevaremconsideraçãoosfatoressituacionaisacaboulevandoao

desenvolvimentoposteriordasteoriascontingenciaisousituacionaisdeliderança.

Vamosanalisaragoraumaquestãosobreogridgerencial?

4.(Cespe/MTE/Administração/2008)Omanagerialgrid(gradegerencial)propostoporBlakee

Moutonpressupõequeoadministradordevasepreocuparapenascomosresultados.

Comovimos,ogrid(ougrade)gerencialdeBlakeeMoutonmostraumarelação

entreduasvariáveisnaliderança:oenfoqueempessoaseoenfoquenaprodução.

Paraosautores,amelhorsituaçãoseriaumaaltapreocupaçãocomosdoisfatores

(produçãoepessoas).Dessamaneira,ogabaritoéquestãoincorreta.

6.7.LiderançasContingenciaiseSituacionais

Oconceitoquebaseiaasteoriascontingenciaisesituacionaiséadequeo

contextodeveserlevadoemconsideraçãoantesdeumaatuaçãodolíder.Dessa

maneira,nãoexistiriaumestilo“melhor”deliderança.Nãoexistiriauma“receita

debolo”quefuncionariaemqualquersituação.

Atenção:aliderançasituacionaloucontingencialindicaquenãoexiste“estilo”melhorde

liderança–dependedasituação.

Aquicaiuporterraatentativaporpartedemuitosestudiososdecriarumestilo

quepudesseserutilizadoportodososadministradoresemtodasasempresase

paratodotipodefuncionário.

Apercepçãodolíderpassaaserfundamentalparaqueelepossadecidircomo

liderarseussubordinados.Semisso,ogestoracabaránãoatuandodaforma

corretaparacadasituação.

Muitasbancasacabamutilizandoessestermos(contingenciaisesituacionais)

comosinônimos.Portanto,vamosveressasteoriasemconjunto,parafacilitara

compreensão.

6.8.ModelodeFiedler

AnoçãobásicadomodelodeFiedleréadequesedeveidenticaroestilodo

lídereencaixá-locomasituaçãoquemaisfavoreceseuestilodeliderança.14

Fiedlerdistingueosestilosdeliderançaemdois:oslideresfocadosnatarefa

(quedariammaisimportânciaaosresultadosemetasdoqueàspessoas)eos

líderesfocadosnorelacionamentopessoal(quedariammaisimportânciaàs

pessoasdoqueaoalcancedasmetas).

Assituações,peloautor,teriamtrêsprincipaiselementos:arelaçãoentreo

lídereseussubordinados,aestruturadatarefa(tarefasestruturadassãomais

rotineirasetarefasmenosestruturadassãomaisligadasaatividadesinovadorase

criativas)eopoderdeautoridadedolíder.Esseselementoscombinados

formariamoitodimensões,damaisfavorávelàmaisdesfavorável.

OqueFiedlerdescobriufoiqueoslíderesfocadosnastarefaserammais

ecazestantonassituaçõesmaisfavoráveisquantonasmaisdesfavoráveis,

enquantooslíderesfocadosnosrelacionamentoserammaisecazesapenasnas

situaçõesintermediárias.

Aimportânciadeseutrabalhofoinãotersebaseadoapenasnosestilosde

liderança,masestudadoaintegraçãodessesestilosdeliderançacomasdiversas

situaçõesqueoslíderesenfrentamnasorganizações.

Figura6.6–TeoriadaContingênciadeFiedler

6.9.TeoriaSituacionaldeHerseyeBlanchard–Ociclodevidada

Liderança

AabordagemdeHerseyeBlanchardseapoianorelacionamentoentrea

maturidadedosempregadoseocomportamentodolíderemrelaçãoao

relacionamentoeàtarefa.15

Deacordocomosautores,osempregadosvariammuitoemseunívelde

maturidade–suahabilidadedefazerseutrabalhodeformaindependente,assumir

responsabilidadeedesejarosucesso.

Imaginequevocêéogerentedeumaconcessionáriadeautomóveis,por

exemplo.Nestasemanafoicontratadoumnovovendedor,de20anosenenhuma

experiênciaprévia.Nesseprimeiromomento,vocêirádizerexatamentecomoesse

novofuncionáriodeverátratarosclientes,comodevepreencherosformulários

etc.

Comotempo,essapessoacomeçaráaseaclimatarnaempresaesesentirmais“à

vontade”noserviço.Jáconheceráosmodelosàvendaeasmaneirasdeseabordar

umcliente,alémdasopçõesdefinanciamentoquepoderáoferecer.

Nessesegundomomento,olíderpoderáiniciarumprocessodeaproximação

pessoalmaiorcomosubordinado,deformaaincentivá-loeabrirumcanalde

respeitomútuoeconançaentreosdois.Entretanto,aindaterádesupervisioná-

lodeperto,poiseleaindanecessitarádeajudaemalgunsmomentos.

Quandomaistemposepassar,oempregadoestarácapacitadopararesolver

praticamentetodososproblemasnoseutrabalhosozinho.Ovendedorjáse

sentirátranquilosemprequeentrarumclientenaconcessionária.

Agora,olíderpoderádeixaroseuvendedormaislivre,semnecessitardeuma

supervisãomaispróxima,apenasmantendoapreocupaçãocomaconstruçãode

umbomrelacionamentoentreosdoisparaqueessefuncionárioconenogerente

elheforneçaasinformaçõesdecomootrabalhoestásendoexecutado.

Finalmente,chegaráummomentoemqueessefuncionáriojáestarámadurono

serviçoequeorelacionamentoentreogerenteeovendedorjáestarásolidicado.

Jánãoexistiránenhumatarefaquesejadifícilparaovendedor,eelejásesentirá

seguroparaexecutarcadapassodoseutrabalho.

Dessaforma,ogerentenãoprecisarámaissepreocuparnemcomatarefaenem

comorelacionamento.Assim,essefuncionáriojápoderácar“livre”parafazero

seutrabalho,sabendoqueterásempreumrelacionamentopositivocomoseu

chefeequecontarácomsuaajudasemprequenecessitar.

Deacordocomesse“ciclodevida”,osautoresentãocriaramquatroestilosde

liderançaadequadosaessesníveisdematuridadedosempregados:

Direção–Quandoosempregadossãoincapazeseindispostosatrabalhar,o

estiloidealseriafocadonastarefas.Nessasituação,ochefemanda,determinao

quedeveserfeito,quandodeveserfeitoecomodeveserfeito.Éoquesechama

decomportamentodealtatarefaebaixorelacionamento;

Persuasão–Nessasituação,oempregadojátemumníveldeprontidão

moderado,temumpoucodeexperiênciae/ousegurançanoquefaz,permitindo

queolídersecomportedemodoafocarnãosónatarefa,masnorelacionamento

comoempregado.Éoquesechamadecomportamentodealtatarefaealto

relacionamento;

Participação–Nessasituação,oempregadojátemmaishabilidadesno

trabalhoeopapeldolíderseriaajudá-loeguiá-loemseutrabalho,masdemodo

participativo.Seriaumlídermaisfocadonorelacionamentodoquenastarefas.É

oquesechamadecomportamentodebaixatarefaealtorelacionamento;

Delegação–Nessasituação,osempregadosjátêmumaltoníveldehabilidades,

desegurançaeiniciativa,possibilitandoaolíderutilizaroestilodedelegação.

Caberiaaolíderdarumametaeaautoridadecorrespondenteaoempregadopara

queestepossafazerseutrabalhocommaiorautonomia.Éoquesechamade

comportamentodebaixatarefaebaixorelacionamento.

Figura6.7–CiclodevidadeHerseyeBlanchard

Assim,olíder,deacordocomHerseyeBlanchard,deveriatratarcada

empregadodemaneiradistinta,conformeseuníveldeexperiência,de

habilidade,deiniciativaederesponsabilidadenotrabalho.

Vamosvercomoessetópicojáfoicobrado?

5.(Esaf/ANA/Analistaadministrativo/2009)Oreexodoexercíciodaliderançaéoresultado

alcançadopelolíderemrelaçãoàspessoasqueinuencia.Paraolíderqueocupaumaposição

formaldentrodaorganização,éumdesaoidenticaroestilodeliderançaquedeveaplicara

cadaumadascircunstânciasquevivencianocotidiano.Considerandoocontextodeliderança,

selecioneaopçãocorreta.

a)Nadivisãodotrabalho,olíderautocráticodeterminaatarefadecadaumecadaqualescolheseu

companheirodetrabalho.Naliderançademocrática,ogrupodecidesobreadivisãodetrabalhoe

sobreoparceirodecadaum.

b)Naprogramaçãodostrabalhos,tantoolíderdemocráticocomooliberalnãointerferemdenenhuma

formanasdecisõesdogrupo.

c)Ascaracterísticascomportamentaispredominantesdossubordinadosdolíderliberaledolíder

democráticosãosimilaresquantoàescolhadoquefazerequandofazer.

d)Ovolumedosresultadosproduzidospeloexercíciodaliderançaautocráticaémaior,poréma

frustraçãoeaagressividadetambém.

e)Grupossubmetidosàsliderançasliberaisedemocráticastendemaoindividualismoeaignorarolíder

comopassardotempo.

AletraAestáerrada,poisnaliderançaautocráticaaspessoasnãoescolhemseus

companheirosdetrabalho.Alémdisso,naliderançademocráticaasdecisõesnão

sãotomadasapenaspelosmembrosdogrupo,masemconjuntocomochefe.

AletraBestáequivocada,poisaliderançademocráticanãosecaracterizapela

nãointerferênciadochefe.Nesseestiloochefedásugestões,auxiliaoprocesso

etc.Domesmomodo,aletraCestáerrada,poisoestilodemocráticoeoliberal

nãosãoidênticos.

AletraDestáperfeitaeéonossogabarito.Entretanto,aletraEestá

equivocada,poisaliderançademocráticanãotendeacriarindividualismos.Pelo

contrário,esseestilogeraumacoesãomaiorentreosmembros.Ogabarito,

portanto,éaletraD.

6.10.LiderançaTransacionalxTransformacional

Grandepartedasteoriasdeliderançasebaseianorelacionamentoentrelíderes

esubordinados.Existem,porém,estudosqueabordamotipoderecompensaque

olíderoferece,emvezdeanalisaremocomportamentodolíderemrelaçãoaos

liderados.Essesestudosapontamdoistiposdelíderes:líderestransacionaise

transformacionais.

Liderançatransacionaléaquelaemqueexisteumarelaçãodetrocaentre

líderesubordinado.Seunomevemexatamentedessatroca,detransação!Olíder

deneasmetasquedevemseralcançadaseprometeos“prêmios”casoosobjetivos

sejamatingidos.

Olídertransacionaldeveentãoesclarecerquaisserãoastarefaseosobjetivos,

motivarseusfuncionáriosparaqueelesatinjamsuasmetasefornecerapoioaos

lideradosnotrabalho,buscandosuprirsuasnecessidades.

Lembre-se:olídertransacionaléolídertradicional,quebuscamotivaratravésde

incentivosmateriais.

Deacordocommuitosautores,aliderançatransacionalfuncionabemquandoa

organizaçãoestáemumambienteestável.Paraqueelasejaecaz,ambososlados

(lídereliderado)devemestar“satisfeitoscomonegócio”,ouseja,deveexistirum

equilíbrioentreasrecompensaseosesforços.

Jánaliderançatransformacional,opapeldolíderédeinspiradordeseus

subordinados.Esselíderbuscaqueseuslideradostranscendamseusobjetivos

pessoaisembenefíciodaorganização!

Elenãoseapoiasomentenasrecompensasmateriaisparamotivarosseus

liderados,masusatambémoutrosaspectos,comoavisão,osvalores

compartilhadoseasideiasparaqueseussubordinadossesuperem.

Existeumaviademãoduplanessetipodeliderança.Háumestímulomútuo,

emqueambasaspartesseincentivamparaquefaçamumesforçoextra,amde

queomaisaltoníveldemotivaçãosejaatingido.

Figura6.8–LiderançaTransacionaleTransformacional

Olídertransformacionaléumagentedemudançaseinovaçõesna

organização.Esselídertemcaracterísticasdolídercarismático,masvaialém

deste!Olídercarismáticopodequererqueoslideradossigamseumodode

pensar,enadaalémdisso.

Nocasodolídertransformacional,elequerqueosseusseguidoressejam

questionadoreseempreendedores,quebusquemonovo,quesejamcriativos!

Assimsendo,aliderançatransformacionalseriaconstruídacombasena

liderançatransacional.Elagerariaesforçoseresultadosacimadosquealiderança

transacionalpoderiaatingir!

Vamosanalisaragoraumaquestãosobreessestiposdeliderança:

6.(Cespe/Anatel/Analista/2004)Característicascomoótimosníveisdecompreensãodarealidade

domundo,capacidadedecompartilharpoderecapacidadedeinspiraredeenvolveros

lideradossãoimportantesatributosdoslíderesdofuturo,ouseja,lideresvisionáriose

transacionais.

Ficafácilverqueabancasimplesmenteinverteuosconceitos,nãoémesmo?A

pegadinhaestáexatamentenofatodequeessadescriçãoserelacionacoma

liderançatransformacional,enãoacomtransacional.Portanto,ogabaritoé

questãoincorreta!

6.11.LiderançaCarismática

Aliderançacarismáticabaseia-senanoçãodequecertoslíderespodem,através

dopoderdeseucarisma,motivaraspessoasaatingirobjetivos.Elesconseguiriam

formargruposcoesosevoltadosparaessesobjetivos,comumaltograude

confiançanolíder.16

DeacordocomCleggetal,17

aliderançacarismáticaéotipodeliderançaquefazumaênfasenaarticulaçãodeumavisãoedeuma

missãoqueprometeumavidamelhor.

Decertaforma,aTeoriadaLiderançaCarismáticaapontaocarismacomouma

característicaquefariamossubordinadossededicaremmaiseseguiremelmente

olíderemtornodeumametaidealizadaporele.

Mascomoumlídertorna-seumlídercarismático?Oumelhor,como

desenvolvemosumcarisma?SegundoumaanálisedeCongereKanengo,as

principaiscaracterísticasdeumlídercarismáticoseriam:18

Autoconfiança–possuemconfiançaplenanoprópriojulgamentoehabilidade;

Visão–apresentamumametaidealizadaquepropõeumfuturomelhorqueostatusquo.Quanto

maioradisparidadeentreessametaidealizadaeostatusquo,maisosseguidorestenderãoaatribuir

aolíderumavisãoextraordinária;

Habilidadedearticulação–elespodemesclarecereformularsuavisãodemaneiracompreensível

paraosdemais,demonstrandoumacompreensãodasnecessidadesdosseguidorese,

consequentemente,atuandocomoforçamotivadora;

Forteconvicção–oslíderescarismáticossãotidoscomofortementecomprometidos,dispostosa

assumirelevadosriscospessoais,arcarcomcustosaltosededicar-seaoautossacrifíciopararealizar

oquepretendem;

Comportamentoforadohabitual–osquetêmcarismaadotamatitudesvistascomomodernas,

nãoconvencionaisecontráriasàsnormas.Quandobem-sucedidos,essetipodecomportamento

despertasurpresaeadmiraçãonosseguidores;

Agentesdemudança–oslíderescarismáticossãopercebidoscomoagentesdemudançaradical,

emlugardemantenedoresdostatusquo;

Sensibilidadeaoambiente–fazemavaliaçõesrealistasdasimposiçõesambientaisedosrecursos

necessáriosparaprovocaramudança.

Dessemodo,oslíderescarismáticossãovistospelosseussubordinadoscomo

obstinados,autoconfianteseagentesdemudançaemsuaorganização.Nãose

comportamcomochefesconservadoresebuscamorisco,onovo,amudança.

DeacordocomRobbins,19existemtrêspassosprincipaisparaqueumlíder

possadesenvolvercarisma:

Naturalmente,existemadeptosecríticosdessetipodeliderança.Pesquisas

mostramquepessoasquetrabalhamcomlíderescarismáticossãomais

motivadasesededicammais.Alémdisso,comogostamdeseuschefes,sãomais

felizesnoambientedetrabalho.20

Entretanto,algunsapontamqueesseestilopodecriarproblemas.A

autoconançaebuscaobstinadaporumobjetivopodem“cegar”olíder,fazendo

comqueelenãoescutecríticasesugestõesdepessoascomoutrasvisões.21Assim,

essetipodelíderseriamaisadequadoaempresasemergentesouquenecessitemde

umagrandemudança.

6.12.LiderançaVisionária

Damesmaformaquealiderançacarismática,opróprionomedaliderança

visionáriaindicaqualéoseuenfoque.Osautoresquecriaramessetermo,Bennise

Nanus,apontaramqueacapacidadedecriarecompartilharumavisãode

futuroseriafundamentalnaatuaçãodeumlíder.22

Masanal,oqueseriaumavisão?Esseconceitoestárelacionadocomodestino

desejado–um“macro-objetivo”.Seria,basicamente,responderàpergunta:“Como

queremosestardaquiacincooudezanos?”

ParaAlbrecht,23parachegaràvisão,umaempresadeveperguntar:como

queremosservistospelaspessoasepelomercado?

Masumavisãonãopodecarsomentena“cabeça”dolíder.Umlíder

visionário“vende”avisãoaosmembrosdaorganização.Eleutilizaesse“alvo”,

essemacro-objetivo,comoumfatormotivadorequecriaumacoesãonogrupo.

Essavisãoserveparaquetodostenhamumanoçãoclaradequaissãoos

objetivosbuscadoseauxilianapriorizaçãodeesforçosederecursos.Deacordo

comBenniseNanus,24

É,portanto,umelementomotivador,alémdeatuarcomocritériodeseleçãoparaaalocaçãode

esforços,ltrarasinformaçõesaseremanalisadasedisciplinarasações,demodoacanalizartodos

osesforçosparaumúnicofim.

Assimsendo,umlídervisionárionãosóescolheumavisãocorretaparaa

empresa,mastambémcomunicaessavisãoatodososmembroseconsegueque

estes“comprem”essavisãoesedediquemaomáximoparaqueelavirerealidade.

QuestõesComentadas

7.(Cespe/Abin/Ocialtécnico/2010)Emorganizaçõesformaiscontemporâneas,osdirigentes

ocupamposiçãoemumahierarquiaregidapornormasimpessoais.Aautoridadeformal

concedidaaessesdirigentesnãogarantealiderançaeaconduçãodepessoas.

Perfeito!Ofatodeumapessoaestarocupandoumcargonãoégarantiadeum

papeldeliderança.Ogabaritoéquestãocorreta.

8.(FCC/Infraero/Administrador/2009)Comrelaçãoàdistinçãoentreliderançaegerência,écorreto

afirmarqueagerência:

a)sepreocupacomobomfuncionamentodaorganização,enquantoaliderançasedenepelabuscada

inovaçãoeamudança;

b)atuasempreapartirdemetasconsensualmenteestabelecidas,enquantoaliderançadeverecorrer

sempreasuaposição;

c)estámaisorientadaparaarealizaçãodasmudançasnopresente,enquantoaliderançasearma

unicamentepelacapacidadedeantecipaçãodastendênciasfuturasedamontagemdecenários

estratégicos;

d)seorienta,emprimeirolugar,paraaformaçãodenovasidentidadesemetasorganizacionais,

enquantoaliderançasepreocupaemgarantirbonsresultadosapartirdosrecursosjáexistentesna

organização;

e)baseiaseupodernocarisma,istoé,narotinizaçãodoseupapelhierarquicamentesuperior,enquantoa

liderançadependeexclusivamentedoconhecimentotécnicoeadministrativo.

AalternativaAestácorreta,eénossogabarito.AalternativaBestáerrada,pois

ogerentenãoatuasempreemmetasconsensuais(negociação),mastambém

atravésdacoerção.AalternativaCestáincorreta,poisaliderançanãoseforma

unicamenteatravésdacapacidadedoslíderesdepreverastendênciasfuturas.

AsopçõesDeEestãocomosconceitosdegerenteselideresinvertidos,

portanto,sãoincorretas.OgabaritoémesmoaletraA.

9.(FCC/TRT/PA/Analistaadm./2010)Asteoriassobreliderançaapresentadasporautores

humanistaspodemserclassificadasemtrêsgrupos:

a)inteligênciageral,interesseseatitudinais;

b)contingenciais,reforçoemotivacionais;

c)traçosdepersonalidade,estilosdeliderançaesituacionais;

d)traçosdecaráter,contingenciaisedeaprendizagem;

e)estilosdepoder,sistêmicasecomportamentais.

Agoracoufácil,nãoémesmo?Asprincipaisteoriasdeliderança,comovimos,

são:teoriadostraçosdepersonalidade,dosestilosdeliderançaeasteorias

situacionais.OgabaritoéaalternativaC.

10.(Esaf/STN/Desenv.institucional/2005)Indiqueaopçãoquecompletacorretamenteafrasea

seguir:

“Umadasteoriasdeliderançabaseia-senascaracterísticasdolíder,oquesignificadizer....”

a)quetraçosfísicos,sociaiseintelectuaisgarantemosucessodaliderança;

b)quevaloresetradiçõesdaempresagarantemosucessodaliderança;

c)queoestiloliberalgaranteosucessodaliderança;

d)queaforçadossubordinadosgaranteosucessodaliderança;

e)queograudeparticipaçãodadoaossubordinadosgaranteosucessodaliderança.

QuestãofácilessadaEsaf,nãoémesmo?Ateoriadostraçoséaqueacredita

queostraçosfísicosedepersonalidadegarantemosucessodeumlíder.O

gabaritoéaletraA.

QuestõesPropostas

11.(Esaf/STN/Desenv.institucional/2008)Considerandoqueosestilosdeliderançapodem-se

desenvolverdeformaautocrática,democráticaouliberal(laissez-faire),assinale,entreas

opçõesaseguir,aquelaqueapontaumacaracterísticacorretanaaplicaçãodeumadessas

formas.

a)Liberal:olíderdeterminaasprovidênciaseastécnicasparaexecuçãodastarefas,cadaumaporvez,na

medidaemquesetornamnecessáriasedemodoimprevisívelparaogrupo.

b)Autocrática:adivisãodastarefascaacritériodoprópriogrupoecadamembrotemliberdadede

escolherosseuscompanheiros.

c)Democrática:oprópriogrupoesboçaasprovidênciaseastécnicasparaatingiroalvo,solicitando

conselhotécnicoaolíder,aquemcabesugerir,senecessário,duasoumaisalternativasparaogrupo

escolher.

d)Democrática:háliberdadecompletaparaasdecisõesindividuaisougrupais,sendomínimaaaçãodo

líder.

e)Autocrática:tantoadivisãodastarefasquantoaescolhadoscompanheiroscamtotalmenteacargo

dogrupo,emdecorrênciadeumaabsolutafaltadeparticipaçãodolíder.

12.(Cespe/BASA/Administração/2010)Naatualidade,inexistesituaçãoquecomporteaaplicação

daliderançaautocráticanoâmbitodeumaorganização,poisessaéumateoriasem

aplicabilidadeprática.

13.(Esaf/RFB/ATRFB–2009)Aooptarpelaliderançasituacional,umgestordepessoasdeve:

a)conduziramaturidadedetrabalhodoliderado,comocuidadodenãointerferiremsuamaturidade

psicológica;

b)adotarumestilodiretivoparaquemapresentamaturidademédiaoualta;

c)identicaramaturidademédiadogrupoe,combasenisso,adotarumestiloúnicoparatodosos

liderados;

d)levarosindivíduosaoseumaiselevadoníveldedesempenho,mediantealiderançaeficazdeumaum;

e)saberquecompetênciaéacombinaçãodamotivaçãoedaconançadoindivíduoemsuacapacidade

deatingirumobjetivoouderealizarumatarefa.

14.(Esaf/AFCCGUDesen.inst.–2008)Liderançaéacapacidadedeexercerinuênciasobre

indivíduosegrupos,necessáriaparaqueorganizaçõesalcancemsuamissãoeobjetivos.

Dasteoriassobreliderança,escolhaaopçãoquecorrespondeàliderançacontingencial.

a)Baseia-seemtraçosdepersonalidadenatos,sejamfísicos,intelectuais,sociaisourelacionadoscoma

tarefa.

b)Adotaoestilodemocráticoaserexercidoportodoprossionaldesucesso,contrapondo-seao

modelomecanicistadegestão.

c)Ocorrepormeiododesenvolvimentodetécnicasaprendidaspelosindivíduosemprogramasde

capacitação.

d)Temcomofontedepoderaliderançaautoritáriaexercidapelolídersobreseussubordinados

imediatos.

e)Dependedarelaçãoentrelíder,lideradosesituação,nãoestandosujeitaaumúnicoestilo.

15.(Cespe/Anatel/Analistaadministrativo/2009)ATeoriadaLiderançaSituacionalprocuradenir

qualestilodeliderançaseajustamelhoracadasituaçãoorganizacional.Paraatingiresse

propósito,deve-se,preliminarmente,diagnosticarasituaçãoexistente.

16.(FCC/PGE/RJ/Auditor/2009)Sobreliderançasituacional:

I.Olíderéaquelequeprocuraadequarasituaçãoaoseuestilodecomando.

II.Quandoastarefassãorotineiraserepetitivas,aliderançaélimitadaesujeitaaocontrole

pelossubordinados,quepassamaatuarnumpadrãoautocrático.

III.Paraummesmosubordinado,olíderpodeassumirdiferentespadrõesdeliderança,

conformeasituaçãoenvolvida.

IV.ATeoriadeLiderançaSituacionaldeHerseyeBlanchardpropõeummodelodeliderança

adequadoparacadaníveldematuridadedosliderados,noqualolíderdeveidenticarem

quenívelseencontramoslideradospara,então,optarporumdosquatroestilosde

liderança.

V.UmproblemadaTeoriadeLiderançaSituacionaldeHerseyeBlanchardéaideiadequeos

subordinados“imaturos”devemsertratadoscomo“pulsoforte”,poisessetipode

comportamentoporpartedaliderançanãoestimulariaodesenvolvimentodos

subordinados.

a)EstãocorretasAPENASasafirmativasIeI.

b)EstãocorretasAPENASasafirmativasI,IeV.

c)EstãocorretasAPENASasafirmativasI,IIeIV.

d)EstãocorretasAPENASasafirmativasIIeIV.

e)EstãocorretasAPENASasafirmativasII,IVeV.

17.(Esaf/CGU–AFC–2006)Escolhaaopçãoquecorretamenteserefereàliderança.

a)Todocontatopessoaleporescritorealizadodeformaunilateral.

b)Conjuntoderazõesinternasqueestimulamumcomportamentoespecífico.

c)Esforçoouaçãodoindivíduoqueinfluenciaseudesempenho.

d)Conhecimentoscomportamentaispresentesnafunçãogerencial.

e)Capacidadedeinfluenciarocomportamentodaspessoas.

18.(Esaf/STN/Desenv.institucional/2005)Assinaleaopçãoincorreta.Daabordagemsituacionalda

liderançadepreende-seque:

a)háumcontinuumdepadrõesdeliderançadadopelograudeautoridadeutilizadopelolídereograude

liberdadedisponívelparaossubordinados;

b)oestilodeliderançaaseradotadodependedasituação,decaracterísticasdogerenteedos

subordinados;

c)paratarefasrotineiraserepetitivaspode-seadotarumestilodeliderançamaiscentradonolíder;

d)oclimaorganizacionaldaempresaindicaoestilodeliderançacentradonolíderparatarefas

operacionaisrepetitivas;

e)olíderpodeassumirdiferentespadrõesdeliderançaparaummesmosubordinadoeparacadaumdos

seussubordinados.

19.(Esaf/Aneel/Analista/2004)Assinaleaopçãoquenãoindicaumavisãocontemporâneade

liderança.

a)Olíderadquirehabilidadesdeliderançapormeiodeprocessosdeensino-aprendizagem,cabendoà

organizaçãoinvestiremcapacitaçãoparaformaçãodelíderes.

b)Oexercíciodaliderançaéprodutodaparticipação,envolvimento,comunicação,cooperação,

negociação,iniciativaeresponsabilidade.

c)Olíderpossuicaracterísticasinatas,comunsemgrandespersonalidadesdomundopolíticoe

empresarial,cabendoàorganizaçãoinvestiremrecrutamentoeseleção.

d)Oexercíciodaliderançapressupõedescobriropoderqueexistenaspessoas,tornando-ascapazesde

criatividadeeautorrealização.

e)Oexercíciodaliderançadependedecondiçõesorganizacionaisedacapacidadedeaprendere

desenvolverhabilidades

20.(Cesgranrio/Funasa/Administrador/2009)Liderançaéumacaracterísticaimportantíssimada

açãogerencial.Háváriasclassificaçõesdetiposdeliderançapropostasporautoresnocampoda

administração,comoKurtLewin,R.LippiteLikert,entreoutros.Aliderançademocráticaéum

dessestipos,caracterizadacomoaquelaemqueolíder:

a)atuademaneiraimpositiva;

b)dispensaumacoordenaçãoefetiva;

c)consultaeorientasubordinados;

d)focaaatençãonatarefaenosresultados;

e)preocupa-secomaspectoshumanosecomaunidadedaequipe.

21.(Cespe/Petrobras/Administrador/2007)Olíderautocráticoéaquelequedelegaaautoridadee

encorajaaparticipaçãodosmembrosdaequipe.

22.(Cespe/Petrobras/Administrador/2007)Identica-seoestilodeliderançadiretivoquandoo

líderexplicaaosdemaismembrosquaissãoastarefasdelesecomodevemexecutá-las.

23.(Cespe/Previc/Anal.adm.–2011)Amaturidadedeumgrupooudeumlideradodeveser

consideradaglobalmente,enãosomenteemrelaçãoàtarefaespecíficaaserrealizada.

24.(Cespe/Previc/Anal.adm./2011)Consideram-secomportamentodetarefaoapoio

socioemocionaleoencorajamentodadopelolíder.

25.(Cespe/Abin/Ocialtécnico/2010)Aliderança,porconstituirtraçodepersonalidade,nãoestá

relacionadaafatoressituacionaisrelativosàexecuçãodetarefasemumaorganização.

26.(Cespe/Ceturb-ES/Administrador/2010)Aoportunidadedeliderarumaequiperepresenta,

muitasvezes,umfatordemotivaçãoparaocolaborador,queconsideraatarefacomoumvoto

deconfiançadosseussuperiores.

27.(Cespe/Ceturb-ES/Administrador/2010)Olídertransacionaléopersonageminspiradorque

recompensaseuslideradoscomoprestígiodepertenceradeterminadogruponaorganização.

28.(Cespe/Min.Esporte–Administrador/2008)Notrabalhoemequipe,oestilodeliderança

democrático,porcompartilharoprocessodecisório,contribuiparaoaumentodamotivação

dosmembrosdaequipe.

29.(Cespe/DFTRANS/Administrador/2008)Aliderançaautocráticatemcomofocoa

automatizaçãodosprocessosdetrabalhoeocompartilhamentodasdecisõescomos

membrosdaequipe.

30.(Cespe/Unipampa/Administrador/2009)Casooadministradoradoteumaliderançaliberal,as

questõesserãodebatidasedecididaspelogrupo.

Gabaritos

1.E

11.C

21.E

2.E

12.E

22.C

3.C

13.D

23.C

4.E

14.E

24.E

5.D

15.C

25.E

6.E

16.E

26.C

7.C

17.E

27.E

8.A

18.D

28.C

9.C

19.C

29.E

10.A

20.C

30.E

Bibliografia

Cavalcanti,VeraLúcia,MarceloCarpilovsky,MyrianLund,eReginaArczynska

Lago.LiderançaeMotivação.3aed.RiodeJaneiro:FGV,2009.

Certo,SamuelC.,eTrevisS.Certo.ModernManagement.10aed.UpperSaddle

River:Pearson-Prentice-Hall,2006.

Chiavenato,Idalberto.AdministraçãoGeralePública.2aed.RiodeJaneiro:

Elsevier,2008.

—.Administraçãonosnovostempos.2aed.RiodeJaneiro:Elsevier,2010.

—.IntroduçãoàTeoriaGeraldaAdministração.8aed.RiodeJaneiro:Elsevier,2011.

Clegg,Stewart,MartinKornberger,eTyronePitsis.Managing&Organizations:An

introductiontotheoryandpractice.ThousandOaks:Sage,2008.

Daft,RichardL.Management.Mason:Thomson,2005.

Robbins,StephenP.OrganizationalBehavior.11aed.UpperSaddleRiver:Pearson-

Prentice-Hall,2004.

Robbins,StephenP.,eMaryCoulter.Administração.5aed.RiodeJaneiro:

Prentice-Hall,1998.

Sobral,Felipe,eAlketaPeci.Administração:teoriaepráticanocontextobrasileiro.

SãoPaulo:Pearson-Prentice-Hall,2008.

Zaleznik,Abraham.ManagersandLeaders–aretheydifferent?HarvardBusiness

Review,1992.

Capítulo7

ControleeAvaliação

Paraquepossamossaberseestamosounãoatingindonossosobjetivos,

precisamosdeumsistemadecontroleecaz.Semcontrole,estamostrabalhando

“àscegas”,ouseja,semsabercomoestáonossodesempenho.

DeacordocomRobbinseCoulter,1ocontrolepodeserdefinidocomo:

“Oprocessodemonitorarasatividadesdeformaaassegurarqueelasestejamsendorealizadas

conformeoplanejadoecorrigirquaisquerdesviossignificativos.”

JáMaximiniano2acreditaqueoprocessodecontroleconsisteemfazera

comparaçãoetomaradecisãodeconrmaroumodicarosobjetivoseos

recursosempregadosemsuarealização.

Portanto,umsistemadecontrolebuscagarantirqueosobjetivosestãosendo

alcançados.Eleéimportanteporqueatéosmelhoresplanospodemdarerrado,

nãoémesmo?

Dessaforma,precisamossaberosmotivosdosacertosedoserrosdecorrentes

dosnossosplanos.Assim,aprendemoscomnossoserroseacertos,corrigimosos

problemaseaumentamosachancedesucessonofuturo.

Imaginequevocêéumgerentedevendasemumalojadeeletrodomésticos.Sua

metadevendasparaomêsdejaneiroeradeR$200mil.Mas,analisandoasvendas

daprimeiraquinzena,vocêconstatouqueasvendassomaramapenasR$70mil.

Alémdessedado,osistemagerencialdalojalheforneceuosdadosdetalhados

devendaporvendedoreportipodeproduto.Comessesdados,vocêpoderá

analisaremqueáreadalojaasvendasnãoacompanharamoprevisto,sealgum

vendedoremparticularnãoatingiusuasmetas,sealgumdiadasemanafoi

particularmenteruimetc.

Osistemadecontrolelhefornece,assim,ummododemonitorarosefeitosdas

decisõeseaçõestomadasecompararcomoquefoiplanejadoanteriormente.

Dessamaneira,suaconclusãopodeatéserdequeoplanejamentofoimalfeitoe

dequeametaeraambiciosademais!

Umbomsistemadecontrolelheproporcionaráentendercomosuaempresa

estáfuncionandoparaquevocêpossatomarasdecisõesnecessáriasdemodoa

atingirseusobjetivos.

Oprocessodecontroleéformadoporquatroetapas:

ØEstabelecimentodospadrões–deniçãodosobjetivosdaatividade,ouseja,

odesempenhoesperado.Semdenirquaissãoasexpectativasdaempresa

paradeterminadaatividadeseriaimpossívelcontrolá-la.

ØMonitoramentododesempenho–acompanhamentodosresultados

decorrentesdasatividades.Basicamenteéumtrabalhodecoletade

informações.Nessafase,iremosdeterminaroqueserámedido,comoiremos

medir(fontesdeinformação)equandoiremosmedirecomquefrequência.3

ØComparaçãodoresultadocomopadrão–análisedosresultadosreaisem

comparaçãocomoobjetivopreviamenteestabelecido.Quasetodasas

atividades,naturalmente,terãoalgumtipodedesvio(sejamdesviospositivos

ounegativos).Entretanto,umaavaliaçãodosmotivosquelevaramaodesvio

tambéméimportante.

ØMedidascorretivas–nessemomento,devemostomarasdecisõesquelevem

aorganizaçãoaatingirosresultadosdesejados,seestesnãotiveremsido

alcançados.Dessaforma,podemacontecertrêscoisas:nãomudarnada,

buscarcorrigirodesempenhooualterarospadrõesdedesempenho(se

tiveremsidomaldefinidos).

Figura7.1–Fasesdoprocessodecontrole

Vamosveragoraumaquestãodeumconcursorecentesobreessetema?

1.(FCC/TRT/PA–Analistaadm./2010)Oprocessoadministrativopossuiquatrofunçõesbásicas−

planejamento,organização,direçãoecontrole.Afunçãodecontroleéumprocessocíclico

compostodequatrofases:

a)análisedosresultados;deniçãodemetas;açõesdereforçoparaaplicaçãodeboaspráticase

acompanhamento;

b)definiçãodeobjetivos;acompanhamentodaexecução;avaliaçãoeaçõesdemelhoria;

c)acompanhamentodaexecução;identicaçãodeinconformidades;deniçãodenovosprocessose

implantação;

d)monitoraçãodosprocessosdefinidos;identificaçãodeinconsistências;implantaçãoeavaliação;

e)estabelecimentodepadrõesdedesempenho;monitoraçãododesempenho;comparaçãodo

desempenhocomopadrãoeaçãocorretiva.

Comovimos,afunçãocontroleenvolvequatrofases:adeniçãodospadrõesde

desempenho,omonitoramentodosresultados,acomparaçãodessesresultados

comopadrãoeasaçõescorretivas.Assimsendo,onossogabaritoéaalternativa

E.

7.1.MomentodoControle

Continuando,ofocodocontrolepodeserfeitoantesdaatividade,durantee

depoisqueelaocorreu.Dessaforma,existemtrêstiposdecontrole(quantoao

tempooutiming):preventivo,prévioouexante,simultâneoeposteriorouex-

post.

Imaginequevocêéogerentedeumhotel.Quandovocêentrevistaum

candidatoaumcargonaempresaouanalisaseosprodutosalimentíciosqueo

fornecedorestáentregandoestãoemboaqualidade,estáfazendoumcontrole

preventivo.

Poroutrolado,quandovocêestásupervisionandootrabalhodascamareiras

(queestãoarrumandoosquartos),estáfazendoumcontrolesimultâneo.

Jáquandoanalisaonúmerodehóspedesdomêspassado,oupedeaopiniãode

umhóspedequesehospedouemseuhotel(umachamadatelefônicapós-venda),

estáfazendoumcontroleposterior.

Dessamaneira,ocontrolepreventivobuscaidenticareprevenirproblemas

antesdeelesefetivamenteocorrerem.4É,assim,ummododecontroleproativo,

poisbuscaevitarqueosproblemasaconteçam.

Ofocodessecontroleéavaliarseosrecursos(humanos,materiaisetc.)quesão

utilizadosnaorganizaçãoestãoembomestadoeprontosparaseremutilizados

pelaempresa.Todosnósfazemosessetipodecontroleemnossodiaadia.Sevocê

vaicalibraropneudeseucarroantesdeumaviagem,estáfazendoumcontrole

prévio,nãoémesmo?

Ocontrolesimultâneoéomonitoramentodasatividadesbuscandogarantir

queestejamsendoexecutadasdeacordocomoplanejado.Diferentementedo

controlepreventivo,éumaformadecontrolereativo.5

Asupervisãodotrabalhodeumapessoaéumexemplodessetipodecontrole.

Comaevoluçãodastecnologiasdeinformação,hojeépossívelocontroleem

temporealdediversasatividades.Omonitoramentodeumcaminhãoporsatélite,

porexemplo,dáaogestorumamaneiradecorrigirosdesviosderotademodo

muitomaisrápido,nãoémesmo?

Jáocontroleposteriortemoobjetivodeanalisaroresultadodeumaatividade

apóssuaocorrência.Portanto,oobjetivonessemomentoéaprendercomos

desvios(sejamnegativosoupositivos)paraqueaorganizaçãonãotenhaos

mesmosproblemasnofuturo.

Aqui,apreocupaçãonãoémaiscorrigirodesempenho,poisofatojáocorreu.

Portanto,devemos,sim,identicarascausasdosproblemas,6ou,ainda,entender

porqueaempresaconseguiuumresultadoexcelenteemalgumaárea.

Figura7.2–Classificaçãodocontroledeacordocomotempo

7.2.ConceitosdeEficiência,EficáciaeEfetividade

Osconceitosdeeciência,ecáciaeefetividadesãotemasdosmais“batidos”

emprovasdeconcursos.

Eficiênciaserelacionacomousodosrecursosdisponíveisparaatingirnossos

objetivos.Portanto,quandofalamosquealguémfoieciente,éporqueessapessoa

utilizouosrecursosquetinhadeformaadequada.7

Seriaentãoamaneiracomofazemosalgobuscandoatingirumobjetivo!

Lembre--sedequeessesrecursospodemservários,comootempo,aspessoas,o

dinheiro,asmatérias-primasetc.

Eficáciaéfazeracoisacerta!Oconceitoérelacionadonãocomautilizaçãodos

recursos,masserealmenteatingimosoobjetivoquetraçamos.

Porexemplo,seMariaplanejoufazerumaviagemdecarrodeBeloHorizontea

Brasíliaelevoutrêsdiasparachegar,dizemosqueelafoiecaz(atingiuoobjetivo

dechegaraBrasília),masnãofoieciente(levoumuitomaistempo–recurso–

queserianormalmentenecessário).

SeMariativesselevadosetehoras,masemvezdeBrasíliativessechegadoaoRio

deJaneiro,teriasidoeficiente(suaviagemdurouotempoplanejado),masineficaz

(nãoatingiuoobjetivo).

Atenção:Existemautoresqueconsideramqueaefetividadeéasomadaeciênciacoma

eficácia.

Aefetividaderefere-seaoimpactodasações!Comoaexecuçãodeum

programapodeounãoalterarumarealidade.8

Imaginequeaprefeituradesuacidadedesejadiminuiramortalidadeinfantil.

Buscandoatingiresseobjetivo,aprefeituracomproudezmilvacinasparapoder

imunizaromesmonúmerodecriançasdomunicípionoanode2010,tendoem

vistaoobjetivodereduziramortalidade.

Senonaldoprogramatodasasdezmilcriançasdomunicípioforam

vacinadas,oprogramafoieciente(utilizoudeformaadequadaosrecursos,como

vacinas,médicosetc.)eeficaz(ascriançasforamvacinadascomoplanejado).

Parasabermosseoprogramafoiefetivo,teremosdepesquisarserealmentea

mortalidadeinfantilfoireduzida,poispodeserqueavacinanãosejaaadequada,

ouquenaverdadeacausadasmortessejaoutroproblemaetc.

Cabeaquilembrarqueesseconceitodeefetividadeémuitoimportante,pois

muitasdasaçõesdoEstadodevemserplanejadaseexecutadastendoemmente

mudaralgumarealidade,comoapobrezaextrema,asdesigualdadesregionaisoua

evasãoescolar.

Portanto,oobjetivoemmentenãodeveserconstruirescolas,masmelhoraro

nívelescolardosalunos.Dessemodo,mediraefetividadedecadaaçãoajudaráa

planejarasaçõesfuturasdoEstadoparacombaterosproblemasdasociedade.

Vamosvercomoissojáfoicobrado?

2.(FGV/Sefaz-RJ–Auditor/2011)Nãoraro,aspessoasconfundemostermoseciência,ecáciae

efetividade.Aecáciaconsisteemfazeracoisacerta(nãonecessariamentedamaneiracerta).

Assim,estárelacionadaaograudeatingimentodoobjetivo.Sedesejamosfazeralgo(ser

aprovadosemumconcursopúblico,porexemplo)elogramosêxitonesseprojeto,somos

eficazes.Dessaforma,evidenciaocumprimentodamissãochegaraoresultadodesejado.

Perfeito.VejamcomoaFGVtrabalhouoconceito:ecáciaérelacionadaao

atingimentodosobjetivospropostos.Dessamaneira,seatingimosoquetínhamos

comoobjetivo,fomoseficazes.Ogabaritoéquestãocorreta.

7.3.UsodeControleseIndicadoresdeProdutividade

Ocontroleéumdosprocessosmaisimportantesnotrabalhodeum

administrador.Paraqueessecontrolepossaserfeito,utilizamosdiversostiposde

indicadoresdegestão.

ParaGraçasRua,9osindicadoressão:

Indicadoresnãosãosimplesmentedados,masumabalançaquenospermite“pesar”osdados

ouumaréguaquenospermite“aferir”osdadosemtermosdequalidade,resultado,impactoetc.

dosprocessosedosobjetivosdoseventos.Osindicadoressãomedidas,ouseja,sãouma

atribuiçãodenúmerosaobjetos,acontecimentosousituações,deacordocomcertas

regras.

Dessamaneira,osindicadoressãomedidasquenosfacilitamentendero

funcionamentoeoandamentodeumprocesso,deumprodutoouserviçosda

organização.Deacordocomamesmaautora:10

Deumamaneirasimplicada,osindicadoressãomedidasquerepresentamouquanticamum

insumo,umresultado,umacaracterísticaouodesempenhodeumprocesso,deumserviço,deum

produtooudaorganizaçãocomoumtodo.

Assim,usamosindicadoresparanosajudarnoplanejamentoenocontroledas

organizações.11Essasferramentasnosauxiliamnabuscadosresultadosena

melhoriadosprocessosdasempresasedosórgãospúblicos.

Alémdisso,ousodeindicadorespodemelhorarmuitoaqualidadedasdecisões

deumgestor.Elepoderáanalisarsesuasaçõesestãosurtindoefeito,seos

processosestãoounãomelhorando,identicaráreaseprocessosproblemáticose

determinarondeosrecursosdevemserempregados.

7.4.IndicadoresdeDesempenho

Antesdefalarmosemindicadoresdedesempenho,vamosconheceroconceito

dedesempenho.Estepodesercompreendidocomoumsomatóriodeesforçose

resultados.12Portanto,sãonecessáriosesforçosnabuscadoatingimentodecertos

resultadosdesejados.

Figura7.3–Desempenho

SegundoRummlereBrache,13umindicadordedesempenhoéaquanticação

dequãobemumnegócio(suasatividadeseprocessos)atingeumametaespecíca.

Semindicadoresdedesempenho,nãoconseguimosmedir.Esemmedir,não

conseguimosgerenciar.14

Entretanto,existemdiversosaspectosenvolvidosnoâmbitodosesforçosedos

resultadosquedevemserconsideradosparaquepossamosdenirummodelode

controledodesempenho.

DeacordocomPalvarini,15ogovernofederalescolheuummetamodelocom

umaconcepçãodeumacadeiadevalorqueidenticaseisdimensõesdo

desempenho.

Umacadeiadevalor,segundoBennetteWholey,16é:

Acadeiadevalorédenidacomoolevantamentodetodaaaçãoouprocessonecessárioparagerar

ouentregarprodutosouserviçosaumbeneciário.Éumarepresentaçãoorganizacionalque

permitemelhorvisualizaçãodovaloroudobenefícioagregadonoprocesso,sendoutilizada

amplamentenadefiniçãodosresultadoseimpactosdeorganizações.

Vamosverumarepresentaçãográficadetodooprocesso?

Figura7.4

Fonte:(MartinseMariniapudMinistériodoPlanejamento,2009)

Nagura,vemosqueoprocessotememseuiníciodiversosinsumos(inputs),

quesãogerenciadosdentrodasorganizaçõesdemodoagerarumasériede

produtos(investigaçõespoliciais,atendimentosmédicos,iluminaçãopública,

serviçosdetelefoniaetc.),que,porsuavez,deverãogerarosimpactos(outcomes)

desejadosnasociedade(melhoriadoensino,diminuiçãodacriminalidadeetc.).

Portanto,segundoomodelodoGespública,asseiscategoriasbásicasde

indicadoresdedesempenhosão:14

7.5.Indicadoresligadosàdimensãoresultado

ØEfetividadesãoosimpactosgeradospelosprodutos/serviços,processosou

projetos.Aefetividadeestávinculadaaograudesatisfaçãoouaindaaovalor

agregadoeàtransformaçãoproduzidanocontextoemgeral.Essaclassede

indicadores,maisdifícildesermensurada(dadosanaturezadosdadoseo

carátertemporal),estárelacionadacomamissãodainstituição.Por

exemplo,seumacampanhadevacinaçãorealmenteimunizarediminuira

incidênciadedeterminadadoençaentreascrianças,acampanhafoiefetiva.

Indicadoresdeefetividadepodemserencontradosnadimensãoestratégica

doPlanoPlurianual(PPA).

ØEficáciaéaquantidadeequalidadedeprodutoseserviçosentreguesao

usuário(beneciáriodiretodosprodutoseserviçosdaorganização).Por

exemplo,se,namesmacampanhacitada,ametadevacinaçãoéimunizarcem

milcriançaseessenúmerofoialcançadoousuperado,acampanhafoiecaz.

IndicadoresdeecáciapodemserdenidosapartirdaCartadeServiçosdo

órgão.

ØEficiênciaéarelaçãoentreosprodutos/serviçosgerados(outputs)eos

insumosutilizados,relacionandooquefoientregueeoquefoiconsumido

derecursos,usualmentesobaformadecustosouprodutividade.Por

exemplo:umacampanhadevacinaçãoémaisecientequantomenorforo

custo,ouseja,quantomenorforocustodacampanha,mantendo-seos

objetivospropostos.Indicadoresdeeciênciapodemserencontradosna

CartadeServiçoscomseuselementosdecustoseeminformaçõesdesistemas

estruturantesdogoverno,comooSiafi.

Aseguirpodemosterumresumodosconceitosvistos:

Figura7.5–Eficiência,eficáciaeefetividade

7.6.IndicadoresLigadosàDimensãoEsforço

ØExecuçãoserefereàrealizaçãodosprocessos,projetoseplanosdeação

conformeestabelecidos.Indicadoresdeexecuçãopodemserencontradosno

monitoramentodasaçõesdoPPA.

ØExcelência

é

aconformidade

a

critérios

e

padrões

de

qualidade/excelênciaparaarealizaçãodosprocessos,atividadeseprojetos

nabuscadamelhorexecuçãoeeconomicidade;sendoumelemento

transversal.Indicadoresepadrõesdeexcelênciapodemserencontradosno

InstrumentodeAvaliaçãodaGestãoPública(IAGP).

ØEconomicidadeestáalinhadaaoconceitodeobtençãoeaousoderecursos

comomenorônuspossível,dentrodosrequisitosedaquantidadeexigidas

peloinput,gerindoadequadamenteosrecursos

nanceirosefísicos.

Indicadoresdeeconomicidadepodemserencontradosnasunidadesde

suprimentos.

7.7.QualidadedosIndicadores

DeacordocomoTCU,18osprincipaisatributosouqualidadesdeumindicador

devemser:

üConfiabilidade:afontededadosutilizadapeloindicadordeveserconável,

fidedigna.

üAdaptabilidade:capacidadederespostaàsmudançasdecomportamentoe

exigênciasdosclientes.Osindicadorespodemtornar-sedesnecessáriosao

longodotempoedevemsereliminadosousubstituídosporoutrosdemaior

utilidade.

üAtualizaçãoperiódica:oindicadordevepermitiratualizaçãodeformaa

representarasituaçãomaisatualpossível.

üRepresentatividade:deveexpressarbemarealidadequerepresentaoumede.

üDisponibilidade:facilidadedeacessoparacoleta,estandodisponívela

tempo,paraaspessoascertasesemdistorções,servindodebaseparaque

decisõessejamtomadas.

üSimplicidade:oindicadordeveserdefácilentendimento,qualquerpessoa

devesercapazdetirarconclusõesapartirdaanálisedoindicador.

üAcessibilidade:oindicadordeveapresentarfacilidadeepossibilidadede

acessoàsinformaçõesprimáriasparasuamedição.

üEconomicidade:oindicadordevemostrar-seeconomicamenteviável,não

devesergastotempodemaisprocurandodados,muitomenospesquisando

ouaguardandonovosmétodosdecoleta.

üEstabilidade:oindicadordevepermanecerestávelaolongodeum

determinadoperíodo,permitindoaformaçãodeumasériehistórica.

üRastreabilidade:facilidadedeidenticaçãodaorigemdosdados,seuregistro

emanutenção.

üPraticidade:oindicadordeverealmentefuncionarnapráticaepermitira

tomadadedecisõesgerenciais.

Vamosvercomoessetópicojáfoicobrado?

3.(FGV/Badesc/Analistaadm./2010)Osindicadoresdeprodutividade,tantonoâmbitoda

administraçãoprivadaquantonodaadministraçãopública,devemrespeitarcertosatributos

geraisemsuaconcepção.Aesserespeito,assinaleaalternativaquecorrespondeaoatributo

deeconomia.

a)Garantiadequeégeradoemrotinasdeprocessoepermaneceaolongodotempo.

b)Garantiadequenãodevesergastotempodemaispesquisandodadosouaguardandonovosmétodos

decoleta.

c)Facilidadeparaidentificaçãodaorigemdosdados,seuregistroemanutenção.

d)Facilidadedeacessoparacoleta,estandodisponívelatempo,paraaspessoascertasesemdistorções.

e)Garantiadequerealmentefuncionanapráticaepermiteatomadadedecisõesgerenciais.

Oatributorelacionadocomaeconomianamontagemdeindicadores

(economicidade)estáligadoaogastodetempoeaoutrosrecursosparaseobter

essasferramentas.

Ouseja,ocustodeseobterosindicadoresdevesermenordoqueseus

benefícios.19

OgabaritoéaletraB.

QuestõesComentadas

4.(FGV/Sefaz-RJ–Auditor/2008)Quantoaousodeindicadoresdeprodutividade,nãoécorreto

afirmarque:

a)éumaferramentaimportanteaplicadaparacompreenderproblemasorganizacionaiseapoiaro

processodedecisãogerencial;

b)éumcomponentedesucessoefatordecompetitividadeentreorganizações,porissoéimportante

oseumonitoramentopormeiodeindicadoresqueapoiemdecisõesadministrativas;

c)éumadasferramentasusualmenteutilizadasemprocessosdequalidade;

d)aprodutividadedeumrecursoéaquantidadedeprodutosouserviçosproduzidosnumintervalode

tempodivididopelaquantidadenecessáriadesserecurso;porisso,variáveiscomocapital,materiais,

mãodeobradiretaegastosgeraisnãodevemserusadasparamediredeterminaraprodutividadeem

umintervalodetempo;

e)éoinvestimentofeitopelasorganizaçõesemtecnologiaetreinamentodosempregados.

Aquestãopedeaalternativaincorreta.Assim,aúnicaalternativaqueestá

erradaéaletraD.Recursoscomo:capital,materiaisemãodeobradevem,sim,ser

usadosparadeterminaraprodutividadedaempresa.

Lembre-sedequeaprodutividadeestárelacionadacomautilizaçãodos

recursosdeumaorganização(eficiência).Dessamaneira,ogabaritoéaletraD.

5.(FGV/SAD/PE/APOG/2008)Comrelaçãoaosatributosdosindicadoresdemensuração

organizacional,oquecaptaasetapasmaisimportantesecríticasdoprocessomensuradoéo

atributoda:

a)representatividade;

b)adaptabilidade;

c)disponibilidade;

d)rastreabilidade;

e)estabilidade.

Essaquestãoéinteressante,poisnos“põeparapensar”!Abancalistoucinco

atributosqueumindicadordeveterepediuo“maisimportante”.Decertomodo,

todoselessãoimportantes.

Entretanto,entreessesatributosomais“crítico”éodarepresentatividade.Se

umindicadornãoexpressaarealidadequesepropõe(imagineumindicadorque

sepropõeamediroensinotabulandodadosdecriminalidade,porexemplo),não

serviráparanada.

Dessamaneira,ogabaritoéaletraA.

6.(FCC/TRT/PR–Analistaadm./2010)Oindicadordedesempenhoqueafereosimpactosgerados

pelosprodutoseserviços,processosouprojetosdeumdeterminadosistema(organização,

programa,políticapública,rede)nobeneficiáriofinalédenominadoindicadorde:

a)efetividade;

b)eficiência;

c)eficácia;

d)economicidade;

e)excelência.

Comojávimos,quandoestamosavaliandooimpactogeradoporumprograma,

projetoouprocesso,estamosnosreferindoàefetividade.Aeciênciaserefereà

utilizaçãodosrecursosdamelhormaneirapossível,portantoaalternativaBestá

incorreta.

NocasodaletraC,aecáciaserefereaosprodutosentregues,aosobjetivos

organizacionais.Érelacionadoa“fazeracoisacerta”.Dessaforma,aletraCestá

errada.

Eeconomicidadeserelacionacomamelhorutilizaçãodosrecursosnanceiros.

Jáaexcelênciaserelacionacomosrequisitosdequalidadequeosclientes

necessitamoudesejam.Portanto,nossogabaritoéaletraA.

7.(FCC/TRT/PR–Analistaadm./2010)Asseiscategoriasdeindicadoresdedesempenhoestão

relacionadasaalgumdoselementosdacadeiadevalor(insumos,processos,produtose

impactos)edividem-senasdimensõesde:

a)execuçãoeexcelência;

b)efetividadeeesforço;

c)outputseoutcomes;

d)resultadoeeficácia;

e)resultadoeesforço.

Comovimos,osseistiposdeindicadoressedividememindicadoresdeesforços

(economicidade,excelênciaeexecução)eindicadoresderesultados(eciência,

eficáciaeefetividade).Assimsendo,ogabaritoéaletraE.

8.(FCC/Sefaz/SP/Fiscalderendas/2009)Umexemplodeindicadordeprodutividadeé:

a)onúmerodesolicitaçõesdereparospelonúmerototaldeunidadesentregues;

b)aporcentagemdefuncionárioscomformaçãodenívelsuperior,emrelaçãoaototal;

c)aporcentagemdeclientesquereclamaram,ounão,quandodaentregadoproduto;

d)oíndicederetrabalhoemrelaçãoaototalproduzidoemumdeterminadoprocessoindustrial;

e)onúmerodehomens/horaparaumaunidadedeserviçoexecutado.

Essaquestãojáfogeumpoucoda“decoreba”comumdaFCC.AalternativaAse

refereaosprodutosentreguesemrelaçãoàssolicitações.Dessaforma,podeserum

indicadordeexcelência,nãodeeciência(produtividade).AletraBnãoserefere

aumindicadordedesempenho.

JáaletraCeaDestãosereferindoaindicadoresquebuscammedira

qualidade,ouexcelência,deumprocesso,portanto,sãoincorretas.Aalternativa

corretaémesmoaletraE.

9.(FCC/TRT/MT/Técnico/2011)Entreasfunçõesadministrativasnoprocessoorganizacional,o

controlecompreendea:

a)emissãodeordens,instruções,comunicação,motivação,liderançaecoordenação;

b)deniçãodeobjetivos,odiagnósticodasituaçãoeumprognósticoapartirdasinformações

diagnosticadas;

c)definiçãodemissão,visão,metasestratégicasecenáriosprospectivos;

d)deniçãodepadrões,avaliaçãododesempenho,comparaçãododesempenhocomopadrão

estabelecidoeaçãocorretiva;

e)definiçãodemetas,controledeprocessos,correçãodeprocedimentosefeedbackdoprocesso.

Maisumavezabancapedeestasquatrofasesdoprocessodecontrole:a

deniçãodepadrões,omonitoramentododesempenho,acomparaçãoeaação

corretiva.Assim,ogabaritoéaletraD.

10.(FGV/Badesc/Analistaadm./–2010)Levandoemconsideraçãoousodecontroleseindicadores

deprodutividadeemumprogramadeeducação,opercentualdecriançasmatriculadasea

avaliaçãodaqualidadepormeiodeexamesnacionaissão,respectivamente,exemplosde:

a)eficáciaeeficiência;

b)eficiênciaeeficácia;

c)efetividadeeeficiência;

d)efetividadeeeficácia;

e)eficáciaeefetividade.

Nessaquestão,teremosdesabercomoclassicardoisindicadores:opercentual

decriançasmatriculadaseaavaliaçãodaqualidadedoensinopormeiodeexames

nacionais.Opercentualdecriançasmatriculadaséumcasotípicodeeficácia.

Setivéssemosfalandodeumindicadorcomonúmerodeprofessoresporaluno,

porexemplo,seriaumcasodemediçãodeeciência(utilizaçãoidealdosrecursos

disponíveis,nocaso,osprofessores).Portanto,oprimeiroindicadorédeeficácia.

Jáosegundoéumcasotípicodeavaliaçãodeefetividade,poistentamosmedir

comoasaçõesnocampoeducacionalafetaramarealidade(nocaso,onível

educacionaldosalunos).Ogabaritoé,então,aletraE.

QuestõesPropostas

11.(FCC/SergipegaS/Assist.adm./2010)Nagestãodaqualidade,medem-seosdesempenhos

medianteoclienteesuasatisfaçãoe,consequentemente,aefetividadedosprocessos,por

meiodos:

a)planos;

b)orçamentos;

c)fluxos;

d)indicadores;

e)mapasderiscos.

12.(FCC/TRT/RS/Anal.adm./–2011)OProgramaNacionaldeGestãoPúblicaidenticaseis

categoriasbásicasdeindicadoresdeavaliaçãodagestãopública:efetividade,ecácia,

eficiência,execução,excelênciaeeconomicidade.

I.Efetividadeestávinculadaaograudesatisfação,ouaindaaovaloragregado,àtransformação

produzidanocontextoemgeral.Estárelacionadacomamissãodainstituiçãoepodeser

encontradanadimensãoestratégicadoPlanoPlurianual.

II.Eficáciaéaquantidadeequalidadedeprodutoseserviçosentreguesaousuário.

III.Eciênciaestáalinhadaaoconceitodeobtençãoeusoderecursoscomomenorônus

possível,dentrodosrequisitosedasquantidadesexigidaspeloinput,gerindo

adequadamenteosrecursosfinanceirosefísicos.

IV.Economicidadeéarelaçãoentreosprodutoseserviçosgeradoscomosinsumosutilizados,

relacionandooquefoientregueeoquefoiconsumidoderecursos,usualmentesobaforma

decustosouprodutividade.

V.Excelênciaéaconformidadeacritériosepadrõesdequalidadeparaarealizaçãodos

processos,atividadeseprojetosnabuscadamelhorexecuçãoeeconomicidade,sendoum

elementotransversal.

ÉcorretooqueconstaAPENASem:

a)II,IVeV;

b)I,IIeIV;

c)I,IeV;

d)I,IVeV;

e)I,IeII.

13.(FCC/Arce/Analistareg./2006)Mensuraçãoecomparaçãododesempenhorealemrelaçãoaum

padrãoetomadadeaçãogerencialparacorrigirdesviosoupadrõesinadequadossãoetapasdo

processode:

a)organização;

b)planejamento;

c)coordenação;

d)controle;

e)liderança.

14.(FGV/Sefaz-RJ/Auditor/2011)Oconceitodeeciênciarelaciona-secomamaneirapelaqual

fazemosacoisa.Éocomofazemos,ocaminho,ométodo.Noprojetodeaprovaçãoemum

certame,seescolhermoscorretamenteomelhormaterial,amelhorequipedocentee

estudarmosdeformaproativa,provavelmenteseremosaprovadosemmenostempo.Aíestáa

eciência:aeconomiademeios,omenorconsumoderecursosdadoumdeterminadograude

ecácia.Algunsautoresassociamoconceitodeeciênciaaodeeconomicidade,como

sinônimos.Apesarde,emsentidolato,nãohaverdiferenças,normalmenteaeconomicidade

estáligadaaomenorconsumoderecursosmonetários(gastar-semenos).Emumalicitação,

porexemplo,aevidênciadocumprimentodoprincípiodaeconomicidadeprevistona

ConstituiçãodaRepúblicasedácomaobtençãodomenorpreçonacontrataçãodeprodutoou

serviçopreviamentedeterminado.

15.(FGV–Sefaz-RJ/Auditor/2011)Aefetividaderessaltaoimpacto,amedidaemqueoresultado

almejado(econcretizado)mudoudeterminadopanorama,cenário.Considerandoaconstrução

deescolaseoincrementononúmerodeprofessorescontratados,aefetividadeevidenciará,

porexemplo,dequemaneiraissocontribuiuparaareduçãodoíndicedeanalfabetismo

(impacto).Nessesentido,háautoresquedefendemqueaefetividadedecorredoalcanceda

ecáciaedaeciência,simultaneamente.Numaoutraacepção,podeserentendida,também,

comosatisfaçãodousuário.Na“pontadalinha”,aefetividadeocorrequandoumprodutoou

serviçofoipercebidopelousuáriocomosatisfatório.

16.(FGV/SAD/PE/Apog/2008)Observeocasoaseguirqueexemplicaaatuaçãodeumgestor

públiconasoluçãodeumproblema.

Ogestorpúblicorealizoudeterminadoprocedimentonabuscadasoluçãodeumanecessidade

dapopulaçãomoradoraemumaáreacarente.Aorealizá-lo,buscouotimizarosrecursos,

evitandoperdasedesperdícios,massemprejuízodoalcancedoobjetivopretendido.Sua

iniciativaatendeuaopúblico-alvo,resolvendooproblemademodosatisfatório.

SobaóticadaAdministração,aatuaçãodessegestorestábaseadanosconceitosde:

a)legalidade,legitimidadeeeconomicidade;

b)moralidade,eficiênciaeprudência;

c)oportunidade,utilidadeeinteressepúblico;

d)eficácia,eficiênciaeefetividade;

e)legalidade,moralidadeeinteressepúblico.

17.(Esaf/MPOG/EPPGG/2009)Aoavaliarumprogramadegoverno,énecessáriolançarmãode

critérioscujaobservaçãoconrmará,ounão,aobtençãoderesultados.Assim,quandose

desejavericarseumprogramaqualquerproduziuefeitos(positivosounegativos)no

ambienteexternoemqueinterveio,emtermoseconômicos,técnicos,socioculturais,

institucionaisouambientais,deve-seusaroseguintecritério:

a)eficiência;

b)eficácia;

c)sustentabilidade;

d)efetividade;

e)satisfaçãodobeneficiário.

18.(Cespe/SegerES/EPPGG/2007)Considerando-seque,conformecritérioscorrentesdeavaliação,

aescolapúblicanoBrasilvenhaformandograndequantidadedealunos,masqueaqualidade

doensinosejabaixa,poisaspessoasnãosaemcapacitadasparaenfrentarosdesaosdo

mercadodetrabalhoeasdiculdadesdeinserçãosocial,écorretoconcluirqueosistemade

ensinotemefetividade,masnãoéeficaz.

19.(FCC/TJ/AP–Analistaadm./2009)Aavaliaçãodaprodutividadenasorganizaçõesenvolve:

a)aresponsabilizaçãodosgestorespelanãorealizaçãodasmetasdefinidapeladireção;

b)oequilíbrioentreosváriosfatoresenvolvidosnoprocessoprodutivo;

c)acomparaçãodocustodeaquisiçãodeprodutoseserviçoscomseuretornoemtermosde

efetividade;

d)aadequaçãodosprocessosemrelaçãoàsespecificaçõestécnicasouàsuperaçãodosmesmos;

e)acomparaçãoentreentradasesaídasdossistemasprodutivos,levando-seemcontaosfatoresde

produção,comomãodeobra,recursosfinanceiroseinsumosfísicos.

Gabaritos

1.E

8.E

15.C

2.C

9.D

16.D

3.B

10.E

17.D

4.D

11.D

18.E

5.A

12.C

19.E

6.A

13.D

7.E

14.C

Bibliografia

Chiavenato,Idalberto.AdministraçãoGeralePública.2aed.RiodeJaneiro:

Elsevier,2008.

Daft,RichardL.Management.Mason:Thomson,2005.

Maximiniano,AntonioCesarAmaru.IntroduçãoàAdministração.4aed.SãoPaulo:

Atlas,1995.

MinistériodoPlanejamento,OrçamentoeGestão,SecretariadeGestão.“Guia

Referencialparamediçãodedesempenhoemanualparaconstruçãode

indicadores.”Brasília,2009.

Palvarini,Bruno.“GuiaReferencialdeMensuraçãodoDesempenhona

AdministraçãoPública.”IIICongressoConsaddeGestãoPública.Brasilia,2010.

Robbins,StephenP.,eMaryCoulter.Administração.5aed.RiodeJaneiro:

Prentice-Hall,1998.

Rua,MariadasGraças.“Desmisticandooproblema:umarápidaintroduçãoao

estudo

dos

indicadores.”http://www.enap.gov.br/downloads/ec43ea4fUFAM-

MariadasGraEstudoIndicadores-novo.pdf(acessoem02deabrilde2011).

Sobral,Felipe,eAlketaPeci.Administração:teoriaepráticanocontextobrasileiro.

SãoPaulo:Pearson-Prentice-Hall,2008.

Souza,DanielLuizde,MarcioLemosSaid,MarkusBuhatanKock,RicardoAraújo

Malachias,eLeonardRenneGuimarãesLapa.“IndicadoresdeDesempenho.”

TribunaldeContasdaUnião.Brasília,2009.

TribunaldeContasdaUnião,SecretariadePlanejamentoeGestão.Indicadoresde

Desempenho.Brasília:TCU,2009.

Capítulo8

Coordenação

AcoordenaçãoadministrativaéumelementodaAdministraçãofundamental

nasempresasmodernas.Quantomaioraempresaemaioraespecialização(divisão

dotrabalhoentreseusmembros),maioranecessidadedecoordenarotrabalho.

DeacordocomGulick,1coordenação“éaobrigaçãoessencialdeinter-relacionar

asváriaspartesdotrabalho”.JáparaFayol,acoordenaçãoé“areunião,aunicação

eaharmonizaçãodetodaaatividadeeesforço”.2

Imagineumaempresaquefabriquemóveisdemadeira.Nãoadiantaquecada

órgãodaempresasejaecienteemseutrabalho.Énecessárioqueosesforçosde

todossejamcoordenados,ouseja,quetodososórgãosestejamfuncionandoem

harmonia,buscandoomesmoobjetivoemconjunto,paraqueessaempresatenha

sucesso.

Dessaforma,nãoadiantaaáreadeproduçãofabricarumexcelenteprodutosea

áreacomercialnãoconseguirvenderessetipodemóvel.Osmóveiscariam

“encalhados”,nãoémesmo?

Aomesmotempo,nãoadiantaaáreacomercialvendermuitosabajuresseaárea

deproduçãonãoconseguiratenderaessespedidosnotemponecessárioou

produzi-loscomaqualidadedesejadapelosclientes.

Assim,todasasáreasdaorganizaçãonecessitamunirseusesforçosparaqueos

resultadossejamosmelhorespossíveis.Entretanto,quantomaiorfora

organização,maisissosetornadifícilefundamental.

Imagine-setrabalhandoemumaempresamultinacional,commilharesde

funcionários.Semuitasvezesjáédifícilse“acertar”comumcolegadetrabalho

queestáaoseulado,imagineharmonizarosesforçosdepessoasquetrabalhamem

paísesdiferentes,falandooutraslínguasetrabalhandoemfusoshorários

diferentes.Complicado,nãoémesmo?

Portanto,éimportantequeaorganizaçãomapeieasmelhoresestratégiaspara

fazeressacoordenaçãoentreprossionaiseáreasdiferentesdaempresa.Vamosa

elas.

8.1.EstratégiasdeCoordenação

DeacordocomMintzberg,3existemcincoprincipaisestratégiasparase

coordenarotrabalhonasorganizações:

8.2.AjusteMútuo

Éaformamais“intuitiva”decoordenação.Ocorreatravésdasrelações

informais,ouseja,dacomunicaçãodiretaeinformalentreaspartesquedevemser

coordenadas.Lembra-sedaqueleditado:“éconversandoqueagenteseentende?”.

Esseéo“fundamento”dessaestratégia.

Dessamaneira,osrepresentantesdasáreasenvolvidasemumproblemapodem

marcarumareuniãopara“acertarosponteiros”.Comisso,ostrabalhospodemser

feitosdemodoamaximizarosresultadoseminimizarosproblemas.

Figura8.1–Ajustemútuo

8.3.SupervisãoDireta

Quandoaorganizaçãocresce,oajustemútuocamuitomaisdifícil!Coordenar

otrabalhodecincopessoasemumdepartamentopequenoérazoavelmente

tranquilodesefazer.

Emumaempresade200pessoas,oajustemútuocamuitomaiscomplexo.Até

acomunicaçãocomtodasessaspessoasaomesmotemposetornaumatarefa

árdua.

Assim,nessaestratégia,umapessoaénomeadaparasupervisionaroucoordenar

ostrabalhos.Dessemodo,nãoocorremaisumprocessoinformaldeajuste,mas

umasupervisãodotrabalhoatravésdeumresponsávelporcobrarresultados.

Portanto,essesupervisorirádeniroqueéesperadodecadafuncionário(ou

órgão)eirámonitorarotrabalhodecadaáreademodoqueoresultadonalseja

positivo.

Figura8.2–Exemplodesupervisão

8.4.PadronizaçãodosProcessosdeTrabalho

Quandootrabalhoérepetitivoeonúmerodepessoasenvolvidasnotrabalhoé

muitogrande,aestratégiadepadronizaçãodotrabalhopodeserinteressante.

Umexemplocomuméalinhademontagemdeumaindústria.Commilharesde

funcionáriosemumalinhademontagem,somenteasupervisãopodenãosermais

suciente.Umapadronizaçãodasatividadeseposturasfacilitariaacoordenação

dostrabalhos.

Portanto,existiriaumaanálisepréviadatarefaaserfeita,sendoessaatividade

assim“manualizada”epadronizada.Nesseprocesso,regraseregulamentosseriam

feitosbuscandoumahomogeneizaçãodaspráticaseprocessosprodutivos.

Comotodososfuncionáriospassariamaseguir“àrisca”todosos

procedimentos,existiriaumamenorpreocupaçãocomomonitoramentodos

trabalhos.

Figura8.3–Padronizaçãodosprocessos

8.5.PadronizaçãodosResultados

Nessasituação,oqueépadronizadoéoresultadodotrabalho.Dessemodo,o

trabalhodevesercoordenadoparaatingirumaqualidadedeterminadaou

quantidadeespecificada.

Seriaocasodeumaequipedevendasquetivesseumametaespecícaparauma

região.Paraogerente,nãoimportariaqualaestratégiaqueaequipeutilize(desde

quesejalegal,claro),contantoqueametasejaatingida.

8.6.PadronizaçãodeHabilidades

Ocorrequandoaorganizaçãojácontrataprossionais“formados”e

especializadosemumamesmahabilidade.Dessamaneira,elesnãonecessitariam

detantotreinamentoemonitoração.

Umexemploseriaacontrataçãodemédicoseenfermeirosparaumaala

cirúrgicadeumhospital.Elesjásabemquetipodetrabalhoéesperadodelese

comodevemproceder.

Apadronizaçãodepessoascomamesmaformaçãoeconhecimentosfacilitariaa

coordenação,aprincípio,dessaspessoas.Assim,otrabalhodecoordenaçãose

tornamuitomaisfácil.

8.7.ConflitosnaCoordenação

Comovimos,aespecializaçãonotrabalhoacabacriandoanecessidadede

coordenaçãodotrabalhodeprossionaiseáreasdiferentesdentrodeuma

organização.

Entretanto,nemsempreesseprocessoésimples.Aspessoas,quandoestão

trabalhandodentrodecertodepartamento,acabamadotandocertas“culturas

dominantes”,ouseja,valoresespecíficosrelativosàquelegrupodepessoas.

Portanto,médicosemumhospitalcostumam“pensar”deumamaneiraqueé

diferente,normalmente,dosadministradoreshospitalaresquetrabalhamna

mesmaorganização.

Médicos,porexemplo,sãomaispreocupadoscomaquantidadeeaqualidade

dosexamesnecessáriosaodiagnósticodospacientes.Issoacontecepoiselessabem

queessesexamesfacilitamseutrabalhodediagnósticoepassaumaideiadeque

sãomaisatualizadosperanteseuspacientes.Alémdisso,nãosãoresponsáveis

diretamentepelocontroledecustosdohospital.

Assim,seusincentivossãorelacionadoscomomaiorpedidopossíveldeexames,

poisestesfacilitamseutrabalhoedãoumamaiorsegurançadequeodiagnóstico

eotratamentoestãocorretos.

Poroutrolado,osadministradoresbuscarãoveralgunsexamesemáquinas

solicitadascomo“gastos”desnecessáriosparaumbomatendimentodomédico.

Comoessesprossionaissãocobradospelosresultadosnanceirosdohospital,

terãoumatendênciamaiordeverosexamescomoumexagerodomédico.

Comisso,adiretoriamédicateriasempreumpotencialconflitocomadiretoria

deadministração.Esseésóumexemplodeáreasquecostumamterdificuldadesde

coordenação.Outrasáreas“famosas”porconitosdecoordenaçãosão:setorde

vendasversussetordeprodução,setordevendasversussetorfinanceiroetc.

QuestõesComentadas

1.(FCC/ICMS-SP/Auditor/2006)Obterintegraçãoestruturaléumdesaoimportantecomquese

defrontamtodososgerentes,obrigando-osatomardecisõesemtornodacoordenaçãodas

relaçõesentrepessoasegruposinterdependentesporelesgerenciados.Omecanismodessa

integraçãoéacoordenaçãocompostadeajustamentomútuo,supervisãodiretae

padronização,assimentendidos:

I.Ajustemútuoéacoordenaçãorealizadaporprocessosdecomunicaçãointerpessoal.

II.Nasupervisãodireta,umapessoaassumearesponsabilidadepessoalpelotrabalhodeum

grupodepessoas,adquirindoaautoridadehierárquicaparadeterminarastarefasaserem

realizadas.

III.Padronizaçãocompreendeacoordenaçãodostrabalhos,fornecendopadrõese

procedimentosqueajudamostrabalhadoresadefiniromododeexecutarsuastarefas.

Estácorretooqueseafirmaem:

a)I,IeII.

b)IeI,apenas.

c)I,apenas.

d)I,apenas.

e)II,apenas.

EssaquestãodaFCCapenasreproduzosconceitosdeMintzbergsobreas

estratégiasdecoordenação.Todasasarmativasestãocorretas.Oajustemútuo

ocorreatravésdacomunicaçãodiretaeinformalentreaspartesquedevemser

coordenadas.

Quandoaempresacresce,oajustemútuocamuitomaiscomplexo.Assim,

podeseradotadaasupervisãodireta.Nessetipo,umapessoaénomeadapara

supervisionarcoordenarostrabalhos.

Jáapadronizaçãoéinteressantequandootrabalhoérepetitivoeonúmerode

pessoasenvolvidasnotrabalhoémuitogrande.Portanto,ogabaritoémesmoa

letraA.

2.(FCC/Bacen/Analista/2006)HenryMintzbergdenequeaestruturaorganizacionaldeuma

empresaenvolveduasexigênciasfundamentais:adivisãodotrabalhoemtarefasdistintasea

realizaçãodacoordenaçãoentreessastarefas.Acoordenaçãoprovouserumtrabalhomais

complicado,queenvolveváriosmeios,denominadosmecanismosdecoordenação.Os

mecanismosdecoordenaçãodenidossãocinco:ajustemútuo,supervisãodireta,

padronizaçãodosprocessosdetrabalho,padronizaçãodosresultadosdotrabalhoe

padronizaçãodashabilidadesdostrabalhadores.

Nessaperspectiva,écorretoafirmarque:

a)osoutputssãopadronizadosquandoosresultadosdotrabalho–porexemplo,asdimensõesdo

produtoouodesempenho–nãosãoespecificados;

b)oajustemútuoobtémacoordenaçãodotrabalhopelosimplesprocessodecomunicaçãoinformal;

c)ashabilidades(eoconhecimento)sãopadronizadasquandootipodetreinamentoexigidoparao

desempenhodotrabalhonãoéespecificado;

d)asupervisãodiretaassumeacoordenaçãoquandoumapessoapassaaserresponsávelpelotrabalho

dasoutras,dispensandoinstruçõesemonitoramentodasações;

e)osprocessosdotrabalhosãopadronizadosquandooconteúdodotrabalhonãoéespecicadoou

programado.

AsletrasAeCestãoincorretas,poisabancainseriuum“não”quealteraos

conceitostantodepadronizaçãodosresultadosquantodapadronizaçãodas

habilidades.

AletraBestáperfeitaeéonossogabarito.Entretanto,naletraDasupervisão

nãodispensaasinstruçõeseomonitoramento.AletraEtambémnãofazsentidoe

estáerrada.Portanto,ogabaritoémesmoaletraB.

3.(Cespe/–Embasa/Analista/2010)Denise,fundadoradeumapequenaempresadeconsultoriade

marketingerelacionamentocomamídia,descobriuquealgunsdosseus15empregados

levavamqueixasaumconsultorsêniordaempresaenãoaela.Sabendoqueeraimportante

compreenderacomunicaçãoqueocorriaentreosempregadoseentendercomoamaiorparte

dainformaçãoeinuênciarealmenteuíaemsuaempresa,alémdeestarperturbadacomo

fatodenãoestarapardasinformaçõesdesuarma,Denisecriounovocargo—planejadorde

avaliações—paramantertodasaspessoasinterligadas.Dessemodo,todosostrabalhadores

daempresaconseguiriamobterasinformaçõesdesejadas.Oplanejadordeavaliaçõestinhaa

funçãodemanterDeniseinformadaacercadosproblemasedouxodeinformações.Além

disso,todosestariamincluídosnaesferadessasinformações.Considerandoessasituação

hipotética,julgueoitemquesesegue.

Osmecanismosinformaisdeajustemútuosãoutilizadosnasorganizaçõespequenase

flexíveis,comoaempresacitada.

Perfeito.Aspequenasempresaseempresasexíveisutilizam,emgrandeparte,o

ajustemútuocomoestratégiadecoordenaçãodostrabalhos.Ogabaritoéquestão

correta.

Gabaritos

1.A

2.B

3.C

Bibliografia

Chiavenato,Idalberto.IntroduçãoàTeoriaGeraldaAdministração.8aed.Riode

Janeiro:Elsevier,2011.

GrahamJr.,ColeBlease,eSteveW.Hays.Paraadministraraorganizaçãopública.

RiodeJaneiro:Ed.JorgeZahar,1994.

Capítulo9

Comunicação

Todoadministradorprecisasecomunicarparaconseguirfazerseutrabalho.

Cercade70%a80%dotrabalhodeumadministradorenvolveacomunicação:1ler

relatórios,mandare-mails,participardereuniões,trabalharemequipe,ouviros

outros,orientareliderarsãotarefasligadasàcomunicação.

Comoninguémdominatodasasinformaçõesnecessáriasparaaempresa,

necessitamosdainteraçãoedatrocademensagensentreosindivíduosparaquea

organizaçãoatinjaseusobjetivos.ParaBerlo,2umaorganizaçãodequalquerespécie

sóépossívelpormeiodacomunicação.

Assim,acomunicaçãopodeserdenidacomoumprocessointerpessoalde

envioerecebimentodesímboloscommensagensatreladasaeles.3

Portanto,éatransmissãodeumamensagemdeumapessoaparaoutra.De

acordocomMacêdoetal.4

Longedeserumprocessounilateral,acomunicaçãoésobretudoumexercíciodemútua

influência,apartirdatransmissãodeinformações,ideiasouemoçõesdeumaparteparaoutra

utilizandocódigocompartilhadospeloemissoreoreceptor.

Atenção:acomunicaçãoéuma“viademãodupla”.

Dessaforma,acomunicaçãoéconsideradaumprocessodemãodupla,pois

nãodependeexclusivamentedapessoaqueenvia,mastambémdapessoaque

recebe.Assim,requeraida(oenviodamensagem)eavolta(oretornodo

recebedor,confirmandooentendimento).DeacordocomMarchiori,5

Comunicaçãoéessencialmenteumapontedesignicadosquecriacompreensãomútuae

conança,jáquecompreensãolevaàaceitaçãoounãodamensagem,eaumaaçãoparaquem

recebeamensagem.

Dessamaneira,nemsemprequefalamosestamossendoouvidos.Oprocessode

comunicaçãoenvolvetambémointercâmbio,oentendimentoeo

compartilhamentodainformação,deformaqueainformaçãosetornecomumàs

duaspessoas.

Deveexistir,assim,umaempatiaentreapessoaqueenviaamensagemeapessoa

quearecebe.Acomunicaçãosóocorrequandoamensagemécompreendida.

Assim,asduaspartesdevemterinteresseemqueacomunicaçãosejafeitade

formaeficaz.

Diversosproblemasqueocorrememumaorganizaçãosãodecorrentesdealgum

problemadecomunicação.

Vamosveraseguirosprincipaiselementosdoprocessodecomunicação:6

ØAfonte–iniciaamensagemcodificandoumainformação.

ØOtransmissor–éomeioquecodicaamensagem,oprodutoda

codificaçãodafonte.

ØOcanal–éomeioqueafonteescolheparaenviaramensagem.

ØOreceptor–éomodoouinstrumentoquedecodificaamensagem.

ØOdestino–éapessoaquedevereceberamensagem.

ØOruído–representaasbarreirasdecomunicaçãoquedistorcemosentido

damensagem.

ØAretroação–éoretornododestinoconrmandoosucessoounãodo

processodecomunicação.

Figura9.1–Processodacomunicação

Imagineumasituaçãodescritanagura:acomunicaçãoentreumpilotode

aviãoeumcontroladordevoo.Opilotonoaviãoéafonte.Parasecomunicar,ele

seutilizadeumaparelhoderádioemsuacabine(transmissor).Esseaparelhovai

transmitiramensagemdopilotoatravésdesuasantenas(canal).

Dooutrolado,ocontroladordevoo(destino)tambémnecessitarádeum

aparelhoderádionatorredecomando(receptor)paradecodicaramensagem

quechegaráàsuaantena.Masacoisanãoétãosimples,poistambémocorreo

ruído,quenessecasopodeseraestática,umainterferênciadeumarádiopirataou

simplesmenteumadificuldadedeexpressãodopiloto(linguagem).

Paratercertezadequefoicompreendido,opilotosolicitaráumaconrmação

damensagem,oquechamamosderetroação(oufeedback).

Vamosvercomoessetemajáfoicobrado?

1.(Cespe/Cespe/MPU/Anal.administrativo/2010)Acomunicaçãoconstituiatividadequedemanda

grandepartedaatençãodequemocupacargogerencial.

Exato.Acomunicaçãoéumadasprincipaispreocupaçõesdeumbomgestor,

poisénecessáriaparaqueelepossacomunicarosobjetivosquedevemser

alcançados,demodoamotivareliderarseusfuncionários,entreoutrastarefas.O

gabaritoéquestãocorreta.

9.1.ComunicaçãoEficienteeEfetiva

Outrosconceitosinteressantessãoosdecomunicaçãoecienteecomunicação

efetiva.7Acomunicaçãoecienteocorrequandoutilizamosomínimode

recursosparanoscomunicarmos,osquaispodemtantosertantonanceirose

materiaisquantootempo.

Quandoumgerenteprefereenviarume-mailemvezdeirpessoalmenteao

destinatárioterumaconversa,estábuscandoexatamenteumaeconomiade

recursosnessacomunicação,nãoémesmo?Elegastaráassimmenostempo

enviandoamensagem.

Entretanto,acomunicaçãosomenteéefetivaquandoamensagemdapessoa

queestáenviando(afonte)écompletamentecompreendidapelodestinatário.

Assim,muitasvezesquandobuscamosumaeciêncianacomunicação(ocasodo

gerentequepreferesecomunicarpore-mails,porexemplo)acabamosnão

conseguindoumaefetividade,ouseja,nossacomunicaçãonãoéplenamente

entendidapelapessoaquearecebe.

Damesmaforma,nemtodamensagemvaleocustodeirpessoalmentefalarcom

cadacolaborador,nãoéverdade?Nãotemos52horasnodia.Otrabalhodeum

administradorenvolveatéatarefadedecidirquaissãoasmensagensquedevem

serpassadaspessoalmenteequaispodemoudevemserenviadasporoutrocanal

menos“rico”(comooe-mail).

Portanto,ogestordeveanalisaraimportânciadamensagemeocusto/benefício

doprocessodecomunicaçãoparadecidirqualcanalutilizar.

9.2.CanaisdeComunicação

Existemdiversosfatoresquepodeminuenciarumaboacomunicação.A

escolhadocanaldecomunicaçãocorretoparaumadadasituaçãoéumdesses

fatores.

Umadministradorcompetentedeveentenderasvantagensedesvantagensde

cadacanalparapoderescolheramelhoralternativaemumasituaçãoreal.

Oscanaisdiferememsuacapacidadedetransmitirinformação.8ParaDa,

quantomaisinformaçãoocanalconseguetransmitir,maisricoeleé.Ariquezade

umcanalseriaaquantidadedeinformaçãoqueeleconseguetransmitiremum

determinadomomento.

Dessaforma,podemosclassicaroscanaisemumahierarquia,mostradana

figuraaseguir:

Figura9.2–Hierarquiadariquezadoscanaisdecomunicação.

(Fonte:Daft2005)

Acomunicaçãopessoal,oucaraacara,éocanaldecomunicaçãomaisrico.

Pensebem,quandoqueremosdizeralgomuitoimportanteparaalguémnão

mandamosume-mail,nãoéverdade?

Quandoestamospessoalmenteconversandocomoutrapessoapodemosverseu

semblante,seapessoaestásorrindoouaborrecida,seugestualetc.Dessaforma,

podemosterumasensibilidademaiordo“estadodeespírito”dooutro,alémde

tirarqualquerdúvidaqueapessoatenha,demodoinstantâneo(feedbackou

realimentação).

Entretanto,umcanalmaisricotambémtemsuasdesvantagens.Umaconversa

pessoalnãodeixaregistro(anãoserquevocêgraveaconversa),deixaespaçopara

oerro(devidoaoseucaráterespontâneo)eédedifícildisseminação,ouseja,não

conseguimosreplicaressaconversademodofácil(teríamosdeconversarcom

cadaumadaspessoasdeumdepartamento,porexemplo)!

Assimsendo,seumamensagemérotineiraedevealcançarumagrandemassade

pessoas(oupessoasqueestãodispersas),ocanalutilizadonãodeveserumdealta

riqueza(comoumaconversaouumtelefonema),masumdebaixariqueza.

Portanto,umcanaldebaixariquezatambémtemsuasvantagensedesvantagens.

Vejanaguraaseguirasprincipaisvantagensedesvantagensdoscanaisdebaixa

riqueza:

Figura9.3–Vantagensedesvantagensdoscanaisdebaixariqueza.

(Fonte:Daft2005)

Dessaforma,oobjetivodeumgestordeveseranalisarotipodeinformaçãoa

sertransmitidaeescolheromelhorcanalparaaqueletipodesituação.

9.3.ComunicaçãoVerbaleNãoVerbal

Acomunicaçãopodedar-sedemodoverbalounãoverbal.Acomunicação

verbalfazusodapalavraparaatransmissãodasmensagens.Jáacomunicaçãonão

verbalutilizaaspectoscomo:otomdevoz,aposturadocorpo,oolhar,osgestos,

entreoutros.

Paraumacomunicaçãoefetiva,necessitamostantodeummodoquantodo

outro.DeacordocomMacêdoetal.9

Pesquisasrevelamqueapenas7%dacomunicaçãointerpessoalpodesertraduzidapor

palavras,pois38%provêmdainexãodavoz,enquantoosrestantes55%resultamdaexpressão

facialedalinguagemcorporal.

Assim,asmensagensnãoverbaisdãocredibilidadeàsmensagensverbais.

Imagineumaentrevistadeempregoemqueocandidatosetremetodoaocontar

umapassagemdesuaexperiência.Vocêacreditarianele?Difícil,nãoémesmo?

Arelaçãointerpessoaldependemuito,portanto,deumacoerênciaentrea

mensagemverbaleamensagemnãoverbal.Umacomunicaçãosóseráecaz

quandoambasconvergiremparaomesmosignificado.10

9.4.ComunicaçãoOrganizacional

Ascomunicaçõesformaisdentrodasorganizaçõesnormalmenteseguemtrês

direções:descendente,ascendenteelateral(ouhorizontal).11

Ascomunicaçõessãodescendentesquandosãoenviadasdascheasparaseus

subordinados,ouseja,quando“descem”dosníveishierárquicosmaisaltosparaos

maisbaixosdaempresa.Podemincluirdesdeinstruções,objetivosaserem

buscados,normasaseremseguidas,resultadosalcançadosenotícias.

Damesmaforma,ascomunicaçõessãoascendentesquandosãoenviadaspelos

subordinadosparaseuschefes.Paraqueosexecutivosdeumaorganizaçãoquem

apardosacontecimentos,umacomunicaçãoascendentelivreémuitoimportante.

Dessaforma,osempregadospodeminformaralgumproblemaquetenha

ocorrido,informaralgumconitoouinsatisfação,fazersugestõesetrazertodo

tipodeinformaçãoda“linhadefrente”paraseuchefe.

Grandepartedasorganizaçõesestimulaessetipodecomunicação.Isso

acontece,poisinstrumentosquefacilitemoenviodeinformaçõesparaascheas

podemevitarumasériedeproblemasemumaempresa.Infelizmente,muitos

chefesresistemaouvirseusfuncionários,eosfuncionáriospodemnãoconaro

bastantenaschefiasparapassaressasinformações.12

Figura9.4–Comunicaçãoascendenteedescendente

Essesuxosdecomunicação(descendenteeascendente)descritossãoosmais

comunsemorganizaçõesverticalizadasehierarquizadas.Entretanto,atualmente

osfluxoshorizontaissãocadavezmaisimportantes.

Acomunicaçãolateral(ouhorizontal)éouxodeinformaçõesentrecolegas

detrabalhoesetoresdeumaempresa.Umacomunicaçãolateralacontecequando

pedimosajudaaumcolegadetrabalhoouquandodoisdepartamentosprecisam

coordenarumatarefa.

Entreosexemplosdecasosemqueacomunicaçãolateralocorre,temos:as

reuniõesentredepartamentosdiferentes,osgruposdetrabalho(ougruposde

“força-tarefa”)easequipesdeprojetos.13Essetipodecomunicaçãoéfundamental

paraumacoordenaçãodeesforçosentreasdiversasáreasdaempresa.

Figura9.5–Comunicaçãolateralouhorizontal

Dessaforma,sejadentrododepartamentoouentredepartamentos,a

comunicaçãohorizontaléfundamentalparaotrabalhoemequipe.Elaévitalem

estruturasmaismodernaseemempresasquenecessitamdeumamaior

flexibilidadeeadaptabilidadeaomeioambiente.

Jáacomunicaçãoinformalocorreforadasnormasedoscargosdeterminados

peloorganogramadaempresa.Essacomunicaçãoacontecenaturalmentepelo

contatoentrepessoas.

Quandoencontramosumcoleganocorredore“trocamosumaideia”,estamos

utilizandoacomunicaçãoinformal.Essetipodecomunicaçãoéimportante,pois

aceleraouxodeinformaçõesentreasáreasepessoasecomplementa,decerta

forma,ofluxodeinformaçõesqueocorrenacomunicaçãoformaldaempresa.

Entreosexemplosdesituaçõesdominadaspelacomunicaçãoinformaltemos:as

conversasinformais(o“rádio-corredor”ou“rádio-peão”),o“giro”dodiretor

pelasinstalaçõesdaempresa(emqueeleconversacomfuncionáriosdobaixo

escalão),ainteraçãoentrefuncionáriosdediferentesáreasnorefeitórioetc.

Vamosvercomoessetópicojácaiuemprovas?

2.(Cespe/Cespe/MPU/Anal.administrativo/2010)Aordeméumexemplotípicodecomunicação

colateralnoprocessoorganizacional.

Umaordeméumexemplodecomunicaçãodescendente,poisseoriginaem

algumnívelhierárquicomaisaltoparaummaisbaixo(vocênãoveráumsoldado

dandoumaordemaumgeneral,nãoéverdade?).Portanto,essaarmativaestá

incorreta.Ogabaritoéquestãoerrada.

9.5.BarreirasàComunicação

Asbarreirasàcomunicação(ouruídos)sãoascausadorasdasdistorçõesque

impedemqueumacomunicaçãosejaefetiva.ParaRobbins,asmaisimportantes

barreirassão:14

üFiltragem–manipulaçãodamensagempelapessoaqueenvia(fonte),para

quesejavistafavoravelmentepelorecebedor.Acontecequandoum

subordinadonãoquerdarumamánotíciaaochefeouquer“dizer”aochefe

oqueelequerouvir.Normalmente,ocorrequandoafonteeodestinatário

estãoemníveishierárquicosdiferentes.

üPercepçãoseletiva–aspessoasquerecebemamensagemasinterpretamcom

baseemsuasnecessidades,experiências,motivaçõeseoutrascaracterísticas

pessoais.Dessaforma,seumentrevistadorderecrutamentotemum

preconceitoemrelaçãoaestrangeiros,porexemplo,tenderáainterpretar

negativamentequalquermensagemqueumangolano(porexemplo)venhaa

dizer.

üExcessodeinformação–todosnóstemosumacapacidadenitadeprocessar

informação.Quandoessacapacidadeésuperada,tendemosaesquecerde

informações,postergaracomunicaçãoouaçãonecessária.Assim,oresultado

éperdadeinformaçãoeumacomunicaçãocomruído.

üEmoções–asemoçõesqueapessoaquerecebeamensagemestásentindoirão

inuenciaromodocomoelainterpretaráamensagem.Seestivermosfelizes,

tenderemosaveramensagemcomopositiva.Jáseestamosdepressivos,

tendemosaverascoisasdemodonegativo,nãoémesmo?Dessaforma,

deixamosdeanalisaramensagemobjetivamenteeusamosnossasemoções,

distorcendoosentido.

üLinguagem–Aidade,oníveleducacionaleaculturadeumapessoa

inuenciamcomoelausaalinguagem.Aspalavrasnãosignicamamesma

coisaparatodomundo.Quandovamosaummédicoeeleusatermos

técnicosparanosexplicarquedoençatemos,podemosnãoentendera

mensagem.Omesmopodeocorrerquandonecessitamosdoserviçodeum

advogado.Aspessoasqueenviamasmensagenscostumamimaginarquetodo

mundocompreendedamesmaformaasualinguagem,masissonãoé

necessariamenteverdade.

üApreensãoouansiedade–Muitaspessoassofremdeumaextrema

dificuldadedesecomunicar.Podemterdificuldadeparafalar,escrever,ouas

duascoisas.Issopodeocorrerpor:timidez,diculdadenafala,fobiasetc.

Dessaforma,tenderãoabuscarreduziraomáximoasuainteraçãocom

outraspessoas.Pessoasquenãogostamdefalarempúblicoouconversarcara

acaratenderãoautilizarcartasoue-mailsquandoumaconversaseriaomais

indicado.

JáMacêdoetal.15classicamasbarreirasqueimpedemumacomunicaçãoecaz

emquatrotipos:asbarreiraspresentesnoemissor,aspresentesnoreceptor,asque

estãopresentestantonoemissorquantonoreceptoreaspresentesnoambiente.

Vejanafiguraaseguirasprincipaisbarreirasdeacordocomessaclassificação:

Barreirasno

Barreirasnoemissor

Barreirasnoreceptor

Barreirasnosdois

ambiente

Usodelinguageme

Desatenção,impaciência

Pouca

símbolos

oupressa.

disponibilidade

Inadequação

inadequados.

Tendênciaaavaliare

detempo.

docanal

Timidez,

julgar.

Interesseem

escolhido.

impaciênciaetc.

Preconceitos.

distorcera

Escolhadeum

Desconfiançaemrelação

mensagem,

Distrações,

momentoimpróprio.

aoemissor.

hostilidade.

ruídos,

Suporqueoreceptor

Resistênciaemaceitara

Diferençana

interrupções

jádominaoassunto

mensagemporexcesso

hierarquiaeno

frequentesetc.

asertratado.

deautoconfiança.

nívelcultural.

Figura9.6–Barreirasnacomunicação.

(Fonte:Macêdo,etal.2007)

Vamosvercomoestetemajáfoicobrado?

3.(Esaf/ReceitaFederal/Analista/2009)Sobreotema‘comunicaçãoorganizacional’,écorreto

afirmarque:

a)tantooemissorquantooreceptorsãofontesdecomunicação;

b)redigircomclarezaécondiçãosuficienteparaqueacomunicaçãosejabem-sucedida;

c)quandooperadaemfluxodescendente,acomunicaçãoéconsideradaformal;

d)acomunicaçãoinformaldeveserevitadaedesprezada;

e)ousodomelhorcanaldisponíveleliminaaocorrênciaderuídos.

Essaquestãofoimuitocriticadanaépoca.Muitosrecursosforamfeitos,masa

bancanãomudouoentendimento.Existemalgunsautoresquesóconsideram

comofonteoemissor.Entretanto,comoacomunicaçãodeveseruma“estradade

mãodupla”,ouseja,deveteroretorno,algunsautoresconsideramtantoo

emissorquantooreceptorcomofontes.

Dessamaneira,abancaconsideroualetraAcomocorreta.

QuantoàalternativaB,redigircomclarezaécondiçãonecessária,masnão

suciente.Portanto,essaopçãoestáincorreta.Nãoéporqueacomunicaçãoé

descendentequetenhadeserformalsempre.Dessaforma,aletraCestáerrada.

NaletraD,acomunicaçãoinformalé,sim,importante.Assimsendo,aopção

estáerrada.Finalmente,oruídonuncaéeliminadototalmenteemuma

comunicação.Portanto,aletraEtambémestáequivocada.Onossogabaritoé

mesmoaletraA.

QuestõesComentadas

4.(FGV/Badesc/Analistaadm./2010)Asalternativasaseguirapresentamexemplodecomunicação

eficaz,àexceçãodeuma.Assinale-a.

a)Osignificadodamensageméconsistente.

b)Háconsequênciasapósacomunicação.

c)Ocanaldecomunicaçãonãotemruído.

d)Odestinatárioforneceretroaçãoaoemissor.

e)Acomunicaçãoétotalmentecompletada.

Nãoexistecomunicaçãosemalgumtipoderuído!Esseelemento,mesmoque

nãoprejudiquedemaisacomunicação,sempreestápresenteemalgumnível.Dessa

maneira,ogabaritoéaletraC.

5.(Esaf/MPOG/EPPGG/2009)Elementobásicoparaainteraçãosocialeodesenvolvimentodas

relaçõeshumanas,acomunicaçãodesempenhapapelfundamentalparaaefetivaçãodeplanos

eprogramasemqualquerambienteorganizacional.Porissomesmo,écorretoafirmarque:

a)acomunicaçãodeveseprestaràdefesaincondicionaldaorganização,semlevaremcontaos

interessesdeseusdiversospúblicos,internoseexternos;

b)emorganizaçõescomnslucrativos,acomunicaçãomercadológicadeveserpriorizadaem

detrimentodascomunicaçõesinstitucionaleinterna;

c)oplanejamentoestratégicodecomunicaçãodeveconsideraraculturaorganizacionalcomoumfator

determinantedosprocedimentosaseremadotados;

d)acomunicaçãoorganizacionaldeveserlevadaaefeito,exclusivamente,porespecialistasdaárea,de

preferêncialotadosemumaassessoriavinculadaàaltagerência;

e)pornãodisponibilizarembenseserviçosaomercado,organizaçõespúblicaspropriamenteditas

devemapenassepreocuparcomacomunicaçãointerna.

AletraAestáincorreta,poisacomunicaçãoorganizacionalnãodeveseprestar

àdefesaincondicionaldaempresa,semlevaremconsideraçãoosdemais“atores”.

Dessaforma,seriaomesmoqueaSouzaCruzdefenderqueofumofazbemà

saúde.Essetipodedeclaraçãosótrariamaiscríticasàprópriaempresa,nãoé

verdade?Assimsendo,osoutrospúblicosdevemserconsiderados.

NaletraB,nãoexisteessaprioridadeparaacomunicaçãomercadológica.

Portanto,aalternativaestáincorreta.AletraCestácorretaeéonossogabarito.

JáaopçãoDestáequivocada,poisacomunicaçãonãodeveserumaexclusividade

dealgumsetorespecífico(comoumaáreadeRelações-Públicas).

Porm,aletraEéamaisfácil,poisobviamenteasentidadespúblicasnãosó

podemcomodevemsecomunicarcomseupúblicoexterno,nãoéverdade?O

gabaritoémesmoaletraC.

6.(Cespe/Cespe/TJCE/Técnicoadm./2008)Oexcessodemensagensenviadasaosdestinatárioseo

usodelinguagemrebuscadaporpartedoemissortendemaprovocarruídosnoprocessode

comunicaçãoorganizacional.

Perfeito.Oexcessodeinformações(pensebem:dequantoscomerciaisde

televisãovocêconsegueselembrardepoisdepassadaumahora?)ealinguagem

sãoruídosquecriambarreirasàcomunicação.Ogabaritoéquestãocorreta.

7.(FCC/Arce/Analistareg./2006)Nalinguagemverbalháhabilidadesdecomunicaçãocodicadorase

decodificadoras.Sãodecodificadorasashabilidadesde:

a)leituraeaudição;

b)escritaeapalavra;

c)leituraeescrita;

d)audiçãoeapalavra;

e)leituraeapalavra.

Maisumaquestãodecodicaçãoedecodicação.Ashabilidadesdeescritaeda

palavrasãorelacionadasàcodicação.Jáashabilidadesdeleituraedeaudiçãose

referemàdecodificação.Dessaforma,nossogabaritoéaletraA.

8.(Cespe/Cespe/TCU/ACE/Educaçãocorp./2007)Autilizaçãodainternetdeveserincentivadana

organização,porserumcanaldecomunicaçãodegrandeefácilalcanceequeseaplicaa

qualquertipodeinformaçãoqueaorganizaçãonecessitadisseminar.

Comovimos,cadacanaltemumpropósitoespecíco.Ainterneté,sim,um

canalquedeveserincentivado,masnãoseprestaatodotipodeinformaçãoque

umaorganizaçãonecessitedisseminar.

Existemdiversoscasosemqueacomunicaçãodeveserpessoal,porexemplo.

Imaginereceberainformaçãodequevocêfoidemitidonositedaempresa.Você

gostariadisso?Assim,ogabaritoéquestãoerrada.

9.(FCC/TJ/AP/Analistaadm./2009)Nointuitodemelhoraracomunicaçãointerpessoaleintergrupal

numaorganizaçãodegrandeportedeve-se:

a)aperfeiçoarosuxosdescendenteseformaisdecomunicaçãoescritaparamelhoraraimagemda

direção;

b)centralizarosfluxosascendenteseinformaisdecomunicaçãooralvisandoelevarocontrolegerencial;

c)estimularosuxoshorizontaisdecomunicaçãoinformaleoralentretodososfuncionáriosemtorno

demetaseprojetos;

d)incentivaracomunicaçãoformaleescritaentreosaltosdirigentesdossetoresdeMarketingeRH;

e)reduzirosuxoslateraisdecomunicaçãoformaleinformalparaimpediraschamadas“centraisde

boatos”.

Paraqueumaorganizaçãodegrandeportepossamelhoraracomunicação

intergrupaleinterpessoal,ointeressanteseriaaumentarouxohorizontal(entre

aspessoaseosdepartamentos)efacilitaracomunicaçãoinformal.Dessaforma,

nossogabaritoéaletraC.

Osuxosverticais(ascendentesedescendentes)eacomunicaçãoformalnãose

caracterizampormelhoraracomunicaçãointergrupal.

10.(FCC/Defensoria/SP/Administrador/2010)Comrelaçãoàimportânciadofeedbacknoprocesso

decomunicaçãointerpessoalnasorganizações,considereasafirmativasaseguir.

I.Paraserefetivoofeedbackdeveserdescritivoaocontráriodeserumprocessodeavaliação.

II.Ofeedbackémaisútilquandosolicitadoeoportuno,istoé,quandofeitonomomentodo

comportamentooudofatoemquestão.

III.Devesercompatívelcomasmotivaçõeseobjetivosoemissor,mesmoquesejaexpressona

formadeumdesabafo.

IV.Deveserdirecionadoàscaracterísticaspessoais,idiossincrasias,limitaçõesderaciocínioe

outrasmanifestaçõesindividuaisquepodemserapontadascomofalhas.

V.Deveserespecíficoaocontráriodeverbalizarumageneralização.

EstácorretooqueseafirmaAPENASem

a)IeV;

b)I,IIeV;

c)I,IeV;

d)II,IVeV;

e)I,II,IVeV.

Aprimeiraalternativaestácorreta,poisaofazerumfeedbackoavaliadordeve

serdescritivoenãofazeruma“crítica”.Noprimeirocaso,ofeedbackpoderiaser

assim:

–“Vocêconseguiualcançar700contratosfechadosnomês,mascoua80%da

metadesejada”.

Jáumacrítica,oufeedbackavaliativo,poderiasoarmal.Imagineumgerente

comunicandoumaavaliaçãoassim:

–“VocêestámuitopiordoqueoJoão,hein?Estáfaltando“garra”..

Vocênãogostariadereceberumfeedbackdesses,nãoémesmo?Dessaforma,o

feedbackdescritivonãodeixaoavaliadocomumaposturadefensiva,ok?

Asegundafrasetambémestácorreta,poisofeedbacknãodeveserimposto,mas

simapresentadonomomentocerto.Jáaterceirafraseestáerrada.Ofeedbackdeve

sercompatívelcomasmotivaçõeseosobjetivostantodoavaliadorcomodo

avaliado.Umdesabafopoderiasermuitoprejudicial,ok?

Imagineouvirdeumchefeestedesabafo:“Masvocês,hein,sãotodosunslerdos!

Sófazemmepuxarprabaixo!”

Vocêgostariadeouvirisso?Seriaprodutivo?

Aquartafrasetambémestáerrada.Osaspectosquedevemserapontadossãoos

quepodemseralteradosemelhorados.Característicaspessoais(altura,pesoetc.)

nãopodemserapontadascomofalhas,ok?

Aquintafraseestáperfeita.Aavaliaçãodeveserespecícaenãogeral.Deve

apontaropontoemqueofuncionáriodeveseaprimorar,enãoumacríticageral.

Imagineouviralgocomo:

–Vocêémuitoteimoso!

Issonãoapontaclaramenteoqueapessoadevemudar,nãoéverdade?Portanto

nãoéprodutivo.Dessaforma,nossogabaritoémesmoaletraC.

11.(Cespe/Cespe/MPU/Anal.administrativo/2010)Umdosobstáculosàcomunicaçãonoprocesso

organizacionaléaavaliaçãoprematuradamensagempeloreceptor.

Atendênciadoreceptordeprejulgaramensagemqueoemissorestábuscando

comunicaréumadasbarreirasmaiscomunsquedicultamaecáciadeuma

comunicação.

Emumasituaçãoassim,apessoaquerecebeamensagemnãocostumase

preocuparmuitoemouvircorretamenteoqueestásendopassado,masemcomo

responderá.Dessaforma,podenãocompreenderperfeitamenteosentidoda

mensagem.Ogabaritoéquestãocorreta.

QuestõesPropostas

12.(Cespe/TRT-16/Técnico/2005)Karen,técnicajudiciária,trabalhanosetordetelefoniadoTRTe

tementresuasatribuiçõesaatividadedeprestaçãodeinformaçõesacercadatramitaçãode

processosnotribunal.Karenémuitoatenciosaecriteriosaeseexcedeaodaros

esclarecimentossolicitados;elaexplicaváriasvezesomesmoassuntoparaqueoclientepossa

entenderbemasuaresposta.Nessasituação,acondutadeKaren,nasuacomunicaçãocomo

público,ébastanteeficiente.

13.(FCC/Bacen/Analista/2005)NaTeoriadaComunicação,umdospontosdemaiorimportânciaéa

preocupaçãocomapessoaqueestánaoutrapontadacadeiadecomunicação:oreceptor.

Trata-sede:

a)empatia;

b)efetividade;

c)atitude;

d)feedback;

e)diretividade.

14.(Cespe/TCU/ACE/2008)Acomunicaçãonoserviçopúblicoestásujeitaaalgumasfalhas

caracterizadaspelosautorescomodistorção,quando,porexemplo,ascheasnãotransmitem

aorientaçãonecessáriaàrealizaçãodastarefasatribuídasaoservidor,ou,então,como

omissão,quandoaquantidadedeinformaçõestransmitidasexcedeacapacidadedo

destinatáriodeprocessá-lasadequadamente.

15.(Cespe/TRE-AL/Técnico/2004)Muitoutilizadaemtarefascomplexas,devidoaseuperlde

descentralização,acomunicaçãoformaltemsidocadavezmaisutilizadaemorganizaçõesque

passamporprocessosdeflexibilizaçãoorganizacional.

16.(Cespe/TRE-AL/Técnico/2004)Acomunicaçãoécapazdeinuenciarosindivíduosnabuscados

objetivosorganizacionaisefazercomqueelessecomprometamcomaorganização.

17.(FCC/Bahiagas/Administrador/2010)Noprocessodecomunicação,apercepçãoeinterpretação,

porpartedoreceptor,dosignificadodamensagemrecebidaédenominada:

a)codificação;

b)feedbackpositivo;

c)decodificação;

d)tautologia;

e)resposta.

18.(Cesgranrio/Eletrobras/Administrador/2010)Umaadequadagestãodepessoasenvolveuma

cuidadosaseleçãodecanaisdecomunicaçãoerelacionamentocomcolaboradores.Oscanaisde

comunicaçãopodemserhierarquizadosemfunçãodesuacapacidadequantoa

•lidarcommúltiplossinais,simultaneamente;

•facilitarumfeedbackrápidodeviadupla;

•estabelecerumfocopessoalparaacomunicação.

O(s)canal(ais)decomunicaçãoqueatende(m)adequadamenteàstrêscapacidadesde

transmissãodeinformaçõesé(são):

a)conversaaotelefone;

b)conversafaceaface;

c)e-maileintranet;

d)relatórioseboletins;

e)memorandosecartas.

19.(Cespe/Previc/Anal.Administrativo/2011)Acomunicação,basedequalquerprocesso

administrativoemumaorganização,inuenciafortementeaimageminstitucional,porisso

devecomporoplanejamentoestratégicodaorganização.

20.(Esaf/AFCCGU–Desen.inst./2008)Acomunicaçãoéumexercíciodemútuainuênciapresente

nasrelaçõeshumanasdetodaordem.Nasorganizações,assumevitalimportânciaparaque

metaseobjetivossejamatingidos.Selecioneaopçãoqueexpressacorretamenteconceitos,

elementos,barreirasoutiposdecomunicaçãonasorganizações.

a)Aescolhadocanal,característicaspessoais,coerênciaentreotomdevozeacomunicaçãoverbal

podemserbarreirasdecomunicaçãopresentesnoreceptor.

b)Areuniãoéummecanismodecomunicaçãoorganizacionalque,alémdeconteúdoclaro,deveser

conduzidadeolhonastarefasenosrelacionamentos.

c)Adecodicaçãodeumainformaçãoestásujeitaaltrosporpartedoemissor,queseleciona,avalia,

interpretaedecideousoquefarádamensagem.

d)Informar,esclarecer,comandar,avaliardesempenhosesituações,motivarepersuadirsãoalgunsdos

requisitosdeumaboacomunicação.

e)Oscanaisdecomunicaçãoinformal,nasorganizações,podemserverticais–descendentese

ascendentes–ouhorizontais.

(Cespe/MPU/Técnicoadministrativo/2010)Paulo,novodiretordeumaorganizaçãopública,

pretendedesenvolverumsistemadecontrolecapazdeapontarerroscometidosdurantea

execuçãodosserviços.Paraaconsecuçãodeseuobjetivo,deniunovasformasdecontrole

combaseeminformaçõesquecoletoupessoalmente,aointeragircomcolaboradoresdetodos

ossetoresdainstituição,semserestringiraosmétodostradicionaisdeobtençãodedados.

Considerandoessasituaçãohipotética,julgueosseguintesitens,quedizemrespeitoao

processoorganizacional.

21.Aocoletarasinformaçõesparadenirasnovasformasdecontrole,Pauloprivilegiououxo

comunicativocircular.

22.Paraatenderàsdemandasmaisinstáveiseurgentesdaorganização,Paulodeveutilizararede

formaldecomunicação.

23.(Cespe/MS/Administrador/2010)Aorganizaçãonãogovernamental(ONG)VivercomSaúde

possuiprogramasdeincentivoàpráticadesportiva,demonitoramentodasaúdebucal,de

divulgaçãosobreaprevençãodedoençassexualmentetransmissíveis,entreoutros.Areferida

ONGatuaexclusivamentenoDistritoFederal(DF),maspretendeampliarsuaáreadeatuação

paraoestadodeGoiásemfacedarealidadeprecáriadosfornecedoresdeserviçosdesaúde

daspequenascidadesdoentornodoDF.Talampliaçãoimplicaráumaumentode70%nos

custosdaorganização.Paratanto,aONGterádetomardecisõesacercadoseuprocesso

organizacional.

Apartirdasituaçãohipotética,julgueoitemaseguircomrelaçãoaoprocessoorganizacional.

Considereaseguintesituaçãohipotética.NonaldasegundaetapadoprogramaSaúdeBucalnas

Escolas,apresidênciadaONGVivercomSaúdedecidiufazerumchurrasco,aproveitandoa

proximidadedasfestasdenaldeanoparacomemorarasuperaçãodametaanualdo

programaem200%.Seupresidenteencaminhouconvitesparacadacheadosvariados

departamentos,solicitandoquerepassassemoconviteparaosseussubordinados.Nessa

situação,oconviteparaochurrascoéconsideradoumacomunicaçãoinformalevertical,pois

foirealizadoentreníveisdiferentes,masdeumamesmaáreadeatuação.

24.(Cespe/TCU/ACE/Educaçãocorp./2007)Alémdefornecerainformaçãodequeaspessoas

necessitampararealizaroseutrabalho,acomunicaçãopodeauxiliaraorganizaçãoacontrolar

ocomportamentodosmembrosdaequipe,alémdemotivá-losparaotrabalhoepossibilitara

catarsedesuasemoções.

25.(Cespe/TCU/ACE/Educaçãocorp./2007)Aorganizaçãodeveestimularasredesinformaisde

comunicaçãoporque,pormeiodelas,podefortalecerasrelaçõesentreasequipesetornaro

processodecomunicaçãomaisvariadoemaisbemadministradopelosgestores.

26.(Cespe/TJCE/Técnicoadm./2008)Énecessárioqueoprocessodecomunicaçãoorganizacional

sedêpeloscanaisformaisestabelecidos,masoscanaisinformaisdecomunicaçãoque

preexistemouquepodemseformarnaorganizaçãodevemserconsideradosnesseprocesso.

27.(Cespe/TJCE/Técnicoadm./2008)Nocontextoorganizacional,oprocessodecomunicação

grupaltemcomoobjetivosatrocadeinformaçõeseacoordenaçãodasatividadesentreos

membrosdogrupo.

28.(Cespe/TRT-16/Técnico/2005)Jorge,técnicojudiciário,atuanaáreadesegurançaetransporte

doTRT.Noexercíciodocargo,Jorgedeve,entreoutrasatribuições,asseguraraintegridade

físicadasautoridades,servidoresedemaispessoasqueconduz,zelarpelamanutençãode

veículoseequipamentossobsuaguarda,atuarnasegurançapatrimonialdasinstalaçõesdo

tribunal,

scalizaracirculaçãodepessoas,controlaraentradaesaídademateriaise

equipamentosnasdependênciasdoTRT.AequipedetécnicosjudiciáriosdequeJorgefaz

parteécompostapor15servidores,dosquaisJorgeéumdosmaisresponsáveiseprocura

exercersuasatribuiçõescomcompetência.AqualidadedotrabalhodeJorgeémais

inuenciadapelaapresentaçãoehigienepessoalquepelahabilidadedecomunicaçãoverbal,

poisesteéumatributodispensávelàsuafunção.

29.(Cespe/TRT-16/Técnico/2005)Daniel,técnicojudiciáriodoTRT,éresponsávelpelarealizaçãode

contatosascendentesedescendentes,internoseexternosdotribunal.Paracumprirbemsuas

atribuições,Danielpodefazerusodevárioscanaisdecomunicação.Nessasituação,aseleção

docanalaserutilizadoporDanielindependedotipodemensagemasertransmitida,mastem

deconsiderarascaracterísticasdoreceptoreadisponibilidadedocanal.

30.(Cespe/TRT-16/Técnico/2005)Considereaseguintesituaçãohipotética.Albertino,servidor

público,aosecomunicarcomseuchefe,manipulaasinformaçõesparaqueelassejammais

bemaceitasporele.Nessasituação,Albertinoestáprejudicandosuacomunicaçãocomochefe,

poisestácolocandoumabarreirainterpessoalnoprocessodecomunicaçãocomele.

Gabaritos

1.C

11.C

21.C

2.E

12.E

22.E

3.A

13.A

23.E

4.C

14.E

24.C

5.C

15.E

25.E

6.C

16.C

26.C

7.A

17.C

27.C

8.E

18.B

28.E

9.C

19.C

29.E

10.C

20.B

30.E

Bibliografia

Chiavenato,Idalberto.Administraçãonosnovostempos.2aed.RiodeJaneiro:

Elsevier,2010.

Daft,RichardL.Management.Mason:Thomson,2005.

Macêdo,IvanildoIzaiasde,DenizeFerreiraRodrigues,MariaElizabethPupe

Johann&NeisaMariaMartinsdaCunha.AspectoscomportamentaisdaGestãode

Pessoas.7aed.RiodeJaneiro:FGV,2007.

Robbins,StephenP.OrganizationalBehavior.11aed.UpperSaddleRiver:Pearson-

Prentice-Hall,2004.

SchermerhornJr.,JohnR.Management.9aed.Hoboken:Wiley&Sons,2008.

Capítulo10

GestãodasMudanças

10.1.Criatividadeeinovaçãoorganizacional

10.1.1.Criatividade

Acriatividadepodeserdescritacomoahabilidadedegerarideiasoriginais

ounovasperspectivasdeideiasexistentes.1Normalmente,relacionamosa

criatividadeaobrasdearteouprojetosartísticos(comoumamúsicaouumlme).

Masacriatividadetambémémuitoimportantenocontextodasorganizações.

Entretanto,aocontráriodomundodasartes,umaideiacriativadevetambémser

útilnoambienteorganizacional,ouseja,deveterumaaplicaçãoprática.

Dessaforma,acriatividadeestáligadaàsnovasvisõesenovasmaneirasdese

analisarumproblemaouumaquestão.2Relaciona-secomarupturados“velhos

modos”eabordagensquepodemnãomaisestarfuncionando.

Osgestoresfrequentementeestãobuscandonovasmaneirasderesolveros

problemasorganizacionais,demodoqueassuasempresaspossamsobrevivere

superarseusconcorrentes.

Eparaissonecessitamtercriatividade.Precisamverasquestõesporoutro

ânguloeternovasideias,queajudemaorganizaçãoaatingirresultadosmelhores.

DeacordocomCerto,3acriatividadeindividualéfunçãodetrêselementos:a

especializaçãooucompetência(expertise),opensamentocriativoeamotivação.

Aespecializaçãooucompetênciaestárelacionadacomosconhecimentose

habilidadesqueoindivíduoadquiriu,sejademodoformalouinformal,dentrodo

contextodoseutrabalho.

Dessamaneira,umgerentedebanco,porexemplo,deveterconhecimentosde

matemáticananceira,decomofuncionaosistemananceiro,dasprincipais

regrasdeimpostoderendaetc.Semconhecerprofundamenteumprocessoou

produto(paraoqueserve,quemsãoseusclientes,comoéfeitoetc.),carámuito

difícilimaginarumanovamaneiradefazê-lo,nãoémesmo?

Jáopensamentocriativoestárelacionadocomacapacidadedaspessoasde

usaressesconhecimentosehabilidadesadquiridosdenovasformas.Quantomais

questionadoroindivíduofor,maiscriativoeletenderáaser.

Assim,umapessoadevesesentir“àvontade”aocontestarummodoarraigado

deconduta,ouumprocessogerencialdeumaempresa.Umgerentedebanco

criativopoderia,porexemplo,“bolar”umanovamaneiradevenderseguros,ou

“sugerir”acriaçãodeumnovoseguro(comoumseguro-atropelamento,por

exemplo!).

Masnadadissoresolvesenãoexistirmotivaçãoparaacriatividade.Ouseja,os

conhecimentosehabilidades,somadosaopensamentocriativo,nãosãosucientes

paraqueexistaacriatividade.Nãobastatercapacidadedecriar,apessoatemde

querercriar!

Essamotivaçãopodeacontecerdeduasformas:demaneiraintrínseca(derivada

deumanecessidadeprópriaderealizaçãodapessoa,deumsentimentodequeseu

trabalhoéimportanteetc.)oudemaneiraextrínseca(atravésdepromoções,

aumentossalariais,prêmios,bônusetc.).

Amotivaçãointrínsecaaparecequandooempregadosentequeseutrabalhoé

importante,queévalorizadopeloschefesecolegas,quandootrabalhoé

desaador,entreoutrosmotivos.Dessamaneira,amotivaçãovemde“dentro”do

indivíduo.

Jáamotivaçãoextrínsecaocorreporincentivosexternos.Quandorecebemos

umbônusparaatingirumameta,porexemplo,somos“motivados”parafazeralgo

quenãoteríamosvontade“interna”defazer.Assim,asorganizaçõesbuscam

motivarosseusfuncionárioscominstrumentostantointrínsecosquanto

extrínsecos.

Figura10.1–Elementosdacriatividade

10.1.2.Inovação

Ainovaçãoemumaempresadependentedacriatividade.Ouseja,asideias

geradaspelacriatividadeserãoutilizadasnoprocessodeinovação.Portanto,a

inovaçãoseriaoprocessodepôrempráticaessasnovasideias.4

DeacordocomCerto,5ainovaçãoconsistenousodeumanovaideiaparaa

utilizaçãoemumnovoprocesso,produtoouserviço(ounamelhoriadeum

processo,produtoouserviçoexistente),demodoaqueaorganizaçãopossa

atingirseusobjetivos.

Ainovação,dentrodocontextoorganizacional,podeocorrerdetrêsformas:6

inovaçãonosprodutoseserviços,inovaçãonosprocessosorganizacionaise

inovaçãonosmodelosdenegócio.

Umareorganizaçãonoprocessodeentregadeumproduto,deformaqueele

cheguemaisrápidoaoclientenal,éumexemplodeinovaçãoemumprocesso

organizacional.

Jáautilizaçãodeumanovatecnologia,comoatelasensívelaotoque,parao

lançamentodeumnovocelularquenãonecessitemaisdeumteclado(comofoio

casodoiPhone),seriaumexemplodeinovaçãoemprodutoseserviços.

Finalmente,autilizaçãodeumnovocanaldedistribuição,marketingevenda,

demodoquesuaempresanãoprecisemaismanterlojasfísicasfuncionandopara

atingirsuaclientela(comofoiocasodaNatura),seriaumexemplodeinovação

nomodelodenegócios.

Figura10.2–Formasdeinovação

10.1.3.InovaçãoIncrementaleRadical

DeacordocomLemos,7existemdoistiposdeinovação:aincrementalea

radical.Ainovaçãoradicalseriaaintroduçãodeumnovoproduto,serviços,

processoouformadeorganização.

Assim,setratadealgoquevenhaa“romper”comoqueexistiapreviamente.De

acordocomaautora,aintroduçãodamáquinaavaporedamicroeletrônicaseria

exemplodeinovaçõesradicais,emcomparaçãocomoqueexistiaanteriormente.

Jáumainovaçãoincrementalseriarelacionadacomasmelhoriascontínuasque

sãonecessáriasparaqualquerorganizaçãosemanteratualizadaecompetitiva.Éa

melhoriadealgoquejáexiste!

Assim,aintroduçãodainjeçãoeletrônicanoscarrosbrasileirosnosanos1990

foiumainovaçãoincremental,poisapenasmelhorou,marginalmente,o

desempenhodosmotoresacombustãoquejáexistiam.

Jáaintroduçãodosmotoreselétricos,quepodemserabastecidosnatomadade

suacasa,seriaumainovaçãoradical,poismudacompletamentecomo“modelo”

existente.

Nãomaisseránecessáriooabastecimentoempostosdegasolina(que

desaparecerãoouserãoreduzidosemgrandenúmero),eaindústriadopetróleo

seráprejudicadaseriamente,porexemplo.

Figura10.3–Tiposdeinovação

Entretanto,nãosedeveteranoçãodequeainovaçãoradicalé“melhor”doque

aincremental.Asduassãoimportantes,poisainovaçãoincrementalé

fundamentalparaqueosprocessos,produtoseformasorganizacionaiscontinuem

sendoaprimoradosenquantonãoocorreminovaçõesradicais.8

Vamosvercomoissojáfoicobrado?

1.(FGV/Sefaz/RJ/Fiscalderendas/2009)Assinaleaalternativaquenãopodeserumacaracterística

deinovaçãoincremental.

a)Avançocontínuo.

b)Criaçãodenovosmercados.

c)Inovaçõescriadaspormeiodasestruturasedecisõesorganizacionaisnormais.

d)Melhoriastecnológicas.

e)Melhoriasemprodutos.

AalternativaAéumexemplodeinovaçãoincremental,poisoavançocontínuo

éumadascaracterísticasdessetipodeinovação.Jáacriaçãodenovosmercadosé

umexemplodeinovaçãoradical.Assim,essaopção(letraB)estácorretaeéo

gabarito.

AletraCestáincorreta,poisserefereainovaçõesemestruturas“normais”,o

quepassaaideiadealgojáexistente.JáasletrasDeEpoderiamser“encaixadas”

emqualquerdosdoistiposdeinovação.

Nomeuentender,essasalternativasapenasconfundemocandidatoedeveriam

tercausadoaanulaçãodaquestão.Apesardisso,ogabaritoémesmoaletraB.

10.2.MudançaOrganizacional

Comooambientequeenvolveasorganizaçõesestásempremudando,é

necessárioqueessasempresasmudemparapoderencararessesdesaos.Novas

tecnologias,novoscompetidores,novasdemandasdosusuários,entreoutras

mudanças,podemgeraranecessidadedemudaralgumaspectodainstituição.

DeacordocomCerto,9oprocessodemudançadeumaorganizaçãodevebuscar

amelhoradesuaefetividade.Essasmudançaspodemenvolverqualquersegmento

daempresa,masgeralmenteafetamaslinhasdeautoridade,osníveisde

responsabilidadedediversosmembrosdaorganizaçãoeaslinhasestabelecidasde

comunicação.

Apesardeconsiderarmosacriatividadeeainovaçãoemumaorganização

fatoresimportantesparaqueelasemantenhacompetitiva,asmudanças

necessáriasemumaempresanemsempresãofáceis.

Muitosfatorespodemfazercomqueoprocessodemudançaorganizacionalnão

sejaexecutadocomsucesso.Paraqueesseprocessoaconteça,énecessárioo

envolvimentodetodos.

10.2.1.AgentesdeMudança

Paraquequalquerprocessodemudançapossatersucesso,éimportanteque

existamlíderescomprometidoscomamudança.Umlíderdessetipodeve

incentivaratomadaderiscosporseusfuncionárioseestarsempreabertoanovas

ideiasecontribuiçõesdeseussubordinados.

Elenãodeveveramudançacomoumaameaçaparasiouparaseuórgão,e,

alémdetudo,deveserproativo.Dessamaneira,nãopodeesperarquealgoocorra

(porexemplo,aentradadeumnovoprodutoconcorrente)parainiciaroprocesso

demudança.

Schermerhorn10comparaesseslíderesagerentesquesãoapegadosàsituação

atual(statusquomanagers,eminglês).Essesgerentessesentemameaçadospelas

novasideiaseporsubordinadosquebuscamo“novo”.

Essasmudançassempreacarretarãomaistrabalhoeriscos.Assim,essegerente

prefere“esperarparaveroqueacontecerá”enãotomarumaatitude.Assim,ele

temumaposturareativa.

Figura10.4–LíderesdemudançaXGerentesdostatusquo.

(Fonte:SchermerhornJr.,2008)

10.2.2.ModelosdeMudança

Oidealéquetodosnaorganizaçãoestejamenvolvidosnoprocessodemudança.

Entretanto,muitasvezesopapeldeiniciadordoprocessocabe,emgrandeparte,

aonívelestratégico.Jáemoutrasempresas,aresponsabilidadeestádistribuídaao

longodosníveishierárquicosmaisbaixos.

10.2.3.MudançadeCimaparaBaixo(Top-Down)

Nessemodelo,asmudançassãoiniciadaspelonívelestratégico.Osdiretores

definemoquedevesermudadoecomodeveserefetuadaamudança.

Esseéométodomaiscomumnasorganizações,mascostumatermuita

diculdadedealcançarsucesso.Issoocorrequandoadireçãodaempresanão

consegue“vender”aideiademudançaaosgerentesefuncionáriosqueterãode

implementarasmudanças.

Assim,quandoosfuncionáriosnãoentendemperfeitamenteoquedeveser

feito,ouquandopercebemasmudançascomoalgoqueseránegativoparaeles,

nãoapoiarãoasmedidas,eoprocessodemudançatenderáanãofuncionar.

10.2.4.MudançadeBaixoparaCima(Bottom-Up)

Nessemodelo,oprocessodemudançasnãoestálocalizadonacúpulada

empresa,masnosseusníveisinferiores.Naverdade,todospodemcontribuircom

novasideiasparaoprocessodemudança.

Assim,umoperárioteriaacapacidadedepropormudançasemsuaáreapara

seugerente.Essemodelotemumagrandevantagemsobreotop-down,poisos

funcionáriosquetrabalhamdiretamentecomastarefastêmumconhecimento

muitomaiordosdetalhesedarealidadedosprocessosatuaiseseusproblemas.

Alémdisso,comoasideiaseasmudançassãoiniciadaspelosmesmos

funcionáriosqueterãodeexecutá-las,nãoétãonecessáriooprocessode

convencimentodosméritosdessasmudanças.

10.2.5.MudançaIntegrada

Nessemodelotenta-sejuntarasvantagensdosdoisoutrosmodelos.Existem

situaçõesemqueaforçadacúpuladaorganizaçãoénecessáriapara“romper”com

certosentravesouparafazerumaalteraçãomaisdrásticanaoperaçãodaempresa.

Domesmomodo,semaparticipaçãodosfuncionáriosnoprocessocadifícil

implantarasmudançasecriarumambientedeaprendizadoconstanteemudanças

sustentáveisemtodaacadeiadevalor.11

Dessaforma,omodelointegradobuscaaliartantoasmudançasincentivadas

pelacúpulaquantooenvolvimentodosfuncionáriosnoprocessodemudanças.

10.2.6.ResistênciasàsMudanças

Asresistênciasaosprocessosdemudançasãoextremamentecomuns.Essas

resistênciaspodemserdesdeasderivadasdafaltadeconhecimentosobrea

extensãodasmudançasatéosimplesmedododesconhecido.

Algumasoutrascausasfrequentesdasresistênciassão:odesconfortocoma

mudançadehábitosjáarraigados,odesgastequeasmudançascausamnos

funcionários(decorrentesdenovosconhecimentosqueserãonecessários,novas

práticasetc.)eacrençadequeasmudançaslevarãoaumasituaçãodesfavorável

aosfuncionários(comoademissãodepessoas).

Oslíderesdevemsaberlidarcomasresistênciasdosfuncionários.Cadatipode

resistênciadeveterumcomportamentoadequado.Dessaforma,nãodevemosagir

demodobruscoquandoomotivodaresistênciaéodesconhecimentodosplenos

demudança,nãoé?

Aomesmotempo,seapessoaaindaéresistentedepoisdeváriastentativasde

convencimentoeincentivo,muitasvezesacoerção(ameaça)éaúnicasaída.Vejaa

seguirosprincipaismodosdeseencararumaresistência:12

ØEducaçãoecomunicação–parafacilitaracompreensãodosfuncionários

sobreosplanosdemudançaesuaimportância.

ØParticipaçãoeenvolvimento–paraincluirsugestõesdosfuncionáriose

aumentarseucomprometimento.

ØFacilitaçãoesuporte–envolvetreinamentoeincentivosparaqueos

funcionáriossesintamcompreendidosemsuasdificuldades.

ØNegociação–ofertadevantagenseincentivosmateriaisemtrocada

aceitaçãodasmudanças.

ØManipulação–tentarinuenciarosfuncionáriosatravésdamanipulaçãoda

informação.

ØCoerção–Quandonadafunciona,podeserocasodeameaçaros

funcionáriosresistentesparaqueelescumpramasordenseimplementemas

mudanças.

10.2.7.MudançasPlanejadas

Comoaspessoassãonaturalmenteresistentesàsmudanças,asorganizações

precisamdesenvolverumaestratégiaparaimplementá-las.Umdessesmétodosfoi

desenvolvidoporKurtLewin:13

Essemétododemudançaplanejadatemtrêsfases:odescongelamento,a

mudançaeorecongelamento.Aseguirvemoscadaumadelasemdetalhes:

Descongelamento–Lewinacreditaqueaempresadeveprimeirotornara

necessidadedamudançaevidenteatodos.Ouseja,comunicarosmotivosdas

mudanças,envolveraspessoasno“problema”ebuscarreduzirasresistências

iniciaisaoprocessodemudança.Assim,amudançapoderiaseralcançadade

modomaisfácil.

Mudança–nessafase,amudançaemsiseriaexecutada.Entreosaspectosque

podemsermudadosestão:astarefas,aspessoas,aculturaorganizacional,a

tecnologiaeasestruturasorganizacionais.

Recongelamento–nãoadiantamudarascoisaspara,apósoesforçoinicial,

tudovoltarasercomoeraantes.Oprocessodeveserconsolidado.Assim,Lewin

indicaquedeveserfeitoumtrabalhodereforçoparaqueamudançaseestabilize.

Paraqueamudançaseconsolide,osnovoshábitosdevemserreforçados,seja

atravésdetreinamentocomoatravésdeincentivos.

Figura10.5–ProcessodemudançaplanejadadeLewin

Vamosvercomoessetemajáfoicobrado?

2.(Esaf/MPOG/APO/2010)Assinaleaopçãoincorreta.

a)Aoadotarummodelodesistemafechado,aorganizaçãotendeaserconduzida,gradativamente,a

níveiscadavezmaisaltosdeentropiaedesagregação.

b)Aspessoasnãoresistemnecessariamenteamudanças.

c)Umaorganizaçãopodemudaremfunçãodoproduto,masnãoemfunçãodomercado,sobpenade

transmutar-seemumanovaorganização.

d)Deumaformageral,asabordagensteóricassobremudançaorganizacionalsugeremanecessidadede

incorporaçãodeprocessosmaisdemocráticoseparticipativos.

e)Mesmoorganizaçõespúblicas,comsuasamarraslegaiseburocráticas,sãopassíveisdemudanças

causadaspeloambiente.

Aprimeirafraseestáperfeita.Umsistemafechadoocorrequandoaorganização

trabalha“voltadaparasimesma”,ouseja,semtrocarenergiacomomeioexterno.

Assim,nãoseadaptaàsmudançasenãoentendequaissãoasnecessidadesde

mudança.

Asegundafraseestácorreta,pois,apesardearesistênciasernormalem

processosdemudança,nãoésemprequeelaocorre.Anal,vocênãoserá,por

exemplo,resistenteaumaideiaquevocêmesmotenha“bolado”,nãoé?

AletraCestáincorretaeéonossogabarito.Asempresaspodemtantomudar

comofoconoprodutoquantoemrelaçãoaomercado.

AletraDestáexata.Comovimos,osprocessosdemudançaqueincluemos

funcionáriostêmmaischancedesucesso,poiselesserãoosresponsáveispela

implementaçãodasmudanças.

AletraEtambémestácorreta,poismesmoasentidadespúblicassãoafetadas

pelasmudançasocorridasnoambienteexternoedevemmudarparaseadaptaraos

novosdesafios.OgabaritoémesmoaletraC.

QuestõesComentadas

3.(FGV/Badesc/Analistaadm./2010)Comrelaçãoàsinovaçõesnosetordeserviços,analiseas

afirmativasaseguir.

I.Asinovaçõesnosetordeserviçospodemserdeprodutos.

II.Asinovaçõesnosetordeserviçospodemserdeprocessos.

III.Asinovaçõesnosetordeserviçospodemserdenaturezaorganizacional.

Assinale:

a)sesomenteaafirmativaIestivercorreta;

b)sesomenteaafirmativaIestivercorreta;

c)sesomenteaafirmativaIIestivercorreta;

d)sesomenteasafirmativasIeIIestiveremcorretas;

e)setodasasafirmativasestiveremcorretas.

Questãotranquila.Comojávimos,asinovaçõespodemserrelacionadasanovos

produtoseserviços,novosprocessosenovosmodelosorganizacionais(formasde

organização).Dessamaneira,todasasarmativasestãocorretas,eogabaritoéa

letraE.

4.(Esaf/MPOG/EPPGG/2009)Aoserdesignada(o)comoresponsávelporumprocessodemudança

organizacional,vocêsaberásercapazdefazê-lodesdequenãoincorranoseguinteerro

conceitual:

a)noâmbitodaadministraçãopública,omarcolegal-burocráticoconstitui-seemumfortelimitador

paraaimplementaçãodemudançasque,deregra,naadministraçãoprivada,dependeriamapenasda

visãodoagentedemudanças;

b)mudançascausamosmaisdiferentestiposdereaçãodosatoresorganizacionais,desdeaadesão

imediataàpropostademudançaatéàresistênciacompletaaqualquertipodemudança,sendoestaa

maisfrequente;

c)aresistênciaàmudançapodesedarnosâmbitosindividualeorganizacional;

d)emborafundamentalemumprocessodemudança,opapeldaaltagerêncianãopodeserconsiderado

comofatordeterminantedesucesso;

e)mudançasqueimplicamalteraçõesnaestruturaorganizacional,comreexonosníveisdeautoridadee

responsabilidade,decorrem,quasesempre,daintroduçãodenovastecnologias.

AletraAestácorreta,poisrealmentearealidadedaAdministraçãoPúblicaé

menospropensaàsmudançasdoqueadaAdministraçãoPrivada.AletraB

tambémnãotraznenhumadiculdadeereeteatendênciaàpossibilidadedeas

mudançastantoseremaceitasquantoseremrejeitadas.

AletraCestáperfeita,poistantoaspessoasquantoasorganizaçõespodem

mudar.AletraDéumpoucoambígua,poisacreditoqueopapeldaaltadireçãoé,

sim,fundamentaledeterminanteparaqueumprocessodemudançamaisgeral

ocorra.Entretanto,mudançaspequenaspodemrealmenteprescindirdeumapoio

daaltacúpuladaempresa.

AletraEestáerrada,poisasmudançasnaestruturanãosãonecessariamente

frutodemudançasnastecnologias.OgabaritoémesmoaletraE.

5.(Esaf/MPOG/EPPGG/2009)Umprocessodemudançaorganizacionalcujoobjetivoprioritárioseja

perseguir,deumlado,amotivação,atitudes,comportamentosindividuais,satisfaçãopessoal

eprossionale,deoutrolado,acoesãoeaidentidadeinterna,introduzindonovosvalorese

hábitosaseremcompartilhadoscoletivamente,deveseranalisadosobaóticadasseguintes

perspectivas:

a)humanaepolítica;

b)culturalepolítica;

c)humanaeestrutural;

d)culturaleestrutural;

e)humanaecultural.

Essaquestãonãotrouxemuitasdiculdadesaoscandidatos.Vejaqueos

aspectoscitadossão:motivação,atitudes,comportamentosindividuais,satisfação

pessoaleprossional(perspectivahumana)eacoesãoeaidentidadeinterna,

introduzindonovosvaloresehábitosaseremcompartilhados(valoresehábitos

compartilhadossãorelativosàculturaorganizacional,nãoémesmo?).

Portanto,ogabaritoéaletraE.

6.(FGV/Badesc/Analistaadm./2010)Comrelaçãoaoprocessodemudançaorganizacional,analiseas

afirmativasaseguir.

I.Odescongelamentosignificaoabandonodevelhaspráticasorganizacionais.

II.Amudançasignificaoaprendizadodenovaspráticasorganizacionais.

III.Orecongelamentosignicaqueasnovaspráticasorganizacionaisnãoforaminternalizadas

pelocomportamento.

Assinale:

a)sesomenteaafirmativaIestivercorreta;

b)sesomenteaafirmativaIestivercorreta;

c)sesomenteaafirmativaIIestivercorreta;

d)sesomenteasafirmativasIeIestiveremcorretas;

e)setodasasafirmativasestiveremcorretas.

Questãotranquilaessaparaquemjáestudouotema!Nodescongelamento

iremosabandonarasvelhaspráticaseconscientizaraspessoasdanecessidadede

mudança.Assim,aprimeirafraseestácerta.

Namudança,iremosimplementarasnovaspráticasnecessárias.Entretanto,no

recongelamento,asnovaspráticasdevem,sim,serinternalizadas,ouseja,

consolidadas.Portanto,onossogabaritoéaletraD.

7.(Cespe/TCU/ACE/Educaçãocorp./2007)Oprocessodemudançaquevisaodesenvolvimento

organizacionaltemcomoestratégia,primeiramente,descongelarostatusquoexistentena

organização,procederàmudançanecessáriae,depois,investirnorecongelamentodanova

realidade.

Perfeito.Comovocêspodemver,oCespeapenasdescreveuomodelode

mudançasplanejadasdeLewin:descongelamento,mudançaerecongelamento.O

gabaritoéquestãocorreta.

8.(Cespe/TCU/ACE/Educaçãocorp./2007)Asestruturasorgânicasinuenciampositivamenteno

processodeinovação

Estruturasorgânicas,emqueaspessoasdetêmmaisautonomiaeliberdadepara

criaresecomunicarcomoutrosfuncionários,realmentesãomaispropensasa

mudanças,aocontráriodasestruturasmecanicistas.Ogabaritoéquestãocorreta.

QuestõesPropostas

9.(FCC/TCE-AP/Analista/2012)Emrelaçãoamudançaorganizacional,considere:

I.Sãopresenciadossimultaneamentetrêscamposdeforçaemumaorganizaçãoemmudança,

quesecaracterizampordesejaramudança,evitaramudançaeimplementá-la.

II.Ahistóriadamudançamostraque,quandoascondiçõesmelhoram,aspessoassemostram

insatisfeitas;nãosecontentamporteremprogredidotantoereclamamsobreoqueainda

devempercorrer.

III.OmodelodemudançadeLewinexpõetrêspassosparaaintroduçãodamudança,

identificadossequencialmentecomomudança,descongelamentoerecongelamento.

IV.Umadiculdadenagestãodamudançaocorrepelofatodeaempresadesejarounecessitar

mudareosindivíduos,emborapossaminicialmentecolocar-seafavor,tendemaresistirà

medidaqueasmudançasocorrem.

EstácorretooqueseafirmaAPENASem

a)I,IeII;

b)I,IIeIV;

c)IeII;

d)IeIV;

e)IIeIV.

10.(FGV/Badesc/Analista/adm./2010)Asalternativasaseguirapresentamfatorescríticosparaas

mudançasorganizacionaiscontemporâneas,àexceçãodeuma.Assinale-a.

a)Velocidade.

b)Integração.

c)Flexibilidade.

d)Inovação.

e)Tamanhoorganizacional.

11.(Cespe/–TCU/ACE/2008)Aconstataçãodequeasintervençõesparaqueaorganizaçãoevolua

noalcancedesuavisãoenvolvemconstantesmudançasdeprocessoseatédeparadigmasfez

osdirigentesdeumaorganizaçãodesenvolveremplanodegestãodamudançaorganizacional.

Nessasituaçãohipotética,aspremissasqueocitadoplanodevepossuirparaobtersucesso

incluemanegociação,noscasosemquearesistênciaàmudançaforumafontedepodere

influêncianaorganização.

12.(Cesgranrio/Bacen/Analista/2010)Amudançaorganizacionaléconsideradaorgânicaporque:

a)osproblemasorganizacionaisnãopodemserresolvidosporpartes,jáqueháumainter-relaçãomuito

forteentreosdiversosprocessos,áreasepúblicosdasempresas;

b)envolveaestruturamecanicista,aestratégiasistêmicaeoambiente;

c)ocorrequandoalteraçõesnoambientejáafetaramodesempenhodaorganização,movidaspor

problemasincrementais,queimitamoutrasmudanças;

d)deveseriniciadapeladireçãoantesqueoproblemaocorra,apartirdaidenticaçãoedacriaçãode

oportunidades;

e)buscaodesenvolvimentodaprevisibilidade,emumambientenoqualoserrosnãosãobemvistose

podemserpunidos.

13.(Cesgranrio/BNDES/Administrador/2008)“(...)contemplananciamentosdelongoprazoe

custoscompetitivos,paraodesenvolvimentodeprojetosdeinvestimentoseparaa

comercializaçãodemáquinaseequipamentosnovos,fabricadosnopaís,bemcomoparao

incrementodasexportaçõesbrasileiras.Contribui,também,paraofortalecimentoda

estruturadecapitaldasempresasprivadasedesenvolvimentodomercadodecapitais.”

Disponívelem:http://www.bndes.gov.br/empresa/default.asp.

OcampodeatuaçãodoBNDES,brevementedescritonotexto,reeteoprocessodemudança

organizacionalpeloqual,emvirtudedaglobalização,muitasempresasbrasileirastiveramde

passar.Nessecontexto,relaciona-secomoprincipalvariávelparaosucessodestapremissade

gestãoo“agentedemudança”,queécompreendidoporconjuntode:

a)práticasorganizacionaisinstitucionalizadasquepermeiamasatitudesdoscolaboradores;

b)forçasqueatuamdinamicamentedentrodasempresas,levando-asàadaptaçãoanovassituações;

c)pessoasqueguiamoprocessoesãoresponsáveispelacriaçãodenovassituaçõesorganizacionais;

d)variáveisexógenasquenorteiamasaçõesdaorganizaçãoenasquaissebaseiamseusprincípios;

e)estímulos,positivosenegativos,queformamabasedesustentaçãodosvaloresorganizacionais.

14.(FCC/MPE-RS/Assessor/2008)Comrelaçãoaoprocessodemudançaorganizacional,assinalea

alternativacorreta.

a)Amudançaorganizacionaléumprocessoinduzidodeforaparadentrodaorganizaçãoparaevitara

resistênciadosseusmembros.

b)TantonaperspectivatayloristacomonaTeoriadaContingência,amudançaorganizacionaldepende

doenvolvimentoeaceitaçãodosnovosobjetivosorganizacionaisporpartedosseusmembros.

c)Omedoeaincertezacomrelaçãoaoprocessodemudançaorganizacionalpodemsemanifestar

comoresistênciaedevemserlevadosemconsideraçãonoseuplanejamento.

d)Aculturaorganizacionaléumobstáculoedevesertransformadapreviamenteparaqueoprocessode

mudançatenhasucesso.

e)Aperspectivadamudançaorganizacionalvêasorganizaçõescomomáquinasfeitasdepartesquese

interligam,cadaumadesempenhandoumpapelclaramentedefinidonofuncionamentodotodo.

Gabaritos

1.B

8.C

2.C

9.D

3.E

10.E

4.E

11.C

5.E

12.A

6.D

13.C

7.C

14.C

Bibliografia

Certo,SamuelC.,eS.TrevisCerto.ModernManagement.10aEd.UpperSaddle

River:Pearson-Prentice-Hall,2006.

Lemos,Cristina.InovaçãonaEradoConhecimento.In:InformaçãoeGlobalização

naEradoConhecimento,porHelenaMariaMartinsLastreseSaritaAlbagli,122–

145.RiodeJaneiro:Campus,1999.

SchermerhornJr.,JohnR.Management.9aed.Hoboken:Wiley&Sons,2008.

Capítulo11

GestãodeConflitos

Emtodasasvezesqueexistircontatoentrepessoaspoderáocorrerumconito.1

DeacordocomautorescomoFreud,2oconitoéalgoinerenteàvidaem

sociedadeeacontecesemprequeexistaminteresses,ideias,sentimentosou

atitudesquesejamantagônicos.

Ouseja,ocorrequandoosobjetivosoudesejosdaspessoassãodiferentesdos

dasoutras.Assim,oconitoéalgonormalnavidaemsociedade.Irmãosse

desentendem,casaisseseparamesóciosterminamasociedade.

Entretanto,osconitospodemserresolvidos.Veremosalgumasabordagensna

resoluçãodosconitosnestecapítulo.Nemsempre,porém,oresultadodessa

resoluçãoéoesperadoouodesejável.

Osconitospodemsersubstantivosouemocionais.Ossubstantivossão

derivadosdedesentendimentossobremetas,objetivos,recursos,políticasetc.

Assim,umconitopotencialpodeocorrersealgumaáreadaorganização

(Finanças,porexemplo)tivercomoobjetivocortarcustosaomáximoeoutraárea

(PesquisaeDesenvolvimento)tivercomoobjetivoaumentarosinvestimentos.

Jáosconitosemocionaissãobaseadosemsentimentosderaiva,medo,

antipatiaetc.Quantasvezesvocêconheceualguémque,semqueessapessoatenha

feitonadademais,você“nãofoicomacara”?

Dessaforma,oconitopodeocorrersemquenadade“concreto”existapara

isso.Oqueexistesãosentimentosconitantesentreessasduaspessoas(ouórgãos)

quelevamadesentendimentoseconflitos.

Figura11.1–ConflitosSubstantivoseEmocionais

Ambosostiposdeconitopodemgerarproblemasparaaorganização!Assim,

osconitosdevemseradministradosdeformaquenãoprejudiquemaobtenção

dosresultadosplanejados.

Oobjetivodogestordeveseradministrarosconitosdeformaquepromovam

acriatividade,ainovaçãoeumaltodesempenho!

Figura11.2

Quandobemadministrado,oconitopodeserpositivoparaaorganização!

Essetipodeconflitoéchamadodeconflitofuncional(construtivooupositivo).

Quandooconitoémoderado,elepromoveummaioresforçonotrabalho,

maiorcriatividadeecooperação.Deacordocompesquisas,umnívelrazoávelde

conitononívelestratégicodeumaorganizaçãopodelevaràtomadadedecisões

maisacertadas.3

Quandooconitoémuitobaixooumuitointenso,eleéchamadodeconflito

disfuncional(destrutivoounegativo).Seoconitoformuitobaixo,pode

ocorrerumacomplacência,umaacomodação.Jáseformuitointenso,poderá

levaraspessoasaperderumtempovalioso,poisestarãoenvolvidasemconitose

nãonotrabalho.

Muitasvezes,buscamosevitaroconito.OautorJerryHarvey4descreveuuma

situaçãovividapelasuafamíliaqueilustraisso.Suafamíliaumdiaviajoupara

umacidadechamadaAbilene(umaviagemde40km)emumdiamuitoquente,

comoarcondicionadodocarroquebrado.Todosestavam“sofrendo”nopasseio.

Apósisso,elepercebeuquenenhumadaspessoasoriginalmentequeriair,mas

aceitaramopasseioparanãocontrariarosoutros.Dessaforma,elecunhouo

termo“paradoxodeAbilene”,queserefereàtendênciadaspessoasdecederaos

outros,nabuscadeevitarconflitos.

Figura11.3–Conflitospositivosenegativos

11.1.CausasdosConflitos

DeacordocomSchermerhorn5eDa,6algumascondiçõespodemaumentarachancedeumconflitoocorrer:

PapelAmbíguo–Ocorrequandooempregadonãosabeoqueéesperadodele,

ouoquedevefazerexatamente.Podelevaraumasituaçãoemqueaspessoas

estejamfazendotrabalhosconitantes,ouestejamemtarefasquelevemaum

conflitocomoutraspessoasnaorganização.

RecursosEscassos–Sefaltamrecursosparatodosnaorganização,alguém

caráinsatisfeito!Disputasporrecursossãocausasmuitocomunsdeconitosem

umaorganização!

Interdependências–Quandodependemosdotrabalhodeoutraspessoaspara

fazernossotrabalho,existeumconitoempotencial!Emtodaorganização

existeminterdependênciasentresetoresepessoas.Dessaforma,umapessoadeve

cumprirseupapeldeformaeficienteparaqueaoutrapossafazeromesmo.

ObjetivosConitivos–Quandoosobjetivosdaorganizaçãonãosãobem

desenhados,ousãoambíguos(setordevendasrecebebônusporvendasmaiores,

massetordemateriaisrecebebônusporinvestimentoemestoquemenor,por

exemplo),existeumachancegrandedehaverconflitosdentrodaorganização.

Diferenciação–Diferençasnaestruturadaorganizaçãoounoperldas

pessoasdeáreasdiferentespodemcausarconitos,poisasvisõesdemundoedo

trabalhopoderãoserconflitantes(porexemplo,imagineumsetordefinançascom

pessoascommédiadeidadede55anosinteragindocomosetordemarketing

recheadodepessoascommédiadeidadede25anos?Adiferençadeidadee

experiênciaporsisópoderágerarconflitos!).

FalhasdeComunicação–Umacomunicaçãodecientepodelevara

desentendimentoseinterpretaçõesincorretasdeparteaparte.Principalmenteem

equipesquetrabalhamadistância,umafalhadecomunicaçãopodegerare

aumentarconflitos.

ChoquedePersonalidades–Muitasvezesaspessoassimplesmentenãose

“bicam”.Personalidades,valoreseatitudesmuitodiferentes(ouiguais)podem

gerarumconitoúnicaeexclusivamentederivadodosestilosdevidadaspessoas.

Muitasvezesaúnicasoluçãoésepararaspessoasenvolvidas.7

ConflitosAnteriores–Quandojáocorreramconitosentrealgumaspessoase

estesnãoforambemresolvidos,existirãoressentimentos.Portanto,estessempre

poderãoressurgiremnovosconflitos!

Figura11.4–Causasdosconflitos

Vamosvercomoessetemajáfoicobrado?

1.(Cesgranrio/CasadaMoeda/AnalistaRH/2009)Aexistênciadeconitoséinevitávelnas

organizações.Porisso,ogestorpodeidenticarformasdeprevençãodeconitos,

conhecendocondiçõesque,viaderegra,conduzemasituaçõesconituosas.Umadessas

condiçõeséainterdependênciadeatividades,queseconstituiquando:

a)expectativassãopoucoclarasouconfusas,levandoaspessoasatrabalharemparapropósitos

incompatíveis;

b)cadagruporealizaoudesenvolvetarefas,objetivos,concepçõeseatitudesdiferenciadas;

c)determinadogruponãoconseguerealizartarefasamenosqueoutrogruporealizeasua;

d)recursosorganizacionaissãolimitadosouescassoseprecisamsercompartilhados;

e)aespecializaçãodetarefasconduzaoestabelecimentodemetasconcorrentes.

Comovimos,ainterdependênciaocorrequandodependemosdotrabalhode

outrapessoaououtrogrupoparaquepossamosfazernossotrabalho.Sempreque

houveruminsucesso,existiráumachancedequeoconflitoaconteça!

Éaquelaestóriafrequenteemtodaorganização:“fulanopisounabolaeagorao

problemaénosso!”.Portanto,aletraCestácorreta.AletraAserefereà

ambiguidadedepapéis.

AletraBnãoserefereaumacausacomumdeconitos.Nocasodaalternativa

D,osrecursossãoescassos.EaletraEserefereametaseobjetivosconitantes.O

gabaritoémesmoaletraC.

11.2.EstilosdeGestãodeConflitos

Aspessoaspodemlidarcomosconitosdediversasmaneiras.Normalmente,os

estiloscombinamumaparceladeassertividade(vontadedesatisfazerseus

própriosinteressesenecessidades)ecooperação(vontadedesatisfazeros

interessesenecessidadesdosoutros).

Afiguraaseguirmostraoscincoestilosdegestãodeconflitos:

Figura11.5–Estilosnagestãodeconflitos

Evitação–Éafugadoconito!Apessoa,portanto,neméassertivanem

colaborativa.Esseestiloémuitocomumquandooassuntoépoucoimportante,

quandonãohápossibilidadedeganharouquandoocustodaderrotaémuito

alto.

Dessaforma,aspessoasngemqueoconitonemexisteeescondemseus

sentimentos!(Quandoevitamosdiscutirreligiãocompessoasdeoutrocredo,por

exemplo,estamosnoestilodeevitação!).8

Acomodação–Reeteumaltograudecooperação,semassertividade.Éa

buscapelaharmonia!Nesseestilobuscamossomentesatisfazerooutro,sem

buscarnossosinteresses!Écomumquandootemaaserdiscutidoémuitomais

importanteparaaoutraparteequeremosgerarum“crédito”deboavontadepara

ofuturo.

Quandoumaesposadeixaseumaridoassistiraojogodefutebolnatelevisãoda

sala,estábuscandoesseestilodelidarcomumpossívelconflito,porexemplo.

Competição–Nesseestilosomosassertivos,ouseja,queremosatodocusto

satisfazernossosinteresses,poucoimportandoosdooutro.Ocorrequando

queremosimpornossaposição!9

Portanto,podeserrecomendávelquandoumadecisãoéurgente,ouquandoo

temaéimpopulareénecessáriaumadecisão.Éumconitoganha-perde,ouseja,

alguémsairávencedoreooutrosairáderrotado!

Seumacategoriaentraemgreveeseuspatrõesdecidemnãooferecernenhuma

concessãoecomeçamacontratarnovosfuncionáriosparaocuparoslugaresdos

grevistas,estábuscandoesseestilo.

Compromisso–Nesseestilo,cadapessoaougrupocedeumpoucoparaque

umasoluçãointermediáriaou“aceitável”aconteça.Dessaforma,cadapartecede

eganhaalgode“valor”.

Entretanto,comocadaparte“perde”algo,pode-sesemearoutroconito

futuro(ouseja,oconitoseriasomente“empurradocomabarriga”).O

importantenessecasoéum“jogodecintura”eumabuscaporumasituação

aceitávelparaasduaspartes.

Colaboração–Necessitatantodecooperaçãoquantodeassertividadepara

funcionar.Oobjetivoéquetodasaspartessaiamganhando,eébaseadona

negociaçãoenocomprometimentodaspartes.

Portanto,podefuncionarquandoasduaspartessãofortes,quandoosobjetivos

deambasaspartespodemseratingidosaomesmotempoouquandotodos

percebemquealgoestáerradoeénecessáriaaparticipaçãodetodosnasolução

doproblema.

Vamosvermaisumaquestãoagora?

2.(FGV/Sefaz/RJ/Fiscalderendas/2009)Comrelaçãoàsfontesdosconitosorganizacionais,analise

asafirmativasaseguir.

I.Osrecursoslimitadossãoumafontedeconflitoorganizacional.

II.Adiferenciaçãoéumafontedeconflitoorganizacional.

III.Aausênciadeinterdependênciadetarefaséumafontedeconflitoorganizacional.

Assinale:

a)sesomenteaafirmativaIestivercorreta;

b)sesomenteaafirmativaIestivercorreta;

c)sesomenteaafirmativaIIestivercorreta;

d)sesomenteasafirmativasIeIestiveremcorretas;

e)setodasasafirmativasestiveremcorretas.

Essaquestãojáfoibemmais“tranquila”.Asduasprimeirasfrasesestãocorretas.

Jáaterceirafrasetemuma“pegadinha”.Nãoéaausênciadeinterdependências,e

simsuapresençaqueéumacausadeconflitos.Portanto,ogabaritoéaletraD.

11.3.AbordagensnaResoluçãodeConflitos

Existemtrêsabordagensparaquepossamosadministrarumconflito:10

11.3.1.AbordagemEstrutural

Quandooconitoécriadoporumasituaçãodediferenciação,derecursos

escassosoudeinterdependência,aabordagemdeveserestrutural.Imagineduas

pessoasquetrabalhemnamesmaempresa:Paulo(umvendedor)eJoão(analista

decrédito).

Portanto,PauloéremuneradoporvolumedevendaseJoãotemcomometa

reduzirainadimplêncianaempresa.AcadavendaquePauloperde,porqueJoão

nãoliberouocrédito,poderáhaveraumdesentendimentoentreosdois!

Aabordagemestruturalbuscaentãomudarumacondiçãoquepredispõeao

conito.Entreasmedidasquepodemsertomadasestãoodesenhode

recompensasgrupaiseobjetivoscomuns(parainterferirnadisputaporrecursos

ounosobjetivosconitantes),eoreagrupamento,aseparaçãoearotaçãode

pessoas(parareduzirainterdependênciaeadiferenciaçãodosgrupos).11

Figura11.6–Abordagemestrutural

Atenção:naabordagemdeprocessooconflitojáestápresente.

11.3.2.AbordagemdeProcesso

Nessaabordagem,ogestorbuscaresolveroconitomudandooprocesso,ou

seja,intervindonopróprioatoquelevouaoconito.Podeserfeitoporuma

pessoaenvolvidanoconflitoounão,comoumconsultor.

Nessescasos,oconitojáexiste.Dessaforma,temosdeatuarsobreoconitojá

existente,enãosomentebuscaratuarsobreassuascausas.

Umadastécnicaséadesescalonização(oudesativação),emqueumadaspartes

reagecooperativamenteaumareaçãoagressivadaoutraparte,buscandocomisso

desarmarooutro.

Éofamoso“ofereceraoutraface”.Comumgestopositivo,buscamos

“desarmar”nosso“oponente”.Imaginequevocêtenhaumchefequeestejalhe

causandodificuldades.

Umamaneiradeutilizaradesescalonizaçãoseriarealçarseuspontospositivos

nasreuniões,comosua“capacidadedeliderança”ousuacompreensãoe

generosidade,embuscadeumamudançadocomportamentodoseuchefe.

Outratécnicaéareuniãodeconfrontação,quedeveocorrerquandoaspartesjá

trabalhamparaumconfrontoaberto.

Nessasituação,devehaverummomentoemquetodospossamexporsuas

necessidadeseemoções,deformaqueasposiçõesdecadapartesejamexpostase

umasoluçãopossaserencontrada.

Aterceiratécnicaéadecolaboração,queocorrequandoasduaspartesbuscam

solucionarosproblemasdemodoqueambaspossamsairganhandoeatinjamseus

objetivos.

Figura11.7–Abordagemdeprocesso

11.3.3.AbordagemMista

Buscaadministraroconitousandotantofatoresestruturaiscomodeprocesso.

Inicialmente,podem-secriarregraseregulamentosquedenamcomoum

conflitodeveserresolvido(fatorestrutural).

Outrainiciativaécriarpapéisdeligação(terceiraspartes)ouintegradores,ou

seja,pessoasqueteriamafunçãodeharmonizarosesforçosdepessoasegrupos

conflitantesdentrodaorganização(fatordeprocesso).

Nocasodeumadisputaentreduaspessoasdeumamesmagerência,opróprio

gerentepoderiaassumiressepapelintegrador.

Figura11.8–Abordagemmista

Vamosvercomoessetemaestásendocobrado?

3.(Esaf/CVM/AnalistaRH/2009)Emrelaçãoàabordagemquantoàadministraçãodeconitos,

marqueoitemqueapresentaastrêsabordagensàdisposiçãodogerente.

a)Espera,tensãoeresolução.

b)Estrutural,mistaedeprocesso.

c)Impasse,vitória-derrotaeconciliação.

d)Ganhar/perder,perder/perdereganhar/ganhar.

e)Diferenciaçãodegrupos,recursoscompartilhadoseinterdependênciadeatividades.

Essaquestãoestábemtranquilaparaquemestudouotema,nãoéverdade?Não

ficadifícilapontaraletraBcomoogabarito.

11.4.EfeitosdoConflito

Osconitospodemgeraraspectospositivosenegativos.12Vejamosas

possibilidades:

Aspectospositivos

ØOconitopode“empurrar”ogrupoparaqueseesforcemaisesejamais

criativoparasolucionarosproblemas,gerandoumaenergiaeumimpulso

inovadormaiores.

ØUmníveladequadodeconitopodepreveniraocorrênciado“pensamento

grupal”,queocorrequandoumgrupoétãocoesoqueaspessoasnãose

sentemàvontadeparaexpressaropiniõescontrárias.13

ØOconitoaumentaacoesãointragrupal,poisgeraumsentimentode

identidadenogrupo.Éafamosasensação“nóscontraeles”,queexisteem

disputasesportivas.

ØOconitoaumentaapercepçãodoserroseproblemasexistentesna

organização,quetalvezficassem“adormecidos”semsuapresença.

Aspectosnegativos

ØBoapartedaenergiageradapeloconitoégastanasuaresolução,ouseja,

nãoécanalizadaparaalgoprodutivo,esimemreuniõesintermináveis,

disputasemocionaisetc.

ØAcooperaçãodeixadeexistireésubstituídaporcomportamentosque

dificultamotrabalhoeobomfuncionamentodaorganização.

ØOconitoimpedequeosesforçosdaspessoasegrupossejamecazes,

gerandosentimentosdefrustração,hostilidade,raivaetc.

Vamosvercomoessetemajáfoicobradoemprovas?

4.(FCC/TRF1aRegião/Anal.adm./2011)Umacausafrequentedeconflitosnasorganizaçõesé:

a)oexcessodeliberdade;

b)aambiguidadedepapéis;

c)aexistênciadeobjetivoscompartilhados;

d)alimitaçãoderecursos;

e)ahierarquiaderesponsabilidades.

AalternativaAestáerrada,poisexcessodeliberdadenãoéumacausade

conitos.AalternativaBestácorreta.Entretanto,aletraCestáincorreta.Uma

causadeconflitospoderiaseraexistênciadeobjetivosconcorrentes!

Infelizmente,aletraDtambémestácorreta.Entretanto,abancaaconsiderou

errada.Adisputaporrecursosescassoséumacausadeconitos.AletraEtambém

nãoserelacionacomascausasdeconflitos.Ogabarito,portanto,éaletraB.

QuestõesComentadas

5.(FGV/Sefaz/RJ/Fiscalderendas/2007)Oalcancedosobjetivosorganizacionaispodeserfacilitado

pelousoadequadodopoderepelasoluçãodosconitosorganizacionais.Sobrepoderegestão

deconflitos,écorretoafirmarque:

a)oconflitoéessencialmentedisfuncionalecausaaestagnaçãodoprogressoorganizacional;

b)nanaturezadosprocessosreivindicatóriosqueemergemnasorganizações,dimensõesindividuais

não

podemserconsideradas;

c)aparticipação,diretaouindireta,constituiumaformaderestauraroequilíbriodepoderedesebuscar

asoluçãodeconflitos;

d)segundoaconcepçãopluralísticadepoder,poderéacapacidadedeinuenciaroutrapessoaase

comportarsempredeacordocomosinteressesdaorganização;

e)oconitosurgedodesequilíbrioentreaspossibilidadeseaspiraçõesindividuais,advindodefatores

internosàorganização,semainfluênciadefatoresexternos.

Essaquestãomisturaostemasdegestãodeconitoscomosdepodernas

organizações.AletraAestáclaramenteerrada,poisosconitospodemser

positivos.

AletraBtambémestáequivocada,poisasdimensõesindividuais,ouseja,os

objetivosindividuais,devem,sim,serobservados.JáaletraCestácorretaeéo

nossogabarito.

AletraDéabsurda,poisessaconcepçãoserefereàdisputadepoderpolíticoe

nãoàobediência“cega”dosindivíduos.NocasodaalternativaE,osfatores

externostambémpodemgerarconflitos.OnossogabaritoémesmoaletraC.

6.(FGV/Sefaz/RJ/Fiscalderendas/2011)Aspessoasnãotêmobjetivoseinteressesidênticos.As

diferençasdeobjetivosedeinteressesindividuaissempreproduzemalgumaespéciede

conito.Oconitoéinerenteàvidadecadaindivíduoefazparteinevitáveldanatureza

humana.Constituioladoopostodacooperação.Alémdadiferençadeobjetivoseinteresses,

devehavernecessariamenteumainterferênciadeliberadadeumadaspartesenvolvidaspara

quehajaconito.Oconitoexistequandoumadaspartes–sejaindivíduoougrupo–tenta

alcançarseusprópriosobjetivosinterligadoscomalgumaoutraparteeestainterferenaoutra

queprocuraatingirseusobjetivos.Ainterferênciapodeserativa(medianteaçãoparaprovocar

obstáculos,bloqueiosouimpedimentos)oupassiva(medianteomissãooudeixardefazer

algo).Asequipes,assimcomoaspessoas,desenvolvemestilosespecícosparalidarcom

conflitos,baseadosnodesejodesatisfazerseusprópriosinteressesversusointeressedaoutra

parte.Sãodescritosaseguircincoestilosdeadministrarconitospormeiodeduas

dimensões,daassertiva(tentativadesatisfazeraosinteressesdasoutraspartes)atéanão

assertiva.UmdelesNÃOestácorreto.Assinale-o.

a)Estilodeacomodação:reeteumaltograudecooperaçãoefuncionamelhorquandoaspessoas

sabemoqueéerrado,quandoumassuntoémaisimportantequeoutrosparacadalado,quandose

pretendeconstruircréditossociaisparautilizaremoutrassituaçõesouquandomanteraharmoniaéo

maisimportante.Onegócioéirlevando.Éassertivo.

b)Estilocompetitivo:reeteassertividadeparaimporoseuprópriointeresseeéutilizadoquandouma

açãoprontaedecisivadeveserrapidamenteimpostaemaçõesimportantesouimpopulares,durante

asquaisaurgênciaouaemergênciasetornanecessáriaouindispensável.Onegocioéganhar.Reetea

assertividadetotal.

c)Estilodeevitaçãoouabstenção:reeteumaposturanãoassertivanemcooperativaeéapropriado(1)

quandoumassuntoétrivial,(2)quandonãoexistenenhumapossibilidadedeganhar,(3)quandouma

demoraparaobtermaiorinformaçãosetornanecessáriaou(4)quandoumdesentendimentopode

sermuitooneroso.

d)Estilodecompromissooutransigência:reeteumamoderadaporçãodeambasascaracterísticasde

assertividadeedecooperação.Éapropriadoquandoosobjetivosdeambososladossãoigualmente

importantes,quandooscomponentestêmigualpodereambososladosqueremreduzirasdiferenças

ouquandoaspessoasprecisamchegaraalgumasoluçãotemporáriasempressãodetempo.O

negócioéterjogodecintura.

e)Estilodecolaboração:reeteumaltograudeassertividadeedecooperação.Oestilocolaborativo

habilitaambasaspartesaganhar,enquantoutilizaumasubstancialparceladenegociaçãoede

intercâmbio.Oestilodecolaboraçãoéimportantequandoosinteressesdeambososladossão

importantes,quandoospontosdevistadaspartespodemsercombinadosparaumasoluçãomais

amplaequandoocompromissodeambososladosrequerconsenso.Onegócioéresolverparaque

ambasaspartesganhemesecomprometamcomasolução.

Bom,iniciamoscomumcomentáriopreliminar.Abancainseriuumenunciado

imensoe,alémdisso,comumadeniçãoincorretadadimensãoassertiva.Ela

signicaexatamenteocontráriodoqueestáescritonaquestão!Ouseja,éa

tentativadesatisfazeraosnossosprópriosinteresses.

Ocandidatoquesebaseounoenunciadopararesponderaquestãocomcerteza

se“enroloutodo”.Oproblemaéqueaquestãoincorreta(quedeveriaser

marcada)éaletraA.Oerrodaquestãoéexatamentedeserelacionara

assertividadeaoestilodeacomodação.

Noestilodeacomodaçãonãobuscamosaassertividade,pelocontrário.Assim,o

gabaritoémesmoaletraA,masseocandidatosebaseassenadeniçãocontidano

enunciadonãoacertariaaquestão.Vidadeconcurseiroéduramesmo!

7.(Cespe/PrefeituraMunicipaldeRioBranco/2007)Aestratégiadecompetiçãodeveserutilizada

quandoanecessidadedeseconstruirumrelacionamentoémaisimportantedoquea

obtençãodeganhosfinanceirosimediatos.

Oestilo(ouestratégia)decompetiçãonãodeveserutilizadoquandotemosa

necessidadedeconstruirumrelacionamento!Nesseestilooimportanteéganhar!

Éimpornossaposiçãoindependentementedosinteressesdaoutraparte.

Obviamenteque,quandoimpomosnossaposiçãosemnosimportarcomo

interessedooutro,nãoteremosmuitasimpatiadaoutraparte,nãoéverdade?

Portanto,afraseestáerrada!

8.(Cespe/Aneel/Anal.adm./2010)Odiretorquepropõeumareuniãodeconfrontaçãovisando

sanearumconflitoestáadotandoumaabordagemestrutural.

Areuniãodeconfrontaçãoocorrequandoqueremosresolverumasituação

especíca.Portanto,nãovisaresolveracausadeumproblema,ouseja,seu

problemaestrutural!

Assimsendo,quandoreunimosaspartesemconitoeforçamosumconfronto,

queremosqueasrazõesemocionaiseospontosdeconitosejamexpostos,de

formaquesejapossívelencontrarmos,juntos,umasoluçãoconciliadora.

Dessaforma,areuniãodeconfrontaçãoéumaabordagemdeprocessoquetenta

atuaremcimadeumepisódioespecíco,enãonascausasdeumconito!Afrase

estáerrada.

9.(Cespe/Aneel/Anal.adm./2010)Odiretorquebuscareduzironíveldoconitocomumareação

cooperativaestáadotandoumaabordagemprocessual.

Umgerentequebusqueatenuarumconitocomumareaçãocooperativa

estariabuscandoatuarnoprocesso,ouseja,emumconitojáexistente.Dessa

forma,adesescalonizaçãoéumadasalternativasdeabordagemdeprocesso.O

gabaritoéquestãocorreta.

QuestõesPropostas

10.(FCC/Alesp/SP/Gestãoprojetos/2010)Comrelaçãoàgestãodeconitosnumaorganização,

considereasseguintesafirmativas.

I.Quandoasmetasdeumgrupointerferemnasdeoutro,équasecertoquehaveráconito

entreeles.

II.Umgerenteeficientedeveresolvertodoequalquertipodeconflitonaorganização.

III.Reconhecerdiantedeumsubordinadoquevocêestavaerradopodedebilitá-locomochefe.

IV.Quandoéprecisoimplantarmedidasimpopulares,nãoéprudentenegociá-laspreviamente

comaspessoasenvolvidas.

V.Aogerirconitos,omelhoréater-seaosfatosesoluçõesenãoàspersonalidadeseaos

comportamentosdaspessoasenvolvidas.

EstácorretooqueseafirmaAPENASem:

a)I,I,IIeV;

b)I,IeII;

c)IeV;

d)IeIV;

e)II,IVeV.

11.(FCC/Arce/Analistareg./2006)Quandooconitoétrivial,quandoosânimosestãoexaltadoseé

precisotempoparaesfriá-los,ouquandoapotencialperturbaçãoprovocadaporumaaçãomais

enérgicarepresentarumcustomaiorqueosbenefíciosdaresolução,aestratégiagerenciala

seradotadaé:

a)acolaboração;

b)aacomodação;

c)aimposição;

d)oacordo;

e)aabstenção.

12.(FGV/Badesc/Analistaadm./2010)Arespeitodagestãodeconitos,analiseasarmativasa

seguir.

I.Oestilodeabstençãoprevêumaposturanãoassertivaenãocooperativa.

II.Oestilodetransigênciarefleteumamoderadaporçãodecooperação.

III.Oestilodeacomodaçãorefleteumaltograudecooperação.

Assinale:

a)sesomenteaafirmativaIestivercorreta;

b)sesomenteaafirmativaIestivercorreta;

c)sesomenteaafirmativaIIestivercorreta;

d)sesomenteasafirmativasIeIestiveremcorretas;

e)setodasasafirmativasestiveremcorretas

13.(FGV/Badesc/Analistaadm./2010)Asalternativasaseguirapresentamcaracterísticasdas

abordagensmistasnagestãodeconflitos,àexceçãodeuma.

Assinale-a.

a)Estabelecimentoderegraseregulamentos.

b)Sistemasderecompensasgrupais.

c)Gruposeequipesdetrabalho.

d)Papéisdeligação.

e)Papéisintegradores.

14.(FGV/Sefaz/RJ/Fiscalderendas/2010)Comrelaçãoaosestilosetécnicasdegestãodeconitos,

analiseasafirmativasaseguir.

I.Adesescalonizaçãoprocuramitigarconitosaoencorajaratitudescooperativasentreas

partes.

II.Oestilodeabstençãoésugeridoquandoumassuntoétrivialouodesentendimentoémuito

oneroso.

III.Aabordagemestruturalfazusodossistemasderecompensascomomecanismodegestão

deconflitos.

Assinale:

a)sesomenteaafirmativaIestivercorreta;

b)sesomenteaafirmativaIestivercorreta;

c)sesomenteaafirmativaIIestivercorreta;

d)sesomenteasafirmativasIeIIestiveremcorretas;

e)setodasasafirmativasestiveremcorretas.

15.(Cespe/Abin/Psicologia/2011)Nasorganizações,osconitosinterpessoaissãoameaças

potenciaisàsobrevivênciadosgruposdetrabalhoe,porisso,devemsereliminados.

16.(Cespe/INCA/GestãodeRH/2011)UmaspectocríticodaARHéasoluçãodeconitosinternos,

sendoque,emgeral,osconflitospessoaissãoprovocadosporconflitosorganizacionais.

17.(Cespe/DPU/Psicólogo/2010)Aausênciadequalquertipodeconitoindicabomambiente,boas

relaçõese,nocasodasorganizações,competência.

18.(Cespe/MPU/Técnicoadministrativo–2010)Nasorganizações,cabeàáreaderecursoshumanos

garantiroequilíbrionasrelaçõesentreosfuncionárioseaorganização,esuaaçãoenvolveo

gerenciamentodepotenciaisconitos,doqueéexemploapercepçãonegativade

funcionáriosquenãosãorecompensadosdeformacompatívelcomoseutrabalhona

organização.

19.(Cespe/–MPU/Técnicoadministrativo/2010)Emsituaçõesvitaisparaobem-estarda

organização,acompetiçãoéaestratégiamaisadequadaparaogerenciamentodeconitono

casodeumadaspartessaberqueestácomrazão.

20.(Cespe/MPU/Técnicoadministrativo/2010)Aquelequeutilizaaestratégiadaacomodaçãopara

gerenciarconflitossatisfazosinteressesdooutroemdetrimentodosprópriosinteresses.

21.(Cespe/MPU/Técnicoadministrativo/2010)Aestratégiadeevitaroconitoéamelhormaneira

degarantiro“ganha-ganha”quandoumadaspartesnãoquernegociar.

22.(Cespe/STM/Analista/2011)Competição,concessãoecolaboraçãoconstituemabordagens

efetivasparaaresoluçãodeconflitos.

23.(Cespe/MPU/Técnicoadministrativo/2010)Denomina-seabordagemprocessualaestratégiade

resoluçãodeconflitoembasadanamodificaçãodascondiçõesantecedentesdesseconflito.

24.(Cespe/TRT-16/Analista/2005)Pedro,analistajudiciáriodeumtribunaldotrabalho,estáem

exercíciohácincoanos.Recentemente,Pedrofoiconvidadoaassumiragerênciadeseusetor

devidoasuacompetênciatécnicaeinterpessoal.Emsituaçõesdeconito,Pedrobusca

identicaroproblemaeresolvê-lopormeiodediscussãoaberta,evitandooenfrentamento

dosenvolvidos,minimizandoasdiferençasentreaspartesconitantes,enfatizandoos

interessescomunsepropondo,quandonecessário,aparticipaçãoemtreinamentoderelações

humanas.Nessasituação,écorretoarmarquePedroéhabilidosonaadministraçãode

conflitosinterpessoais.

25.(Cespe/TRT-16/Analista/2005)Osconitosinterpessoaissãodenaturezadisfuncional,

traduzidapelaintensidade,estágiodeevolução,contextoeformacomosãotratadose,por

taismotivos,essesconflitosdevemserevitadosnocontextodetrabalho.

Gabaritos

1.C

10.C

18.C

2.B

11.E

19.C

3.B

12.C

20.C

4.B

13.B

21.E

5.C

14.E

22.E

6.A

15.E

23.E

7.E

16.E

24.C

8.E

17.E

25.E

9.C

Bibliografia

Aronson,Elliot,TimothyD.Wilson,eRobinM.Akert.SocialPsychology.5aed.

UpperSaddleRiver,NJ:EditoraPrenticeHall,2004.

Chiavenato,Idalberto.AdministraçãoGeralePública.2aed.SãoPaulo:Elsevier,

2008.

—.GestãodePessoas:eonovopapeldosrecursoshumanosnasorganizações.2aed.

RiodeJaneiro:Elsevier,2004.

Daft,RichardL.Management.Mason:Thomson,2005.

Robbins,StephenP.,eMaryCoulter.Administração.5aed.RiodeJaneiro:

Prentice-Hall,1998.

SchermerhornJr.,JohnR.Management.9aed.Hoboken:Wiley&Sons,2008.

Capítulo12

GerenciamentodeProcessos

Antesdetudo,oqueéumprocesso?DeacordocomDavenport,1umprocessoé

umasériedeatividadesestruturadasparaproduzirumprodutoouserviçoaum

clienteoumercadoemparticular.

Atenção:umprocessoéumconjuntodeatividadesinter-relacionadasquebuscagerar

valoraocliente.

Dessamaneira,todotipodetrabalhoimportanteemumaorganizaçãofazparte

dealgumprocesso.2Nãoexisteprodutoouserviçofornecidosemqueexistaum

processoorganizacionalportrás.

Harringtondeneprocessocomoautilizaçãoderecursosdaempresapara

oferecerresultadosobjetivosaosseusclientes.3Assimsendo,oprocessoseriaum

uxodetrabalho,emqueexistiriamosinputs(materiais,informação,

equipamentosetc.)queseriamtrabalhados,deformaaagregarvalor.Dessaforma,

ouxoresultariaemumasériedeoutputs(produtoseserviçosdesejadospelos

clientes).

Vamosimaginarumcasoprático.Vocêéogerentedeumapizzariadeentregas

adomicílio.Seusclientesligamparasuaempresaefazemopedido.Apartirdesse

momento,existirãodiversastarefasenvolvidasnoprocessodeproduzireentregar

pizza.

Entretanto,essasatividadesisoladasnãosignicamnadaparaocliente.Oque

elerealmentequeréasuapizza,quentinha,noprazoesperado.Ouseja,o

importanteéfocarnasnecessidadesedesejosdocliente.

Naguraaseguirteríamosumaideiaresumidadosprocessosenvolvidosno

atendimentodessecliente:

Figura12.1–Processosimplificadodeentregadapizza

Portanto,cadaatividadedessaspodeagregarvalorounãoaoprocesso,poisum

errooudemoraemumadelasacabaráporprejudicarocliente.Seapessoaque

atendeoclienteanotaropedidoerrado,porexemplo,farácomquesejafeitauma

pizzaerrada,nãoéverdade?Dessamaneira,oclienteficaráinsatisfeito.

Portanto,quandopensamosemprocessotemosdeentenderqueasatividades

estãointerligadasequenãoadiantaumadelassermuitobemfeitaseoutrafor

deficiente.

Dessaforma,agestãoporprocessosimplicaumaênfaseem“como”oproduto

ouserviçoéfeito,aocontráriodofocono“que”éfeito,característicadas

organizaçõestradicionais.

Vamosvercomoessetópicojáfoicobrado?

1.(FGV/SAD/PE/APOG/2008)Assinaleaalternativaquenãocorrespondaàdeniçãodeum

processo.

a)Conjuntodefinidodepassosparaarealizaçãodeumatarefa.

b)Grupodeatividadesrealizadasemumasequêncialógica.

c)Ordenaçãoadhocdeatividades.

d)Qualqueratividadequerecebeuminput,adicionavaloreforneceumoutput.

e)Estruturadeação.

AúnicaalternativaqueestáincorretaéaletraC.Adhocéumaexpressãolatina

quesignica“paraisso”.Dessaforma,querdizeralgumaatividadeespecíca,ou

queseráanalisadacasoacaso.

Ouseja,seriamalgumasatividadesousériedeatividadesquenãosão

estruturadas,ouqueserãofeitasapenasumavez.Portanto,nãoserelacionamcom

umprocessoestruturado.Ogabaritoé,assim,aletraC.

Continuando,vamosagoraentendermelhoroqueéagestãoporprocessos.As

empresastradicionais,emsuagrandemaioria,aindautilizamumaestrutura

baseadanadivisãodotrabalhodeacordocomfunções.

Assimsendo,asorganizaçõestradicionaisseestruturambaseadasemdiversos

pressupostosquejánãofazemmaissentido.Nessetipo,asáreasousetores

trabalhamisoladosunsdosoutros.Portanto,têmpoucainteraçãoea

coordenaçãodostrabalhosédifícil.

Asorganizaçõestradicionaissãomaisvoltadasparasimesmas,preocupadas

comseusproblemasinternos.4Aescolhaporumagestãoporprocessosimplica

umanovavisão.Dessamaneira,énecessárioofoconasnecessidadesdosclientes,e

quaissãoosprocessosimportantesequegeramvalorparaele.

DeacordocomNunes:5

Omodelodeorganizaçãoorientadoporprocessospassouaserconsideradocomoalternativa

maisadequadaparapromoverumamaiorefetividadeorganizacional.Opressupostofoide

quenessaformadeorganizaçãoocorresseumaeliminaçãodebarreirasdentrodaempresa,

possibilitandoavisualizaçãodaorganizaçãocomoumtodoeumamaiorinter-relaçãoentre

osdiferentesagentesdacadeiadevalor(cliente,fornecedor,executoresdoprocesso).

DeacordocomHammereStanton,6aoprocurarseestruturarporprocessos,as

empresasacabamdescobrindoqueéimpossívelsobreporumprocessointegradoa

umaorganizaçãofragmentadapelodesenhofuncionaltradicional.As

organizaçõesestruturadasporprocessosprecisamserredesenhadasparapoderem

passarasergerenciadasporprocessos.

Portanto,amudançadeumaestruturafuncionalparaumaestruturavoltada

paraosprocessosimpõeadeniçãodasresponsabilidadespeloandamentodos

processos,aminimizaçãodastransferênciasinternas(atrocadeinformaçõese

materiaisentreossetores),amaximizaçãodoagrupamentodeatividades

conexaseadiminuiçãodogastodetempoeenergia(queocorrecomareunião

daspessoaseatividadesquedependerãoumasdasoutras,reduzindoassimaperda

detempoeocustocomtransporteetc.).7

Dessemodo,asempresasestãobuscandomudarseumodeloparaagestãode

processosparaobteremmaioreciêncianaproduçãoeprestaçãodeserviços,a

mdeseadaptaremmelhoràsmudançasnoambiente,paraconseguiremuma

maiorintegraçãodeseusesforçoseampliaremsuacapacidadedeaprendizado.8

12.1.CadeiadeValor

Cadeiadevalor,paraMichaelPorter,éoconjuntodeatividadestecnológicase

econômicasdistintasqueumaorganizaçãoutilizaparaentregarprodutose

serviçosaosseusclientes.

Cadaumadessasatividades(produção,distribuição,comercializaçãoetc.)deve

entregaralgum“valor”.Quantomaisvaloragregado,maiscompetitivacaa

empresa.Esseéumconceitorelacionadocomavantagemcompetitiva.

Deacordocomele,existemtrêsvantagensdautilizaçãodametodologiada

cadeiadevalor:9apossibilidadedeverificarosvaloresagregadosemcadaetapado

processonalísticodaempresa;aapresentaçãodarelevânciadosprocessosde

suporteparaqueosnalísticossejamplenamenteexecutados;eacapacidadede

poderrepresentarorelacionamentoentreaestratégia,dopontodevistada

excelênciaoperacional,eosprocessos,oquepermiteelevaravantagem

competitivadasorganizações.

Vamosvercomoessetemajácaiuemprovas?

2.(Cespe/TCDF/ACE/2012)Deacordocomaabordagemporprocessos,ofatodeumauditor

procurarreduziroscustosnanceirosdosinsumosenvolvidosnodesenvolvimentodesuas

atividadesdescalizaçãoconstituiumexemplodeauditorquebuscamelhoraracadeiade

valordoseuprocessodefiscalização.

Otermovalorestárelacionadoaoqueé“entregue”paraocliente,sejainterno

ouexterno.Otermo“cadeiadevalor”,paraMichaelPorter,éoconjuntode

atividadestecnológicaseeconômicasdistintasqueumaorganizaçãoutilizapara

entregarprodutoseserviçosaosseusclientes.

Assimsendo,cadaumadessasatividades(produção,distribuição,

comercializaçãoetc.)deveentregaralgum“valor”.Quantomaisvaloragregado,

maiscompetitivacaaempresa.Esseéumconceitorelacionadocomavantagem

competitiva.

Dessamaneira,nãoéocustoquedenea“cadeiadevalor”,ouseja,quaissãoos

benefíciospercebidospelosclientes.Umcustomaisbaixopodesignicarquea

organizaçãoestásendoeciente,masnãoqueestáoferecendoummelhorserviço

aocliente.Ogabaritoémesmoquestãoincorreta.

12.2.TiposdeProcessos

DeacordocomGonçalves,10existemtrêstiposdeprocessosempresariais:os

processosdenegócios(oudeclientes),osprocessosorganizacionaiseos

processosgerenciais.Aseguirpodemosverasprincipaiscaracterísticasdecada

um:

Figura12.2–Tiposdeprocessos.

(Fonte:Gonçalves,Processo,queprocesso?,2000)

Assimsendo,osprocessosdenegóciosãoosquegeramosprodutoseserviços

desejadospelosclientes.Estãoligadosàcadeiadevalor,negóciodaempresaou

corebusiness.Portanto,sãoconsideradosprocessosessenciaisparao

funcionamentodaorganização.

DeacordocomMartin,11sepensarmosnageraçãodevalorparaocliente,os

processospodemserconsideradosprimáriosoudesuporte.Osprocessos

primáriossão,portanto,osprocessosdenegócio,poissãoosque“entregam”

valorparaosclientes.

Jáosprocessosdesuportesãorelacionadosaosprocessosorganizacionaiseaos

gerenciais.Assimsendo,essesprocessosservemdebaseparaofuncionamentodos

processosprincipais.

Utilizandoumexemploprático,seapizzariaquemencionamosantesnãotiver

umprocessodecontrolenanceiro(processoorganizacional),teráproblemas

paraexecutarseuprocessodeatendimentoaocliente(processoprimário,oude

negócio),poisacabaráfaltandodinheiroparacomprarosmateriaisnecessários,

bemcomopagaraopizzaioloseusalárioetc.

VejaumresumodadefiniçãodeMartin:

Figura12.3–Processosquantoàgeraçãodevalor

Osprocessosorganizacionaisegerenciaissãouxosdeinformaçãoedecisão.

Essesprocessostambémpodemserclassicadoscomohorizontaiseverticais.12Os

processosverticaissãorelacionadoscomoprocessodeplanejamento,de

orçamentoedeorientação.

Jáosprocessoshorizontaisserelacionamcomouxodotrabalhoecoma

coordenaçãoentreosdiversosdepartamentoseáreasparaqueasatividadessejam

feitasdamelhormaneirapossível.

Osprocessostambémpodemserinternosouexternos.Sãointernosquando

começameterminamdentrodoambientedaempresa.Portanto,senãoexistirem

dessaformaserãoconsideradosexternos.

Outracaracterísticaimportantedosprocessosdenegócioéasuacaracterística

de“atravessar”diversossetoresdaempresa,eatédeoutrasempresasparceiras.

Dessaforma,sãochamadosdeprocessostransversais,ouseja,quenãosão

efetuadosemumasóáreaousetordaempresa,atravessando-a.

Outradistinçãoquepodemosfazeréreferenteaosprocessos-chaveparauma

organização.Elesproduzemresultadosquetrarãoaltoimpactoaosclientes.Dessa

forma,sehouveralgumafalhanessesprocessos,poderácomprometero

desempenhoglobaldaorganização.Portanto,sãocríticosparaqueaorganização

consigaatingirseusobjetivosestratégicos.

12.3.NíveisdeDetalhamentodosProcessos

Emtodaempresa,existemalgunsprocessosmaiscomplexoseoutrosmais

simples.Alémdisso,existemprocessosmaisimportanteseoutrosmenos

importantes.Oníveldedetalhamentodeumprocessoestárelacionadocoma

complexidadedeumprocesso.

Quantomaiscomplexo(maisatividades,entradasouprodutosresultantes),

maisprovávelquetenhamosde“decompô-lo”emsubprocessosparaquesejam

maisfáceisaanáliseeocontrole.

Adecomposiçãodeumprocessosegueestalógica:

ØMacroprocesso–éumprocessoqueenvolve,normalmente,maisdeuma

áreadaempresaegeraumimpactoconsiderávelnaorganização.

ØProcesso–éumconjuntodeatividadesrelacionadasesequenciaisque

recebeentradas,agregavaloreproduzsaídas(produtoseserviços).

ØSubprocesso–éumprocesso“dentro”deoutroprocesso.Assim,possibilita

obomfuncionamentodoprocessomaior.

ØAtividade–sãotrabalhosexecutadosnosprocessosousubprocessospara

atingirumresultadodesejado.

ØTarefa–éomenor“elemento”deumprocesso.Podeserumaparte

específicadaatividadeouumasubdivisãodealgumtrabalho.

12.4.MapeamentodeProcessos

Paraquepossamosmelhorarumprocessonecessitamosantesconhecê-lo.Dessa

maneira,precisamosanalisaroprocesso,deformaaentenderouxodetrabalho

envolvido,quaissãoossetoresepessoasenvolvidoseasdecisõesquedevemser

tomadasduranteoprocesso.

Portanto,otrabalhode“entender”evisualizarumprocessodetrabalhoé

chamadodemapeamentodeprocessos.Essetrabalhoéexecutado,normalmente,

atravésdeumaferramentachamadade

uxograma,queseráanalisada

posteriormente.

12.5.TécnicasdeMapeamento,AnáliseeMelhoriadeProcessos

Paraqueumprossionalpossamapearumprocesso,eledeveprimeiro

compreendê-lo.Paraisso,existemdiversastécnicasquepodemserutilizadas.As

principaissão:

Øentrevistasereuniões;

Øobservaçãodasatividadesinloco;

Øanálisedadocumentaçãoedossistemasexistentes;

Øcoletadedadoseevidências.

Aomapearmosumprocesso,eleserádescritodesdeoinício,deformaa

representarcadaatividadeedecisãoenvolvidanele.Dessaforma,apessoaque

estiverfazendoomapeamentodeverácompreenderoselementosdoFEPSC

(fornecedores,entradas,processo,saídaseclientes)demodoadescrevertodosos

aspectosdoprocesso.

Entreosbenefíciosqueumaorganizaçãopodeteraomapearseusprocessos,

temos:

ØCompreenderoimpactoqueoprocessotemparaaorganizaçãoeseus

clientes.

ØEntenderarelaçãodedependênciaentreossetoresnoprocesso.

ØCompreenderquaissãoos“atores”envolvidosnoprocesso.

ØAnalisarseoprocessoénecessárioeseébemexecutado.

ØPropormudançasnoprocesso.

ØIdentificarquaissãoosfatorescríticosnoprocesso.

12.6.ProjetodeMapeamentoeModelagemdeProcessos

OBPMN(BusinessProcessModelingNotation)éopadrãoutilizadoparao

desenho(oumodelagem)dosprocessosemumaorganização.Consisteemum

conjuntodenotaçõesgrácas,ouseja,umconjuntodesímbolospadronizados

queservemparaquepossamosdescrevereredesenharumprocesso.Essediagrama

quenospermitevisualizarumprocessotambéméconhecidocomofluxograma.

Assim,essaéaferramentautilizadaparaefetuaromapeamentoeamodelagem

dosprocessos.Dessaforma,eleéutilizadoparadescrever,demodográco,um

processoatravésdousodesímboloselinhas.

Dessamaneira,comelecamaisfácilvisualizareconhecermelhorum

processo,deformaquepossamospropormelhoriasemudanças.Oobjetivo,

portanto,dautilizaçãodessaferramentaésimplicarotrabalhodemapeamentoe

facilitaroentendimentodecomoosprocessosdetrabalhofuncionam.

Aseguirpodemosverumexemplodeuxograma(docasodoatendimentoda

pizzariaquecitamos):

Figura12.4–Fluxogramabásico

Quandoestamosprocurandomelhoraralgumprocesso,normalmentefazemos

primeiroummapeamentodoprocessodaformaatual(quechamamosdeAS-IS,

ouseja,“comoestá”,eminglês).Apósisso,montamosoutrouxogramacomas

alteraçõespropostas.

Esseoutrouxogramaéumredesenhooumodelagemdoprocessooriginal.

Assim,échamadodeTO-BE.Issoocorreporqueessenovouxogramaincorpora

asmelhoriasemudançasnecessáriasparaqueoprocessofuncionemelhor.

Assim,camaisfácilparaosgestoresvisualizaremasdiculdadesexistentesno

processoatualeasmelhoriaspossíveiscomaproposta.

Vamosvercomoissopodesercobrado?

3.(Cespe/Ceturb-ES/Administrador/2010)Os

uxogramasdaempresacontribuemparauma

gestãodeprocessosdequalidade.

Ouxogramaéumaferramentaquepossibilitaaogestormodelargracamente

(ok,desenhar..parafalarmaisfácil!)umprocessoqualquer,portantoajudana

gestãodeprocessosdequalidade.Ogabaritoéquestãocorreta.

12.7.ProcessoseCertificaçãoISO9000:2000

Agestãoporprocessosestámuitoligadaàgestãodaqualidade.Paraqueuma

organizaçãoconsigatrabalharcomqualidadeefornecerbenseserviçosque

atendamaosdesejosenecessidadesdeseusclientes,precisagerenciarseus

processosdetrabalho.

SegundoaISO9000,13gestãodaqualidadeéoconjuntode

“atividadescoordenadasparadirigirecontrolarumaorganizaçãonoquedizrespeitoàqualidade”

Bom,vamosentãoentenderoqueéISO9000?

ISOéonomedeumaorganização(eminglês:InternationalOrganizationfor

Standardization).Essainstituiçãofoicriadaem1947e,atualmente,contacom

maisde130paísesmembros.NoBrasil,érepresentadapelaAssociaçãoBrasileira

deNormasTécnicas–ABNT.

AsnormasISO9000foramdisponibilizadasem1987etinhamcomoalvoa

deniçãodepadrõesparaagarantiadaqualidadenasempresas.Em2000,foi

lançadaumaatualização,aISO9000:2000.

ConformeanormaeditadapelaABNT,existemoitoprincípiosdegestãoda

qualidadequedevemserseguidospelosgestoresparamelhorarodesempenho:14

ØFoconocliente

ØLiderança

ØEnvolvimentodepessoas

ØAbordagemdeprocesso

ØAbordagemsistêmicaparaagestão

ØMelhoriacontínua

ØAbordagemfactualparatomadadedecisão

ØBenefíciosmútuosnasrelaçõescomosfornecedores

Aversão2000trouxeumfocomaiornosprocessos.DeacordocomZacharias,15

ositensmaisimportantessão:

Øsatisfaçãodocliente;

Ømelhoriacontínua;

Øprocessos.

Assimsendo,oenfoquenagestãoporprocessosévistocomoumpilardagestão

daqualidade.Esseenfoqueconsistenavisãodeumaempresabaseadaemseus

processos,enãobaseadaemseusdepartamentos(visãotradicional).

Muitasvezes,asempresastêmproblemasnacoordenaçãoentreasdiversasáreas

responsáveisporumprocessodetrabalho.Cadasetoracabatendo“objetivos”

diferentesediculdadesdesecomunicarcomosoutros,causandodefeitos,

atrasoseaumentodecustos.

Issoéderivadodavisãotradicional,emqueaspessoassãovistascomo

pertencentesaoseu“setor”enãocomopeçasdeum“todo”.Assim,acabamse

preocupandosomentecom“suaparte”enãoentendemqueseosetorseguinte

nãotiverumbomdesempenhoaempresacomoumtodoirásofrer.Ficaaquele

“jogodeempurra”,emqueumpassaaculpaparaooutro.

Noenfoquedeprocessos,todososprossionaiseasáreasenvolvidasemum

processodevemtrabalharjuntoseconhecerasdemandaseosproblemasdecada

área.Éoquesechamadevisãohorizontal,aocontráriodeumavisãovertical

(baseadaemdepartamentosehierarquias).

Alémdisso,asnormasISO,comsuapreocupaçãocompadrõesdequalidade,

facilitamaadoçãodediversaspráticaseprodutosquepodemsermaisfacilmente

compradosevendidosemtodoomundo,poissãointercambiáveis.

DeacordocomValls,16

AsnormasNBRISOsérie9000compõemumconjuntodenormastécnicasquetratam

exclusivamentedegestãodaqualidade,nasuaexpressãomaisgeralesistêmica.Suaadoçãopassoua

serreconhecidapelomercadocomoum“atestadodegarantiadaqualidade”,eoconsumidornal,

cadavezmaisatentoaosaspectosdequalidadeesegurança,tendeaidenticareprivilegiaras

organizaçõesquedispõemdecerticação,porconsideraressefatoumsinônimodeseriedadee

confiabilidade.

Dessamaneira,asnormasISO9000buscam,atravésdapadronizaçãodos

processos,umamaiorprevisibilidadenofornecimentodebenseserviços,a

reduçãodoscustosdeoperaçãoedosriscosenvolvidosnonegócio.

Essasnormasforam“desenhadas”combasenaexperiênciadetécnicosdetodos

ospaísesmembros,sendoconsideradasum“consenso”mundialarespeitodas

melhorespráticasnagestãodaqualidade.

Apesardisso,umaempresaquesigaasnormasnãoestá“isenta”deproblemas.

Ouseja,aISO9000nãoéuma“panaceia”queresolvetodososproblemasdeuma

empresa.Emprovasdeconcurso,émuitocomumqueabancaarmealgocomo:a

observânciadasnormasISO9000“garante”umprodutodequalidade.Prestem

atenção,poisessaéuma“pegadinha”comum!

AsnormasISOnãogarantemporsisósaqualidade,massãonormasquebuscam

umagestãodaqualidade.Ouseja,denemosrequisitosnecessáriosparaqueuma

empresapossaobterumníveldequalidadedesejadoparaseusprodutoseserviços.

Essasnormassãoaplicáveisaorganizaçõesdediversossetores(comoindústria,

serviços,comércioetc.)epodemserutilizadasporinstituiçõesdediversos

tamanhos.

Vamosvercomoessetópicojácaiuemprovas?

4.(Cespe/TCDF/ACE/2012)AorganizaçãoquesealinhaaosprincípiosdacerticaçãoISO9000:2000

devebuscarreduzirospreçosdecompra,visandonegociardeformaquesejamaisvantajosa

parasidoqueparaseusfornecedores.

AISO9000:2000buscaauxiliarosgestoresnaimplementaçãodeumsistemade

qualidade,indicandoosprincípiosefundamentosdessesistema.Dentrodessa

ideiadegestãodaqualidade,anegociação“ganha-perde”comosfornecedores

nãoéincentivada,muitopelocontrário.

Oqueévistocomoidealnessesistemaéumrelacionamento“ganha-ganha”

comosfornecedores,emqueexistemumatrocamaiordeinformaçõeseum

relacionamentomaisharmoniosoedelongoprazo.Dessemodo,ogabaritoé

mesmoquestãoerrada.

12.8.NoçõesdeEstatísticaAplicadaaoControleeàMelhoriade

Processos

Oconhecimentodeestatísticaéfundamentalnaanáliseenamelhoriade

processosorganizacionais.Somentecombaseemopiniõeseno“achômetro”dos

gestores,émuitodifícilfazerumdiagnósticocorretosobreosresultadosdeum

processo.

Parapodermosanalisarumprocessocorretamente,necessitamosdeumaanálise

combaseemdadoseindicadoresdedesempenho.Paraisso,utilizamosdiversas

ferramentasestatísticas,queiremosapresentaraqui.

Nossointuitonãoserá,naturalmente,esgotaroassuntoetrabalharasequações

matemáticasenvolvidasnosestudosestatísticos,massomenteexplicarcomoa

estatísticapodeauxiliarotrabalhodosadministradoresequaissãoosprincipais

conceitos.

12.9.PopulaçãoeAmostra

Quandoqueremosestudaralgumfenômeno,devemosentendereidenticara

populaçãoenvolvida.Esseconceitosignicatodosos“eventos”ouintegrantesdo

fenômenotrabalhado.Algunsautorestambémchamamesseconceitodeuniverso.

Assimsendo,seestivermosanalisandoocomportamentodosbrasileirosperante

ocasamento,porexemplo,ouniversoenglobariatodososcidadãosbrasileiros.

Quandoestamosnosreferindoaumprocessodetrabalho,apopulaçãopoderia

serdescritacomotodososresultadosapresentadospeloprocesso(produtos,

serviçosetc.).

Naturalmente,ocontroleouaavaliaçãodetodaumapopulaçãoseria

extremamentecaroedemorado(comoocorrenoCenso).Comisso,utilizamos

ferramentasestatísticasparapodermosanalisarpartesdapopulação,chamadasde

amostras.

Umaamostraé,portanto,umsubconjuntoqualquerdapopulação.17Deacordo

comMaranhãoeMacieira,

Usualmentehágrandediculdadeeporvezesatéháimpossibilidadedetratarcomtodaa

população.Issosedáporumaquestãodecustosouatémesmoporimpossibilidadetécnica.

Assim,aoanalisarmosumprocesso,deveremosescolherumaamostraque

representedemaneiracorretaoprocessocomoum“todo”.

Figura12.5–Populaçãoeamostra.

(Adaptadode:MaranhãoeMacieira,2010)

Acaracterísticamaisimportanteentãoaoanalisarmosumaamostraéasua

representatividade.Umaamostraérepresentativaquandotemasmesmas(ou

semelhantes)característicasdapopulaçãoouuniversodoqualfoiretirada.

Dessemodo,emumapesquisadeopiniãodevemosterocuidadodeescolher

umnúmerodeentrevistadosque,somados,sejamumreexodototaldaspessoas

envolvidas.

12.10.MédiaeVariabilidade

Quandoestudamososprocessos,temosdoistiposdemedidasestatísticas

importantes:

asmedidasdetendência(ouposição)easmedidasde

variabilidade(oudispersão).

Asprincipaismedidasdetendênciasão:

Ømoda;

Ømédia;

Ømediana.

Amodaéoresultadomaisfrequentedeumroldedados.Pelopróprionome,é

oresultadoquemaisaparece.Vamosverissonaprática?Imaginequeestamos

analisandootempodeprodução,emminutos,deumbrinquedo.Osresultados

analisadossão:

32,35,36,38,38,38,39,42,45

Vejamqueoresultadoquemaisapareceéode38minutos.Essaseriaamoda

dessesdados.Jáamédiamostraopontoexatoemqueasomadosvaloresà

esquerdaseriaamesmadasomadosvaloresàdireita.Asomadosresultadosdo

processocitadoseria:

32+35+36+38+38+38+39+42+45=38,11

Jáamedianaapontariaoponto“médio”dosresultados.Adiferençaparaa

médiaédequeaquinãoestamosfalandodosresultadossomados,masda

quantidadedosresultados.Ouseja,senossoroldedadosapontanove“resultados”

oudados,amedianaéoquintodadodaamostra.

Assimsendo,oquintodadoseriaa“posiçãocentral”dosdadosdaamostra.

Portanto,nossamedianaseria38minutos,assim:

32,35,36,38,38,38,39,42,45

Jáasmedidasdevariabilidadeapontariamquantoosdadossãodispersosem

relaçãoàmédia.18Assim,quantomais“espalhados”,maiorsuavariabilidade.As

medidasdevariabilidademaisimportantessão:aamplitude,avariânciaeo

desviopadrão.

Esseúltimoéconsideradoumindicadorimportantenaanálisedosprocessos.

DeacordocomMaranhãoeMacieira,

Quandoosvaloresdosresultadosdeumprocessosãosignicativamentedispersosemtornoda

média,podemossuspeitarqueodesviopadrãoé“grande”(ouavariabilidadedoprocessoé

“grande”)equeoprocessoéimprecisoouirregular.

Dessamaneira,aoanalisarmososresultadosdeumprocesso,temosdeavaliaros

dadosdamédiaedodesviopadrão,poisissonosforneceumdiagnósticodecomo

estãoosresultadosmédiosequaléaprecisãodoprocesso(ouqualéasua

regularidade).

12.11.MelhoriadosProcessos

Paramelhorarmosumprocesso,temosdemelhorarasuamédia(aumentarou

reduzir,deacordocomasituação)esemprereduzirsuavariabilidade.19

Vamosvoltaraonossocasocitado?Seoprocessodefabricaçãodeum

brinquedoestáapresentandoosresultadosanalisadosaseguir,umdenossos

objetivosserábaixarovalordamédia.

32+35+36+38+38+38+39+42+45=38,11

Dessaforma,oidealéproduziremmenostempo,nãoémesmo?Assim,ao

reduzirmosotempomédiodefabricação,ganharemoseficiência.

Outroobjetivoimportanteéodareduçãodavariabilidade.Issodecorredofato

dequenãonosadiantamuitoterumaboamédiaseosresultadosvariammuito.

Nocasoemquestão,areduçãodavariabilidadedotempogastonaprodução

nosforneceriaumamaiorprevisibilidadedoprocessodefabricação–auxiliando

noplanejamento.

Alémdisso,aprevisibilidadetambéméfundamentalquandoestamosfalando

dosrequisitosdequalidadeedotempodeentregaaocliente,porexemplo.

Amodernagestãodaqualidadeéfocadaexatamentenareduçãoda

variabilidadedosprocessosdetrabalho.Dentreosprogramasmaisconhecidosque

focamnesseponto,podemoscitaroSeisSigma,programadesenvolvidona

Motorolanosanos1980quetemcomoobjetivoreduzironúmerodeproblemas

paramenosde3,4partespormilhão.

12.12.Reengenharia

DeacordocomHammereChampy,20areengenhariaé:

orepensarfundamentaleareestruturaçãoradicaldosprocessosempresariaisquevisamalcançar

drásticasmelhoriasemindicadorescríticosecontemporâneosdedesempenho,taiscomocustos,

qualidade,atendimentoevelocidade.

Portanto,oconceitodareengenhariaserelacionaamudançasdrásticasno

mododesefazeremascoisasdentrodeumaorganização.Muitasteoriase

técnicasadministrativassepreocupamcomamelhoriadosprocessoseatividades

jáexistentesdentrodeumaempresa.

Jáareengenhariapartedopontozero,ouseja,ogestordeve“começarde

novo”.Oproblemaaquinãoéabuscademelhoraroquejáexiste,masquestionar

oqueéfeito,porqueméfeito,porqueéfeito,paraquemetc.

Muitasorganizaçõesatuaisforamcriadasváriasdécadasatrás.Assim,muitasde

suasnormasregras,eprocessosdetrabalhoforamdesenvolvidoscombaseem

demandasedesafiosquenãomaisexistem.

Portanto,ospressupostosquelevaramacertasatividadeseprocessosatuaisjá

nãomaissejusticam.Dessaforma,asempresasgastamtempoeenergiacom

atividadesquenãoagregammaisvaloraoclienteeàprópriaorganização.

Atenção:Muitasbancasbuscamconfundirocandidatoarmandoqueareengenharia

buscaumamelhoriacontínua–nãocaianessa!

Comotempo,muitaspessoasjánemsedãomaiscontadequecertasatividades

não“servem”maisparanada.Assim,busca-semelhorarouaperfeiçoaratividades

quenãodeveriammaisestarsendofeitas!DeacordocomHammereChampy,a

reengenharianãobuscaconsertarnada!21Elabuscamelhoriasradicaisnos

processos.

Figura12.6

Dessaforma,todososprincipaisprocessosdaorganizaçãodevemserrevistos.

Assim,osprocessosdevemser“montados”deacordocomosdesaosedemandas

atuais(emcontrastecomosprocessosjáexistentes,baseadosemdesaos

passados).

Assim,osfundamentos(alógica)decadaprocessodevemseranalisados.O

gestornãodeveiniciarseutrabalhopensando:“comopossomelhorarocontrole

dosestoques?”,poisessaperguntapartedoprincípiodequeosestoquesdevemser

controlados.Talvezocustodocontroleestejasendomaiordoqueovalordos

estoques.

Paraentenderareengenharia,temosdeconhecerosconceitosdeprocessos.A

reengenhariatratadeserepensaremosfundamentosrelacionadosaosprocessos

organizacionais,demodoqueaorganizaçãopossaredesenharseusprocessospara

melhorardrasticamenteseusresultados.

Aqui,nãosebuscammelhoriasmarginaisecontínuas.Oobjetivoéconseguir

umaumentograndenodesempenho.Deacordocomosautores:22

areengenharianãodizrespeitoamelhoriasmarginaisoudepequenasquantidades,masasaltos

quânticosdedesempenho.

Dessamaneira,seaempresaestánecessitandodeumaumentodedesempenho

pequeno,umajusteno,nãodeveiniciarumprocessodereengenharia.A

reengenhariasódeveacontecerquandoaorganizaçãoestáquerendodestruiro

processoqueexisteeconstruiralgototalmentenovonolugar,ok?

Paraatingiressasmudançasdrásticasnosprocessos,asempresasaproveitamas

novastecnologiasdainformaçãodisponíveis.Dessaforma,muitasatividades

podemdeixardeserexecutadasporqueouxodeinformaçãopassaa“correr”

maisfacilmentedentrodaorganizaçãodoqueantes.

Deacordocomosautores,23“atecnologiadainformaçãoagecomoum

capacitador,permitindoàsorganizaçõesrealizarotrabalhodeformas

radicalmentediferentes”.

12.13.OqueaReengenhariaNãoÉ?

Normalmenteoconceitodereengenhariaéconfundidocomoutrastécnicas

administrativasouprogramasdemelhoriasnasorganizações.Muitasbancas

buscamconfundirocandidatocomperguntasquemisturamareengenhariacom

essesoutrosconceitos.

Porexemplo,apesardeseutilizardastecnologiasdeinformação(TI),a

reengenharianãoéumsinônimodeautomaçãodeprocessos.Umprocesso

desnecessáriopodeserautomatizado,sendofeitodemodomaiseciente,nãoé

mesmo?Masissonãoéreengenharia!

Portanto,aautomaçãopodeserumaformadesefazeralgoquenãodeveria

estarsendomaisfeito(atividadeserradasoudesnecessárias)demodomais

eciente.Areengenhariavaimaisafundo,questionandoaprópriaexistênciado

processo,suanecessidade.

Areengenhariatambémnãoéumdownsizingouumareestruturaçãopurae

simples.Odownsizingéumtermonovoqueserelacionaaumareduçãodepessoal

eestruturafísica(ouseja,decapacidade)paraqueaempresaseadapteauma

situaçãodedemandamenor.

Deacordocomosautores,24odownsizingéfazermenoscommenos.Jáa

reengenhariaéfazermaiscommenos!

Aempresapodereduzirseusníveishierárquicos,porexemplo,econtinuar

fazendoseusmesmosprocessosantigoseobsoletos.Dessaforma,lembre-sedeque

areengenhariapartedoprincípiodequeosproblemasdasempresasserelacionam

comosprocessosdetrabalho,enãocomsuaestruturaorganizacional.

Outraconfusãocomumécomosprogramasdequalidade.Areengenharianão

serelacionacomaGestãodaQualidadeTotal.Deacordocomosautores:25

osprogramasdequalidadetrabalhamdentrodaestruturadosprocessosexistentesemuma

empresaeprocurammelhorá-losatravésdoqueosjaponeseschamamdekaizen,oumelhoria

gradualecontínua.Oobjetivoécontinuarfazendooquejáéfeito,sóquemelhor.Amelhoriada

qualidadeprocuramelhoriasgraduaiseconstantesnodesempenhodoprocesso.

Areengenharianãobuscamelhoraroqueexiste,esim“começardozero”!

Portanto,nãoserelacionacomosprogramasdemelhoriadaqualidade.

Vejaasprincipaiscaracterísticasdareengenharia:

Figura12.7–Característicasdareengenharia

Vamosvercomoessetemajáfoicobrado?

5.(Cespe/Ceturb-ES/Administrador/2010)Areengenhariadeprocessosobjetivaareduçãodo

númerodecolaboradorespormeiodoredesenhodosfluxosprocessuaisdoempreendimento.

Areengenharianãoobjetivaareduçãodepessoal.Elatemcomometasa

reduçãodecustos,areduçãodotempogastoeaumentodaqualidade.Aredução

depessoalpodeeventualmenteocorrer,masnãoéoobjetivo!Portanto,o

gabaritoéquestãoerrada.

QuestõesComentadas

6.(FCC/TJ-AP/Analista/2009)ISO9000:

a)éumconjuntodenormasquesópodemserutilizadasporempresasgrandesdecaráterindustrial;

b)éumpacotedesoftwaresorientadoparaimplantaçãodesistemasdequalidadeemempresasdosetor

deinformática;

c)conferequalidadeaumproduto(ouserviço),garantindoqueoproduto(ouserviço)apresentará

sempreasmesmascaracterísticas;

d)forneceumcerticadosegundooqualoprodutoavaliadoterámaiorqualidadequeumoutrosimilar

semacertificação;

e)dizrespeitoapenasaosistemadegestãodaqualidadedeumaempresa,enãoàsespecicaçõesdos

produtosfabricadosporessaempresa.

Aprimeiraarmativaestáerrada,poisasnormasISOpodem,sim,serutilizadas

porempresasdetodososportesesetoreseconômicos.NaletraB,abancaestá

lançandouma“cascadebanana”paraoscandidatosmenosatentos.AISOnãose

tratadeumpacotedesoftware.

NocasodaletraC,abancaestáutilizandoumafamosa“pegadinha”.Asnormas

ISOnãogarantem,porsisós,aqualidadedeumaempresa.Omesmoocorrecoma

letraD,poisacerticação,porsisó,éapenasumindicativodequeaquela

empresatemumsistemadegestãodaqualidademaiseciente.Finalmente,a

alternativaEestácorretaeéonossogabarito.

7.(Esaf/MPOG/EPPGG/2009)Sobreotema“gerenciamentodeprocessos”,écorretoafirmarque:

a)apartirdaestratégia,aorganizaçãodeveidenticarseusprocessoscríticos,gerenciando-oscombase

emumenfoquedesistemaaberto;

b)osesforçosdemapeamentodevemserabrangentes,demodoapermitirumacompletavarredura

emtodososprocessosdaorganização;

c)umbommapeamentodeprocessosdecorredesesaberqueaabordagemverticalémaisimportante

queaabordagemhorizontal;

d)todososprocessosorganizacionaisdevemserotimizados,mesmoaquelesquenãoagregamvalor;

e)consideradoummemsimesmo,omapeamentodeprocessosémaisimportantequeo

cumprimentodasmetaseobjetivosestabelecidos.

Aprimeirafraseestáperfeita.Deacordocomsuaestratégia,aorganizaçãodeve

“mapear”seusprocessoscríticosparaquepossaatingirseusobjetivos.NaletraB,

existeuma“pegadinha”.Nãosãotodososprocessosquedevemsermapeados,

somenteosrelevantes.

AletraCinverteoconceito,poisaabordagemhorizontalémaisadequadaà

gestãoporprocessos.AletraDtambémvainalinhadaletraBeestáequivocada.

Finalmente,aletraEéabsurda,poisobviamenteoqueimportasãoosobjetivos,

enãoomapeamentodosprocessosporsisós!Assim,ogabaritoémesmoaletraA.

8.(Esaf/CVM/Analista2010)Entreasdezmudançasfrequentesqueocorremnasempresascoma

reengenhariadosprocessos,temos:

a)asunidadesdetrabalhomudam,passandodeequipesparadepartamentos;

b)oscritériosdepromoçãomudamdedesempenhoindividualparadesempenhogrupal;

c)asestruturasmudamdehierárquicasparapessoais;

d)apreparaçãodosempregadosparaoserviçomuda,deixandodesertreinamentoparaserinstrução;

e)osserviçosdaspessoasmudam,passandodetarefassimplesparatrabalhosmultidimensionais.

EssaquestãosebaseouemumartigodeJoséErnestoGonçalvesnaRAP.26De

acordocomoautor,asdezmudançasmaisfrequentesqueocorremnasempresas

comareengenhariadosprocessossãoasseguintes:

Asunidadesdetrabalhomudam,passandodedepartamentosfuncionaisparaequipesdeprocesso;

Osserviçosdaspessoasmudam,passandodetarefassimplesparatrabalhos

multidimensionais:

Ospapéisdaspessoas,antesdefinidosecontroladospelosgerentes,passamaserdesenhadospelos

seusprópriosocupantes;

Apreparaçãodosempregadosparaoserviçomuda,deixandodesertreinamentoparasereducação;

Oenfoquedasmedidasdedesempenhoedaremuneraçãosealtera,daatividadeparaoresultado;

Oscritériosdepromoçãomudamdodesempenhoindividualparaahabilidade;

Osvalores,antesprotetoresdaorganização,passamainspiraraprodução;

Osgerentesmudamdesupervisoresparainstrutoresdeseustimes;

Asestruturasorganizacionaismudamdehierárquicasparaachatadas;

Osexecutivosdeixamdesercontroladoresdoresultadoparaseremlíderes.

Vejamqueabancaapenas“bagunçou”osconceitosexpostospeloautorpara

fazeraquestão.Paraquemnãotinha“decorado”otexto(quemdissequevidade

concurseiroéfácil?),sobrouafamosatécnicada“eliminação”.

AletraApoderialogosereliminada,poisumamudançadeumaestruturapor

equipes(maismoderna)paraumaestruturadepartamentalnãofariamuito

sentido,nãoémesmo?

Asegundaalternativajáseriamais“adequada”aocontextodareengenhariae

deixariamuitoscandidatoscomdúvidas.Entretanto,Gonçalvesapontaamudança

paraocritériodahabilidade.AletraCéconfusaenãofazsentido.

JánaletraDocandidatoteriadeteralgumconhecimentoemGestãode

Pessoas.Otreinamentoéfeitotendoemvistaocurtoprazo,asnecessidadesjá

aparentes.Aeducaçãoeodesenvolvimentosãovoltadosàsnecessidadesfuturase

potenciais.Finalmente,aletraEéonossogabarito.

9.(Cespe/FUB/Engenheiro/2011)OconjuntodenormasISO9000representaumaespéciede

consensointernacionalarespeitodasboaspráticasdeadministraçãodaqualidade.

Perfeito.AsnormasISO9000sãoconsideradasumconsensointernacionala

respeitodasmelhorespráticasdagestãodaqualidade,poisforamgeradasatravés

dodebateetrocadeexperiênciasdetécnicosdediversosórgãosquecompõema

ISO.Ogabaritoémesmoquestãocorreta.

10.(Cespe/Min.Esporte/Administrador/2008)Umadasprincipaispráticasdareengenharia

organizacionaléaeliminaçãodasbarreirasfuncionais,pormeiodareorganizaçãodas

atividadesemprocessos.

Reengenhariaéumprocessodereexaminartodososprocessoscríticosda

administraçãodesdeopontozero.Ouseja,pensarsobre:seráqueesseprocessoé

necessário?Seráqueamaneiracomoéfeitoéaquemaisagregavalorà

organização?Seráqueocontrolequeestásendofeitofazsentido?

Dessemodo,aorganizaçãodeixadeser“pensada”emtermos

departamentais/funcionaisparaseranalisadasobopontodevistadosprocessos.O

gabaritoéquestãocorreta.

QuestõesPropostas

11.(FCC/Bahiagas–Administrador/2010)Umgrupodeatividadesrealizadasnumasequêncialógica

comoobjetivodeproduzirumbemouserviçoquetemvalorparaumgrupoespecícode

clientesédenominado:

a)processo;

b)cicloPDCA;

c)kaizen;

d)fluxograma;

e)just-in-time.

12.(Cespe/Ibram/Administrador/2009)Pode-seconsiderarcomoprocessoorganizacionalo

conjuntodeatividades,mesmoquenãorelacionadasentresi,quetransformaminsumos

(entradas)emprodutosouserviços(saídas)quetêmvalorparaumgrupoespecícode

clientes.

13.(FCC/Infraero/Administrador/2009)Sãoobjetivosdagestãoporprocessos:

a)simplicarprocessos,condensandooueliminandoatividadesquenãoacrescentamvalor,eadotarum

enfoquepontualdosnegóciosdaorganização;

b)elevarsensivelmenteaprodutividade,comeciênciaeecácia,eaumentarovalordoprodutoou

serviçonapercepçãodocliente;

c)determinarmetasdelongoprazoeadotarcursosdeaçãoealocaçãoderecursosnecessáriosà

consecuçãodessasmetas,alémdeestabelecerumplanomestrequeespeciquecomoaorganização

alcançarásuasmetas;

d)organizarasatividadesemfunçãodastarefasenãodassaídasdeumprocesso,comoaconteceem

organizaçõestradicionais,ereduzirníveishierárquicosparafacilitaraimplementaçãodeumaestrutura

matricial;

e)identicarosresponsáveisoulíderesdosprocessos,paraeliminaranecessidadedepontosde

controle,eadotarumenfoquesistêmicodosprocessos.

14.(Esaf/ANA/Analistaadm./2009)AGestãodeProcessosenvolveomapeamentoeaanálisepara

amelhoriadeprocessosorganizacionais.Entreasopçõesaseguir,selecioneaincorreta.

a)AmodelagemdeprocessoséfeitapormeiodeduasgrandesatividadesintituladasAsiseTobe.

b)Afaseinicialdamodelagemdeprocessostenta,entreoutrosobjetivos,entenderoprocesso

existenteeidentificarsuasfalhas.

c)OredesenhodeumprocessoéexecutadonafaseAsisdomapeamento.

d)Amelhoriacontínuaéumadasmetodologiasderacionalizaçãodeprocessos.

e)Ametodologiadeinovaçãodeprocessoscaracteriza-seporserumaabordagemradicaldemelhoria

deprocessos.

15.(Cespe/–MPS–Administrador/2010)Umuxogramadentrodeumaorganizaçãomostracomo

sefazotrabalhoeanalisaproblemascujasoluçãoécompatívelcomoexercíciodeuma

administraçãoracional.Trata-sedarepresentaçãográcasequencialdeumtrabalho,

caracterizandoasoperaçõeseosresponsáveispeloprocesso.

16.(Cespe/MPS/Administrador/2010)Oestudodeprocessoscríticosnaorganizaçãotemoobjetivo

deassegurara

uidezdamovimentaçãoentrepessoaseunidadesdaorganização,

estabelecendolimitesdedecisãoquepermitamoaumentodaecáciaedaeciênciadessas

açõessistemáticas.

17.(Cespe/TJ-ES/Analista/2011)Emrazãodeenfatizarocliente,umadasprincipaiscaracterísticas

dareengenhariaconsistenaadoçãodemudançasradicaisemprocessoseprodutosda

organização.

18.(Esaf/MTE/–AFT/2006)Indiqueaopçãoquecorrespondecorretamenteàfraseaseguir:“Éo

repensarfundamentaleareestruturaçãoradicaldosprocessosempresariaisquevisam

alcançardrásticasmelhoriasemindicadorescríticosdedesempenho.”

a)Trata-sedadefiniçãodeprocessosdequalidade.

b)Trata-sedeprincípiosquenorteiamabuscademaioreficácia.

c)Trata-sedadefiniçãodereengenharia.

d)Trata-sedecaracterísticasdeumprocessoderesponsabilidadesocial.

e)Trata-sedadefiniçãodeprodutividade.

19.(Esaf/MTE/AFT/2003)Umdospontos-chavedareengenhariaérepensardeformafundamental

ereprojetarradicalmenteosprocessosparaconseguirmelhoriasdrásticas.Indiqueaopção

queexpressacorretamenteaideiacontidanessaafirmativa.

a)Diminuiçãodrásticadospostosdetrabalho.

b)Terceirizaçãodosserviçosnãoessenciaisaonegóciodaorganização.

c)Fusãodeunidadesorganizacionaisedeempresas.

d)Requalificaçãodamãodeobranabuscadeempregabilidade.

e)Análisedosclientes,insumos,informaçõeseprodutos.

20.(FCC/TJ/AP/Analistaadm./2009)Quandoumaorganizaçãodeciderealizarométodode

reengenharia,elaprecisaemprimeirolugar:

a)avaliarosmétodosdeengenhariaempregadosemsuarotinadeprocessos;

b)repensarradicalmenteosprocessosutilizadoseosresultadosalcançados;

c)racionalizarosprocessosdecompraparaeconomizarrecursosfinanceiros;

d)formularnovosprodutoseserviçosdemandadospelosclientes;

e)analisarecompararseusprodutoseserviçoscomosdoslíderesdomercado.

21.(FCC/PGE/RJ/Auditor/2009)“Umaempresadescobriuquesuaconcorrenteutilizavacincovezes

menosempregadosparaoperaroseudepartamentodecontabilidadedeformaeciente.A

soluçãoapresentadapelosconsultoresfoi:‘jogarfora’osatuaisprocessosecomeçardenovo,

utilizandoopoderdamodernatecnologiadainformaçãopararedesenharcompletamenteos

processos,deformaaalcançarprofundosmelhoramentosnasuaperformance.”Ométodo

gerencialescolhidopelosconsultoresfoi:

a)oEmpowerment;

b)aReengenharia;

c)oBalancedscorecard;

d)aQualidadeTotal;

e)okaizen.

22.(FCC/TCE/RO/Auditor/2010)Quandoaplicadaaosprocessosdereformadagestãopública,a

reengenhariatemcomoobjetivo:

a)transformarradicalmenteaculturadasorganizações,apoiando-senumavisãointegradaquevincula

mudançasestruturaisemudançascomportamentais;

b)reestruturarosprocessosorganizacionais,enfatizandoacentralizaçãoeaespecializaçãofuncional;

c)redeniraestruturadepartamentaldasorganizações,preservandoosconitoseaspolarizações

existentesentreeles;

d)descentralizaragestãodoconhecimentotácitodasorganizações,visandoumamaiorinteraçãoentre

direçãoeestruturasoperacionais;

e)introduzircontinuamentemudançasorganizacionaisvisandoareduçãodecustos,aracionalizaçãodos

recursoshumanoseamaioreficiênciadasorganizações.

23.(FCC/ISS-SP/AFTM/2007)Constituiprincípiobasilardoconceitooriginaldereengenharia:

a)oestabelecimentodealiançasestratégicasparasegmentaçãodomercado;

b)odrásticoenxugamentodoquadrodepessoal;

c)afusãodeestruturaseeliminaçãoderedundâncias;

d)aradicalredefiniçãodosprocessosembasezero,conhecidocomoprincípioda“folhaembranco”;

e)aterceirizaçãoemlargaescala.

24.(FGV/Sefaz/RJ/Auditor/2009)Comrelaçãoaosprocessosorganizacionais,analiseasafirmativasa

seguir.

I.Osprocessosorganizacionaisenvolvempessoas,procedimentos,recursosetecnologia.

II.Osprocessosorganizacionaisnãoafetamaspectosrelacionadosàcapacitaçãodasempresas.

III.Osprocessosorganizacionaissãorealizadosdediversasmaneirascomresultadosdiferentes

emtermosdecusto,valor,serviçoouqualidade.

Assinale:

a)sesomenteaafirmativaIestivercorreta;

b)sesomenteaafirmativaIestivercorreta;

c)sesomenteaafirmativaIIestivercorreta;

d)sesomenteasafirmativasIeIIestiveremcorretas;

e)setodasasafirmativasestiveremcorretas.

25.(FGV/Senado/Analistaadm./2008)Agestãodeprocessos(ougestãoporprocessos)éhojeuma

realidadeconcretaedesejadapororganizaçõesemtodoopaís.Autilizaçãodeuxogramasé

antiganomundodasorganizaçõesnopaís.Jánasdécadasde1950e1960surgiaouxograma

comouminstrumentoútilnoestudoenaaçãodasorganizações.Contudo,ousode

fluxogramasnaatualgestãodeprocessossofrerestrições.Umadessasrestriçõesé:

a)terumasimbologiaincompatívelcomamodernagestãodeprocessos,porseremcompostosporum

númeroexpressivodesímbolosquepoucosbenefíciostrazem,emgeral,aosgestores;

b)ofatodequeosmodernosmanuaisvirtuaistornaramofluxogramauminstrumentoobsoletoquando

sedesejacompreenderofluxodepessoasepapéisnumdadomomento;

c)anaturaldiculdadequefuncionáriosdeapoiotêmnainterpretaçãodossímbolostípicosdos

fluxogramas;

d)ofatodequenãoexistegestãodeprocessoscomasimplesaplicaçãodouxograma,poiseleapenas

registra,namaioriadosmodelosdeuxogramas,umúnicoprocesso,eraramentemencionaalguma

correspondênciacomoutrosprocessos;

e)que,curiosamente,consultoresegestorestêmcertaaversãoaosuxogramasporque,emgeral,eles

apresentamlongasrotinasquedificultamsuaprontaanáliseeimediatasimplificação.

26.(FGV/Senado/Analistaadm./2008)Comoformademudançaorganizacional,areengenhariade

processossecaracterizapeladrásticamudançaemmuitasestruturasesistemas.Segundo

ThomasDavenport(1994),amudançabaseadanareengenhariadeprocessosdeveser

compreendidaemtermosdecertascaracterísticas,dentreasquaisnãosedestaca:

a)magnitudegeraldamudançanecessária;

b)níveldeincertezaquantoaosresultadosdamudança;

c)amplitudedamudançanaseentreorganizações;

d)níveldepenetraçãonecessáriadasatitudesecomportamentosindividuais;

e)irrelevânciadoprocessodemudança.

27.(Cespe/MPU/Anal.administrativo/2010)Osprocessosorganizacionaisprescindemde

alinhamentocomaestratégiadaorganização.

28.(Cespe/MPU/Anal.administrativo/2010)Agestãoorganizacionalcombaseemprocessos

pressupõeaestruturaçãodaorganizaçãoemtornodomododerealizaçãodotrabalho,coma

preocupaçãovoltadaparaovalorquecadaatividadeagregaàanterior.

29.(Cespe/MPU/Anal.administrativo/2010)Gestãodeprocessoseaçõesdegestãodequalidade,

comoaspreconizadaspelasnormasISO,nãoguardamentresiqualquercorrelação.

30.(Cespe/Abin/Ocialtécnico/2010)Agestãodeprocessosinclui-senavisãosistêmicade

administraçãopública,naqualpredominaofoconaestruturadaorganização.

31.(Cespe/Abin/Ocialtécnico/2010)Umprocessopodeseraprimoradopelareduçãodonúmerode

etapasqueocompõemouderecursosqueutiliza.

32.(Cespe/Abin/Ocialtécnico/2010)Sematecnologiadeinformaçãoéimpossívela

implementaçãodagestãodeprocessos.

33.(FCC/MP/SE/Administrador/2009)Aogrupodeatividadesrealizadasemumasequêncialógica

comoobjetivodeproduzirbemouserviçoquetemvalorparaumgrupoespecícodeclientes

atribui-seonome:

a)insumos;

b)processo;

c)just-in-time;

d)qualidadetotal;

e)kaizen.

34.(FCC/Infraero/Administrador/2009)Emtermosdecapacidadedegeraçãodevalor,osprocessos

emumaorganizaçãopodemserclassificadoscomoprocessos:

a)primários,quecorrespondemaatividadesquegeramvalorparaocliente,edesuporte,quegarantem

oapoionecessárioaofuncionamentoadequadodosprocessosprimários;

b)nalísticos,destinadosàgeraçãodosprodutosnais,eintermediários,destinadosàgeraçãode

produtosparaclientesinternos;

c)verticais,relativosaoplanejamentoeorçamentoempresarialequeserelacionamcomaalocaçãode

recursosescassos,ehorizontais,quesãoaquelesbaseadosnofluxodotrabalho;

d)deentrada,quedenemaquantidadedosinsumoseseusfornecedores,edesaída,quedenema

quantidadedosprodutosfinaiseosrespectivosclientes.

e)demapeamento,queidenticamosmacroprocessos,eessenciais,queestãoassociadosàsregras

básicasdonegócio.

35.(FCC/TCE/GO/Desenv.Org./2009)Emumaorganização,osprocessos:

a)podemserorganizadosquantoàáreadeatuaçãoouotipodecliente,sendo,portanto,ligadosà

essênciadaorganização,eresultamnoprodutoouserviçoqueérecebidopelocliente;

b)responsáveispelouxodeinformaçãoetomadadedecisãoqueresultamemaçõesdemediçãoe

ajustedodesempenhodaorganizaçãosãodenominadosprocessosorganizacionaisoudeintegração;

c)queviabilizamofuncionamentocoordenadodosváriossubsistemasdeumaorganizaçãoembuscade

seudesempenhoglobalegarantemosuporteaosprocessosdenegóciosãodenominadosprocessos

gerenciais;

d)verticaissãobaseadosnouxodotrabalhoeenvolvemacoordenaçãodasatividadesespalhadaspor

váriasunidadesorganizacionais;

e)horizontaisreferem-seaoplanejamentoeorçamentodaorganizaçãoeàalocaçãoderecursos

escassos,comofinanceirosehumanos,sendoimportantesparaacoordenaçãolateral.

36.(FCC/Defensoria/SP–Administrador/2010)Opontocrucialnaanálisedeprocessosdetrabalhoé

adeterminação:

a)dosprocessoscríticosdaorganização;

b)dalacunadecompetências;

c)doorganogramafuncionaldaorganização;

d)doclimaorganizacional;

e)dospontosforteseoportunidadesdenegócios.

37.(FCC/TRF4aRegião/Anal.adm./2010)Osprocessos-chavedenegócioestãodiretamente

relacionadosàsatividades-fimesãocríticosparaosucessoorganizacional.

Analise,comrelaçãoataisprocessos:

I.Seusresultadosproduzemaltoimpactoparaosfornecedores.

II.Falhasnessesprocessoscomprometemodesempenhoparcialdosistema.

III.Nãosãocríticosparaaexecuçãodaestratégiadaorganização.

IV.Éprecisohaverconvergênciadoesforçoorganizacionaldemodoaminimizarriscos,tempo

edesperdíciosderecursosemaximizarsinergia.

V.Todososprocessosdasunidadesdenegóciosdevemestarintegradosnosistemadegestão.

ÉcorretooqueconstaAPENASem

a)IVeV.

b)IeIV.

c)II,IVeV.

d)I,IeII.

e)IeV.

38.(Cespe/Correios/Administrador/2011)Oprocessodereengenhariaorganizacionalresultaem

mudançasinternas,debaixoparacima,nasestruturasorganizacionais.

39.(Cespe/Correios/Administrador/2011)Reengenhariarefere-seàreformulaçãoquevisa

incrementarosprocessoscomerciaisdeumaorganização,deformaaagregarvaloraocliente.

40.(Cespe/Correios/Administrador/2011)Entreosobjetivosdareengenhariaorganizacional,

destacam-seasmelhoriasemqualidade,custo,serviçoevelocidadedosprocessos

organizacionais.

Gabaritos

1.C

11.A

21.B

31.C

2.E

12.E

22.A

32.E

3.C

13.B

23.D

33.B

4.E

14.C

24.D

34.A

5.E

15.C

25.D

35.A

6.E

16.C

26.E

36.A

7.A

17.C

27.E

37.A

8.E

18.C

28.C

38.E

9.C

19.E

29.E

39.E

10.C

20.B

30.E

40.C

Bibliografia

ABNT–AssociaçãoBrasileiradeNormasTécnicas.NBRISO9000.Sistemasde

gestãodaqualidade–Fundamentosevocabulário.Dezembro2000.

Davenport,omasH.ProcessInnovation–reengineeringworkthroughinformation

technology.1aed.Boston:HarvardBusinessSchoolPress,1992.

Gonçalves,JoséErnestoLima.Asempresassãograndescoleçõesdeprocessos.

RevistadeAdministraçãodeEmpresas–RAE,Jan/Mar.2000:6-19.

Gonçalves,JoséErnestoLima.Processo,queprocesso?RevistadeAdministraçãode

Empresas–RAE,Out/Dez.2000:8-19.

Gonçalves,JoséErnestoLima.Processo,queprocesso?RevistadeAdministraçãode

Empresas–RAE,Out/Dez.2000:8-19.

Gonçalves,JoséErnestoLima.Reengenharia:umguiadereferênciaparao

executivo.RevistadeAdministraçãodeEmpresasV.34(Jul/Ago1994):23-30.

Hammer,Michael,eJamesChampy.Reengenharia:revolucionandoaempresaem

funçãodosclientes,daconcorrênciaedasgrandesmudançasnagerência.Riode

Janeiro:Campus,1994.

Maranhão,Mauriti,eMariaElisaBastosMacieira.Oprocessonossodecadadia:

modelagemdeprocessosdetrabalho.2aed.RiodeJaneiro:Qualitymark,2010.

Martins,HumbertoFalcão,eCaioMarini.Umguiadegovernançapararesultados

daadministraçãopública.Brasília:PublixEditora,2010.

Nunes,SirleiMariadeSouza.Avaliaçãodagestãoporprocessosnocontextode

modernizaçãodaadministraçãofiscaldoCeará.Fortaleza:UECE,2010.

Valls,ValériaMartin.OenfoqueporprocessosdaNBRISO9001esuaaplicação

nosserviçosdeinformação.CiênciadaInformação.V.33,No2(Mai-Ago2004):

172-178.

Capítulo13

GestãodaQualidade

Antesdevermosoqueégestãodaqualidade,precisamossaberexatamenteo

queéqualidade.Oconceitodequalidadeébastanteantigo,masapenas

recentementeassumiuseussignificadosatuais.Consideremosalgumasdefinições:

Tuchman–“Qualidadeéatingiroubuscaropadrãomaisaltoemvezdese

contentarcomomalfeitooufraudulento.”1

Edwards–“Qualidadeconsistenacapacidadedesatisfazerdesejos.”2

Gilmore–“Qualidadeéograuemqueoprodutoespecícoestádeacordocom

oprojetoouespecificação.”3

Inicialmentesepensavaemgestãodaqualidadecomoalgoreativoevoltado

parainspeções.Atualmente,essasatividadesestãomaisabrangentesesãovistas

comocentraisparaaestratégiadeumaorganização.4

DeacordocomGarvin,5aevoluçãodaqualidadepodeserclassicadaem

quatroeras.Vamosvê-lasemdetalhes?

13.1.ErasdaGestãodaQualidade

13.1.1.EradaInspeção

ComaintroduçãodaproduçãoemmassaeaindustrializaçãonoséculoXIX,

cresceuovolumedaproduçãoepassouasernecessárioocontroledaqualidade.

Asindústriascaramcadavezmaiores,eexistiamcentenasdetrabalhadoresna

“linhademontagem”.

Nesseprincípiodaindustrialização,aqualidadeeraanalisadaapósaprodução,

poruminspetor.Dessaforma,todososprodutoseraminspecionadosapóso

processoprodutivo,semumapreocupaçãocomaprevençãodedefeitos.

Umainspeçãoeraefetuadadurantealgumaetapadoprocessoprodutivo(a

checagemdopesocorretodeumapeça,porexemplo)ouquandooprodutojá

estavafinalizado.

Basicamente,aequipedeinspeçãocomparavaalgumascaracterísticasdos

produtosemanálisecomalgunspadrõespreestabelecidos.Sealgumprodutonão

estivessedeacordocomopadrão,descartava-seoprodutoouvendia-secom

algumdesconto.

13.1.2.EradoControleEstatísticodaQualidade

Depoisdealgumtempo,asempresasperceberamqueagestãodaqualidade

deveriareceberumaatençãomaiordoqueaestavasendofornecida.

Paraentendermelhoroqueestavacausandoosdefeitosedesperdícios,os

controlesdeveriamsermaisdetalhados.Atravésdousodemodelosmatemáticos,

iniciou-seocálculodospercentuaisdefalhasedelimitesaceitáveis.

Alémdisso,ousodaamostragempassouaserutilizadonocontroleda

qualidade,facilitandoebarateandoocontrole.Ocontroleestatísticopossibilitou

ummaiorconhecimentodosprocessosdetrabalhoequaiseramosfatoresque

impactavammaisnaqualidadedosprodutoseserviços.

Dessaforma,comocontroleestatísticoosprocessosproblemáticoserammais

facilmentedescobertos.OobjetivodosgestoresnestaEraédescobrirecorrigiros

errosnosprocessos.6

13.1.3.EradaGarantiadaQualidade

NaEradagarantiadaqualidade,oobjetivopassouaseraprevençãodos

defeitosedosdesperdícios.Oplanejamentodasaçõeseapreocupaçãoem

aprendercomosproblemasencontradostornam-semaisimportantes.

Alémdisso,otrabalhodegestãodaqualidadepassouaabrangeroutras

ferramentas:ocálculodocustodaqualidade,ocontroletotaldaqualidade,

engenhariadaconfiabilidadeeamentalidade“zerodefeito”.

JuraneDemingforamautoresimportantesnessaépoca.7Apreocupaçãopassou

aserglobaleholística(quevêaorganizaçãocomoumtodo),englobandotodosos

aspectosdofuncionamentodeumaorganização.8

Basicamente,ummaiorfocoédadonoplanejamentodaqualidade,no

treinamentodosfuncionários,namelhoriadodesigndosprodutoseprocessos,na

eficiênciadoscontrolesenamotivaçãodosempregados.9

13.1.4.EradaGestãoEstratégicadaQualidadeouGestãodaQualidadeTotal

Aqualidadepassaaservistacomofundamentalnagestãoestratégicadeuma

organização.Aqualidadedeumaempresaévistacomodiferencialcompetitivo

pelosseusclienteseparceiros.

Oambientecadavezmaiscompetitivoeumaclientelacadavezmaisinformada

levamàapariçãodenovasleisdeproteçãoaoconsumidoredenormais

internacionaisdequalidade,comoaISO9000.

NaGestãodaQualidadeTotal,aqualidadeévistacomofatorprincipalno

atendimentodedesejosenecessidadesdosclientes.Assim,agestãodaqualidade

deveenglobartodososaspectosdaorganização,inclusiveasrelaçõescomos

fornecedoreseclientes.

Entretanto,

não

devemos

satisfazer

somente

os

clientes

externos

(consumidores),mastambémosinternos,ouseja,oscolaboradoresinternosque

dependemdenossotrabalho.

DeacordocomanormaISO8402,aGestãodaQualidadeTotalé:10

umaabordagemgerencialdeumaorganizaçãocentradanaqualidade,baseadanaparticipaçãode

todososseusmembros,buscandoumsucessodelongoprazoatravésdasatisfaçãodeseusclientes

ebeneficiandotodososmembrosdaorganizaçãoeasociedade.

Aorganizaçãoévistademodoabrangente,poisnãoadiantaumsetorter

qualidade(produção,porexemplo)eoutrosetor(entregas,porexemplo)deixaro

clientena“mão”,nãoémesmo?

Figura13.1–ErasdaGestãodaQualidade

DeacordocomCrosby,11paraobtermosqualidadenãopodemos“aceitar”que

existampequenasfalhas.Paraoautor,devemosbuscaro“defeitozero”:

Opropósitodaqualidadenãoéacomodarascoisaserradas,éeliminá-lasparaevitartais

situações.Éexatamenteamesmacoisadasfalhasdesoldagemqueencontreiquandotrabalhavana

Martin.Enquantomantínhamosumnívelaceitáveldelas,elasexistiam,quandosetornaram

inaceitáveis,desapareceram.

OmovimentodagestãodaqualidadefoiimpulsionadoinicialmentenoJapão

comainuênciadetécnicosamericanos,comoDemingeJuran,nosanos1950e

1960,apósopaístersuainfraestruturadestruídanaSegundaGuerraMundial.12

Comacontribuiçãodessesautores,asempresasjaponesaslargaramoantigo

modelodecontroledaqualidadeporinspeçãoparaummodeloqueenvolvesse

todososempregadosnabuscadaprevençãodeproblemasenabuscada

qualidade.

Dessaformaosvaloresdabuscadaqualidadesãoincutidosemcadaempregado

enaexecuçãodecadatarefaeprocesso.Oselementosmaisimportantesdagestão

pela

qualidade

são:envolvimentodosempregados,foconocliente,

benchmarkingeaevoluçãocontínua.13

AGestãodaQualidadeTotal(TQM–TotalQualityManagement,eminglês)

demandaquecadafuncionárioestejaenvolvidonoprocessodebuscadoaumento

daqualidade,enãoapenasosempregadosquelidamdiretamentecomosclientes

ounaproduçãodosprodutos.

Opessoaldalimpeza,dosetordefaturamento,dosetordecontratoseoutros

setoresdeveentenderqueseutrabalhocontribuiparaqueaempresasejaeciente

econsigasatisfazerseusclientesesercompetitiva.Dessaforma,aTQMsignicao

comprometimentodeumaorganizaçãocomaqualidadeatravésdamelhoria

contínuaemcadaatividade.14

Todosdevemteremmenteasnecessidadeseosdesejosdosclientes.As

organizaçõesqueaplicamaqualidadetotalprocuramdescobriroqueseusclientes

querem,parapoderofereceressesprodutoseserviçosesatisfazê-los.Deacordo

comColtro,15

nota-sequea“grande”qualidadetemumenfoquebastanteabrangenteetotalemrelaçãoà

organização.Esseenfoqueexigeumamudançaagudanalosoatradicionalquesepraticaem

termosdeproduçãonasempresas,umamudançadeumfocobaseadoemcustose

produtividadeparaumfundadoemqualidadeenavisãodocliente/consumidor.

Veremosaseguiraimportânciadotrabalhodessesautoresnaevoluçãoda

gestãodaqualidade.

13.2.PrincipaisAutoresou“Gurus”daQualidade

13.2.1.Shewart

WalterShewartéconsideradopormuitoso“paidocontroleestatísticoda

qualidade”.Esseengenheiroamericanofoiograndeinspiradordosestudosdos

maisconhecidos“gurus”daqualidade:DemingeJuran.

FicoutambémconhecidopeloseutrabalhonacompanhiaBellTelephone

Industries,ondetrabalhoupormaisde30anos.Entreoutrosimpactosnagestão

qualidade,ShewartcriouocicloPDCA(quedepoiscouconhecidopelonome

decicloDeming)eográficodecontrole.VamosconhecerocicloPDCA?

13.2.2.CicloDemingouPDCA

OciclodeDeming(oudeShewart,seucriador)ou,comoémaisconhecido,

PDCA,éumaferramentanabuscadamelhoriacontínua,dokaizen.Oobjetivo

daferramentaésimplicaroprocessodemelhoriadosprocessoseacorreção

deproblemasefazercomquequalquerfuncionáriodaorganizaçãopossa

participardesseprocessoemelhoraraqualidadedaorganização.

Échamadodeciclo,poisnãodevenuncaterminar,sendoseuprocessosempre

reiniciado.Osresultadosdeumapassagemdociclosãoconsideradosno

planejamentodapróximapassagem.

DeacordocomJunioretal,16ocicloPDCAéummétodogerencialparaa

promoçãodamelhoriacontínuaereete,emsuasquatrofases,abasedalosoa

domelhoramentocontínuo.

EssemétodofoidesenvolvidoporShewartapartirdaanálisedequeamaioria

dosproblemasemumprocesso,consideradoscomooportunidadesdemelhoria,é

causadapeladiferençaentreasnecessidadesdosclienteseodesempenhodo

processo.

Ouseja,quedeveriaexistirumamelhoriacontínuadosprocessosparaqueestes

conseguissem“entregar”exatamenteoprodutoouserviçonecessárioaocliente.

Assimsendo,seuspassosprincipaissãovistosnafiguraaseguir.

Figura13.2–CicloPDCA

AsetapasdoPDCAsãoasseguintes:

Plan(planejar)–nestaetapadevem-seestabelecerosobjetivoseasmetas,de

formaquesejaestabelecidoquaisserãoasaçõesemétodosparaqueosobjetivos

sejamalcançados.

Do(executar)–nestafaseoquefoiplanejadoseráexecutado,ouseja,entrará

emação.Alémdisso,nestafasedeverãosercoletadososdadosparaquepossamos

analisarosdadosnapróximafase.Alémdessefato,nestafasedevem-setreinaros

funcionáriosnasatividadesetarefasespecíficasquedevemexecutar.

Check(verificar)–nestaetapaexisteavericaçãodosresultadosdasações

implementadasnafaseanterior.Paraisso,seráutilizadaumasériedeferramentas

paraatomadadedecisão,comohistogramas,diagramasdeIshikawa,cartasde

controle,entreoutras.

Act(agirdeformacorretiva)–nestaetapapodemocorrerduascoisas

distintas.Seosresultadosforembons,deveexistirumesforçodepadronizaçãodas

açõesedosplanejamentosadotados,jáquealcançaramoobjetivo.Entretanto,se

osresultadosforemruins,devem-sebuscarasrazõesparaasfalhas,demodoa

revisaroprocessoeevitarqueosproblemasvoltemaacontecer.

UmdosobjetivosdoPDCAé,portanto,aumentaraprevisibilidadenos

processosorganizacionaiseaumentarachancedesucessodaempresa.Essa

previsibilidadeocorrepelapadronizaçãodosprocessosdesucesso.17

Vamosvercomoissojáfoicobrado?

1.(Cespe/Correios/Analista/2011)OPDCAéumaferramentadequalidadequerepresentaum

conjuntodeaçõesdemelhoriacontínuanosprocessosorganizacionais.

Correto.EstaéumadeniçãocorretadoquerepresentaoCicloDeming,ou

PDCA.Ogabaritoéquestãocorreta.

13.2.3.Deming

W.EdwardsDemingfoiumdosintrodutoresdomovimentodaqualidadeno

Japão.EssetécnicoamericanofoiconvidadoparavisitaropaísapósaSegunda

GuerraMundialparaajudá-losnareconstruçãopós-guerra.18Opaísestava

totalmentedestruído,esuasempresastinhamdiversasdiculdadesparacompetir

comosprodutoresestrangeiros.

Dessaforma,necessitavamaproveitarseusrecursosdamelhormaneirapossível

eganharcompetitividade.Ocaminhofoiinvestiremqualidade.

ParaDeming,aqualidadedevetersempreoclientecomofocoprincipal.

Portanto,comoosdesejoseasnecessidadesdosclientesvãosealterandocomo

tempo,tambémdevemseralteradososparâmetrosdequalidade.

Dessaforma,asideiasdeDemingforammuitoimportantesparaaevoluçãodo

controledequalidade.Deacordocomele,existem14princípios:19

1.Criarumaconstânciadepropósitodeaperfeiçoamentodoprodutoedo

serviço,amdetorná-loscompetitivos,perpetuá-losnomercadoegerar

empregos.

2.Adotaranovalosoa.Vivemosnumanovaeraeconômica.Aadministração

ocidentaldevedespertarparaodesao,conscientizar-sedesuas

responsabilidadeseassumiraliderançaemdireçãoàtransformação.

3.Acabarcomadependênciadeinspeçãoparaaobtençãodaqualidade.

Eliminaranecessidadedainspeçãoemmassa,priorizandoainternalizaçãoda

qualidadedoproduto.

4.Acabarcomapráticadenegóciocompensadorbaseadoapenasnopreço.Em

vezdisso,minimizarocustototal.Insistirnaideiadeumúnicofornecedorpara

cadaitem,desenvolvendorelacionamentosduradouros,calcadosnaqualidadee

naconfiança.

5.Aperfeiçoarconstanteecontinuamentetodooprocessodeplanejamento,

produçãoeserviço,comoobjetivodeaumentaraqualidadeeaprodutividade

e,consequentemente,reduziroscustos.

6.Fornecertreinamentonolocaldetrabalho.

7.Adotareestabelecerliderança.Oobjetivodaliderançaéajudaraspessoasa

realizarumtrabalhomelhor.Assimcomoaliderançadostrabalhadores,a

liderançaempresarialnecessitadeumacompletareformulação.

8.Eliminaromedo.

9.Quebrarbarreirasentreosdepartamentos.Oscolaboradoresdossetoresde

pesquisa,projetos,vendas,comprasouproduçãodevemtrabalharemequipe,

tornando-secapazesdeanteciparproblemasquepossamsurgirdurantea

produçãoouduranteautilizaçãodosprodutosouserviços.

10.Eliminarslogans,exortaçõesemetasdirigidasaosempregados.

11.Eliminarpadrõesarticiais(cotasnuméricas)paraochãodefábrica,a

administraçãoporobjetivos(APO)eaadministraçãoatravésdenúmerose

metasnuméricas.

12.Removerbarreirasquedespojemaspessoasdeorgulhonotrabalho.A

atençãodossupervisoresdevevoltar-separaaqualidadeenãoparanúmeros.

Removerasbarreirasqueusurpamdoscolaboradoresdasáreasadministrativas

edeplanejamento/engenhariaojustodireitodeorgulhar-sedoprodutodeseu

trabalho.Issosignicaaaboliçãodasavaliaçõesdedesempenhooudeméritoe

daadministraçãoporobjetivosoupornúmeros.

13.Estabelecerumprogramarigorosodeeducaçãoeautoaperfeiçoamento

paratodoopessoal.

14.Colocartodosdaempresaparatrabalhardemodoarealizaratransformação.

Atransformaçãoétarefadetodos.

AgoraquejávimososprincípiosdeDeming,vamosvercomoissotemsido

cobrado?

2.(Cespe/TJ-ES–Analista/2011)Determinadotribunalpretende,pormeiodaadoçãodepráticasde

qualidadeeprodutividade,aperfeiçoaramaneiracomoserealizamasatividadesemsetor

ondeseobservouqueosservidoresgastammuitotempoemtarefasmenosrelevantesparao

públicoeondesãocometidoserrosrecorrentesrelativosàtramitaçãodeprocessosjudiciais.O

diretor,então,adotouapráticadexarumasériedecartazesquevisamestimularos

servidoresatomarmaiscuidadonoqueserefereàexpediçãoeàrecepçãodedocumentose

que,alémdisso,contêmmetasdiáriasreferentesaoencaminhamentodeprocessospara

diversossetores.Ogerentederecursoshumanosdotribunal,porsuavez,defendeaideiade

queosservidoresdevemsertreinadoscontinuamenteparautilizarem,deformaadequada,as

novastecnologiasempregadasnosetor.

Combasenessasituaçãohipotética,julgueositensseguintes.

Aoproporotreinamentocontínuodosservidores,ogerentederecursoshumanoscontrariaos

princípiosdeDeming.

Negativo.OtreinamentocontínuoéumdosprincípiosdeDeming.Ogabaritoé

questãoerrada.

13.2.4.Juran

JosephJuranfoiumimigranteromenoradicadonosEstadosUnidos.Foi,junto

comDeming,umdosimpulsionadoresdarevoluçãodaqualidadenoJapãono

períodoposterioràSegundaGuerraMundial.

SuaobramaisconhecidafoiolivroQualityControlHandbookquefoipublicado

em1951.Consultoreprofessor,elefoiocriadordoInstitutoJuran.

Entreosprincipaisimpactosdeseutrabalho,podemoscitaraconhecida

“trilogiadaqualidade”eamudançadoenfoquedagestãodaqualidade--saindo

doplanooperacionalparaoplanoestratégico.

Atrilogiadaqualidadeteriaosseguintesprincípios:

Figura13.3–TrilogiadaQualidadedeJuran

NoPlanejamentoteríamosdeidenticarasnecessidadesdosclientes;projetar

produtosadequadosaessesclienteseplanejarprocessosadequadosaessesmesmos

produtos.

JánoControledaQualidadebuscaríamosavaliarodesempenhorealda

qualidadenaorganização;compararessedesempenhocomasmetasepropor

medidascorretivas,quandonecessário.

Finalmente,noAperfeiçoamentodeterminaríamosoqueénecessáriopara

melhorarcontinuamenteaqualidade;denirosprojetosdemelhoriaedetalhar

osseusresponsáveis,alémdetreinar,motivareapoiarasequipes.

13.2.5.Feigenbaum

ArmandFeigenbaumcouconhecidoporseutrabalhonaempresaGeneral

Electric(GE).SuaprincipalobrafoiolivroTotalQualityControlouControleda

QualidadeTotal.

Entreseusprincipaisimpactosnagestãodaqualidade,buscouevidenciarquea

qualidadedeveserumesforçosistêmico.Sendoassim,nãoadiantariasomenteo

setorde“produção”sepreocuparcomaqualidade.Todosdevemseengajarna

buscapelaqualidade.

Alémdisso,elediziaqueaparticipaçãoeoconhecimentodosistemade

qualidadeporpartedosgestoresdaorganizaçãoseriamfundamentais.

Semconhecerosistema,essesgestoresnãopoderiamcomunicaraos

subordinadosaimportânciadeseguireimplementarosprincípiosbásicos.

Outroimpactodelefoioestudodoscustosdaqualidade.Deacordocomele,

existemdiversoscustosenvolvidosnagarantiaenafaltadequalidadedentrode

umaorganização.Existiriamosprincipaiscustos:

ØCustosdaPrevenção;

ØCustosdaAvaliação;

ØCustosdasfalhasinternas;

ØCustosdasfalhasexternas.

13.2.6.Crosby

PhilipCrosbyfoiumescritoramericanoqueintroduziuaideiado“defeito

zero”,ouseja,anoçãodeintolerânciacom“padrõesdequalidade”.

Paraele,aqualidadedeveriaserfocadanasnecessidadesenasespecicações

desejadaspelosclientes.Qualidadeseria,portanto,atenderaconformidadecom

certosrequisitosnecessáriosedesejadospelosclientes.

Outroconceitoassociadoaoseunomeéodese“fazercertodesdeaprimeira

vez”.Assim,todososfuncionáriosdeveriampreocupar-seemgarantirqueseus

processoseatividadesfossemsemprefeitoscomqualidade.

13.2.7.Ishikawa

KaoruIshikawafoiumdosgrandestradutoresdasteoriasamericanasparaa

realidadejaponesa.Ficouconhecidopeladifusãodoscírculosdaqualidade

(CQC)epelodiagramadecausaeefeito–quelevouseunome.

Essescírculosdecontroledaqualidade–CQCs–sãopequenosgruposde

empregadosqueconduzemocontroledaqualidade.Decertomodo,podemos

dizerqueisso“democratizou”agestãodaqualidadenasempresas.

13.3.Ferramentasdegestãodaqualidade

Asferramentasdaqualidadesãoinstrumentosquepossibilitamaogestor

monitoraremelhorarosprocessosdetrabalho.Cadaumdelestemumafunção

especíca.Entretanto,podemserutilizadosemconjunto.Asferramentassãoas

seguintes:

Ferramentas

PrincipalFunção

DiagramadeCausaeEfeito

Levantarpossíveiscausasparaproblemas

FolhadeVerificação

Coletardadosrelativosànãoconformidadedeumproduto

Histograma

Identificarcomquefrequênciacertodadoapareceemumconjuntodedados

GráficodePareto

Distinguir,entreosfatores,osessenciaiseossecundários

DiagramadeCorrelação

Estabelecercorrelaçãoentreduasvariáveis

Fluxograma

Descreverprocessos

GráficodeControle

Analisaravariabilidadedosprocessos

Figura13.4–FerramentasdaQualidade.

(AdaptadodeMelo,2012)

13.4.DiagramadeCausaeEfeitooudeIshikawa

OdiagramadeIshikawa,quetambéméconhecidocomoespinhadepeixeou

diagramadecausaeefeito,éumaferramentaquenospossibilita“entender”

melhorumprocessoouumproblema.

Nodiagrama,pode-seestruturarehierarquizarasprincipaiscausasquepodem

estargerandoumdeterminadoefeitoquequeremosestudar.Ouseja,analisamos

osefeitosequaisseriamasprincipaiscausasdele.

Vamosimaginarumcasoconcreto?Caiuumaviãoemumaeroportoevocêfoi

escolhidoparainvestigaroacidente.ComodiagramadeIshikawa,comovemosa

seguir,teríamosdedenirqualfoioefeito.Oefeitonessecasofoi:oaviãoteve

umacidente.

Apósdenirmosoqueestudaremos(oefeitoesuaspotenciaiscausas),temosde

“mapear”osprincipaisfatoresquepodemterlevadoaessasituação.Nocaso,

escolhiosfatores:ainfraestrutura,osequipamentos,opessoaleocontroledevoo.

Cadafatorpodeconterdiversosaspectosdiferentes,queserãoanexadosabaixo

dosfatoresprincipais(ex.:Pistadefeituosaestá“inserida”dentrodofator

infraestrutura).Dessaforma,cadafatorprincipalpodeterdiversosfatores

“secundários”.

Nocasoquedescrevi,dentrodeumfator(opessoal,porexemplo)podem

existirdiversasdeciências.Nocasodofatorpessoalpoderiamser:afaltade

motivação,otreinamentoinadequado,umperlinadequadodaspessoas,afalta

depessoal,ocansaçoetc.

Portanto,quandomontamosumdiagramadesses,temoscondiçãodeestruturar

ascausasprováveisdeumproblema.Vejamográcoaseguirepercebamporque

éapelidadode“espinhadepeixe”:

Figura13.5–GráficodeIshikawa

Vamosvercomoissojáfoicobrado?

3.(Cespe/Embasa/Administração/2010)Odiagramaespinhadepeixe,oudiagramadeIshikawa,é

umaferramentadaescoladaqualidadequemapeiaacorrelaçãoentrecausaseefeitosdeum

processo.

Exato.Nessaferramentadegestãodaqualidade,podemosestruturare

hierarquizarasprincipaiscausasquepodemestargerandoumdeterminadoefeito

quequeremosestudar.Esseefeitopodeser,porexemplo,osdefeitosnoprocesso

deprodução.Ogabaritoé,assim,questãocorreta.

13.5.FolhadeVerificação

Umafolhadevericaçãoéuminstrumentodecoletadedadosreferentesauma

situaçãoespecíca.Quandoqueremosentendermelhorumprocesso,precisamos

sabermaissobreele.

Normalmente,afolhadevericaçãoéopontodepartidadagestãoda

qualidade,poisosdadospodemserutilizadospelasoutrasferramentas.

Suaaplicaçãoémuitosimples.Vamosverumcasoprático?Imaginequevocê

precisaanalisaroprocessodefabricaçãodecalçadosemumafábrica.Parapoder

saberquaissãoastarefas,materiais,pessoasouqualqueroutroaspectoqueestão

trazendoproblemasnaqualidade,devemosprimeirocoletarosdadosdedefeitos

noprocesso.

Aseguir,umexemplodessesdados:

Processo:

FabricaçãodeCalçados

Lotedeprodução:

3600pares

Datadeverificação:

12/08/2012

Frequência:

Semanal

Problema

Quantidade

Costuramalfeita

2

Soladomalcolado

3

Corborrada

4

Defeitonapalmilha

1

Linguetadisforme

8

Figura13.6–Exemplodefolhadeverificação

Sabendoquaissãoosprincipaisdefeitos,porexemplo,jápoderíamosbuscaras

causasdoproblema.Portanto,afolhaéuminstrumentoutilizadoemquasetodos

osprocessos.

13.6.Histograma

Ohistogramaéumgrácodebarrasqueauxilianaanálisedafrequênciados

dados.Qualquerpessoaquejátenhafeitoumgrácoemumaplanilhaeletrônica

jádevetervistoumgráficodebarrasdestes:

Figura13.7–Exemplodehistograma

Aanálisedotempodeesperadeclientesemumserviçodeatendimentoca

mais“intuitiva”emumgrácodoqueemumalistaderesultados(veraseguir),

nãoéverdade?

Seissojáocorreemumalistasimplescomoessadonossoexemplo,imagineem

umalistagrandededados.

TempodeEsperanoSAC/Minutos

Quantidade

0-5minutos

230

5-15minutos

582

15-25minutos

679

25-35minutos

234

35-45minutos

124

Figura13.8–Listadedados

Comisso,ohistogramaserveparafacilitaraanálisedafrequênciadosdadosem

umprocesso.Ouseja,nosauxilianaidenticaçãodafrequênciacomquecerto

dadoapareceemumconjuntodedados.

13.7.GráficodePareto

OprincípiodeParetoéumadastécnicasmaisutilizadasnogerenciamento.É

umconceitoantigo20(baseadonostrabalhosdeVilfredoPareto,noséculoXIX),

masmuitoutilizadoatéhoje.Muitostambémoconhecemcomoaregrado

“80/20”.

Eleémuitobomparapodermosidenticarquaissãoositensmaisimportantes

emumasituação,equaissãoosaspectospoucoimportantes.

Mascomofunciona?

Imaginequevocêtemumalojaderoupas.Vocêprecisasaberquaissãoos

modelosquemaisvendemegerammaisretornoparasuaempresa.Dessaforma,

vocêsempreestaráde“olho”nessesprodutos,ouseja,buscaránãodeixarque

faltemnoestoque!

Assimsendo,vocêfaráumaanálisedequantocadaitemvendeunosúltimosseis

meses,econstruiráumgráficoquepoderáficarcomoeste:

Figura13.9–GráficodePareto(80/20)

OqueParetodescobriuéquenormalmente20%dositensgeram80%dos

resultados(portanto,sãomuitoimportantes)eosoutros80%dositensgeram

apenas20%dosresultados–porissoonomederegrados“80/20”!

Essaregranãoserveapenasparaositensdeumaloja,mascostumaserepetirem

outrosfatores(20%dasdoençasmatam80%dospacientesemumhospital,20%

dosvendedoresgeram80%dasvendasetc.).Portanto,aferramentanospossibilita

“focar”nosaspectosquemaisimpactamnossonegócio.

Vamosvercomoissotemsidocobrado?

4.(Cespe/Basa/Administração/2010)OdiagramadeParetopodeseradotadoquandoseestádiante

deproblemasebusca-seidentificarquaissãoositensresponsáveispelamaiorparceladeles.

Certo.OdiagramadeParetobuscaexatamenteapontarquaissãoosprincipais

fatoresemumdeterminadoproblemaouprocesso.Éumaferramentaquenos

proporcionaumanoçãodequaissãoosaspectosquedevemseranalisadoscom

maiorcuidado,poisgeramamaiorpartedosimpactos.Assim,ogabaritoé

questãocorreta.

13.8.DiagramadeCorrelaçãoouDispersão

Odiagramadedispersãopossibilitaanalisaracorrelaçãoentreduasvariáveis.

Ouseja,podeajudaraovermoscomoumamudançaocorridaemumfatorpode

afetaroutrofator.Portanto,podemosveratravésdessegrácoaalteraçãoemuma

variávelquandooutrasemodifica.

Vamosimaginarumcasoconcreto?OMinistériodaSaúdepoderiaquerer

avaliarseosinvestimentosemsaúdeefetuadospelasdiversascidadesestãoounão

elevandoaestimativadevidadapopulação.Ouseja,seráqueoinvestimento

maioremsaúdegerariaumaexpectativamaiordevidanapopulaçãoda

comunidade?

Dessamaneira,ográcoaseguirmostrariaqueexiste,sim,umacorrelação,ou

seja,queascidadesqueinvestirammaisemsaúdepassaramaterumaestimativa

maior.Jáascidadesqueinvestirammenosestavamcomumamenorexpectativade

vida.

Figura13.10–Gráficodedispersão

Vamosvercomoissojáfoicobrado?

5.(Cespe/Inca/GestãoPública/2010)Odiagramadecausaeefeito,tambémchamadodediagrama

deIshikawa,busca,apartirdeumaanálisegráca,mostrarasrelaçõesentredoisconjuntosde

dadosassociadosqueocorremaospares.

Negativo.Aferramentadegestãodaqualidadequebuscamostrararelação

entreduasvariáveiséodiagramadedispersãooudecorrelação.Ogabaritoé

questãoerrada.

13.9.Fluxograma

Ouxogramaéaprincipalferramentademapeamentoedesenhodeprocessos.

Aferramentaconsisteemumconjuntodenotaçõesgrácas,ouseja,umconjunto

desímbolospadronizadosqueservemparaquepossamosdescrevereredesenhar

umprocesso.

Assim,essaéaferramentautilizadaparaefetuaromapeamentoeamodelagem

dosprocessos.Dessaforma,eleéutilizadoparadescrever,demodográco,um

processoatravésdousodesímboloselinhas.Comelecamaisfácilvisualizare

conhecermelhorumprocesso,deformaquepossamospropormelhoriase

mudanças.Oobjetivo,portanto,dautilizaçãodessaferramentaésimplicaro

trabalhodemapeamentoefacilitaroentendimentodecomoosprocessosde

trabalhofuncionam.

Semconhecercomoosprocessosfuncionam,nãoteríamoscomogerenciá-los,

nãoémesmo?Aseguirpodemosverumexemplodefluxograma:

Figura13.11–Exemplodefluxograma

13.10.GráficodeControle

Osgrácosdecontroleservemparamedirmosavariabilidadedeumprocesso.

Atravésdadeterminaçãodelimitesmínimosemáximosde“tolerância”,podemos

analisarocomportamentodeumprocessoespecífico.

Nocasoaseguir,teríamosonúmerodedefeitosemumprocessoYemcada

mês.Semprequeoprocessomostrarumcomportamentoatípico,comonosmeses

defevereiroesetembro,porexemplo,devemosanalisarofuncionamentodo

processocommaisrigor.

Figura13.12–Gráficodecontrole

Portanto,essegráconosmostraseexistealgumfatorinuenciandodemodo

especialaqualidade.Quandoosvaloresestiveremdentrodafaixaentreolimite

inferioreolimitesuperior,poderíamosdizerqueoprocessoestá“sobcontrole”.

13.11.Programa5S

Oprograma5Sébaseadofortementenaculturajaponesaefocadonaordeme

nalimpezanoambientedetrabalho.Paraosjaponeses,umambientelimpoe

organizadoajudanagestãoenapromoçãodaqualidade.

Oprogramaéaparentementesimples,masdemandaumagrandemudançade

mentalidadenosmembrosdaorganização.21Seunomeébaseadonasiniciaisdas

palavrasseiri,seiso,seiton,seiketsueshitsuke.

Aprimeirapalavra,seiri,signicaorganização.Masnãosomentenosentido

popularqueconhecemos–deorganizaroqueestá“bagunçado”.Oconceitovai

maisalém,dedarumdestinoaoquenãonostrazmaisutilidade,depriorizaro

queémaisimportante,desómanteremestoqueoquerealmenteseránecessário.

Jáapalavraseitonnormalmenteétraduzidacomoarrumação.Nessepasso,

devemosmanterascoisasnosseusdevidoslugares,paraquesempresaibamos

ondepodemosencontrarcadapapelouferramenta.

Nocasodotermoseiso,opassosignicaalimpezadoambientedetrabalho.

Ninguémgostadetrabalharemumlugarsujoemalcuidado,nãoémesmo?Os

japonesesacreditamqueumlocallimpoeasseadogeraummelhor

relacionamentodaspessoascomseuambientedetrabalhoeimpacta

positivamentenaqualidadeenaprodutividade.

Porisso,devemosmanterasferramentasemáquinaslimpas,alémdeproduzir

menosresíduosedetritosnotrabalho,paraquenãosejanecessáriotantotrabalho

paralimparoambiente.

Opróximotermo,seiketsu,estátambémrelacionadocomosanteriorese

signicahigiene.Aquinãofalamossomentedahigienepessoaledalimpezado

ambiente,mastambémdaeliminaçãodetudoquepossasignicarriscoparaa

saúde.22

Assim,envolveumaboasinalizaçãodosriscosdoambiente,reduçãodos

diversosruídosnoambiente,adevidautilizaçãodosaparelhosdesegurança,entre

outrosfatores.

Finalmente,otermoshitsukesignicadisciplinaparamanteraexecuçãodos

passosanterioresemanteroambientesaudável.Denadaadiantafazeressespassos

duranteumasemanaedepois“relaxar”evoltarafazerascoisascomo

antigamente.

Semdisciplina,nenhumprogramadequalidadepodefuncionar.Dessemodo,o

programa5S,apesardesuasimplicidade,demandaumgrandeesforçoparamudar

aculturaorganizacionalemproldeumambientedetrabalhomaislimpo,seguro

eagradávelparatodos–oquedeveimpactarpositivamentenasaúdee

produtividadedostrabalhadores.

Aseguirpodemosveroscincopassosdoprograma5S:

Figura13.13–Programa5S

13.12.SeisSigma

OprogramaSeisSigma(SixSigma)foidesenvolvidopelaempresaMotorolanos

anos1980.Apósessaempresa,muitasoutrasadotaramoprograma,pelasua

capacidadedereduzircustos,aumentaraqualidadeeatingirresultados

significativos.

Basicamente,oobjetivodiretodoprogramaéareduçãodavariabilidadedos

processosdetrabalho,demodoqueestesatinjamumareduçãosignicativados

seusdefeitos.

AexpressãoSeisSigmaindicaumobjetivoprincipaldoprograma,queéatingir

umníveldedefeitosde3,4peçaspormilhão,ouseja,quasedefeitozero.23

Quantomaioronúmerode“sigmas”,menoronúmerodeproblemasnoprocesso.

Oprogramabuscagerarumprocessocomumpercentualde99,9997%de

sucesso.24

AimplantaçãodoSeisSigmaenvolveumasériedeprojetosdemelhoriaque

utilizaumatécnicachamadaDMAIC(quederivadaspalavrasinglesas:define,

measure,analyze,improveecontrol).Essaéumatécnicamuitosemelhanteao

PDCA,quejávimos.DeacordocomCarvalhoePaladini,25

DiversasferramentassãoutilizadasdemaneiraintegradaàsfasesdoDMAIC,constituindoum

métodosistemático,disciplinado,baseadoemdados,enousodeferramentasestatísticasparase

atingirosresultadosalmejadospelaorganização.

Naturalmente,asfasesdoDMAICpodemdemandarferramentasestatísticase

deanálisediferentes,deacordocomasituaçãoeespecicidadesdosprocessosde

trabalhoaseremmelhorados.Basicamente,todasasferramentasdagestãoda

qualidadepodemserutilizadasnoDMAIC,comoodiagramadeIshikawa,o

diagramadePareto,ográficodecontrole,entreoutros.

AseguirvocêspodemverosprincipaispassosdoDMAIC:

Figura13.14–DMAIC

(Baseadoem:Melo2011)

Asequipesdetrabalhoqueserãoresponsáveispeloprogramasãoformadaspor

diversosníveis“hierárquicos”deacordocomaexperiênciaeosconhecimentosdo

programa.Osprincipaisníveissão:26Sponsors,Champions,MasterBlackBelts,Black

Belts,GreenBeltseWhiteBelts.

Figura13.15–NíveisdoSixSigma

Essahierarquiaderesponsabilidadesoudepapéisdeneaparticipaçãodecada

membronoprograma,masmesmoosquenãoestãodiretamenteenvolvidosno

SeisSigmadevemparticipardabuscapelaqualidade,poisestadeveenvolver

todososmembrosdaorganização.

13.13.Kaizen

Oconceitodekaizenestámuitoligadoaoconceitodequalidade.Kaizenem

japonêsseriatraduzidocomo“melhoramento”ou“mudançaparamelhor”.Éuma

buscadoaprimoramentocontínuo,incremental,detodososintegrantesde

umaorganização.Esseaprimoramentonãoselimitaaoâmbitodaprodução,mas

constituiumaformadevidaecomportamento,dentroeforadaorganização.27

Osjaponesespensamaqualidadecomoumtrabalhodetodos,portantoo

controledaqualidadeédescentralizado,eimpactanãosóaqualidadedos

produtosmasoseucusto.Dessaforma,tantotrabalhadoresenvolvidosnalinhade

produçãoquantotrabalhadoresqueatuamemáreasadministrativaseoschefes

devemestarenvolvidosnoprocesso.

Assim,otrabalhodeumoperárioemumaprensaseriatãoimportantequantoo

deumserventequelimpaochãoouodeumcontadorqueorganizaasnançasda

empresa.Tudodeveserfeitocomtotalzeloequalidade.Portanto,todososníveis

hierárquicosdevemestarenvolvidos.

Assimsendo,umdosaspectosimportantesquedevemosobservaréoaspecto

humanodokaizen.Nassociedadesocidentais,aqualidadeémuitasvezesvista

comorelacionadaàstécnicasemáquinasenvolvidasnaproduçãodeumproduto

ouserviço.

Entretanto,paraosjaponesesaqualidadeéfazeralgomelhortodososdias,ou

seja,acadadiatodososempregadosdevembuscarfazeralgomelhor.Issoabrange

desdeastécnicasdeproduçãoàmaneiradesefazerumaatividade.

DeacordocomLima,okaizenéumadiretrizculturalquepermeiatodosos

métodosdeproduçãoorientais:28

Okaizené,portanto,umadiretrizcultural,umvalorquedeterminaoesforçodeaprimoramento

contínuo.Oquenosremeteàbuscadaperfeição,nuncaatingidamassempredesejada.

Éumprocessoconstante,efetuadoportodosnaempresa,quebuscaaumentara

produtividade,diminuirodesperdício,ostressereduzirosacidentesdetrabalho.

Paraosjaponeses,apalavrakaizenéumconceito“guarda-chuva”,29poisengloba

diversosconceitosdequalidadeconhecidos,como:controletotaldaqualidade,

kanban,defeitozeroetc.

Vamosvercomoissojáfoicobrado?

6.(FCC/TCE/SP/Auditor/2008)Oconceitojaponêskaizenéabaselosócaparadiversastécnicas

atuaisdequalidadenaadministraçãocontemporânea.Sintetizacorretamenteosprincípiosdo

kaizen:

a)preocupaçãocontínuacomaelaboraçãodepadrõesdeexcelênciaparamedirprodutos,serviçosou

processoscomrelaçãoaosconcorrentesmaisfortes;

b)preocupaçãoconstantecomaatribuiçãodemaispodereautonomiaaostrabalhadores,visando

partilharresponsabilidadescomrelaçãoàprodutividadedaempresa;

c)abordagemsistêmicadoprocessodeaprendizagemvoltadaaoaperfeiçoamentocontínuoda

inovaçãodentrodaempresa;

d)preocupaçãocontínuacomoaperfeiçoamentotantodeprodutoseserviçoscomodos

procedimentosehábitosdeexecutivosetrabalhadoresnaempresa;

e)foconocliente,suasnecessidadesepreferências,eapreocupaçãosistemáticacomareduçãodos

desperdícioseaagregaçãodevaloraosprodutoseserviços.

Aprimeirafraseestáincorreta,poisapreocupação,nokaizen,nãoécom

padrõesdeexcelência,masemfazersempreomelhorpossível.Quandotemos

padrõesaceitáveispodemosnosacomodaracumprirsomentea“meta”,nãoé

verdade?

Asegundafrasetambémestáincorreta,poisaatribuiçãodemaispoderaos

trabalhadoreseresponsabilidadenãosintetizacorretamenteoconceitodokaizen,

queémuitomaisabrangente.

Aterceirafrasetambémestáincorreta,poisokaizenémaisabrangentedoqueo

processodeaprendizagemcontínuodainovação.JáaletraDestácorreta,poisé

umadeniçãocorretadokaizen.Dessaforma,aletraEestáincorreta,poisnão

abrangeoaspectohumano,alosoadevidaqueéokaizen.Nossogabaritoé

mesmoaletraD.

13.14.ModelodaFundaçãoNacionaldaQualidade

AFundaçãoNacionaldaQualidadeéumainstituiçãovoltadaparaa

disseminaçãodoconhecimentosobreaExcelênciaemGestãoparaas

organizações.Alémdisso,elainstituiuoPrêmioNacionaldaQualidade(PNQ),

quepremiaasmelhorespráticasdegestãodasorganizaçõesbrasileiras.

EssainstituiçãocriouoModelodeExcelênciadaGestão–MEG.30Essemodelo

estábaseadoemumconjuntodeprincípioseelencadiversoscritérioserequisitos

necessáriosaexcelêncianagestão.Alémdisso,oMEGutilizaumaferramentade

aprendizadoemelhoriacontínua,ocicloPDCL(Plan,Do,Check,Learn).

DeacordocomaFNQ,osconceitosprincipais(oufundamentos)são:31

1.PensamentoSistêmico–Entendimentodasrelaçõesdeinterdependênciaentre

osdiversoscomponentesdeumaorganização,bemcomoentreaorganizaçãoe

oambienteexterno.

2.AprendizadoOrganizacional–Buscaealcancedeumnovopatamarde

conhecimentoparaaorganizaçãopormeiodapercepção,reexão,avaliaçãoe

compartilhamentodeexperiências.

3.CulturadeInovação–Promoçãodeumambientefavorávelàcriatividade,à

experimentaçãoeàimplementaçãodenovasideiasquepossamgerarum

diferencialcompetitivoparaaorganização.

4.LiderançaeConstânciadePropósitos–Atuaçãodeformaaberta,

democrática,inspiradoraemotivadoradaspessoas,visandoodesenvolvimento

daculturadaexcelência,apromoçãoderelaçõesdequalidadeeaproteçãodos

interessesdaspartesinteressadas.

5.OrientaçãoporProcessoseInformações–Compreensãoesegmentaçãodo

conjuntodasatividadeseprocessosdaorganizaçãoqueagreguemvalorparaas

partesinteressadas,sendoqueatomadadedecisõeseexecuçãodeaçõesdeve

tercomobaseamediçãoeanálisedodesempenho,levando-seemconsideração

asinformaçõesdisponíveis,alémdeincluirosriscosidentificados.

6.VisãodeFuturo–Compreensãodosfatoresqueafetamaorganização,seu

ecossistemaeoambienteexternonocurtoenolongoprazo.

7.GeraçãodeValor–Alcancederesultadosconsistentespeloaumentodevalor

tangíveleintangíveldeformasustentadaparatodasaspartesinteressadas.

8.ValorizaçãodasPessoas–Criaçãodecondiçõesparaqueaspessoasserealizem

prossionalehumanamente,maximizandoseudesempenhopormeiodo

comprometimento,dodesenvolvimentodecompetênciasedeespaçospara

empreender.

9.ConhecimentosobreoClienteeoMercado–Conhecimentoeentendimento

doclienteedomercado,visandoacriaçãodevalordeformasustentadaparao

clientee,consequentemente,gerandomaiorcompetitividadenosmercados.

10.DesenvolvimentodeParcerias–Desenvolvimentodeatividadesemconjunto

comoutrasorganizações,apartirdaplenautilizaçãodascompetências

essenciaisdecadauma,objetivandobenefíciosparaambasaspartes.

11.ResponsabilidadeSocial–Atuaçãoquesedenepelarelaçãoéticae

transparentedaorganizaçãocomtodosospúblicoscomosquaiselase

relaciona.Refere-setambémàinserçãodaempresanodesenvolvimento

sustentáveldasociedade,preservandorecursosambientaiseculturaispara

geraçõesfuturas,respeitandoadiversidadeepromovendoareduçãodas

desigualdadessociaiscomoparteintegrantedaestratégiadaorganização.

OmodelodaFNQapresentaoscritériosmaisimportantesdomodelocomoum

sistemaglobal.Odiagramaaseguirdescreveoscritériosdomodelo:aliderança,

asestratégiaseosplanos,osclientes,asociedade,asinformaçõeseo

conhecimento,aspessoas,osprocessoseosresultados.

Figura13.16–ModelodeGestãodaFundaçãoNacionaldaQualidade.

(Fonte:<http://www.fnq.org.br/site/376/default.aspx>)

Naturalmente,nenhumaorganizaçãoconseguirásucessosenãoatenderos

desejosenecessidadesdeseusclientes.Alémdisso,deverápercebereatenderas

necessidadesedemandasdasociedadecomoumtodo,tendoumcomportamento

éticoebuscandoumdesempenhosustentável.

Paraqueissoocorra,énecessáriaumaliderançaforteeadequadaaosobjetivos

daorganização.Essaliderançaformulaeacompanhaasestratégiasque

possibilitemàorganizaçãoumamelhoriadesuasvantagensestratégicas.

OmodeloaplicatambémaferramentaPDCA,oucicloDeming.Atravésdela,

temosasfases:Planejamento,Execução,ControleAvaliaçãoeAção.Estas

englobamumagestãoeficazdaspessoasedosprocessosdetrabalho.

Alémdisso,devemincluirocontroledosresultadosobservadospela

organização.Essesresultadosdevemsermonitoradosatravésdeindicadoresque

possibilitemaanálisedasdiversasperspectivasnecessáriasaomodelo.

Assim,oprocessodecontroledosresultadosgerarádados,informaçõese

conhecimentoquepossibilitarãoacorreçãodosdesviosnegativoseuma

aprendizagemorganizacionalefetiva.

13.15.QualidadeeProdutividade

Comovimosantes,aqualidadehojeemdiaéfundamentalparaosucessode

qualquerempresa.Masoutropontoimportanteéaprodutividade!Nãoadianta

termosqualidadesemprodutividade,poisnossosprodutoscariampouco

competitivosnomercado.Dessaforma,qualidadeeprodutividadesãoconceitos

entrelaçados.

Masafinal,oqueéprodutividade?

Aprodutividadeserelacionacomamelhorutilizaçãopossíveldosrecursosna

produçãodebenseserviços,ouseja,narazãoentreassaídaseasentradasdeum

processo.

Dessaforma,imaginequeduasempresasdiferentestêmomesmonúmerode

empregadosedispõemdamesmaquantidadedemateriaisdisponíveis.Entretanto,

umaproduzmaisprodutosdoqueaoutranofinaldeumperíodo.

Dessamaneira,podemosdizerqueessaempresatemmaiorprodutividadedo

quesuaconcorrente.Dessemodo,aempresamaisprodutivateriamaior

competitividade,poisteriacustosunitários(porproduto)maisbaixos.Essa

empresa,comosmesmosrecursos,produziumais!

Antigamenteapreocupaçãodosgestoreserasomentecomaprodutividade,mas

hojedevemostertantoqualidadecomoprodutividade.Umprodutodebaixa

qualidadeterádiculdadedeseraceitopelomercado,nãoémesmo?Damesma

forma,umprodutodeexcelentequalidadeextremamentecaroterápoucos

clientes.

Vamosvercomoissojáfoicobradoemprovas?

7.(FCC/Defensoria/SPAdministrador/2010)Comrelaçãoàqualidadeeàprodutividadenas

organizações,considereasafirmativasaseguir.

I.Aprodutividadesóéobtidaseosprocessostiveremqualidadeemseusinsumoseoperações,

poisdenadaadiantaqualidadesemprodutividade.

II.AQualidadeTotalémuitoabrangenteesededicaaestudarasatisfaçãodosclientes

externoseosclientesinternos.

III.Aprodutividadenãodeveserentendidasomentecomorazãoentreresultadoserecursos,

massimdeumaformamaisampla,levandoemcontatodasasvariáveisexistentesnos

processos,focandonasnecessidadesdetodasaspartesinteressadasnonegócioebuscando

melhoriaseresultadosrelevantesparatodos.

IV.Noiníciodaeraindustrial,apreocupaçãodosgestoresestavavoltadaparaosvolumesde

produção.Comoaumentodacompetitividadesurgiuamediçãodaprodutividade,queatéa

SegundaGuerraMundialeraentendidacomorazãoentreovolumeproduzidoeotempo

gastoparaestaprodução.

V.Umadasformasmaisabrangentesdesedenirprodutividade,atualmente,écomosendo

razãoentreasaída,ouoresultadonal,deumprocessoeaentrada,querepresentaos

recursosnecessáriosàobtençãodasaída.

Estácorretooqueseafirmaem:

a)I,I,II,IVeV;

b)II,IVeV,apenas;

c)I,II,IV,apenas;

d)I,IeII,apenas;

e)I,I,IIeIV,apenas.

Aprimeirafraseestácorreta.Seosprocessosprodutivosnãotiveremqualidade

seráimpraticávelatingirumnívelidealdeprodutividade.Ouseja,parauma

empresaserprodutiva,deveterqualidadeemseusprocessos.

Asegundatambémestáperfeita,poisapreocupaçãonãodevesersomentecom

osclientesexternos,mastambémcomosclientesinternos(osoutrossetorese

pessoasquedependemdenossotrabalhodentrodeumaorganização).

Aterceirafrasetambémestácerta.Quandopensamosemprodutividade

devemossemprelevaremcontaasnecessidadesdosclientesedosdemais

stakeholders(todosquedecertamaneirasãoafetadospelasoperaçõesdaempresa).

Nãoadiantaproduzirmuitooquenãoénecessário,desejável,ouproduzirmos

deformaquenãosejasustentável.Aquartafrasedescrevecorretamenteaevolução

dapreocupaçãohistóricacomaprodutividade.

Aúltimafrasetambémestáperfeita,edescreveumadeniçãodeprodutividade.

Essaquestãoéinteressantepornosdarumaideiadoqueabancaconsidera

correto,nãoémesmo?OgabaritoéaletraA.

QuestõesComentadas

8.(Esaf/CGU–Analista/2006)Indiquequaldasopçõesaseguirexplicitacorretamentepremissasde

umprogramadequalidade.

a)Processodemelhoriacontinua.Satisfaçãodosclientes.Aresponsabilidadepelaqualidadecabeàárea

deprodução.

b)Fazerbemaprimeiravez.Aresponsabilidadepelaqualidadecabeàáreadeprodução.Bonsmateriais

garantemqualidade.

c)Processodemelhoriacontinua.Fazerbemaprimeiravez.Eliminaçãodedesperdício.

d)Satisfaçãodosclientes.Bonsmateriaisgarantemqualidade.Eliminaçãodedesperdício.

e)Aresponsabilidadepelaqualidadecabeàáreadeprodução.Bonsmateriaisgarantemqualidade.

Eliminaçãodedesperdício.

AletraAestáequivocada,poisaresponsabilidadepelaqualidadenãoérestrita

aosprossionaisquetrabalhamnaáreadeProdução,esimdetodosos

funcionários.AletraBrepeteesteerroeanexamaisum:bonsmateriaissão

importantes,masnãogarantemaqualidadedeumprodutoouserviço.

AletraCestáperfeitaeéonossogabarito.JáaletraDrepeteoequívocoda

letraB(materiaisnãogarantemaqualidade),ealetraEtambémestáerrada,pois

repeteoserrosdasalternativasanteriores.Ogabaritoé,portanto,aletraC.

9.(Esaf/MTEAuditor/2006)Indiqueaopçãoquecompletacorretamenteafraseaseguir:Os

programasdequalidadetêmcomofocoasnecessidadesdos.....................,buscamamaior

..........................dosprocessos,evitando........................Caracterizam-sepela.....................e

..................

a)clientesinternoseexternosdaorganização/eciência/desperdícios./participaçãodetodaaequipe/

melhoriacontínua;

b)fornecedores/efetividade/arepetiçãodetarefas./horizontalizaçãodasestruturasorganiz

acionais/

mudançasdrásticas;

c)acionistas/eficiência/desperdícios./subordinaçãoàsdecisõesdadiretoria/melhoriacontínua;

d)clientesinternosdaorganização/ecácia/ahorizontalizaçãodastarefas./mudançafundamentaldos

processos/melhoriacontínua;

e)Stakeholders/efetividade/arepetiçãodetarefas./buscadepadrõesdedesempenho/mudanças

fundamentais.

Questãobastantetranquila.Aprimeiralacunajápraticamente“mata”aquestão.

Agestãodaqualidadedeveterumenfoqueematenderaosdesejosenecessidades

dosclientes,sejamelesinternosouexternos.

Lembre-sedequeseoseutrabalhoforimportanteparaalgumórgãoda

empresa,esseórgãoseráumclienteseu(nocaso,umclienteinterno).

Asegundalacunaépreenchidacoma“eciência”.Agestãodaqualidadepreza

processosdetrabalhoquebusquemreduzirodesperdício,queevitemdefeitos.Ou

seja,comprocessoseficientes.

Orestocoufácil,nãoémesmo?Jáfalamossobreareduçãodosdesperdícios,e

agestãodaqualidadedeveserresponsabilidadedetodososfuncionáriosda

organização.Alémdisso,amelhoriacontínuadosprocessoséumdosprincípios

fundamentaisdagestãodaqualidade.OgabaritoémesmoaletraA.

10.(Cespe/MMA–Analista/2011)OPDCA,tambémchamadoderodadeDeming,éumcicloquenão

para,porsetratardeumasequênciadeatividadesquesãopercorridasdemaneiracíclica,para

melhorarasatividades.

Exato.OPDCAdeveserconstantementeaplicado.Dessamaneira,éum

processodinâmicoecontínuodemelhoriadosprocessos.Ogabaritoéquestão

correta.

QuestõesPropostas

11.(FCC/TRF/4aRegião/Anal.Adm./2010)Nagestãodaqualidade,aferramentaqueauxiliaogestor

avisualizaraalteraçãosofridaporumavariávelquandooutrasemodificaédenominada:

a)diagramadedispersão;

b)histograma;

c)diagramadecausaeefeito;

d)cartadecontrole;

e)gráficodePareto.

12.(Cespe/TJ-ES–Analista/2011)Entreosdenominados14pontosdeWilliamDeming,quetiveram

inuênciadeterminantenaescolajaponesadaqualidade,inclui-seaimplementaçãoda

administraçãoporobjetivos.

13.(Cespe/Inca/GestãoPública–2010)Odiagramadecausaeefeito,tambémchamadode

diagramadeIshikawa,busca,apartirdeumaanálisegráca,mostrarasrelaçõesentredois

conjuntosdedadosassociadosqueocorremaospares.

14.(Cespe/Embasa/Administração/2010)Odiagramaespinhadepeixe,oudiagramadeIshikawa,é

umaferramentadaescoladaqualidadequemapeiaacorrelaçãoentrecausaseefeitosdeum

processo.

15.(Cespe/STF/Técnico/2008)AtécnicadeParetoéutilizadaparaavaliaremelhoraragestãode

processos,poisajudaadetectarascausasessenciaisquerespondempelosresultadosmais

importantes.

16.(Cespe/FUB/SecretárioExecutivo/2011)OsensodearrumaçãoouSeitonrefere-seàlimpezada

áreadetrabalhoeinvestigaçãodasrotinasquegeramsujeira,objetivandomodificá-las.

17.(Cespe/FUB/SecretárioExecutivo/2011)Osensodeutilizaçãoouorganização,denominado

Seiri,

umdoscincoSdoprograma,consisteemumatécnicaparaidenticaçãoeeliminaçãode

objetoseinformaçõesdesnecessáriasexistentesnolocaldetrabalho.

18.(FGV/SAD/PE/APOG/2008)Assinaleaalternativaqueapresentecorretamenteanalidadedo

gráficodeIshikawa.

a)Fornecerumalistadeitenspormeiodaqualépossívelumacoletarápidadedadosparaanálise

quantitativa.

b)Representargraficamenteosrelacionamentosentreumefeito(problema)esuacausapotencial.

c)Capacitaracomparaçãodequantidadesdedadosrelativosacategoriasdiversas.

d)Monitorarodesempenhodeumprocesso.

e)Representarospassosdeumprocesso.

19.(FCC/Defensoria/SPAdministrador/2010)Entreasferramentasdecontroledaqualidade,aquela

quepermiteestruturarhierarquicamenteascausaspotenciaisdedeterminadoproblemaou

oportunidadedemelhoria,bemcomoseusefeitossobreaqualidadedosprodutos,éo:

a)diagramadePareto;

b)histograma;

c)diagramadecausaeefeito;

d)gráficodeDispersão;

e)fluxograma.

20.(UFF/Administrador/2009)Ométododemelhoriacontínuadaqualidadequevisaàreduçãodas

variabilidadesédenominado:

a)kaizen;

b)5S;

c)diagramadecausaeefeito;

d)reengenharia;

e)SeisSigma.

21.(Esaf/CGU–Analista/2006)OsprogramasdequalidadeseconsolidaramnoJapãovisando

basicamenteproduzirganhosdecompetitividadeparaasindústrias;noentanto,asideiase

métodosdaqualidadeseexpandiramparaoutrasrealidades.Assinaleaopçãoquenão

correspondeaoenfoquedadoporumprogramadequalidade.

a)Aimplantaçãodeumprogramadequalidadetemcomofocogarantiraqualidadedoproduto,

promovermelhoriascontínuaspormeiodeestímuloàinovação,atenderàsexpectativasdosclientes,

mantendo-ossatisfeitos.

b)Aimplantaçãodeumprogramadequalidadetemcomofocopadronizaraprodução,promover

melhoriaspormeiodeumprocessodeespecializaçãoecriarunidadesdeatendimentoaocliente.

c)Aimplantaçãodeumprogramadequalidadetemcomofocoproduzirumúnicoproduto

padronizado,promovermelhoriaspormeiodeumprocessodeespecializaçãoecrernadelizaçãodo

mercado.

d)Aimplantaçãodeumprogramadequalidadetemcomofocogarantiraqualidadedoproduto,

promovermelhoriascontínuaspormeiodadiminuiçãodedesperdícios,atenderàsexpectativasdos

clientes,mantendo-ossatisfeitos.

e)Aimplantaçãodeumprogramadequalidadetemcomofocoproduzircertonaprimeiravez,

promovermelhoriascontínuaspormeiodeestímuloàinovação,criarunidadesdeatendimentoao

cliente.

22.(Esaf/CGU/Analista/2004)OsprogramasdequalidadeseconsolidaramnoJapãovisando

basicamenteproduzirganhosdecompetitividadeparaasindústrias;noentanto,asideiase

métodosdaqualidadeseexpandiramparaoutrasrealidades.Assinaleaopçãoquenão

corrrespondeaoenfoquedadoporumprogramadequalidade.

a)Aimplantaçãodeumprogramadequalidadetemcomofocogarantiraqualidadedoproduto,

promovermelhoriascontínuaspormeiodeestímuloàinovação,atenderàsexpectativasdosclientes,

mantendo-ossatisfeitos.

b)Aimplantaçãodeumprogramadequalidadetemcomofocopadronizaraprodução,promover

melhoriaspormeiodeumprocessodeespecializaçãoecriarunidadesdeatendimentoaocliente.

c)Aimplantaçãodeumprogramadequalidadetemcomofocoproduzirumúnicoproduto

padronizado,promovermelhoriaspormeiodeumprocessodeespecializaçãoecrernadelizaçãodo

mercado.

d)Aimplantaçãodeumprogramadequalidadetemcomofocogarantiraqualidadedoproduto,

promovermelhoriascontínuaspormeiodadiminuiçãodedesperdícios,atenderàsexpectativasdos

clientes,mantendo-ossatisfeitos.

e)Aimplantaçãodeumprogramadequalidadetemcomofocoproduzircertonaprimeiravez,

promovermelhoriascontínuaspormeiodeestímuloàinovação,criarunidadesdeatendimentoao

cliente.

23.(Esaf/MTE–AFT–2003)Agestãocomfoconaqualidadeganhavisibilidadeapartirdas

experiênciasimplantadasnoJapão.Aseguirseapresentaumparaleloentreosprincípiosque

norteiamumagerênciatradicionaleaquelesquenorteiamumagerênciavoltadaparaa

qualidade.Identiqueasfrasesquecorrespondemàgerênciatradicionaleàgerênciada

qualidade.Assinaleaopçãocorreta.

I.Errosedesperdícios,senãoexcederemlimitespadrão,sãotolerados.

II.Ênfasenotrabalhoemequipe,incluindofornecedoreseinstituiçõescoligadas.

III.Produtoseserviçosdefinidosdeformasequencial,maspordepartamentos.

IV.Osusuáriosdosprodutoseserviçosdefinemoquequerem.

a)I–tradicionalI–qualidadeII–tradicionalIV–qualidade

b)I–tradicionalI–qualidadeII–qualidadeIV–tradicional

c)I–qualidadeI–tradicionalII–qualidadeIV–tradicional

d)I–tradicionalI–tradicionalII–qualidadeIV–qualidade

e)I–qualidadeI–tradicionalII–tradicionalIV–qualidade

24.(Esaf/MPOG/Enap/2006)Indiqueaopçãoqueapresentaumprincípioquenãoestárelacionado

comprogramasdequalidade.

a)Fazerprodutossemdefeitosdefabricação.

b)Atenderàsnecessidadesespecíficasdocliente.

c)Redesenhardeformaradicalosprocessos.

d)Eliminardesperdícios.

e)Planejaremanterumprocessodeaprimoramentocontínuoegradual.

25.(Cespe/TJ-ES–Analista/2011)Entreosdenominados14pontosdeWilliamDeming,quetiveram

inuênciadeterminantenaescolajaponesadaqualidade,inclui-seaimplementaçãoda

administraçãoporobjetivos.

26.(Cespe/Correios/Analista/2011)Asaçõesdequalidadedesenvolvidascomointuitodeassegurar

asatisfaçãodosclientesdevem-selimitaraosprocessosproblemáticosdasorganizações.

27.(Cespe/MPSAdministrador/Administrador/2010)Oambientenoqualestãoinseridasas

organizaçõesestáemconstantemudança,fazendo-senecessáriasaobservaçãocautelosada

realidadeeaboaadaptabilidade.Essassãoascaracterísticascentraisdaadministraçãoda

qualidadetotal,naqualaflexibilidadeorganizacionaléumpontoessencial.

28.(Cespe/MPE-PI/Técnico/2012)Aobservaçãodiretadoprodutoouserviçoaonaldoprocesso

produtivo,ainspeçãocombaseemamostraseosurgimentodosdepartamentosdequalidade

sãoasprincipaiscaracterísticasdaeradaqualidadetotal.

29.(Cespe/MPE-PI/Técnico/2012)OciclodeShewhartoudeDeming,clássicaferramentadaescola

daqualidade,proporcionaaoprogramadequalidadeumametodologiadecaráterpontual

sobreasatividadesdeplanejamento,execução,checagemecorreção.

30.(Cespe/MPE-PI/Técnico/2012)Entreos14princípiosquecaracterizamalosoadaqualidadede

Deming,encontra-seaeliminaçãodaadministraçãoporobjetivos.

31.(Cespe/MPE-PI/Técnico/2012)OdiagramadeIshikawa,oprincípiodeParetoeosmodelos

estatísticossãoinstrumentosdegestãoutilizadospelodepartamentodequalidade,quedeve

seroresponsávelúnicopelosprogramasdequalidadedaorganização.

Gabaritos

1.C

11.A

23.A

2.E

12.E

24.C

3.C

13.E

25.E

4.C

14.C

26.E

5.E

15.C

27.E

6.D

16.E

28.E

7.A

17.C

29.E

8.C

18.B

30.C

9.A

19.C

31.E

10.C

20.E

22.C

21.C

Bibliografia

Carvalho,MarliMonteirode,eEdsonPachecoPaladini.GestãodaQualidade.2a

ed.RiodeJaneiro:Abepro,2012.

Coltro,Alex.“Agestãodaqualidadetotalesuasinuênciasnacompetitividade

empresarial.”CadernodePesquisasemAdministraçãoV.1,n.2(1oSem.1996).

Crosby,PhilipB.Qualidade–falandosério.1aed.Tradução:JoséCarlosBarbosa

dosSantos.SãoPaulo:McGraw-Hill,1990.

Daft,RichardL.Management.Mason:Thomson,2005.

Dale,BarrieG.ManagingQuality.3aed.Oxford:BlackwellPublishers,1999.

Garvin,DavidA.Managingquality–thestrategicandcompetitiveedge.1aed.New

York:TheFreePress,1988.

Junior,IsnardMarshall,AglibertoAlvesCierco,AlexandreVarandaRocha,

EdmarsonBacelarMota,eSérgioLeusin.Gestãodaqualidade.9aed.Riode

Janeiro:FGV,2008.

Lima,CarlosAlbertoNogueirade.AdministraçãoPúblicaparaconcursos.Riode

Janeiro:Elsevier,2005.

Mello,CarlosHenriquePereira.GestãodaQualidade.SãoPaulo:Pearson

EducationdoBrasil,2011.

Sobral,Felipe,eAlketaPeci.Administração:teoriaepráticanocontextobrasileiro.

SãoPaulo:Pearson-Prentice-Hall,2008.

Capítulo14

GestãodoConhecimento

14.1.ConceitosdeDados,InformaçãoeConhecimento

Paraquepossamosentenderoqueégestãodoconhecimento,necessitamos

compreenderosconceitosdedado,informaçãoeconhecimento.

Umdadopodeserdescritocomoamatéria-primadainformação.1Éa

informaçãobruta–semsignicado,semumcontexto.Jáainformaçãoseriam

essesdadostrabalhados,demodoquetenhamumsignicado.Dessaforma,as

informaçõesadquiremumarelevânciamuitomaior.

DeacordocomDavenportePrusak:2

oconhecimentoédecorrentedainformação,que,porsuavez,derivadeconjuntodedados.Os

dadossãosériesdefatosoueventosisolados.Sãoregistrossemsignicadoinerente,quese

transformameminformaçõesaoadquiriremsignicado.Asinformaçõessãodadosque,

percebidospeloindivíduo,têmrelevância,propósitoecausamimpactoemseujulgamentoou

comportamento.Oconhecimentopodeservistocomoconjuntodeinformaçõesreconhecidase

integradaspelapessoadentrodeumesquemapreexistente.

Dessaforma,apenasosdadosnãonosservemdemuitacoisa.Imaginequeeulhe

forneçaumnúmero:26-05.Isso,porsisó,nãolhe“diz”muito,nãoéverdade?

Entretanto,seagregarmosoutrosdadose“trabalharmos”essesdados,eles

adquirirãoumsignicado.Porexemplo:nodia26-05sairáumeditaldonovo

concursodoSenadoFederal.Essejánãoésomenteumdadoisolado,nãoé

mesmo?Passouaserumainformação(hipotética,poisnãoéverdadeira!)que

podeserutilizadaporqualquerpessoa.

Assimsendo,asinformaçõespodemseranexadaseinseridasemumcontexto,

quesetornaráconhecimentoatravésdaintervençãodaspessoas,queutilizarãoas

informaçõesemseucontexto.

Dessaforma,podemosveraseguirumgrácoquesimbolizaessarelaçãoentre

osdados,asinformaçõeseoconhecimento:

Figura14.1–Dados,informaçãoeconhecimento

DeacordocomSantos,3oconhecimentoderivadainformação,assimcomo

estaderivadosdados.Oconhecimentonãoépuronemsimples,maséuma

misturadeelementos;éuidoeformalmenteestruturado;éintuitivoe,portanto,

difícildesercolocadoempalavrasoudeserplenamenteentendidoemtermos

lógicos.

Portanto,oconhecimentoexisteemcadaumdenós,dentrodaspessoas.Assim,

eleébastantecomplexo.DeacordocomNonakaeTakeuchi,4“oconhecimento,

diferentementedainformação,refere-seacrençasecompromisso”.Paraos

autores:5

Primeiro,oconhecimento,aocontráriodainformação,dizrespeitoacrençase

compromissos.Oconhecimentoéumafunçãodeumaatitude,perspectivaouintençãoespecífica.

Segundo,oconhecimento,aocontráriodainformação,estárelacionadoàação.Ésempreo

conhecimentocomalgumm.Eterceiro,oconhecimento,comoainformação,dizrespeitoao

significado.Éespecíficoaocontextoerelacional.

Portanto,osvaloresindividuaiseascrençasirãointegraroconhecimento,pois

irãoafetaromodocomoosindivíduosirãocaptar,analisaretirarconclusõesdas

informaçõesquereceberem.Émuitocomumquepessoasdiferentesreajamde

maneiradistintaàmesmainformação,nãoé?

Issoocorre,poistemosvaloresecrençasdiferenciados.Essasdiferençasnos

fazeminterpretarosdadosdemaneiradiferente,eporconsequênciaagir

diferente.

14.2.GestãodoConhecimento

Agestãodoconhecimentoéumprocessoqueseinicianaidenticaçãodos

conhecimentosestratégicosdeumaorganização.Portanto,incluitambémas

diversasformasdecriaçãodoconhecimentodentrodaempresa,bemcomoa

evoluçãoearenovaçãodosconhecimentosexistentes,demaneiraquese

mantenhamválidoserelevantes.

Finalmente,oprocessosefechanadevidaaplicaçãodosconhecimentosnas

situaçõespráticasdavidaorganizacional.DeacordocomQuandt:6

Agestãodoconhecimentonasorganizaçõespodeserentendidacomoumprocessoqueseiniciana

identicaçãodosobjetivosestratégicosorganizacionaiseseguepelaspráticasgerenciaisutilizadas

paraaidenticação,desenvolvimento,capturaedisseminaçãodoconhecimentoútil.Esseprocesso

degestãovisaaperfeiçoarodesempenhodaorganizaçãopormeiodaaplicaçãodeconceitos,

procedimentoseferramentasdetecnologiadeinformaçãoparaapoiarpráticascoletivasdecriação

ecompartilhamentodoconhecimentodaempresa.

AimportânciadagestãodoconhecimentofoiressaltadaporDrucker,7que

armouqueestávamosentrandoemumanovaera:aSociedadedo

Conhecimento.Essasociedadeestaria,portanto,substituindoasociedade

industrial.Deacordocomoautor,

“hojeorecursorealmentecontrolador,ofatordeproduçãoabsolutamentedecisivo,nãoéo

capital,aterraouamãodeobra.Éoconhecimento”.

Damesmamaneira,Castells8tambémanalisouestanovasociedadebaseadano

conhecimento.Paraele,asmudançastecnológicasestãomudandoabasematerial

dasociedadeereestruturandoosistemacapitalistadesdeonaldoséculo

passado.Paraoautor:

“nonovomodoinformacionaldedesenvolvimento,afontedeprodutividadeacha-sena

tecnologiadegeraçãodeconhecimentos,deprocessamentodeinformaçõesedecomunicação

desímbolos.Contudo,oqueéespecícoaomodoinformacionaldedesenvolvimentoéaaçãode

conhecimentossobreosprópriosconhecimentoscomoprincipalfontedeprodutividade”.

Dessaforma,agestãodoconhecimentobuscafacilitaroprocessodegeraçãoe

distribuiçãodoconhecimentodentrodeumaorganização.

Oobjetivoéqueoconhecimentosejaacessadoeutilizadoportodosos

funcionáriosemsuasatividadesdiárias,demodoaaumentarodesempenhodas

pessoasedasorganizaçõescomoumtodo.DeacordocomTerra:9

“agestãodoconhecimentopodeserconsideradaoesforçoparamelhorarodesempenhohumano

eorganizacional,pormeiodafacilitaçãodeconexõessignificativas”.

Vamosveragoracomoessetemajáfoicobrado?

1.(Esaf/STN/Desenv.Institucional/2005)Escolhaaopçãoquecompletacorretamenteafrasea

seguir:

Porgestãodoconhecimentoseentendeumprocessointegradoquesedestinaa:

a)criar,organizar,disseminareintensicaroconhecimentoparamelhorarodesempenhoglobalda

organização;

b)criarhabilidadeseorganizartreinamentosemserviçoparamelhorarodesempenhoglobalda

organização;

c)criar,organizaredisseminarinformaçõesparamelhorarodesempenhoglobaldosempregados;

d)criarhabilidadeseorganizartreinamentosemserviçoparamelhorarodesempenhoglobaldonível

gerencial;

e)criar,organizar,disseminareintensicaroconhecimentoparamelhorarodesempenhoglobaldonível

gerencial.

Essaquestão,apesardenãosermuitocriativa,nospossibilitaentendero

posicionamentodabancaquantoàdeniçãodegestãodoconhecimento.Dessa

forma,esteserefereàcriação,organização,disseminaçãoeintensicaçãodo

conhecimentoparamelhorarodesempenhoglobaldaorganização.

Assimsendo,aletraAestácorreta.AletraBestáincorreta,poisalterao

conhecimentoportreinamentoemserviço.JáaletraCtrocaconhecimentopor

informações.EmrelaçãoàletraD,estatambémnãoserefereaosconhecimentos.

Portanto,tambémestáerrada.

Porm,aletraEestáequivocada,poisagestãodoconhecimentonãodeve

ocorrerapenasnonívelgerencial.OgabaritoémesmoaletraA.

14.3.ConhecimentoTácitoeExplícito

Umadasprincipaisnoçõesquetemosdeteremgestãodoconhecimentoéa

diferençaentreconhecimentotácitoeexplícito,derivadadostrabalhosde

Polanyi.10

Deacordocomoautor:

Oconhecimentoexplícitooucodicadoépassíveldetransmissãosistemáticapormeioda

linguagemformal,relacionadoaeventoseobjetos,independentementedecontexto.Deoutro

lado,oconhecimentotácitoépessoal,relacionadoaumcontextoespecícoedifícildeser

formalizadoecomunicado.Representaoconhecimentoproduzidopelaexperiênciadevida,

incluindoelementoscognitivosepráticos.

Assimsendo,oconhecimentotácitoéoquevemdaexperiênciadecadapessoa.

Eleésubjetivo,poisdecorredosvaloresedavivênciadecadaindivíduo.Essetipo

deconhecimentoédifícildesertransferidoparaalinguagemformal,escrita.

Muitasvezes,“sabemosmaisdoqueconseguimosdizer”.11

Portanto,essetipodeconhecimentoéconsideradoumimportantequesitona

competitividadedasorganizações,esóépossívelavaliá-lopormeiodaação.Éo

chamadoknow-how,ousaberfazer.

Imaginequevocêsaibadirigirautomóveis.Dezanosatrás,seupaiolevouauma

fazendaemostrouosprimeiroscuidadosquevocêdeveriaternadireção.Coma

prática,vocêfoievoluindoehojejádirige,naturalmente,muitobem.

Portanto,esseconhecimentoquevocêadquiriuétácito!Elenãofoi

adquirido,nemestáinserido,emummanualdedireçãodeautomóveis.Eleestána

suacabeça!Portanto,foifrutodeumasériedeexperiênciasquevocêtevedurante

suavida.

Decertamaneira,muitasvezesnemsabemosquetemoscertoconhecimento

tácito,ouqueelepodeservirparaalguém.

Jáoconhecimentoexplícito,muitovalorizadoemnossaculturaocidental,éo

conhecimentoquejáfoitransformadoparaalinguagemformal,atravésde

manuais,normas,textos,equaçõesmatemáticasetc.

Esseconhecimentodecertaformajáfoiexplicado,mapeadoeestáprontopara

serreproduzidoetransferidodeformamuitomaisfácilentreaspessoas.

Imaginealeidagravidade,porexemplo.MuitoantesdeNewton(quefoiquem

aconceituou,transformandooconhecimentotácitoemexplícito),osseres

humanosjáaconheciamdemaneiratácita,nãoé?

Ouseja,mesmoantesquealeidagravidadefosseestudadacienticamente,ea

suadinâmicaexplicadaemequações,osindivíduosjásentiamqueexistia“algo”

queospuxavaparabaixo,poissentiamessaforçaemsuavivência.

Outroexemploconhecidoéodosgrandescozinheiros.Pormaisquevocêtente,

mesmoseguindoaquelareceitadolivro“àrisca”opratonãocatãobonitoe

apetitosocomonorestaurante,nãoémesmo?

Issoaconteceporqueareceitaéoconhecimentoexplícito,masprovavelmente

faltoualgumconhecimentotácitoavocênomomentodaexecuçãodoprato.Esse

conhecimentotácitoéasensibilidadedocozinheiro,suaexperiência,seus“anos

decozinha”.

Habilidadeeexperiênciasãomuitodifíceisdeseremtransformadasem

conhecimentoexplícito.Muitasvezes,nemquequeira,ocozinheiroconseguirá

lhepassaresseconhecimento.

Cabelembrarqueosconhecimentostácitoseexplícitossãocomplementares,

poisnãoconseguiríamosisolá-los.Ainteraçãoenteeleslevaráumaorganização

agerarmaisconhecimento.

Vejaaseguirumresumodosconceitos:

Figura14.2–Conhecimentotácitoeexplícito

Vamosanalisaragoraoutraquestãosobreessetema?

2.(FGV/Badesc/Anal.Administrativo/2010)Comrelaçãoaoconhecimentotácitoeaoconhecimento

explícito,analiseasafirmativasaseguir.

I.Oconhecimentotácitoésimplesdeserarticuladonalinguagemformal.

II.Oconhecimentotácitopossuinaturezaintangívelepessoal.

III.Osconhecimentostácitoeexplícitosãocomplementaresesuasinteraçõesproporcionam

dinamismoàsorganizações.

Assinale:

a)sesomenteaafirmativaIestivercorreta;

b)sesomenteaafirmativaIestivercorreta;

c)sesomenteaafirmativaIIestivercorreta;

d)sesomenteasafirmativasIeIestiveremcorretas;

e)sesomenteasafirmativasIeIIestiveremcorretas.

Comovimosantes,oconhecimentotácitoémuitodifícildeserarticuladona

linguagemformal,portantoaprimeiraarmaçãoestáincorreta!Jáasegunda

armaçãoestácorreta,poisoconhecimentotácitoébaseadonavivênciae

experiênciadecadapessoa.Comissoeleéintangível(nãopodesertocado)e

pessoal.

Emrelaçãoàterceirafrase,osconhecimentostácitoeexplícitosãorealmente

complementares.Dessaforma,écombasenasuainteraçãoqueasorganizações

gerammaisconhecimento.Assimsendo,aterceirafrasetambémestácorreta.O

nossogabaritoéaletraE.

14.4.AprendizagemOrganizacional

Aaprendizagempodeocorreremdiversosníveis.Noplanoindividual,a

aprendizagemocorreatravésdeumprocessodeinterpretaçãoeassimilaçãode

informações.

aaprendizagemorganizacionaltemcomoobjetivodesenvolveras

habilidadeseconhecimentosdosfuncionários,deformaqueestespossam

desempenharseutrabalhomelhoresolucionarproblemas.12

Dessaforma,aaprendizagemorganizacionalseapoianaaprendizagem

individual,poisnascedaacumulaçãoeposteriordisponibilizaçãodos

conhecimentosadquiridosnoplanopessoal.

DeacordocomArgyriseSchon:13

todaaçãodeliberadateveumabasecognitiva,quereetenormas,estratégiasesuposiçõesou

modelosdomundo..eaprendizagemindividualpelaaquisiçãodoconhecimentocriaumafundação

paraaprendizagemorganizacional.

Oconceitodeaprendizagemorganizacionalrelaciona-seaumprocessode

acumulaçãoeutilizaçãodoconhecimentoadquiridopelaexperiênciadeseus

funcionários.

Outroconceitoéodeaprendizagemgerencial.Essetipodeaprendizado,de

acordocomMotta,14éoprocessopeloqualumapessoaadquirenovos

conhecimentos,atitudesevaloresemrelaçãoaotrabalhoadministrativo;fortalece

suacapacidadedeanálisedeproblemas;tomaconsciênciadealternativas

comportamentais;conhecemelhorseusprópriosestilosgerenciaiseobtém

habilidadesparaumaaçãomaiseficienteeeficaz.

Paraoautor,esseprocessodeaprendizadogerencialécompostodequatro

dimensõesbásicas:cognitiva,analítica,comportamentalehabilidadedeação.A

seguirpodemosvê-lasemdetalhes:15

ØCognitiva–érelacionadaaoprocessodeserinformadoedeaprendersobre

administraçãobaseadonomontanteestoquedeconhecimentosjáexistentes,

desdeadeniçãodeobjetivoseaformulaçãodepolíticasatéoarsenalde

ideias

existentes

sobre

estruturas,

processos

e

comportamentos

organizacionais.Seria,portanto,maisligadaaoconhecimento“puro”.

ØAnalítica–érelativaaoprocessodeaprenderaidenticareadiagnosticar

problemasadministrativosdecompondo-osemdiferentespartes,para

rearrumá-losnabuscadenovassoluções.Dessamaneira,aprende-sea

estabelecerrelaçõesentrefatoresorganizacionaiseaidenticarosmais

importantes,alémdesepoderverapotencialidadedetécnicase

instrumentosadministrativosnasoluçãodeproblemas.Assimsendo,se

referemaisàcapacidadedeentenderosdesaosemontarcenários

complexos.

ØComportamental–serelacionaaoprocessodeaquisiçãodenovasmaneiras

deinteraçãohumana,entrepadrõesalternativosconhecidosevalidados

socialmente.Portanto,abrange,porexemplo,novasformasdecomunicação,

deinteragirempequenosgrupos,deexerceroudelidarcompodere

autoridade.É,decertomodo,ligadaàshabilidadeshumanas,comoa

oratória,afacilidadedecomunicaçãoeaempatia.

ØHabilidadedeação–serefereàcapacidadedealterarintencionalmentea

realidade,ouseja,acapacidadedetransformarconhecimentosealternativas

comportamentaisemformasefetivasdeação.Assimsendo,abrangeuma

melhorcompreensãodesipróprio,deseupapelorganizacional,dos

objetivosedocontextoemqueseatua,alémdocomprometimentocoma

missãodaempresaemquesetrabalha.Essadimensãoéimportante,pois

passaaideiadequesomenteoconhecimentonãogeraresultados–é

necessário“pôremprática”.

Vamosverumaquestãosobreessetema?

3.(FCC/Arce/AnalistaReg./2006)Noprocessodeaprendizadogerencial,serinformadoeaprender

sobreadministraçãoapartirdoestoquedeconhecimentosexistentes,desdeadeniçãodos

objetivoseaformulaçãodepolíticasatéoarsenaldeideiasexistentessobreestruturas,

processosecomportamentosorganizacionaissignificadesenvolverahabilidade:

a)sistêmica;

b)analítica;

c)comportamental;

d)deação;

e)cognitiva.

Comovimos,adimensãoqueserelacionacomo“conhecimentopuro”,ouseja,

comacapacidadedeaprendercomo“estoque”deconhecimentosjáexistentes,éa

dimensãocognitiva.OgabaritoéaletraE.

Continuando,outrosconceitosquetemosdecompreendersãoosde

aprendizagemformaleaprendizageminformal.

Aaprendizagemformalacontecequandoexisteumprocessodeaprendizagem

estruturadoeformatado,comumobjetivoespecíco.Assim,umcurso

universitárioéumprocessodeaprendizagemformal,poiscontacomumagrade

curricular,umasériedeprofessorestreinadosecontratadosparalecionar

matériasespecíficasetc.

Alémdisso,oobjetivodessecursouniversitáriojáestáespecicadoantes

mesmoquevocêsematricule.Decertaforma,éumprocessopadronizadode

aprendizado.

Jáaaprendizageminformalocorredemaneiranatural,semumobjetivo

denidoouumapadronização.Entretanto,nossasociedadenãocostumadar

muitovaloraoaprendizadoinformal.Porexemplo,vocêfariaumacirurgiacom

algumenfermeiro,oqualjáviutantasoperaçõesquejá“sabe”tudo?

Achopoucoprovável,nãoémesmo?Apesardisso,nenhumaorganização

sobreviveriasemaaprendizageminformal.

Dessaforma,esseaprendizadoestásempreocorrendo,poisestamossempreem

contatocompessoasesituaçõesdiferentes,demodoquevamos“aprendendo”

coisasnovasatodotempo.Dentrodoambienteorganizacional,eleémuito

comumeimportante.

Issoocorre,poisémuitoraroqueexistamprocessosdeaclimataçãoe

treinamentodosnovosintegrantesdeformaqueelesjáiniciemsuasatividades

“prontos”paraotrabalho.Outrofatorimportanteéatrocadeexperiênciasentre

ostrabalhadores,demodoqueoconhecimentosejamaisrapidamentetransmitido

e,atémesmo,maisfacilmentegerado.

DeacordocomMarsickeWatkins:16

adiferençaentreestasduasformasdeaprendizagem,équenainformalocontroledoqueestá

sendoaprendidoestáexclusivamentenasmãosdopróprioindivíduo.

Entretanto,nãopodemosimaginarque,porfaltadecontrole,aaprendizagem

informalsejaindesejadapelasorganizações.Existemdiversasorganizaçõesque

estimulamesseprocesso,poisconsideramfundamentalparaoprocessocriativoe

paraodesenvolvimentodoknow-how,ousaberfazer.

Outropontoimportanteéquepraticamenteinexisteumprocesso“puro”de

aprendizagem.Decertomodo,sempreexistirãofatoresinformaiseformaisemum

processodeaprendizagem.Penseemsuaprópriasituação:quandoestavana

faculdade,aprendeusomentenasaulas?Noslivros?Comcertezanão!

Muitasvezes,mesmoemprocessosformaisdeaprendizagem,aprendemosmais

comatrocadeexperiênciasentreoscolegasouatémesmodosprofessores.De

acordocomMalcom,HodkinsoneColley:17

todas

(ou

quase

todas)

as

situações

de

aprendizagem

contêm

atributos

de

formalidade/informalidade;essesatributosestãointerligadosdeformasdiferentesnasdiversas

situaçõesdeaprendizagem.

Paranalizar,podemosveraseguir,nagura,asprincipaisdiferençasentreos

doistiposdeaprendizagem:

Figura14.3–Aprendizagemformaleinformal

Portanto,vamosveragoraumaquestãodessetema?

4.(Cespe/Inmetro/RecursosHumanos/2009)Aprenderfazendooprópriotrabalhoéumexemplode

aprendizagemformalouinduzida.

Aprendizagemformaléaquelaestruturadaanteriormente,comoocorreem

umafaculdade(existeumagradecurricular,comosobjetivosdoensinoetc.)ou

emumcursotécnicoouadistância.

Quando,porexemplo,aprendemosatravésdenossoprópriotrabalho,através

dosconselhosdealguém,esseprocessodeaprendizageméinformal(nãoestava

formatado,padronizado,comumobjetivoanteriormentedenido).Portanto,essa

questãoestáincorreta.

Continuando,outrofatoréoespaçoemqueosconhecimentoseinformações

sãocaptados.Estespodemseradquiridosinternamenteouexternamente.

Aorganizaçãopodeadquirirconhecimentosinternamenteatravésde

processosdetreinamento,deumareuniãoemqueocorraumbrainstormingoude

qualquerexperiênciaindividualougrupalqueocorradentrodaorganização.

Jáaaquisiçãoexternapodeocorrerdediversasformas.Quandouma

organizaçãoenviaumfuncionárioaumcursoexterno,estáocorrendoessa

aquisição,poisofuncionáriovoltarácomconhecimentosnovosparaoseioda

organização.

Outrapossibilidadeéacontrataçãodeespecialistasouconsultoresexternos,

quetrarãoconhecimentosnovosparaaempresa.Aobservaçãodecompetidores

ououtrosparceiros(comoocorrenobenchmarking)tambémseinserenesse

contexto.Portanto,étodomododeaquisiçãodeconhecimentosqueestá

disponívelforadoambientedaorganização.

14.5.EspiraldoConhecimentodeNonakaeTakeuchi

DeacordocomNonakaeTakeuchi,18oprocessodecriaçãodeconhecimento

nasorganizaçõesserelacionacomainteraçãodosconhecimentostácitose

explícitos–oqueosautoreschamamdeconversãodeconhecimento.

Essemacroprocessopodeserdesdobradoemquatromodosdesecriar

conhecimento:

asocialização,

acombinação,

aexternalizaçãoea

internalização.Naguraaseguirpodemosverumesquemaquesintetizaa

relaçãoentreessesmodos:

Figura14.4–Conversãodoconhecimento.

(Fonte:Nonaka1994)

Oprimeiromodo,chamadodesocialização,compreendeaconversãode

conhecimentotácitoemoutroconhecimentotácito.Esseprocessoocorreatravés

daprópriainteraçãoentreaspessoasnoambientedetrabalho.

Quandoumfuncionáriomaisantigoestápassandosuaexperiênciaparaum

maisnovo,esseprocessoestáacontecendo.Assim,elepodeocorrerpormeioda

linguagem,bemcomoatravésdaobservação,daimitação,entreoutrasformas.

Achavenesseprocesso,segundoNonaka,19éaexperiência.Semummínimode

experiênciascompartilhadasentreduaspessoas,émuitodifícilquealguémpossa

aprendercomoaoutrapessoapensaeporquetomacertasdecisõesouagede

algumaforma.Portanto,temosde“conhecer”umpoucoooutroantesde

aprendercomele.

Osegundomododeconversãoocorrequandotransformamosconhecimento

explícitoemoutroconhecimentoexplícito.Dessaforma,aomudarmoso

contexto,recategorizarmosouaumentarmosumconhecimentoexplícito,estamos,

decertaforma,transformando-o.

Porexemplo,decertamaneiraestoufazendoissoagora,poismeutilizode

diversostextosdeoutrosautoresparaescreverumlivrovoltadoparaconcursos

públicos,nãoéverdade?Damesmaforma,quandovocêsfazemumresumodeum

livro,ouummapamental,tambémestãoseutilizandodesseprocessode

combinação.

Assimsendo,acombinaçãoéumprocessoquetransformaconhecimento

explícitoemoutroconhecimentoexplícito.

Oterceiroeoquartomodosereferemaumaconversãoentreoconhecimento

tácitoeoexplícito.Decertaforma,baseiam-senanoçãodequeesses

conhecimentossãocomplementares,equepodemseexpandircomessainteração.

Assim,oprocessodetransformaçãodeumconhecimentotácitoem

conhecimentoexplícitoéoterceiromodo.Quandoumcozinheirofamosolança

umlivrodereceitas,porexemplo,estábuscandofazerisso.Nonakachamouesse

processodeexternalização.

Finalmente,existeoquartoprocesso.Esteenvolveatransformaçãode

conhecimentoexplícitoemconhecimentotácito.Decertomodo,identica-se

comoconceitocomumde“aprender”.

Ouseja,quandoestamoslendoummanual,umlivro,ouvendoumavideoaula,

estamosbuscandoconverterconhecimentoexplícitoemconhecimentotácito,não

émesmo?Esseprocessoéchamadodeinternalização.

Outronomemuitoutilizadoéode“espiraldoconhecimento”,poisserefereà

ideiadeque,comacontínuainteraçãoentreosconhecimentostácitose

explícitos,potencializa-seodesenvolvimentoetransmissãodessesconhecimentos.

QuestõesComentadas

5.(Cesgranrio/BancoCentral/AnalistaÁrea4/2010)Emumaempresaquetemprocessosdegestão

doconhecimento,quandoumgerentedecomunicaçãoparticipadeumseminárioexterno,ao

retornaràssuasatividades,eledeve:

a)avaliaroquepodeounãoserimplantadonaorganizaçãonocurtoprazo;

b)registrarasinformaçõeserepassá-lasatodososinteressadosnaqueleassunto;

c)elaborarrelatóriossobresuaexperiênciafora,paraseremarquivados,documentandoaatividade;

d)reportaroqueaprendeudenovoaosseussuperioresimediatos;

e)reunir-secomaequipeparacontarasinovaçõesquepresencioueaprendeu.

Agestãodoconhecimentodevefacilitarodesenvolvimentoeadistribuiçãodas

informaçõeseconhecimentosparatodaaorganização.Dessaforma,nocaso

citado,ofuncionáriodeveriaregistrarasinformaçõesrecebidasedisponibilizara

todaaempresa.Portanto,aletraBespecificamelhoresseprocesso.

AletraAestáincompleta,poisserefereapenasàanálisedasinformações.Jáa

letraCtambémabordasomenteoacúmulodeconhecimentos,semsereferiràsua

distribuição.

Entretanto,oserrosdasletrasDeEestãorelacionadosàdistribuiçãodas

informações.Elasnãodevemestarrestritasaoschefeseàsprópriasequipesde

trabalho.Dessamaneira,devemserdisponibilizadasatodososfuncionáriosque

delasnecessitem.Portanto,onossogabaritoémesmoaletraB.

6.(Esaf/MPOG/APO/2008)Emboranãosejaumaabordagemnova,a“gestãodoconhecimento”

aindaprovoca,aomesmotempo,umintensodebatesobreoseusignicado,epoucoecono

âmbitodasorganizaçõespúblicasbrasileiras.Osconceitosapresentadosaseguirsãocentraisà

compreensãodessenovomodelodegestão.Examineosenunciadoseindiquearesposta

correta.

1.Dadossãoumconjuntodefatosdistintoseobjetivos,relativosaeventoseque,emum

contextoorganizacional,sãoutilitariamentedescritoscomoregistrosestruturadosde

transações,masnadadizemsobreaprópriaimportânciaourelevância.

2.Informaçõessãodadosinterpretados,dotadosderelevânciaepropósito,portanto,

signicado,ouseja:ainformaçãovisaamodelarapessoaquearecebenosentidodefazer

algumadiferençaemsuaperspectivaouinsight.

3.Oconhecimento,aocontráriodainformação,dizrespeitoacrenças,compromissoseação.O

conhecimentoéfunçãodeumaatitude,perspectivaouintenção,éespecícoaocontextoe

érelacional.

4.Agestãodoconhecimentoestáintrinsecamenteassociadaàgestãodainformação,que,por

suavez,refere-seàsferramentas,metodologiasetécnicasutilizadasparaacoleta,o

armazenamento,oprocessamento,aclassificaçãoeautilizaçãodedados.

a)Todososenunciadosestãocorretos.

b)Somenteoenunciado3estáincorreto.

c)Osenunciados2e3estãoincorretos.

d)Somenteoenunciado4estáincorreto.

e)Osenunciados1e4estãoincorretos.

Essaquestãoébeminteressanteparaquepossamossaberaposiçãodabanca

quantoaessesconceitos.Nocaso,abancasomenteinseriuuma“pegadinha”na

últimafrase,deformaatorná-laincorreta.Nessecaso,elasebaseouemumartigo

deMadureiraCoelho,20quecitoaseguir:

Deformacontrastanteaotermo“gestãodoconhecimento”contrapõe-seoconceitode“gestão

dainformação”,queestáintrinsecamenteassociadoàexpressão“tecnologiadainformação”,

provenientedaáreade“processamentoeletrônicodedados”,que,porsuavez,refere-seàs

ferramentas,metodologiasetécnicasutilizadasparaacoleta,oarmazenamento,oprocessamento,

aclassificaçãoeautilizaçãodedados.

Entretanto,acreditoqueaquestãonãofoimuitofeliz.Se,porumlado,está

corretaaarmaçãodequeosconceitossãodiferentes,elesnãodeixamdeser

ligados.

Agestãodainformaçãoserelacionamaisaoquechamamosdetecnologiada

informação(quetratadacoletaeanálisededados),enquantoagestãodo

conhecimentoestáemumplanosuperior,poislidacomaidentificação,acaptura,

odesenvolvimentoeadisseminaçãodoconhecimento.21

Dessamaneira,comoabancaconsiderouastrêsprimeirasfrasescorretas,o

gabaritoéaletraD.

7.(FCC/TCE/SP/Auditor/2008)SegundoosnovosmodelosdasorganizaçõesnachamadaSociedade

doConhecimento,constituemasprincipaiscaracterísticasdasempresas:

a)elevadaespecializaçãodefunções,hierarquiasreduzidasefoconosresultados,visandoàcrescente

produtividade;

b)capacidadedeadquirireintegrarnovosconhecimentos,visandoaoequilíbrioentreprodutividade,

qualidade,inovaçãoeresponsabilidadesocial;

c)aumentodaqualicaçãoformal,melhoriassalariaisemaiorautonomiaparaosfuncionários,visandoà

realizaçãomaiseficientedesuasmetasestratégicas;

d)aumentoeimplementaçãodaparticipaçãosocialemsuasdeniçõesestratégicas,visandoàmelhoria

daimagemeàsatisfaçãodosclientes;

e)prossionalizaçãodagestãoefoconainovaçãotecnológicaassociadaàproduçãomassicada,

visandoaoacessouniversaldapopulaçãoaosbensdeconsumoduráveis.

Aprincipalcaracterísticadasorganizaçõesdasociedadedoconhecimentoéa

capacidadededesenvolveredisseminarosconhecimentosdentrodaorganização.

Dessamaneira,ogabaritoéaletraB.

8.(Esaf/MPOG/EPPGG–2009)Nocampodagestãodainformaçãoedoconhecimento,écorreto

afirmarque:

a)oconhecimentotácitopode,notodoouemparte,serexplicitado;

b)classifica-secomotácitooconhecimentodisponívelemlivros,revistaseoutrosmeiosdefácilacesso;

c)todoconhecimentoexplícitotendeasetransformaremconhecimentotácito;

d)agestãodoconhecimentoestávoltada,prioritariamente,paraotratodoconhecimentoexplícito;

e)agestãodainformaçãoestávoltada,prioritariamente,paraotratodoconhecimentotácito.

Aprimeirafraseestácorretaeégabarito.Oconhecimentotácitopode,

naturalmente,serexplicitado.Nahoraquealguémbuscatransformarsua

experiência(adquiridadurantesuavida)emalgummanual,textoouqualquer

formatoqueseja,estátransformandoumconhecimentotácitoemconhecimento

explícito.

Jáasegundaalternativaestáincorreta,poisoconhecimentodisponívelem

livros,apostilasetc.éoconhecimentoexplícito.AletraCtambémestáerrada,

poisnemtodoconhecimentoexplícitotendeasetornarconhecimentotácito.

Emrelaçãoàsduasúltimasalternativas,aEsaftrocouosconceitos.Agestãoda

informação,porlidarcomaanáliseecoletadedados,estámaisligadaao

conhecimentoexplícito.

Entretanto,agestãodoconhecimento,porserelacionarcomoconhecimentoe

comoaprendizado,estámaisligadaaoconhecimentotácito.Portanto,ogabarito

éaletraA.

9.(Cespe/Inmetro/RecursosHumanos/2009)Acontrataçãodeespecialistaseconsultoresé

consideradaumprocessodeaquisiçãoexternadeconhecimentosporváriosteóricosdaárea.

Aaquisiçãodeconhecimentospodeocorrerdemaneirainterna(atravésde

pesquisasedesenvolvimento)edemaneiraexterna,comonocasodeuma

contrataçãodeconsultoreseespecialistasdeforadaempresa.Dessaforma,a

questãoabordaumaaquisiçãoexternadeconhecimentoseestácorreta.

10.(Cespe/Inmetro/Recursos

Humanos/2009)

Reuniõesdetrabalhosãoconsideradas

oportunidadesparadiscussãoeanálisedeeventospassados,constituindo-seemexemplosde

aprendizagempelaexperiência.

Beleza.Atravésdaanálisedoseventospassados,asorganizaçõespodemgerar

umbancode“melhorespráticas”ecomportamentosedecisõesquedevemser

evitados.Asreuniõesdetrabalhosãoeventosquepodemrealmenteservirpara

estaaprendizagem.

Comovimos,esseéummétododeaquisiçãointernadeconhecimentos.O

gabaritoéquestãocorreta.

QuestõesPropostas

11.(Cespe/Inmetro/RecursosHumanos/2009)Atividadesorganizacionaisdepesquisae

desenvolvimentosãoprocessosdeaquisiçãointernadenovosconhecimentos.

12.(FMP/TCE-RS/Auditor/2011)Noqueserefereàgestãodoconhecimento,assinaleaalternativa

CORRETA.

a)Aconversãodeconhecimentotácitoemconhecimentotácitoédenominadaexternalização.

b)Aconversãodeconhecimentotácitoemconhecimentoexplícitoédenominadasocialização.

c)Aconversãodeconhecimentoexplícitoemconhecimentoexplícitoédenominadainternalização.

d)Aconversãodeconhecimentoexplícitoemconhecimentotácitoédenominadacombinação.

e)“Espiraldoconhecimento”éoprocessoemqueainteraçãocontínuaedinâmicaentreo

conhecimentotácitoeoconhecimentoexplícitoémoldadapelasmudançasentreosdiferentes

modosdeconversãodoconhecimento.

13.(FCC/MP/RS/Administrador/2008)Aconcepçãoatualdeaprendizagemorganizacionalpressupõe

acompetênciaparacriareutilizarconhecimento,tornando-oumaimportantefontede

vantagenscompetitivasparaasorganizações.SegundoNonakaeTakeuchi,oprocessopelo

qualasorganizaçõescriamconhecimentoé:

a)aAutopoiesi,quesedesdobraemExtroversão,SistematizaçãoeSocialização;

b)aIniciativaCriadora,complementadapelaCatalisação,CristalizaçãoeConversão;

c)aGeraçãodeConhecimento,quecompreendeacirculação,aapropriaçãoeocompartilhamento;

d)oBrainstorming,seguidodoEmpowerment,doBenchmarkingedoJobEnrichment;

e)aConversãodeConhecimento,atravésdoqualoconhecimentotácitoeexplícitoéexpandido

qualitativaequantitativamente.

14.(Cespe/Ibram/Administrador/2009)ConformeatipologiadeconhecimentodenidaporNonaka

eTakeuchi,asocializaçãoéatransformaçãodeconhecimentotácitoemconhecimento

explícito.

15.(Esaf/STN/Desenv.Institucional/2008)Nadisseminaçãodoconhecimento,asinteraçõesque

ocorrementreoconhecimentotácitoeexplícitosãoclassicadasem:socialização,

externalização,combinaçãoeinternalização.Analiseasopçõesqueseseguemeassinalea

correta.

a)Nasocializaçãoenacombinação,aconteceaconversãodoconhecimentoexplícitoparatácito.

b)Nainternalização,assimcomonaexternalização,oconhecimentoseconvertedetácitoparaexplícito.

c)Nasocializaçãoenainternalização,nãoacontececonversãodeconhecimentoporinteraçõestácitas.

d)Nainternalizaçãoenacombinação,oconhecimentoconvertidoéoexplícito.

e)Nacombinaçãoenaexternalização,aconteceaconversãodoconhecimentoexplícitoparatácito.

16.(Cespe/Seapa-DF/Administrador/2009)Omapadoconhecimentovisaidenticaredisseminar

osconhecimentosexistentesnaorganização,sendovitalqueotrabalhoseconcentre

exclusivamentenaidenticaçãoenasistematizaçãodoconhecimentoexplícitoexistente,

identificadonosdocumentosebasesdedadosdaempresa.

17.(Cespe/Seapa-DF/Administrador/2009)Denomina-seespiraldoconhecimentoarepresentação

grácadoprocessodeinternalizaçãoesocializaçãodosconhecimentosnaorganização,

caracterizadoporumdinâmicoecontínuoprocessodetransformaçãodeconhecimentos

tácitosemexplícitos.

Tabela14.5

(J.S.LoiolaE.Nérise;A.V.B.Bastos.Aprendizagememorganizações:mecanismosquearticulamprocessos

individuaisecoletivos.In:Educaçãoemorganizaçõesetrabalho:fundamentosparaagestãodepessoas.

PortoAlegre:Artmed,2006,p.124(comadaptações).)

(Cespe/Inmetro/RecursosHumanos/2009)Apartirdaguraapresentadaeacercadosprocessosde

aprendizagememorganizações,julgueospróximositens.

18.Osprocessosdeconversãodeconhecimentosemorganizaçõesincluemotreinamentoexterno

eaparticipaçãodeempregadosemcongressoseseminários.

19.Osprocessosdeaquisiçãointernadeconhecimentosincluemasatividadesdetreinamento

internoeoaprenderfazendo.

20.Umexemplodesocialização,denidocomooconjuntodemecanismosadotadospelas

organizaçõesparadifundiroconhecimento,tácitoouexplícito,entreseusmembros,demodo

aconvertê-loemconhecimentoorganizacional,éasoluçãocompartilhadadeproblemas.

21.Certificaçõessãoexemplosderesultadosdeprocessosorganizacionaisdecodificação.

22.Aguraanteriorresumeedescreveosmecanismosindividuaisdeaprendizagemocorridosem

ambientesorganizacionais.

23.(Cespe/Petrobras/Administrador/2007)Aaprendizagemorganizacionaléumfenômenoque

ocorretantononívelindividualquantonodaorganização.

24.(Cespe/Petrobras/Administrador/2007)Asorganizaçõesdeaprendizagemapresentamcultura

direcionadaàconservaçãodosconhecimentosanteriormenteproduzidos.

25.(Esaf/ANA/Analista/2009)“Gestãodoconhecimentosignicaorganizarasprincipaispolíticas,

processoseferramentasgerenciaisetecnológicosàluzdeumamelhorcompreensãodos

processosdegeração,identicação,validação,disseminação,compartilhamento,proteçãoe

usodosconhecimentosestratégicosparagerarresultados(econômicos)paraaempresae

benefíciosparaoscolaboradoresinternoseexternos(stakeholders).”(Terra,2005,p.8).

Selecioneaopçãoincorreta.

a)Oprocessodegerenciamentodoconhecimentoconsisteemumciclodegeração,codicaçãoe

coordenaçãoedisseminaçãodoconhecimento.

b)Adisseminaçãodoconhecimentoconsistenapráticadatransferênciadoconhecimento,podendoser

pelacontrataçãodepessoas,pelasconversasinformaisenãoprogramadas,ouporreuniõeseações

estruturadasquepossibilitamamobilidadedoconhecimentopelaorganização.

c)Nadisseminaçãodainformação,oconhecimentoexplícitoéformalmentecapturadoecompartilhado

pormeiodatecnologiadainformação,enquantootácitonão.

d)Adisseminaçãodainformaçãoéumaaçãoglobalorganizacional:todooconhecimentodeveser

transmitidoparatodaaorganizaçãoindependentementedaabsorçãodecadaum.

e)Aamplitudedadisseminaçãodeveestaremacordocomaestratégiaorganizacional,comaspolíticas

depessoas,comomodelodeestruturadaempresaecomatecnologiaexistente.

26.(Esaf/RFB/Analista/2009)Noâmbitodagestãodainformaçãoedoconhecimento,écorreto

pressuporque:

a)ousointensivodatecnologiadainformaçãovisaàcriaçãodeumrepositóriodesoluçõesprontas,a

seremaplicadaspelaorganizaçãonaresoluçãodenovosproblemas;

b)oconhecimentopodeserobtidonoambienteexternodaorganização,inclusivejuntoaorganizações

concorrentes;

c)aadoçãodeumabasetecnológicadeprimeiralinhagaranteumaculturadoconhecimentonoseioda

organização;

d)oconhecimentotácitodizrespeitoaosindivíduos,nãodevendoseralvodeinteressedaorganização;

e)àmedidaqueorepositóriodeconhecimentoséampliado,aexperimentaçãopassaaser

desnecessária.

Gabaritos

1.A

10.C

19.C

2.E

11.C

20.C

3.E

12.E

21.C

4.E

13.E

22.E

5.B

14.E

23.C

6.D

15.D

24.E

7.B

16.E

25.D

8.A

17.C

26.B

9.C

18.E

Bibliografia

Alves,AnaPaula,AnaCarolinaGursendeMirandaOliveira,eCarlosOlavo

Quandt.GestãodoConhecimentonoBrasil:ummapeamentodaspublicaçõese

autoresde1998a2008.VIISimpósiodeExcelênciaemGestãoeTecnologia.s.d.

Antonello,ClaudiaSimone,eArildaSchmidtGodoy.Aencruzilhadada

aprendizagemorganizacional:umavisãomultiparadigmática.Revistade

AdministraçãoContemporânea14,n.2(Mar/Abr2010):310-332.

Carbone,PedroPaulo,HugoPenaBradão,JoãoBatistaDinizLeite,eRosaMaria

dePaulaVilhena.Gestãoporcompetênciasegestãodoconhecimento.3aed.Riode

Janeiro:FGV,2009.

Coelho,EspartacoMadureira.GestãodoConhecimentocomosistemadegestão

paraosetorpúblico.RevistadoServiçoPúblico,n.1(Jan-Jul2004).

Fernandes,PriscilaNery.Redessociaiseprocessosdeaprendizageminformalem

umaempresadetelecomunicações.Brasilia,2009.

Moresi,EduardoAmadeuDutra.InteligênciaOrganizacional:umreferencial

integrado.CiênciadaInformaçãoV.30,n.2(2001).

Motta,PauloRoberto.FormaçãodeLiderança.CadernodeAdministraçãoda

Justiça–PlanejamentoEstratégico2009:Módulo6.PortoAlegre:TRF–4aregião,

2009.

Nonaka,Ikujiro.Dynamictheoryoforganizationalknowledgecreation.

OrganizationScienceV.5,n.1(February1994).

Santos,AntonioRaimundodos.GestãodoConhecimento:umaexperiênciaparao

sucessoempresarial.Curitiba:Champagnat,2001.

Capítulo15

ProcessoDecisório

15.1.Estruturadasdecisõesempresariais

Todosnóstomamosdiversasdecisõesdurantenossasvidas.Essasdecisões

podemsercomplexasousimples.Podemserdifíceis(qualseránossaprossãoou

comquemcasaremos)oudecisõesdiárias,comoqualseráamarcaderefrigerante

queescolheremosnoalmoço.

Quandovocêdecidiuadquirirestelivro,tomouumadecisão,nãoéverdade?O

mesmoocorreuquandodecidiuestudarparaconcursospúblicos.

Dentrodasorganizações,tambémtomamosdecisões.Decertaforma,boaparte

dotrabalhodeumadministradoréatomadadedecisões.Deacordocom

Maximiniano:1

oprocessodetomardecisõeséumingredientesubstancialeinseparáveldasatividadesde

planejamento,organização,direçãoecontrole.

Vamosverentãoquaissãoosprincipaistiposdedecisão?

15.1.1.Tiposdedecisões

15.1.1.1.DecisõesProgramadaseNãoprogramadas

Decidirnadamaisédoqueescolherentrealternativasparapoderresolverum

problemaouatingirumobjetivo.Entreosdiversostiposdedecisões,podemos

classificá-lascomoprogramadasenãoprogramadas.2

Decisõesprogramadassãodecisõesrotineiras,ouseja,decisõessobre

assuntosquejáforamanalisadosanteseque,dessaforma,jáexisteumaresposta

“padrão”paraeles.Umadasformasdeseganhareciênciaemumaempresaé

relacionandoessas“decisõespadronizadas”acertosproblemasrotineiros.3

Imaginequevocêestudeemumafaculdadeepercaalgumaprovamarcada.

Quandovocêseapresentarnasecretaria,arecepcionistanãoiráfazerumareunião

comosfuncionáriosdosetorparadecidirsobreseucasoespecíco,nãoé

verdade?

Provavelmente,jáestaráprevistanomanualdafaculdadequalapena(ou

multa)quevocêterádepagarparafazeruma“segundachamada”.Dessaforma,a

recepcionistasimplesmenteiráaplicaroquejáestava“decidido”paracasosiguais

aoseu!

Dessamaneira,issoéoquechamamosdedecisãoprogramada,umadecisãoque,

detãorotineira,mereceserpadronizada.Assimsendo,seganhamuitotempo

dentrodeumaorganização,alémdeseevitarquesedecidamcoisasdiferentes

paraumamesmasituação.

Jáasdecisõesnãoprogramadassãotípicasdesituaçõesemqueoproblemaé

novo,oupoucofrequente,eogestordeveanalisarosdadosàsuadisposição

(muitasvezesincompletos)paraquepossatomarumadecisão.4

Pensenocasodeumdiretordeumacervejaria.Deumahoraparaaoutra,ele

descobrequeumconcorrentelançaráummodelodecervejaemseumercadoe

venderáaumpreçoabaixodoquesuaempresaestápraticando.

Antesdedecidiroquefazer,odiretorterádeanalisartodasasinformações

importantes,nãoémesmo?Osfatoresqueelelevariaemconsideração

provavelmenteseriamestes:Oconcorrenteéfortenanceiramente?Amarcada

cervejaéconhecida?Seucanaldedistribuiçãoéabrangente?

Portanto,eleteráentãode“pensar”oproblema,ouseja,analisarasituaçãoa

“fundo”.Dessaforma,nãohácomoseterumadecisãojá“pronta”paraumcaso

novo,nãoéverdade?Issoentãoéachaveparavocêdecidirseumadecisãoé

programadaounão!

Seoproblemaforrepetitivo(rotineiro),adecisãodeveserprogramada.

Entretanto,seoproblemafornovooupoucocomum,adecisãodevesernão

programada.

Vamosanalisarcomoessetópicojáfoicobrado?

1.(FGV/Sefaz/RJ/FiscaldeRendas/2010)Asalternativasaseguirapresentamtécnicasdeapoioà

decisãoprogramada,àexceçãodeuma.Assinale-a.

a)Modelosmatemáticos.

b)Sistemasdeapoioàdecisãocorporativa.

c)Planilhas.

d)Orçamentos.

e)Pesquisaoperacional.

Questãointeressante.Senosreferirmosàsferramentasdeauxílioàdecisão

programada,nãotemoscomoconsiderarossistemasdeapoioàdecisão

corporativa(nometécnicodosprogramascorporativosquegerenciamdadose

proporcionamrelatóriosdegestãoaosadministradores,facilitandoatomadade

decisão)comodesuporteàdecisãoprogramada!

Pensebem,sevocênecessitaanalisarosdadosdevendasjuntoaoscustosde

entreganaRegiãoSulparatomarumadecisãoéporqueestáanalisandoo

problema,nãoémesmo?

Asoutrasferramentascitadassãomesmomaisadequadasàsdecisões

programadas.Portanto,esseéocasodasplanilhasemquepodemosprogramar

fórmulasqueapontemquandosedevecomprarumprodutoouvenderumativo

etc.Dessaforma,ogabaritoéaletraB.

15.2.RacionalidadeeRacionalidadeLimitada

Omodeloracionaldetomadadedecisões(oumodeloclássico)sebaseavana

ideiadequeosgestorestomamdecisõeslógicas(utilizandosomentearazão)eque

buscammaximizarosresultadosdasempresas.5

Dessamaneira,omodeloracionaltraziaoconceitodecomoasdecisões

deveriamsertomadas,nãodecomoelasrealmenteeramtomadas.

Issoderivadaspremissasqueestavamatreladasaomodelo.Asquatroprincipais

premissaseram:

ØOgestortemoproblemadefinidoeumobjetivoclaro.

ØOgestortemtodasasinformaçõesdequenecessitaetodasasalternativas

bemcalculadas.

ØOscritériosparaavaliarasalternativassãodefinidoseconhecidos.

ØOgestorélógicoebuscamaximizarosresultadosdaorganização.

Essateoriateveoobjetivodeaumentararacionalidadedosgestores

encarregadosdetomardecisão,masdescreviamalorealprocessodetomadade

decisão.

Figura15.1–Modeloracional

Infelizmente,essaspremissasnãoocorremnavidareal.Comisso,apareceuum

teórico(HerbertSimon)quenostrouxeoutromodelo:odaracionalidade

limitada.Simonobservouque,narealidade,aspessoasnemsebaseiam

inteiramentenarazãonemsebaseiamtotalmentenaintuição.

Dessaforma,oautorindicouquetemoslimitesàracionalidade.Quenão

conseguimossertotalmenteracionais.6Arealidadeémuitocomplexae,portanto,

temosdeusaraintuiçãoemconjuntoàrazão.

Figura15.2–Razãoeracionalidadelimitada

ParaSimon,aracionalidadelimitadaespelhamelhorascondiçõesdetomadade

decisão:aspessoassãoracionaisapenasatécertoponto,principalmentenos

aspectosemqueelasconseguemperceberouinterpretar.7

Ouseja,temosdiversaslimitaçõesquenosimpedemdeutilizaromodelo

racional.Desdelimitaçõesnocontextodoproblema(comopoucotempo,

informaçõesimperfeitasouvariáveisdemaisparaanalisar)atélimitaçõespessoais

(comodificuldadedeanalisarmuitosdadosaomesmotempo,preconceitosetc.).

Pensebem.Quandovocêfoipelaúltimavezaoshoppingecomprouaquela

camisa(oubolsa,ouperfumeetc.)sebaseouapenasemdadosconcretos?Ouseja,

tomouumadecisãoapenasracional,ousebaseoutambémnoseugostopessoal,na

suapreferênciaanteriorounaopiniãodeumcolega?

Navidaempresarialocorrealgosemelhante.Muitasvezes,temosdedecidir

compoucosdadosounãotemoscomo“gastar”tempodemaisparaanalisar

extensamenteoproblema.

Figura15.3–Limitaçõesaomodeloracional

Vamosimaginaroutrasituação?Vocêpassouemumconcursoefoiescolhido

comogestordeumaempresapública.Essaempresanecessitadeumempréstimo

paraampliarsuasoperações.

Assim,vocêdevetomardecisõessobrequetipodenanciamentoescolher.Terá

deanalisaratendênciadosjurosnacionais,docrescimentodaeconomia

brasileira,dacompetiçãonoramoemqueaempresaatua,entreoutrosaspectos.

Muitosdessesdadosnãoestãodisponíveisousãoextremamentecomplexos,não

éverdade?Nãodáparatomarumadecisãoapenasbaseando-senosdados

concretos,nasinformaçõesdisponíveis.

Assim,navidareal,nosutilizamosdeummodelodetomadadedecisãoquese

aproximamaisdateoriadeSimon.

Ooutroconceitoéodaintuição.Aintuiçãonãoéocontrárioda

racionalidade(ouseja,airracionalidade).Elasebaseiaemexperiências

anteriores,nosnossoshábitosenosnossospensamentossubliminares(ouseja,no

nosso“subconsciente”).

Figura15.4–Intuição

Vamosvercomoessetópicojáfoicobrado?

2.(FGV–Sefaz/RJ–FiscaldeRendas/2007)Atomadadedecisõeséumelementocríticonavida

organizacional.Sobreesseprocesso,analiseasafirmativasaseguir:

I.Nomodeloracionaldetomadadedecisão,oproblemaaserresolvidodeveserdenidode

formaclaraesemambiguidades;devem-selistarasalternativasviáveiseescolheraque

resultenovalormáximopercebido.

II.Aspessoastomamdecisõesdentrodeumaracionalidadelimitada–elasconstroemmodelos

simplicadosqueextraemosaspectosessenciaisdoproblema,semcapturartodaasua

complexidade.

III.Atomadadedecisãointuitivaéumprocessoinconscientegeradopelasexpectativas

vividas,sendoumaalternativacomplementaraométodoracional.

Assinale:

a)senenhumaafirmativaforcorreta;

b)sesomenteasafirmativasIeIIforemcorretas;

c)sesomenteasafirmativasIeIforemcorretas;

d)sesomenteasafirmativasIeIIforemcorretas;

e)setodasasafirmativasforemcorretas.

EssaquestãonosserveparasabercomoaFGV“encara”essesconceitos,pois

todasasarmativasestãocorretas.Imaginoque,apósaleituradosconceitos,não

tenhasidodifícilacertaressa.OgabaritoéletraE.

15.3.Certeza,RiscoeIncerteza

Outrosconceitosquedevemosconhecerparaanossaprovaéodecerteza,risco

eincerteza,muitocobrados.Começocomacerteza.

Acertezaéasituaçãoemquetemosinformaçõesseguraseconáveisquenos

permitem“saber”quaisserãoosresultadosdasalternativasquenossãopropostas.

Assim,umasituaçãodecertezaocorrequandosabemosoqueocorreráemcada

conjuntura–emcadaalternativadeação.8Porexemplo,sesoubermosoquanto

gastamosemumamáquinapormês,podemostercertezadaeconomiaque

teremossedeixarmosdeusá-la,nãoéverdade?

Atéaquifoifácil,nãoémesmo?Masqualéadiferençaentreriscoeincerteza?

Emumasituaçãoderisco,podemoscalcularaprobabilidadedealgum

resultado!

Ouseja,temosdadossucientesquenospermitemteruma“ideia”doresultado

decadaalternativa.Quandoummédico,porexemplo,nosdizqueumacirurgia

tem90%dechancesdesucesso,elenosinformaoriscodesseprocedimento.

Assim,eletemohistóricodediversoscasosdepessoasemumasituação

semelhanteàdanossaeoresultadodecadaumadessascirurgias.Comessesdados,

elepodecalcularaprobabilidadedesucessodanossacirurgia,ok?

Comaincerteza,essesdadosnãoexistem!Assim,nãoconseguimoscalculara

probabilidadedeumadecisãoserfavorávelounão.Muitasvezes,nãotemosacesso

aosdadosquegostaríamosemumadeterminadasituação.

Assim,temosdenosutilizardanossaintuição,denossasexperiênciaspassadas,

parabuscaramelhordecisãopossível.

Figura15.5–Certeza,riscoeincerteza.

Vamosvercomoessetemapodesercobrado?

3.(Esaf/RFB/ATRFB/2009)Umaadequadacompreensãodotema‘processodecisório’implicater

comocorretasasseguintesafirmações,exceto:

a)umproblemacujasoluçãonãodispõedealternativasjáestá,porsisó,resolvido;

b)umúnicoproblemapodeserpercebidodeformasdiferentespordiferentesindivíduos;

c)oprocessoracionaldetomadadedecisãonãoexcluiousodasubjetividade;

d)mesmoamelhordecisãopodeacarretarumresultadodesastroso;

e)atomadadedecisãoemequipeépreferívelàtomadadedecisãoindividual.

Aprimeiraalternativaestácorreta,poisquandonãoexistesoluçãopossível

paraumproblemaeleestá,naprática,“resolvido”,ouseja,nãoháoquefazer.

AletraBtambémestácertaeébemintuitiva,nãoémesmo?Umproblemapode

servistocomosimplesparaumapessoaemuitocomplexoparaoutra,

dependendodesuaexperiência,conhecimentosetc.

Mesmoemumprocessoracionaldetomadadedecisões,nuncatemostodasas

informações.Assim,asubjetividadesempreestápresente,mesmoqueemumgrau

pequeno.

NocasodaletraD,podemostersomentealternativas“ruins”aonossoalcance.

Nessescasos,teríamosdeescolhera“menos”negativa.JáaletraEestáincorretae

éonossogabarito.Umatomadadedecisãoindividualpodeserindicadaemcasos

emquetemospoucotempo,porexemplo.

15.4.ProcessoLineareSistêmico

Oprocessodecisóriopodeservistoporduasóticas:adopensamentolinearea

dopensamentosistêmico.Opensamentolinear(oucartesiano)éumateoriamais

antiga,quederivadeAristóteles.

Dentrodessamaneiradepensar,osproblemasdevemserdecompostosem

diversaspartesecadaáreaanalisadaemseparado.Édaíqueasespecializaçõesnas

diversascarreirasapareceram.

Umgestorquetrabalhenaáreadenançastenderáasósepreocuparcomos

aspectosnanceiros,porexemplo.Assim,teráemmenteosproblemaseobjetivos

daquelaáreaespecíficadaorganização.

Oproblemaéqueumaorganizaçãoécompostadediversasáreas,quesão

interligadaseinterdependentes.Assim,umadecisãonãopodeseateraumsó

aspecto,poisnãoadiantasomenteumaáreatersucesso.

Osucessodaempresadependedetodasasáreas“funcionarem”.Esteéo

conceitodepensamentosistêmico–osproblemassãointer-relacionadoseo

ambienteécomplexo.

Figura15.6–Pensamentolinearesistêmico

Portanto,naatualidade,opensamentosistêmicoémaisvalorizado,pois

englobaasdiversasmudançasnoambienteexternoeacomplexidadedaoperação

deumaorganizaçãomoderna.

Vamosverumaquestãoqueabordaisso?

4.(FGV/Badesc/AnalistaAdm./2010)Comrelaçãoaoprocessodetomadadedecisãoorganizacional,

analiseasafirmativasaseguir.

I.Oprocessodecisórioélinear.

II.Oprocessodecisóriodependedascaracterísticasindividuaisdotomadordedecisão.

III.Oprocessodecisóriodependedocontextoespecíficodecadasituação.

Assinale:

a)sesomenteaafirmativaIestivercorreta;

b)sesomenteaafirmativaIestivercorreta;

c)sesomenteaafirmativaIIestivercorreta;

d)sesomenteasafirmativasIeIIestiveremcorretas;

e)setodasasafirmativasestiveremcorretas.

Aprimeirafraseestáincorreta,poisoprocessodecisóriodentrodocontexto

organizacionaldevesersistêmico,enãolinear.Jáasduasoutrasarmativasestão

perfeitas.

Oprocessodecisóriodependetantodascaracterísticasdotomadordedecisão

quantodasituaçãoespecífica.OgabaritoéaletraD.

15.5.TécnicasdeAnáliseeSoluçãodeProblemas

Qualquerproblemanecessitadeumadecisãoparaserresolvido,masnemtoda

decisãobuscaresolverumproblema.9Muitasdecisõesserãoutilizadaspara

abordarumanecessidade,uminteresseouumaoportunidade.

Dessaforma,oprocessodecisórioécompostoporcertasetapas:identicação

dasituaçãooudoproblema,diagnósticodasituação,desenvolvimentoeavaliação

dasalternativaseaescolhadaalternativa.

Figura15.7–Processodetomadadedecisão

Aidenticaçãodoproblemaéoprimeiroestágio.Nessafase,oproblema(ou

oportunidade)passaaserpercebido.Ouseja,toma-seconhecimentodaexistência

dealgumimpasse,algumaescolhaquedeveráserfeita.

Jáodiagnósticodoproblemaéumadasfasesmaisimportantes.Nesse

momento,oadministradorterádedenirseusobjetivosequaissãoasprincipais

causasquegeraramoproblema.Decertamaneira,nessafaseocorreo

“mapeamento”doproblema.

Vamosverumaquestãoqueabordaessetema?

5.(FGV/Sefaz/RJ–FiscaldeRendas/2009)Emumprocessodecisório,umaoportunidadediz

respeitoà(s)seguinte(s)fase(s):

a)identificaçãodasituação;

b)diagnósticodasituação;

c)desenvolvimentodeoportunidades;

d)avaliaçãodealternativas;

e)seleçãoeimplementação.

Pararesolvermosessaquestãotemosdereetir:setivermosumaoportunidade

(saiuoeditaldeumconcursonovo,porexemplo)nasmãos,emquefasenos

encontramos?

Essaoportunidadeestárelacionadacomaidenticaçãodasituação.Ouseja,no

momentoemqueumaoportunidadeaparece,estamosnafasedaidenticação

dessanossasituação.OgabaritoéaletraA.

Continuando,paraotrabalhodemapeamentodeumproblematemosumasérie

deferramentas,queanalisaremosaseguir.

15.6.FerramentasdeDesenvolvimentodeAlternativas

15.6.1.BrainstormingeBrainwriting

Umadasprincipaisferramentasparaodesenvolvimentodealternativaséo

brainstorming,ou“tempestadedeideias”.Oobjetivodessaferramentaécriarum

ambienteemqueascríticassejam“suspensas”equeexistam,portanto,um

encadeamentodeideias.

Dessamaneira,umgrupodepessoassereúneemumambienteecomeçaa

“jogarsoluçõesaovento”,ouseja,cadapessoapodesugerirqualquerideia,por

mais“louca”quepossapareceràprimeiraanálise.

Quandoumnúmerorazoáveldealternativastenhasidogerado,esseprocessose

interrompe.Portanto,opróximopassoacontececomaseleçãodasmelhores

alternativasgeradasnobrainstorming.

DeacordocomMaximiniano:10

Osprincípiosbásicosnageraçãodeideiassãodois:asuspensãodojulgamentoeareaçãoem

cadeia.Essesdoisprincípiosasseguramquepessoasseexprimamlivremente,semreceiodecríticas,

equeasideiasseassociemegeremnovasideias,numprocessoemqueoobjetivoéassegurar

primeiroaquantidadedealternativas.

Vamosvercomoissojáfoicobrado?

6.(FCC/TCE/GO/Desenv.org./2009)Obrainstorming,outempestadecerebral,éumatécnicade

trabalhoemgruponaqualosparticipantesapresentamideiasdeformaespontâneasobreum

determinadoproblema,e,numaprimeiraetapa,énecessárioenfatizaraqualidadedasideiase

nãoaquantidade.

Afraseestáincorreta,poisnaprimeiraetapadobrainstormingoimportanteé

buscaraquantidadedasideias,enãosuaqualidade.

Portanto,oconceitoprincipaldobrainstormingécriarumprocessoemqueas

pessoassejamlivresparaemitirsuasideias,semseremcriticadasporisso,emum

curtoespaçodetempo.11

Entretanto,nemsempreissoacontecedemaneirafácil.Existeumaalternativa

queéderivadadobrainstormingequecostumafuncionarbem:obrainwriting.

Nessaferramenta,asideiasnãosãopassadasdemodooral,esimatravésdepapéis

quesãotrocadosentreaequipe,semqueaspessoasquemsabendoquemescreveu

cadasugestão.

Vamosimaginarumasituação.Vocêacaboudesercontratadoparauma

empresaeemseuprimeiromêsdeserviçoéchamadoparaumareuniãocomtodos

osseuschefes.Nessareunião,éapresentadoumproblemaeodiretorpedea

opiniãodaspessoaspresentesnasala.

Pormaisquetenhaumaideiaquesejainteressante,vocêvaipensarduasvezes

antesdefalar,nãoémesmo?Omedodesercriticadoouridicularizadoserá

grande.Dessamaneira,muitasvezesobrainstormingnãofuncionaporessereceio

dacrítica.

Comobrainwriting,issonãoocorre.Comoassugestõessãopassadasdemodo

anônimo,aspessoasnãocamsabendoquemdisseoque,facilitandoageraçãode

ideias.

15.6.2.AnálisedoCampodeForças

Oconceitoportrásdessaferramentaédequeumcomportamentoqualqueré

derivadodoequilíbrio,ounão,deduasforçasopostas.12Dessamaneira,para

cadasituaçãoexistiriamasforçaspropulsoras,queestimulariamaquele

comportamento,easforçasrestritivas,queseoporiamàquelecomportamento.

Vamosimaginarasituaçãodevocês.Parapassaremumbomconcurso,é

necessáriomuitoesforço,nãoémesmo?Dessaforma,todosvocêsdevemter

encontradoforçaspropulsoraserestritivasemsuasjornadasdeconcurseiros.

Sevocêtemumanamorada(ounamorado)queécompreensivaelhedáforça

emseumomentodeestudo,elaéumaforçapropulsora.Jáaqueleamigoquete

ligatodahoraparatomarumacervejaéumaforçarestritivaaoseuesforçode

preparação,nãoéverdade?

OqueLewindisse,então,équeseumaforçaformaiordoqueaoutra,

inuenciaráocomportamentonessadireção.Portanto,emumaempresa,

utilizamosessaferramentaparaentenderquaissãoosfatoresfavoráveisequaissão

contráriosaqualquerdecisãoquenecessitamostomar.

Dessamaneira,nãosóentendemosquaissãoasforçasqueoperamemuma

situaçãocomofacilitamosotrabalhodesuperarasresistênciasdaspessoasou

gruposcontrários.13

Figura15.8–Análisedocampodeforças

15.6.3.DiagramadeÁrvoredeDecisão

Essaferramentabuscadetalharedesdobrarcadadecisãopossível,deformaa

quepossamosanalisartodasaspossibilidadeseseusimpactos.Deacordocom

Sobral,essediagramapermiteavisualizaçãográcadasalternativas,naqualcada

umadelasérepresentadacomoumramodeumaárvore.14

Essaferramentaéútilquandoenfrentamosdecisõescomplexasemambientesde

incerteza,comoenvolvimentodeumasérieprogressivadedecisõespossíveis.

Portanto,essaferramentatemessenomepormostrarasdecisõescomouma

hierarquiadenósinternoseexternosconectadosporramos,semelhantesauma

árvore.

Assim,nessediagrama,buscamosentenderquaisseriamosimpactosdecada

alternativadedecisão,demodoquepossamosavaliarqualseriaamelhor

alternativaemcadasituação.

Vamosvercomoessetópicojáfoicobrado?

7.(Esaf/STN/Desenv.Institucional/2008)Obrainstorming,tambémconhecidocomotempestade

cerebraloutempestadedeideias,éummétodoqueproporcionaumgrandenúmerodeideias,

alternativasesoluçõesrápidas.Sendoumexcelenteexercíciodedebatecriativoeinovador,

possibilitaumgrandeusodacriatividade,constituindo-seemtécnicabastanteaplicávelà

seguintefasedoprocessodecisório:

a)identificaçãodoproblemaouoportunidade;

b)diagnóstico;

c)geraçãodealternativas;

d)escolhadeumaalternativa;

e)avaliaçãodadecisão.

Comovimos,obrainstormingserveparagerarmosumnúmerograndede

alternativas,demodoapodermosselecionarasmelhorespossibilidades.Portanto,

érelacionadocomageraçãodealternativas.OgabaritoéaletraC.

QuestõesComentadas

8.(Cespe/Embasa/Administração/2010)Asdecisõesprogramadasmaximizamanecessidadede

gerentesouadministradoresexercitaremseuscritériosparatomadadedecisão.

Asdecisõesdifíceisequedemandammuitodosgerentessãoasnão

programadas!Lembre-sesempredisto:decisãopadronizada,“manualizada”,é

programada!Decisãonova,difícilequedemandaumaanálisemaiscomplexaé

nãoprogramada.Ogabaritoéquestãoerrada.

9.(Cespe/Embasa/Administração/2010)Apolíticadaempresaservedeguiaparaatomadade

decisõesprogramadas.

Apolíticadaempresa(assimcomoasregraseosprocedimentos)éumdos

balizadoresdaatuaçãodeumgestorparaatomadadeumadecisãoprogramada,

poisforneceum“norte”paraadecisão.Ogabaritoéquestãocorreta.

10.(FGV/Senado/Administrador/2008)Noprocessodetomadadedecisões,osadministradores

devempesaralternativas,muitasdasquaisenvolvemacontecimentosfuturosdifíceisde

seremprevistos.Porisso,assituaçõesdetomadadedecisãosãofrequentementeclassicadas

emumcontinuumquevaidacerteza(altamenteprevisível)àturbulência(altamente

imprevisível).ConformesugeridoporJamesA.StonereEdwardR.Freeman(1992),orisco

refere-seàcondiçãoparatomadadedecisão:

a)naqualosadministradoresenfrentamsituaçõesexternasimprevisíveisounãotêmasinformações

necessáriasparaestabeleceraprobabilidadededeterminadoseventos;

b)emqueosadministradorestêminformaçõesprecisas,mensuráveiseconáveissobreosresultados

dasváriasalternativasqueestãosendoconsideradas;

c)emqueosadministradoresconhecemaprobabilidadedequeumadeterminadaalternativaleveaum

objetivoouresultadodesejado;

d)queocorrequandoasmetasnãosãoclarasouquandoomeioambientemudamuitorapidamente;

e)naqualosadministradorestêminformaçõesinsucientes,porémconáveis,sobreosresultados

daquiloqueestásendolevadoemconsideraçãonomomentodatomadadedecisões.

Agoracoumaisfácil,nãoémesmo?Aquestãopedeaopçãoqueserelaciona

aorisco.

Aprimeiraalternativaestásereferindoaoconceitodeincerteza.Jáasegunda

alternativaserefereaoconceitodecerteza.Onossogabaritoé,portanto,aletraC.

Comovimos,oriscoéassociadoàsituaçãoemqueogestorpodecalculara

probabilidadedesucessodeumaalternativa.

QuestõesPropostas

11.(Esaf/STN/Desenv.Institucional/2008)Merarepresentaçãográcadeumatabeladedecisões,

consistindoemumahierarquiadenósinternoseexternosconectadosporramos,aseguinte

técnicapermitevisualizartodososresultadosdasdecisõesquepodemsertomadasparalidar

comsituaçõesincertas.Emfacedoexposto,indiqueaopçãocorrespondente.

a)Análisedocampodeforças.

b)Árvorededecisões.

c)Brainwriting.

d)PrincípiodePareto.

e)DiagramadeIshikawa.

12.(Esaf/MPOG/EPPGG/2009)Nocontextoorganizacional,aoparticiparmosdeumprocesso

decisório,éincorretoafirmarque:

a)aracionalidade,porsisó,insinuaserpossívelodomíniodefatoresnãocontroláveiseaeliminaçãode

riscoseincertezas;

b)emnossosdias,aoladodaracionalidadegerencial,tambémseaplicamelementoscomoosaspectos

comportamentais,osensocomum,ojuízodaspessoaseanegociaçãopolítica;

c)aestruturadoscanaisdeinformaçãoededisseminaçãodoconhecimentoexercegrandeinuência

sobreoprocessodecisório;

d)obrainstormingéatécnicaadequadaparaidentificarproblemas;

e)nonívelsuperior,háopredomíniodasdecisõesestratégicas,quetratamdasligaçõesentrea

organizaçãoeoambienteexterno.

13.(Esaf/STN/Desenv.institucional/2005)Hádoismodelosbásicosqueidenticamo

comportamentoadotadoemumprocessodetomadadedecisão.

Indiqueaopçãoqueidentificacorretamenteosdoismodelos.

a)Ocomportamentoracionaléoquesegueumprocessonãototalmenteconscienteesebaseiana

sensibilidadeepercepção,enquantoocomportamentointuitivoéoquesegueumaordemlógicaese

baseiatotalmenteeminformações.

b)Ocomportamentoracionalaplica-seaambientescomplexosedinâmicos,emqueasinformaçõessão

limitadas,enquantoocomportamentointuitivoseaplicaaambientessimpleseestáveis,ondehá

grandedisponibilidadedeinformações.

c)Ocomportamentoracionaléoquesegueumaordemlógicaesebaseianasensibilidadeepercepção,

enquantoocomportamentointuitivoseaplicaaambientescomplexosedinâmicosemquehágrande

disponibilidadedeinformações.

d)Ocomportamentoracionalaplica-seaambientessimpleseestáveis,emqueasinformaçõessão

limitadas,enquantoocomportamentointuitivoéoquesegueumaordemlógicaesebaseia

totalmenteeminformações.

e)Ocomportamentoracionaléoquesegueumaordemlógicaesebaseiatotalmenteeminformações,

enquantoocomportamentointuitivoéoquesegueumprocessonãototalmenteconscienteese

baseianasensibilidadeepercepção.

14.(Cespe/Embasa/Administração/2010)Viésespsicológicos,pressõesdetempoerealidades

sociaisinfluenciamasdecisõesreais.

15.(Cespe/Embasa/Administração/2010)Aeciênciaorganizacionaléfacilitadapelousode

decisõesnãoprogramadas.

16.(Cespe/Embasa/Administração/2010)Nauniversidade,odesenvolvimentodenovosrecursos

didáticosparausonassalasdeaulaéumexemplodedecisãoprogramada.

17.(Cespe/Basa/Administração/2010)Oconceitoderacionalidadelimitadaevidenciaa

impossibilidadedeseconceberaorganizaçãocomoumempreendimentoestritamente

racional.

18.(Cespe/MS/Administrador/2010)Considereaseguintesituaçãohipotética.Emumhospital

privado,ogestordecidiuadquirirumsistemademonitoramentodecâmerasecatracas

eletrônicasealterartodososprocedimentosrelativosàsegurança.Taldecisãoocorreuapósa

denúnciadeduaspacientesdequeteriamsidomolestadasporummédico.Apósapuraçãodos

fatospelaautoridadepolicial,concluiu-sequesetratavadeumfalsomédicoqueteveacessoa

essaspacientesemumúnicodia,noreferidohospital.Apesardenuncaterocorridotalfato

anteriormente,taisdecisõesforamconsideradasimportantes.Nessasituação,adecisãode

adquirirmaisumitemparaopatrimônio,alémdaalteraçãodosprocedimentosdesegurança,

caracteriza-seporserprogramada.

19.(Cespe/Hemobras/Administrador/2008)Considerequeumdiretordeumaorganizaçãodeva

decidiracercadaconstruçãooualugueldeumnovodepósitodemercadoriasparaa

organização,masnãodispõedetodasasinformaçõesarespeitodospossíveisimpactosquea

atualcrisenanceirainternacionalpodeprovocarnosnegócios.Nessecaso,oprocesso

decisóriodessediretortemaincertezacomoumacaracterística.

20.(Cespe/Hemobras/Administrador/2008)Considerequeumdiretoroptepelautilizaçãodeum

grupodeempregadosnatomadadedecisão,acercadeassuntocrucialparaaorganização.

Nessecaso,suaopçãotrarácomovantagenstermaisinformações,maispontosdevistaa

respeitodoassuntoemaiorcomprometimentocomadecisãotomada.

21.(Cespe/MPS–Administrador/2010)Oprocessodecisórioécompostopelasseguintesfases:

identicaçãodoproblemaouoportunidade,diagnóstico,geraçãodeopçõeseescolhadeuma

opção.

22.(Cespe/Previc/Tec.adm./2011)Umainformaçãogerencialservedeinsumoparaatomadade

decisãoporpartedegerentes,devendo,porisso,suainterpretaçãoserorientadaaocontexto

daorganização.Esseprocessodeveocorrerdeformatotalmenteisentaesemconsiderara

naturezadaorganização.

23.(Cespe/Correios/Administrador/2011)Osaspectosemocionaiseafetivosinuenciamo

processodecisórioerestringemacapacidaderacionaldosindivíduos.

Gabaritos

1.B

9.C

17.C

2.E

10.C

18.E

3.E

11.B

19.C

4.D

12.D

20.C

5.A

13.E

21.E

6.E

14.C

22.E

7.C

15.E

23.C

8.E

16.E

Bibliografia

Chiavenato,Idalberto.AdministraçãoGeralePública.2aed.SãoPaulo:Elsevier,

2008.

_______.IntroduçãoàTeoriaGeraldaAdministração.8aed.RiodeJaneiro:

Elsevier,2011.

Daft,RichardL.Management.Mason:Thomson,2005.

Junior,IsnardMarshall,AglibertoAlvesCierco,AlexandreVarandaRocha,

EdmarsonBacelarMota,eSérgioLeusin.Gestãodaqualidade.9aed.Riode

Janeiro:FGV,2008.

Maximiniano,AntonioCesarAmaru.IntroduçãoàAdministração.4aed.SãoPaulo:

Atlas,1995.

Robbins,StephenP.,eMaryCoulter.Administração.5aed.RiodeJaneiro:

Prentice-Hall,1998.

Sobral,Felipe,eAlketaPeci.Administração:teoriaepráticanocontextobrasileiro.

SãoPaulo:Pearson-Prentice-Hall,2008.

Capítulo16

GestãoporCompetências

Agestãoporcompetênciasestáinseridaemumcontextodemudançasno

modelodegestãodepessoas.Tradicionalmente,osmodelostradicionaisdeGP

estavambaseadosnocontrolecomoformaderelacionamentoentreaorganização

eoindivíduoefocadosemescolherapessoacertaparaolugarcerto.1

Entretanto,oambienteemqueasorganizaçõesoperammudoumuitonas

últimasdécadas.Aconcorrênciacrescenteeademandaporprodutoseserviços

commaiorvaloragregadodemandampessoascomcapacidadedeiniciativae

maiorcriatividade.

Alémdisso,asnovasestruturasorganizacionais,comoasorganizaçõesemredee

virtuais,sãomaisdescentralizadas.Paraessasempresas,sãonecessáriaspessoasque

saibamtomardecisõesequeconsigamexecutarumautogerenciamento.

Dentrodessenovocontexto,asorganizaçõespassaramarepensaramaneira

comocontratavamegerenciavamseusempregados.Avisãodequeoimportante

erasomenteo“estoque”deconhecimentosehabilidadesdoempregadocaiupor

terra.

Oconceitodecompetênciasbuscaentendereconheceraaplicaçãodesses

conhecimentosehabilidadesnocontextodotrabalho,ouseja,naprática.Assim,

aliadoaoconceitodecompetênciaestáode“entrega”.Devemossaberoquanto

cadapessoaagregadevalornaorganização.DeacordocomLeBoterf,2

oconceitodecompetênciaestáassociadoàideiadevaloreentregaadeterminadocontextode

formaindependentedocargo,istoé,apartirdaprópriapessoa.

Então,parapodermosentendermelhorcomofuncionaagestãopor

competências,devemossaberoquesãocompetências!

Grandepartedosautoresatualmenteconsideracompetênciaosomatóriode

conhecimentos,habilidadeseatitudes(CHA–paraajudaraxar!)nocontexto

dotrabalho,ouseja,acombinaçãodestesparaobterumbomresultadoprático,

queadicionevaloràorganização.

Vejacomoessetemajáfoicobradonaseguintequestão:

1.(Cespe/TCU/ACE/GestãodePessoas/2008)Competênciasrepresentamcombinaçõessinérgicas

deconhecimentos,habilidadeseatitudes,expressasnodesempenhoprossionalem

determinadocontexto.

AfraseestácorretaeéretiradadostrabalhosdeCarbone.3Essaéumadenição

decompetênciasbastanteatual,queconsideracompetêncianãosomenteum

estoquedeconhecimentos,habilidadeseatitudes,masaaplicaçãodessesCHAsno

ambientedetrabalhoreal,trazendoresultadospráticosnocontextoda

organização.

Vamosentendermelhoroquesãooselementosdagestãoporcompetências?

Conhecimentos–Estárelacionadoaosaberdapessoa!Sãotodasas

informações,osdadoseconceitosqueoindivíduoarmazenouequepodemser

usadosporeleemseutrabalho.

Habilidades–Éosaberfazer!Acapacidadequeapessoatemdeutilizaros

conhecimentosquepossuinaprática.Umaclassicaçãomuitoconhecidade

habilidadeséadeKatz,quemencionaashabilidadestécnicas,humanase

conceituais.

Atitudes–Éoquererfazer!Estãorelacionadasaomodocomoapessoalida

comseussentimentosemaneirasdepensarecomoestesafetamseu

comportamentoemrelaçãoaotrabalhoeseuscolegas.Existempessoasque

gostamdeliderar,decomandaralgumaação.Outrasnãosesentemàvontade

nessepapel,eterãodiculdadeemassumirqualquerposiçãodecheaemuma

organização.4

Dessaforma,ascompetênciassãoosomatóriodessesfatores:

Figura16.1–Elementosdascompetências

Imagineumcasoprático:penseemumsimplesvendedordeautomóveis.Ele

precisaterconhecimentosobreoscarrosqueestarávendendo(consumo,cores,

modelos,acessóriosetc.),habilidadesdecomunicaçãoenegociaçãoparaque

possaargumentarcomoconsumidoreumaatitudepositivadiantedotrabalho

devendaseemrelaçãoaoatendimentoaoclienteparaquepossatersucessoem

suaprofissão,nãoémesmo?

Dequeadiantariaumvendedorquesabemuitosobreautomóveis,temuma

excelentehabilidadedesecomunicar,masestásempredemauhumoroudetesta

trabalharcomovendedor?

Assim,acompetênciaexistequandocombinamosconhecimentos,habilidadese

atitudesparatrazerresultadosconcretosparaaorganizaçãoemquetrabalhamos.

16.1.CompetênciasHumanaseOrganizacionais

Atéaquifalamosdascompetênciaspessoais,masoutrosautoresligaramo

conceitodecompetênciaaosgruposouorganizações,comoPrahaladeHamel.5

Essesautoresconsideramcompetência(corecompetences,oucompetências

essenciais)acapacidadedasempresasdeagregarvalorqueépercebidopelos

clientes,demododifícildeserimitadoporseusconcorrentes,gerandoassim

vantagemcompetitiva(acapacidadedaPetrobrasdeextrairpetróleodeáguas

profundasseriaumexemplodessascompetências).

Então,ascompetênciaspodemserclassicadasemhumanas(ligadasacada

indivíduo)eorganizacionais(queserelacionariamàcapacidadedaempresa),

sendoqueascompetênciashumanasafetamasorganizacionais,evice-versa.6

Existemváriasoutrasclassicaçõesdecompetências.Vejamosalgumasdasmais

importantes:

Ascompetênciashumanaspodemserclassicadasdeacordocomopapelque

apessoaocupanaorganização.Ascompetênciashumanasseriamentãotécnicas

(relacionadasàsposiçõesoperacionaisoudeassessoria,quenãoexerceriam

formalmentealiderança)egerenciais(necessáriasemocupantesdecargosde

chefiaedireção).

Outraclassicaçãodascompetênciashumanas(ouindividuais)éadivisãoentre

ascompetênciasgerais(quetodosfazem)eascompetênciasespecíficas(quesão

relacionadasàstarefasecargosdaspessoasdentrodaorganização).

Jáascompetênciasorganizacionaispodemserclassicadascomobásicas

(ligadasaofuncionamentodaorganização,masquenãodiferenciariamaempresa

deseusconcorrentes)easessenciais(asquerepresentamaspectosque

diferenciariamaempresadeseusconcorrentes–corecompetences).7

Outraclassicaçãocomumdascompetênciasorganizacionaiséemrelaçãoà

importânciadecadaumacomopassardotempo.Elasseriamclassicadascomo

emergentes(quetendemasermaisimportantesnofuturo),declinantes(que

perderãoimportânciacomotempo),estáveis(manteriamsuarelevânciaatual)e

transitórias(queseriamrelevantessomenteemumperíodo,comoemumacrise).

OutrosconceitosdecompetênciasorganizacionaissãodescritosporMills:8

Competênciasessenciais–fundamentaisparaasobrevivênciadaorganizaçãoe

centraisemsuaestratégia;

Competênciasdistintivas–reconhecidaspelosclientescomodiferenciaisem

relaçãoaoscompetidores;conferemàorganizaçãovantagenscompetitivas(vejam

queéomesmoconceitoutilizadoanteriormenteparaascompetênciasessenciais!

Issoéutilizadonasprovasparaconfundirocandidato);

Competênciasdeunidadesdenegócio–pequenonúmerodeatividades-chave

(entretrêseseis)esperadaspelaorganizaçãodasunidadesdenegócio;

Competênciasdesuporte–atividadesqueservemdealicerceparaoutras

atividadesdaorganização.Porexemplo:aconstruçãoeotrabalhoecientesem

equipespodemtergrandeinuêncianavelocidadeequalidadedemuitas

atividadesdentrodaorganização;

Capacidadedinâmica–condiçãodaorganizaçãodeadaptarcontinuamente

suascompetênciasàsexigênciasdoambiente.

Vamosverumaquestãorecente?

2.(FGV/Sefaz/RJ–FiscaldeRendas/2009)Emgestãoporcompetências,acondiçãodaorganização

deadaptarcontinuamentesuascompetênciasàsexigênciasdoambienteéumacaracterística

das:

a)competênciasessenciais;

b)competênciasdistintivas;

c)competênciasdeunidadesdenegócio;

d)competênciasdesuporte;

e)capacidadesdinâmicas.

DeacordocomadeniçãodeMills,acondiçãodeumaorganizaçãodese

adaptarcontinuamenteàsmudançasdoambienteexternosechamacapacidade

dinâmica.Portanto,ogabaritoéaletraE.

Bom,agoraquevimososconceitosdecompetência,vamosconhecercomoa

gestãoporcompetênciasfunciona.

DeacordocomCarbone,9asempresastêmadotadoagestãoporcompetências

comomodelodegestão,visandoorientarseusesforçosparaplanejar,captar,

desenvolvereavaliar,nosdiferentesníveisdaorganização(individual,grupale

organizacional),ascompetênciasnecessáriasàconsecuçãodeseusobjetivos.

Basicamente,agestãoporcompetênciaséummodelodegestãoque,

considerandoaestratégiadaorganizaçãocomouma“bússola”,busca

identicaredesenvolverascompetênciasnecessáriasparaqueelapossa

atingirseusobjetivosestratégicos.

Dessamaneira,umdosobjetivosprincipaisnagestãoporcompetênciasé

conseguiralinharascompetênciasindividuaisdecadafuncionáriocoma

estratégiaorganizacional.

16.2.EtapasdaGestãoporCompetências

Agestãoporcompetênciaséumprocessocontínuoe,segundoCarbone,10tem

comofaseinicialaformulaçãodaestratégiadaorganização(missão,visãoe

objetivosestratégicos).Apartirdessemomento,poderíamosdenirquaisseriam

ascompetênciasnecessáriasparaqueaorganizaçãoalcanceseusobjetivos.

AseguirpodemosverasetapasdaGestãoporCompetências:

Figura16.2–Etapasdagestãoporcompetências

Entretanto,algumasbancastêmconsideradoqueaetapainicialéo

mapeamentodecompetências,ouatéaidentificaçãodascompetências!

Vamosvercomoessetemajáfoicobradopelasbancas?

3.(FCC/Defensoria/SP/Administrador/2010)Agestãodecompetênciastemcomoetapainicial:

a)pesquisarascompetênciasmaisvalorizadaspelomercado;

b)desenvolverascompetênciasnecessáriasparaosucessodaorganização;

c)estabelecerosobjetivosemetasaseremalcançadospelaorganização;

d)aidentificaçãodalacunadecompetênciasdaempresa;

e)captarascompetênciasinternasdisponíveisnaorganização.

Nestaquestãode2010,aFCCpediuaetapainicialdagestãoporcompetências

econsideroucorretaaalternativaD.Essaalternativaserelacionacoma

identicação

dogapdecompetências,ouseja,apósomapeamentodas

competênciasdaorganização.

Entretanto,amesmaFCCjáconsiderouomesmotemademaneiradiferente.

Vamosàquestão?

4.(FCC/PGE/RJ/Auditor/2009)Agestãoporcompetências,alternativaaosmodelosgerenciais

tradicionalmenteutilizados,constituiumprocessocontínuoquetemcomoetapainicial:

a)aimplementaçãodeinstrumentosdegestãododesempenho;

b)aformulaçãodaestratégiaorganizacional;

c)adefiniçãodeindicadoresdedesempenhononívelcorporativo;

d)aidentificaçãodoseugapoulacunadecompetências;

e)oplanejamentodeaçõesparacaptaredesenvolverascompetênciasnecessárias.

Jánessaquestão,aFCCconsiderouoconceitodeCarbonecomocorreto,ou

seja,dequeagestãoporcompetênciascomeçacomaformulaçãodaestratégia

organizacional.

VejaqueaalternativaDéexatamenteamesmaarmaçãoqueaFCCtinha

consideradocorretanaquestãoanterior.Masnessaprovaabancaconsideroua

alternativaerrada.Vidadeconcurseiroédifícil,nãoé?Assim,ogabaritoda

questãoéaletraB.

Mas,mestre,oqueeuescolhonahoradaprova?

Pessoal,achoumabsurdoasbancasmudaremtodahoradeposicionamento,

masinfelizmenteéanossarealidadenomundodosconcursos.Nahoradaprova,

antesdetudo,analisemseexistemaisdeumaalternativapossívelnaquestão.

Seefetivamenteexistir,vocêterádeescolherumadelas!Euescolheriaa

deniçãodoCarbone,ouseja,dequeagestãoporcompetênciascomeçacoma

formulaçãodaestratégiaorganizacional,poistemsidoamaiscobrada.

Vamosvoltaraonossoconteúdo?

Afasedomapeamentodecompetênciaséumaetapaimportantíssimana

gestãoporcompetências,poiséessafasequeirásubsidiarasaçõesdecaptaçãoe

desenvolvimentodecompetências,bemcomoasetapasdeavaliaçãoede

retribuiçãoaosfuncionários.

Omapeamentodecompetênciasbasicamentevisaidenticarosgaps(ou

lacunas)decompetênciasnaorganizaçãoatravésdaidenticaçãodas

competênciasnecessáriasparaqueamesmaatinjaseusobjetivosestratégicoseda

análisedascompetênciasjáexistentesnaorganização,deformaqueessesgaps

sejamreduzidosoueliminados.11

Dessamaneira,aorganizaçãopoderáprogramartodasasaçõesde

desenvolvimentoecaptaçãodecompetências.

Opassoinicialdomapeamentoéaidenticaçãodascompetências

necessáriasparaqueaorganizaçãoatinjaseusobjetivosestratégicos.Esse

trabalhoéfeitocomautilizaçãodediversasferramentas,eemgeraléencadeado

destemodo:12

ØNormalmenteseiniciaotrabalhocomumaanálisedosprincipais

documentosestratégicos,comoamissão,avisãoeosobjetivosestratégicos,

bemcomooutrosimportantesdocumentos(políticas,normasetc.).

ØSãorealizadasentrevistascompessoas-chavenaorganização,demodoque

sepossamedirapercepçãodecadaumsobreasnecessidadesdecompetência

internanaorganização,ou,seotempoforcurto,érealizadaumaentrevista

coletiva(focusgroup,ougruposfocaisoudediscussão),bemcomoa

utilizaçãodequestionáriosestruturados.

ØOutraferramentapodeseraobservaçãodosprópriosfuncionáriosno

ambientedetrabalho,deformaquesepercebamcertosdetalhesquenão

foramcaptadosnostrabalhosanteriores.

Osegundopassodomapeamentodecompetênciaséoinventáriodas

competênciasjáexistentesnaorganização,quenormalmenteseutilizados

instrumentosdeavaliaçãododesempenhodaorganização(instrumentoscomoa

avaliação360ºouoBalancedScorecard,porexemplo)paraquesepossaentão

mediradistânciaentreascompetênciasnecessáriaseasexistentesatualmentena

organização.13

Portanto,ogapnadamaisédoqueadiferençaentreascompetências

necessáriaseasexistentes!

Comessegap(oulacuna)mapeado,pode-seiniciaroplanejamentodo

desenvolvimentoedacaptaçãodessascompetênciasqueserãonecessáriaspara

sereduziroueliminarogapdecompetências.

Aseguirpodemosverasfasesdomapeamento:

Figura16.3–Fasesdomapeamento

Vejaumaquestãoquecobraessetema:

5.(Cespe/TCU/ACE/GestãodePessoas/2008)Omapeamentodecompetênciasenvolve

macroetapas,entreasquaissepodemcitaraidenticaçãodascompetênciaseaelaboraçãode

planodedesenvolvimentodecompetências.

Aquestãoestácorreta.Esseplanodedesenvolvimentodecompetênciasdeverá

possibilitaraumentaracapacidadedosfuncionáriosdedominarascompetências

necessáriasàorganização!Entreasdecisõesquedeverãosertomadasnessafase,

incluem-seaanáliseeaseleçãodasmelhoresalternativasdeaprendizagem

disponíveisnomomento.

Portanto,aaplicaçãodosresultadosdomapeamentopermitiráoplanejamento

dasaçõesdecaptaçãoeodesenvolvimentodascompetências,deformaquea

organizaçãoconsigareduziralacunaentreascompetênciasexistenteseas

necessárias.14

Cabelembrarque,comooambienteexternodasorganizaçõesestásempre

evoluindo,ascompetênciasnecessáriasestarãotambémmudando,deformaque

esseprocessodemapeamento,captaçãoedesenvolvimentodecompetênciasdeve

sercontínuo,ouseja,devesempreestarsendorefeito!

Acaptaçãodecompetênciasacontecequandoaorganizaçãoseleciona

competênciasexternasebuscatrazê-lasparaoambientedaempresa.Essacaptação

podeocorrerdeduasmaneiras:nonívelindividual(atravésdorecrutamentoe

seleçãodepessoal)enonívelorganizacional(atravésdeparceriasejointventures

comoutrasorganizações).

Odesenvolvimentodecompetênciastambémpodeacontecertantononível

individualquantoorganizacional.15Osindivíduospodemconquistaras

competênciaspormeiodaaprendizagem,enquantononívelorganizacionalos

investimentosempesquisaéquegeramascompetências.

Porm,temosafasedaavaliaçãodosresultados,demodoquesepossa

repensarasaçõestomadasecorrigiralgumdesvionoprocesso,ouseja,ofeedback

ourealimentaçãodesseprocesso.

16.3.HabilidadesAdministrativasdeKatz

Osadministradoresutilizamdiversashabilidadesparapodergeriruma

organização.Duranteadécadade1970,opesquisadorRobertL.Katzrevelouque

osadministradoresprecisamdetrêshabilidadesessenciaisparadesempenharbem

seupapel:habilidadestécnicas,humanaseconceituais.16

Ashabilidadestécnicassãoasnecessáriasparaexecutarmosumatarefa.São

relacionadascomacapacidadedeutilizarumaferramenta,terconhecimentos

especializadosemumaindústriaeexecutarumprocedimento,porexemplo.

Quandoumsupervisormontaumaplanilhadecontroledeestoque,por

exemplo,eleestáutilizandoumahabilidadetécnica.Usamosumahabilidade

técnicatambémquandoconstruímosumorganograma,umuxogramaouum

fluxodecaixa.

Essashabilidadessãonormalmenteadquiridasatravésdotreinamentoformale

depoissãodesenvolvidasduranteaexperiênciaeavivênciadoprossional.De

certaforma,sãoasmaisimportantesnocomeçodacarreiradeumadministrador.

Jáashabilidadeshumanassãorelacionadascomacapacidadedogestordese

relacionaredecooperarcomoutraspessoas.17Quandonecessitamosmotivar,

comunicarelideraroutraspessoas,essashabilidadessãofundamentais!

Semprequeprecisarmosatingirobjetivosemumaorganizaçãoatravésdo

trabalhodeoutrosindivíduos,habilidadesserãocruciais.

Sevocêtemfacilidadedeinteragircomseuscompanheirosdetrabalho,dese

fazerentendere“vender”suasideias,comcertezajápossuihabilidadeshumanas

emseurepertório!Umgerentequetenhareceiodefalarempúblico,deinteragir

comseuscomandados,terámuitadificuldadeemseutrabalho!

Finalmente,ashabilidadesconceituaissãorelacionadasàcapacidadede

pensar,montarconceitoseanalisarsituaçõesabstratasecomplexas.Quandoum

gestornacúpuladaempresaestáanalisandoaconjunturaeconômicaeasaçõesde

diversosconcorrentesparamontarumplanejamentoestratégico,porexemplo,ele

estáutilizandoahabilidadeconceitual.

Quandotemosacondiçãodepensarcriticamente,deanalisarproblemas

complexos,estamosutilizandoashabilidadesconceituais.Acapacidadedepensar

demodosistêmico,porexemplo,sópodeocorrerquandotemostaishabilidades.

Imaginequevocêestáanalisandocomodiferentesfatospodemafetarsua

organização(porexemplo:umanovalegislaçãodogoverno,umatecnologiaque

sejalançada,umacatástrofeambientaletc.).Sevocêconsegueentendercomoesses

fatoresimpactariamsuaempresanofuturo,vocêtemhabilidadesconceituais.

OutracontribuiçãodeKatzfoioentendimentodeque,apesardetodosos

administradoresutilizaremenecessitaremdessashabilidades,elasseriammais

oumenosimportantesparaosadministradoresdeacordocomsuaposiçãona

hierarquiadaorganização.

Figura16.4–Níveishierárquicosehabilidades

Osadministradoresdoníveloperacionalutilizariammaisashabilidades

técnicas,poisestãotrabalhandoematividadeseprocessosespecícos(um

analistananceirodevesabermontarumaplanilhaeoperarumacalculadora

financeira,porexemplo).

Jáosadministradoresnoníveltático(ougerencial)necessitariamteras

habilidadeshumanasmuitobemdesenvolvidas,poisentresuasprincipais

atividadesestãoincluídasamotivaçãoealiderançadosseussubordinadosem

buscadosobjetivosorganizacionais.

Finalmente,osadministradoresdonívelestratégiconecessitariamtermais

desenvolvidasashabilidadesconceituais,poisestãonormalmenteenvolvidos

comproblemasmaiscomplexosequedependemdediversasvariáveisemseu

trabalho.

Comodevemdecidirsobreosrecursoseobjetivosdeumaorganizaçãocomo

umtodo,acapacidadedepensaraempresademodoglobaléfundamental

(analisandotodassuasdivisões,áreaseasrelaçõescomoseuambienteexterno)!

Vamosvercomoessetemajáfoicobrado?

6.(Cespe/AGU/Anal.adm./2010)Ashabilidadesecompetênciasadministrativasdenemoscargos

deacordocomoconhecimentonecessárioàexecuçãodasatividadesdecadanívelgerencial;

sendoassim,pode-serelacionaroníveldegerênciaoperacionalaumagrandeconcentraçãode

habilidadesconceituais.

Essasquestõesquetratamdashabilidadessãogeralmentebemsimples!Abanca

quasesempretrocaosconceitos,portantoprestembastanteatenção!

Nessecaso,caclaroqueashabilidadesconceituaissãomaisnecessáriasno

nívelestratégico!Oníveldegerênciaoperacional(vejamcomoabancabusca

confundirvocê!)serelacionacomashabilidadestécnicas.Ogabaritoéquestão

incorreta.

QuestõesComentadas

7.(FCC/TCM/CE/Auditor/2010)Agestãodepessoasporcompetênciasenvolve:

a)treinarosfuncionáriosemconhecimentostécnicosessenciaisparaoaumentodaprodutividade;

b)mobilizarecolocarempráticaconhecimentos,habilidadeseatitudesdoscolaboradoresdeuma

organização;

c)selecionaroscandidatoscomosmelhorescurrículosoumaiorcompetênciaprofissional;

d)desenvolvertraçosdepersonalidademaisadequadosaoclimaorganizacional;

e)adequaraculturadaorganizaçãoparaproporcionarumazonadeconfortoparacadacolaborador.

Essaquestão,quecaiuemumconcursobastanteconcorrido,mostraos

principaiselementosdascompetências.Comojávimos,ascompetênciasenvolvem

conhecimentos,habilidadeseatitudes.Dessaforma,aalternativacorretaéaletra

B.

AletraAnãoserefereàgestãoporcompetências,portantoestáerrada.Aletra

Ctampoucoserelacionacomotema,poisagestãoporcompetênciasémuito

maisdoqueumprocessodeseleção.

NaalternativaD,ostraçosdepersonalidadenãosedesenvolvemparaseadaptar

aoclimaorganizacional.JáaletraEétotalmentedescabidaeirreal.Nosso

gabaritoémesmoaletraB.

8.(FCC/ALESP/SP/GestãoProjetos/2010)Umdosprincipaisobjetivosdagestãodepessoaspor

competênciasé:

a)permitirsepararosaspectosdehabilidades,potencial,desempenhoedesenvolvimentodo

profissional;

b)proporcionaroalinhamentodascompetênciasindividuaisàsestratégiasorganizacionais;

c)possibilitarmaiorespecializaçãodeatuação,atendendo,pormeiodarestriçãodamobilidade

funcional,àsnecessidadesdanovagestão;

d)priorizarocargoemvezdoprossional,demaneiraaaproveitarosrecursosdisponíveisaos

resultadosdonegócio;

e)direcionarofocodeatuaçãonosprocessosenodesenvolvimentodaorganização.

Comovimos,umdosobjetivosprincipaisnagestãoporcompetênciaséo

alinhamentodascompetênciasindividuaiscomasestratégiasdaempresa.O

gabaritoéaletraB.

9.(Cespe/Anal.gestãopessoas/Serpro/2008)Agestãoporcompetênciadeveserimplantadaa

partirdomapeamentodascompetênciasorganizacionaiseprossionaisdesejadasbemcomo

dascompetênciasexistentes.

Aquestãofoiconsideradacorreta.Dessaforma,paraoCespe,omapeamento

dascompetênciaséopassoinicialdagestãoporcompetências!

10.(Esaf/CGU/AFC/2004)Deumaformageral,ascompetênciasgerenciaissãoclassicadasemtrês

categorias:conhecimentos,habilidadeseatitudes.Estassãonecessáriasparaocuparumcargo

degerenteedependemdonívelhierárquicodocargo,dastarefasaseremdesenvolvidaspelo

gerenteedotipodeorganizaçãoentreoutrosfatores.Escolhaaopçãoquefazumarelação

corretaentrehabilidadeenívelhierárquico.

a)Quantomaisaltoonívelhierárquicodocargoaserocupado,maishabilidadestécnicasserão

requeridas.

b)Habilidadeshumanassãorequeridasmaisintensamentedosgerentesqueocupamoscargosde

gerênciatáticaeintermediária.

c)Quantomaisoperacionalocargoaserocupado,maishabilidadesconceituaisserãorequeridas.

d)Habilidadesconceituaissãorequeridasnamesmaproporçãonostrêsníveishierárquicos.

e)Quantomaisoperacionalocargoaserocupado,maishabilidadestécnicasserãorequeridas.

Ashabilidadesconceituaissãomaisutilizadasporgestoreslocalizadosnonível

estratégico.Jáashabilidadeshumanassãomaisutilizadasporadministradoresno

nívelgerencial(outático).Finalmente,ashabilidadestécnicassãomaisutilizadas

noníveloperacional.

AbancaapontoualetraEcomoaopçãocorreta.Entretanto,pensoquealetra

Btambémestácorreta.Assim,aquestãodeveriatersidoanuladaporconterduas

opçõescorretas.Ogabaritodabancafoi,assim,aletraE.

QuestõesPropostas

11.(Cespe/TCU/ACEgestãodepessoas/2008)Umdosobjetivosdomapeamentodecompetências

éidenticarogap,oulacuna,decompetências,ouseja,identicaradiscrepânciaentreas

competênciasnecessáriasparaconcretizaraestratégiacorporativaeascompetências

internasexistentesnaorganização.

12.(Cespe/TCU/ACEgestãodepessoas/2008)Entreasestratégiasquepodemserutilizadaspara

identicarascompetênciasorganizacionaisehumanas,incluem-seapesquisadocumental,a

coletadedadoscompessoas-chavedaorganização,asentrevistas,aobservação,osgrupos

focaiseoquestionárioestruturado.

13.(FCC/TRE/PB/Analistaadm./2007)Umprogramasistematizadoedesenvolvidoparadenirpers

prossionaisqueproporcionemmaiorprodutividadeeadequaçãoaonegócio,identicando

pontosdeexcelênciaepontosdecarências,suprindolacunaseagregandoconhecimentoe,

tendoporbasecertoscritériosmensuráveisobjetivamenteédenominado:

a)gestãosocial;

b)gestãodoconhecimento;

c)educaçãocorporativa;

d)desenvolvimentoorganizacional;

e)gestãoporcompetências.

14.(FCC/TRF1aRegião/Analadm./2011)Gestãoestratégicaporcompetênciasimplica:

a)incorporaraoplanejamentoestratégicodaorganizaçãoascompetênciasdasorganizaçõesparceiras

comfoconamissãoprincipal;

b)mapearascompetênciastécnicasdosfuncionárioserealocarcadaumdeacordocomsua

especialização;

c)focaroplanejamentoestratégicodaorganizaçãonamelhoriadocomportamentodosfuncionários

diantedosdesafiosdaglobalização;

d)integraraoplanejamentodaorganizaçãoosconhecimentos,ashabilidadeseasatitudesnecessáriasà

realizaçãodassuasmetas;

e)adequarapolíticaderemuneraçãodaorganizaçãoaoníveldecompetênciaformaldosfuncionários,

deacordocomomercado.

15.(FCC/Infraero/Administrador/2009)Umórgãopúblicoinstituiurecentementeummodelo

gerencialderecursoshumanosqueestabelecearemuneraçãovariávelcomoprincípiobásico

damotivaçãodosservidorespúblicosedeascensãonascarreiras.Paraalcançarseuobjetivo,

deniuumconjuntodeconhecimentos,habilidadeseatitudesindispensáveisaoexercíciode

umcargooufunção,estabelecidoapartirdavisãoestratégicadeestado,daanálisedocargoe

mapeamentodessesatributos,assimcomoasistemáticadeevoluçãonacarreira,mediante

comprovaçãodequeoocupanteefetivamenteaspossui.Nessecaso,trata-sedaaplicaçãode

ummodelode:

a)Empowerment;

b)GerenciamentoporCompetências;

c)Reengenharia;

d)GestãodaQualidadeTotal;

e)JobEnrichment.

16.(FCC/Sefaz/SP/Fiscalderendas/2009)Étarefaessencialdagestãodecompetências:

a)articularcompetênciastécnicasecomportamentaisdosindivíduoseintegrá-lasaosobjetivos

estratégicosdaorganização;

b)descobrirotalentonaturaldecadapessoa,talentosquepodemviraseraprimoradospormeiode

estratégiasdecoaching;

c)procurardesenvolverascapacidadesprodutivasinerentesaumapessoa.Essascapacidadesdevemser

relacionadascomodesempenhodeumatarefaespecífica;

d)enfatizarosconhecimentosformaisecomportamentaisqueapessoaprecisaadquirirpara

desempenharumatarefa;

e)aperfeiçoar,nosgerentes,ashabilidadespráticasrelacionadascomaadministraçãocompetentedos

funcionáriosdeumaorganização.

17.(FCC/TRT/24aRegião/Anal.jud./2011)Nagestãoestratégica,éimportanteumaavaliaçãodas

competênciasdaspessoas,poisestasimpulsionamasorganizaçõese,quandoreconhecidas

pelosclientescomodiferenciaisemrelaçãoaoscompetidores,sãodenominadas:

a)distintivas;

b)essenciais;

c)desuporte;

d)dinâmicas;

e)deunidadesdenegócio.

18.(Esaf/AFCCGU/Desen.inst./2008)Aadoçãodagestãoporcompetênciaspressupõeo

desenvolvimentodeumprocessoconstituídodeetapasoufases.Indiqueaopçãoque

descreveação(ões)identificada(s)comafaseformulaçãodeestratégiasorganizacionais.

a)Levantamentodascompetênciasindividuaisegrupaisexistentesecomparaçãodestascomas

competênciasrequeridaspelaorganização.

b)Captaçãodecompetênciasexternaspormeioderecrutamentoeseleçãodepessoaseo

estabelecimentodealiançasestratégicas.

c)Identicaçãodecompetênciasorganizacionaisnecessáriasaocumprimentodamissãoealcancedos

objetivosestratégicosdefinidospelaorganização.

d)Desenvolvimentodeprogramasdecapacitaçãovisandooaprimoramentodascompetênciasinternas

existentes.

e)Estabelecimentodeplanosoperacionaisdetrabalho,indicadoresdedesempenhoederemuneração

deequipeseindividuais.

19.(Esaf/MPOG/EPPGG/2009)Emumambienteorganizacionalondesepraticaagestãopor

competências,écorretoafirmarque:

a)acompetênciaflui,naturalmente,pelosimplesexercíciodasfunçõesatribuídasaocargo;

b)osprocessosderecrutamentoeseleçãotendemasetornarmaissubjetivos;

c)omapeamentodoscargoséfeitodeformaaatenderaosobjetivosdaorganização;

d)investiremtreinamentosetornadesnecessário,jáquetodossãocompetentes;

e)ossaláriossãomaiselevadosqueamédiadomercado.

20.(Esaf/AFCCGUDesen.inst./2008)Adécadadosanos1990émarcadapelaincorporaçãodenovos

paradigmasnasorganizações,demodoafazerfaceàbuscadecompetitividadenosetor

empresarialemaioreciênciaeecácianosetorpúblico.Umadessasmudançaséa

incorporaçãodoconceitodecompetênciasnasorganizações.

Indiqueaopçãoqueapresentacorretamenteosignicadodecompetênciaentreos

profissionaisderecursoshumanosnasorganizações.Competênciaestárelacionadaa:

a)conhecimentos,habilidadeseatitudesdecadaindivíduoeàtarefaassociadaaresultados;

b)conhecimentos,habilidadeseatitudesdecadaprofissionaleàsuacapacidadedeaprendizagem;

c)capacidadedeoindivíduoagregarvaloreconômicoeagirdeformaautônoma;

d)habilidadedesaberfazercorretamenteecomeficiênciaumatarefaeaodesenvolvimentodeatitudes

reativas;

e)formaçãoprossional,frutodeconhecimentosadquiridosnotrabalho,eàpráticadetarefas

rotineiras.

21.(Esaf/STN/Desenv.institucional/2008)Considerandoosistemadegestãodepessoaspor

competências,assinaleaopçãoquemelhorcaracterizaosprocedimentosrelativosàseleção

deumperfilprofissionalporcompetências.

a)Apartirdocurrículoprossional,émapeadacompetênciaqueseráincluídaentreasexistentesna

organização.

b)Apartirdolevantamentodascompetênciasnecessáriasaotrabalhodaorganização,seleciona-seo

currículocujoperfildecompetênciasatendeàquelasnecessidades.

c)Apartirdacompetênciacomprovadapelocurrículodofuncionárioquesequercontratar,se

estabeleceocargoaocupar.

d)Apartirdacomprovaçãodecompetênciaspelocandidatoaocargo,faz-seaadequaçãodas

competênciasrequeridas.

e)Apartirdacompetênciacomprovadapeloperldocandidatoaocupardeterminadocargo,refaz-seo

mapeamentointernodaorganização.

22.(Cespe/Ceturb-ES/Administrador/2010)Omodelodegestãoporcompetênciaéfocadono

treinamentoenodesenvolvimentocontínuodocolaboradorparaqueesteseaperfeiçoee

adquiraashabilidadesnecessáriasàexecuçãodastarefasnaorganização.

23.(Cespe/TCU/ACEgestãodepessoas/2008)Competênciasfuncionaisdeumaorganizaçãosão

aquelasqueadiferenciamdeseusconcorrenteseclientes.Constituemarazãodesua

sobrevivênciaedevemserdesempenhadasemtodasasáreas,porgruposepessoasda

organização,emboraemníveisdiferenciados.

24.(Cespe/TCU/ACEgestãodepessoas/2008)Ascompetênciasessenciaissãoaquelasespecícas

decadaumadasáreasvitaisdeumaempresaesãodesempenhadasporgruposepessoasde

cadaumadelas.

25.(Cespe/TCU/ACEgestãodepessoas/2008)Ascompetênciasindividuaispodemser

corretamenteclassicadasemgerais,quedevemserdesempenhadasportodosos

colaboradores,eespecíficas,quesãoprópriasdanaturezaedaáreadeatuaçãodoprofissional.

26.(Cespe/TCU/ACEgestãodepessoas/2008)Entreascaracterísticasdasorganizaçõescomfocona

gestãoporcompetênciasincluem-seautilizaçãodeabordagensorganizacionaisexíveis,o

foconosclientes,odesmembramentodaestruturaorganizacionaltradicional—reduçãode

níveishierárquicos—easequipesmatriciais.

27.(Cespe/TCU/ACEgestãodepessoas/2008)Aáreadegestãodepessoas,nocontextodagestão

decompetências,deveestarvoltadaparafacilitarodesenvolvimentodascompetências

individuais,dasexistentes,dasnecessáriasparaaimplementaçãodeumaestratégia,bem

comoolevantamentodenovascompetênciasorganizacionais.

28.(Cespe/Anal.gestãodepessoas/Serpro/2008)Agestãoporcompetênciaévoltadaparao

desenvolvimentosistemáticodecompetênciasprofissionaisdasequipesedosempregados.

29.(Cespe/Anal.gestãodepessoas/Serpro/2008)Agestãoporcompetênciavisasuprirlacunasde

competênciasexistentesnaorganização.

30.(Cespe/Anal.gestãodepessoas/Serpro/2008)Ascompetênciashumanassãoexternadas

quandoaspessoasrelatamverbalmenteoquefazemnaorganização.

31.(Cespe/Anal.gestãodepessoas/Serpro/2008)Nasorganizações,oenfoquecombaseem

competênciaspossibilitaumavisãomaisclaradaspossibilidadesdeascensãodoempregado,

namedidaemqueespecicaoconjuntodecompetênciasparadiferentesníveisdecarreirado

mesmoeixo,oumesmoparaeixosdistintos.

32.(FGV/Badesc/Analistaadm./2010)Assinaleaalternativaqueapresenteadenominaçãoda

competênciaaomesmotempocentralparaasestratégiasesobrevivênciadasorganizações.

a)Distintiva.

b)Desuporte.

c)Essencial.

d)Dinâmica.

e)Deunidadedenegócio.

33.(Cespe/MPS–Administrador/2010)Omodelodegestãoporcompetênciaséconsideradoum

fatormotivacional,jáqueincentivaodesenvolvimentoprofissionaldoscolaboradores.

34.(Cespe/MPS–Administrador/2010)Umadasvantagensdagestãoporcompetênciaséa

possibilidadedesetrabalharcomosrecursoshumanosestratégicos.

Gabaritos

1.C

13.E

25.C

2.E

14.D

26.C

3.D

15.B

27.C

4.B

16.A

28.C

5.C

17.A

29.C

6.E

18.C

30.E

7.B

19.C

31.C

8.B

20.A

32.C

9.C

21.B

33.C

10.E

22.C

34.C

11.C

23.E

12.C

24.E

Bibliografia

Brandão,HugoPena,eCarlaPatríciaBahry.Gestãoporcompetências:métodose

técnicasparamapeamentodecompetências.RevistadoServiçoPúblico5,n.2

(Abr/Jun2005):179-94.

Brandão,HugoPena,eTomásdeAquinoGuimarães.Gestãodecompetênciase

gestãodedesempenho:tecnologiasdistintasouinstrumentosdeummesmo

construto.RevistadeAdministraçãodeEmpresas41,n.1(Jan/Mar2001):8-15.

Brandão,HugoPena,etal.Gestãodedesempenhoporcompetências:integrando

agestãoporcompetências,oBalancedScorecardeaavaliação360graus.Revistade

AdministraçãoPública42,n.5(Set/Out2008):875-98.

Bundchen,Edson,eAnielsonBarbosadaSilva.Propostadeumplanode

desenvolvimentodecompetênciasindividuaisgenéricasalinhadoàestratégia

empresarial.RevistadeCiênciasdaAdministração7,n.13(Jan/Jul2005):87-110.

Carbone,PedroPaulo,HugoPenaBradão,JoãoBatistaDinizLeite,eRosaMaria

dePaulaVilhena.Gestãoporcompetênciasegestãodoconhecimento.3aed.Riode

Janeiro:FGV,2009.

Dutra,JoelSouza.Competências:conceitoseinstrumentosparaagestãodepessoasna

empresamoderna.SãoPaulo:Atlas,2010.

Robbins,StephenP.,eMaryCoulter.Administração.5aed.RiodeJaneiro:

Prentice-Hall,1998.

Sobral,Felipe,eAlketaPeci.Administração:teoriaepráticanocontextobrasileiro.

SãoPaulo:Pearson-Prentice-Hall,2008.

Capítulo17

Trabalhoemequipe

Amaioriadasatividadesdentrodeumaorganizaçãovaialémdacapacidadede

umaúnicapessoarealizarotrabalho.Asevidênciassugeremqueasequipesde

trabalhonormalmentesuperamosindivíduoseosgruposquandoatarefa

necessitademúltiplashabilidades,capacidadedeanáliseeexperiência.1

Figura17.1–Evoluçãodotrabalhocoletivo

Alémdisso,asequipessãomaisexíveiseadaptam-semelhoraosdesaosdo

queosgruposdetrabalhonormais.Vamosverentãoadiferençaentreumgrupoe

umaequipedetrabalho?

Umgruponormalmenteédenidocomoumsomatóriodeduasoumais

pessoasqueinteragemparaatingirumobjetivoespecíco.Essaspessoassão

interdependentesetrocaminformaçãoparaquecadamembroconsigaatingiros

objetivosdesejados.

Osgruposemumaorganizaçãopodemserformaiseinformais.Osgrupos

formaissãodenidospelaestruturadecargosdaempresa.Assim,sevocêtrabalha

emumdepartamentodevendasdeumaempresa,formaráumgrupoformalcom

seuscolegasquepertencemaomesmodepartamento.

Jáumgrupoinformalnãoaparecenoorganogramadaempresa,ouseja,não

estáformalmenteorganizadonemestruturado.Eleseformanaturalmentepela

necessidadequetemosdeinteragircomoutraspessoasnoambientedetrabalho.

Dessaforma,oscolegasdetrabalhoquenormalmentealmoçamjuntos

(independentementedetrabalharounãonomesmosetor)podemser

consideradosumgrupoinformal,ouseja,pessoasquenãonecessitariamestar

juntaspeloseucargoesuastarefas,masquetêmafinidadeumascomasoutras.

Aformaçãodeumgrupodetrabalhopodeserclassicadaemumprocessode

quatroacincofases(nocasodegrupostemporários):

Aprimeirafaseéadaformaçãodogrupo,emqueaspessoasseconhecemeas

regrasdefuncionamentoecomportamentodentrodogruposãoestabelecidas.A

segundafase,aerupção,consistenaformaçãodosprimeirosconitos,geralmente

relacionadoscomadisputapelocontroledopoderdentrodogrupoecomas

restriçõesaoindividualismonogrupo.

Figura17.2–Fasesdaformaçãodeumgrupo

Aterceirafaseéchamadadenormalização.Nessaetapajáexisteumacoesão

dentrodogrupoeorelacionamentojáestámaisrme.Aestruturadogrupojáse

solidicoueasregrasdecomportamentojáseinternalizaram,ouseja,jáestáclaro

paraseusmembrosquaissãooscomportamentosaceitáveisdentrodogrupo.

Naquartafase,derealizaçãooudesempenho,aestruturadogrupojáestá

plenamentefuncional,easenergiasjásãogastasnarealizaçãodasatividades.Para

osgrupospermanentesessaéaúltimafase.

Entretanto,nocasodegrupostemporários,comocomitêsouforças-tarefa,

aindaexistemaisumafase:oencerramento.Nessaetapaogrupojáseprepara

paraterminarseutrabalhoeentregaroresultadonal.Aseguir,podemosverum

resumodessasfases:

Figura17.3–Resumodasfasesdeformaçãodegrupos

Jáumaequipe,deacordocomChiavenato,éumgrupodepessoascom

habilidadescomplementaresequetrabalhamemconjuntoparaalcançarum

propósitocomumpeloqualsãocoletivamenteresponsáveis.2

Umaequipedetrabalhogerasinergiapositivaatravésdeumacoordenaçãode

seutrabalho,ouseja,osomatóriodeseuresultadoémaiordoqueseriao

somatóriodosresultadosisoladosdeseusmembros.

DeacordocomClockeGoldsmith,3paraqueumaequipetenhasucessosão

necessáriasalgumashabilidades:

HabilidadesdeComunicação–semumacomunicaçãoecazentreos

membrosdaequipe,serámuitodifícilatingirosresultadosdesejados.Oscanaisde

comunicaçãodevemestarsempreabertos.

HabilidadedeAutogerenciamento–Aequipedeve,emconjunto,ultrapassar

obstáculospormeiodaconstruçãodeumsensodepropriedade,responsabilidade,

compromissoeeciênciadecadamembro,encorajandoatotalparticipaçãoea

autocríticaparamelhorarincessantementeascondiçõesdetrabalho.

HabilidadesdeLiderança–Devemexistiroportunidadesparaquetodos

exerçamaliderança.Cadamembrodeveaprenderaorganizar,colaborar,planejar,

facilitar,relacionareservircomocoachementor.

HabilidadesdeResponsabilidade–Cadamembrodaequipeéresponsávelnão

sópeloseutrabalho,mastambémpelotrabalhodosseuscolegas.A

responsabilidadedotrabalhoécompartilhadaportodos.

HabilidadedeApoioàDiversidade–Quantomaisdiversicadaaequipe,

tantomaiorsuahabilidadederesponderanovosproblemaseapresentarnovas

soluções.Ospreconceitosdevemserevitados.Pessoascom“pers”diferentes

trazemnovasideiasepontosdevistaquepodemacrescentareenriquecero

trabalhodaequipe.

HabilidadedeRetroaçãoeAvaliação–Semaprendercomoserrospassados,

nenhumaequipecresce.Devemosincentivaraautocríticaeabuscapelo

autoaprendizadoconstante.

HabilidadedePlanejamentoEstratégico–Emvezderesponderaproblemas

comrespostasisoladas,aequipedeveutilizaroplanejamentoestratégicopara

mapearosdesafioseasoportunidadesdemodoparticipativo.

HabilidadedeConduzirReuniõesBem-sucedidas–Nãodeveexistirperdade

tempocomreuniõeslongasepoucoprodutivas.Aequipedeveaprenderautilizar

técnicasdemodoqueasreuniõessejamcurtaseprodutivas.

HabilidadedeResolverConitos–Aequipedeveaprenderaresolver

problemas,negociarcolaborativamente,responderasituaçõesdifíceiseresolver

conflitosinternos.

HabilidadesdeDesfrutar–Otrabalhonãodeveserencaradocomouma

“pena”paraosmembrosdaequipe.Aprenderagostardotrabalhoqueéfeitoe

desfrutardosmomentosjuntoséimportanteparaqueaequipetenhasucesso.

Figura17.2–Diferençaentregruposeequipes

(Fonte:Chiavenato,Administraçãonosnovostempos,2010)

Dessaforma,asorganizaçõesbuscamacriaçãodasequipesdetrabalho,pois

estaspossibilitamageraçãoderesultadosmaiorescomosmesmosrecursos

aplicados.

DeacordocomRobbins,existemquatrotiposprincipaisdeequipes:4

ØEquipesdesoluçõesdeproblemas–sãoformadasgeralmenteporpessoas

domesmodepartamento,quesereúnemparaanalisarmelhoriasde

qualidade,noambientedetrabalhooudeeciência.Osmembrostrocam

informação

e

sugerem

alternativas

aos

problemas

encontrados.

Normalmente,essasequipesnãocontamcomaautonomiaparaimplementar

assugestõessozinhas.

ØEquipesmultifuncionais(cross-functional)–equipesformadaspor

membrosdeváriosdepartamentos,normalmentedomesmonível

hierárquico,quesejuntampararealizarumaatividade.Umexemplodesse

tipodeequipesãoasforças-tarefa.

ØEquipesautogerenciadas–sãoformadasporfuncionáriosqueexecutam

atividadesrelacionadasouinterdependentes.Essesmembrosdaequipe

tomammuitasdasresponsabilidadesdeumsupervisor,poisplanejame

distribuemastarefasenvolvidasnotrabalho,controlamosresultadose

chegamatéaescolherosmembrosdaequipe.

ØEquipesvirtuais–osmembrosdessetipodeequipeutilizamastecnologias

deinformaçãoparatrabalharjuntoseatingirseusobjetivos.Atravésde

instrumentoscomoe-mails,videoconferênciaseainternet,conseguemtrocar

informaçõeseinteragir.Essaformapossibilitaquepessoaspossamtrabalhar

dequalquerlugardomundo.Podeconterapenaspessoasdedentroda

mesmaorganizaçãoouintegrarpessoasdeorganizaçõesdiferentes.

Vamosvercomoessetópicojáfoicobrado?

1.(FCC/TJ/AP/Analistaadm./2009)Comrelaçãoaosfatoresessenciaisàecáciadotrabalhoem

equipe,analiseasseguintesafirmativas.

I.Oidealéquecadamembrodaequipeseespecializeemumadashabilidadesnecessáriasao

trabalho,semredundânciadecompetências.

II.Cadamembrodaequipedeveagircomoassessor,estandoatentoaoqueosoutrosestão

fazendoe,senecessário,informandoosdemaissobremétodosmaisecienteseecazes

detrabalho.

III.Éprecisoquealguémseencarreguedeidenticareexplorarnovasoportunidadesparaa

promoçãodaequipe.Issoexigepoderdepersuasãoeinfluênciasobreaaltadireção.

IV.Umfatoressencialdesucessoéacapacidadedecoordenaçãoeintegraçãodotrabalhodos

membrosdaequipepelaaltadireçãodaorganização.

V.Todaaequipeéigualmenteresponsávelpeladeniçãoemanutençãodeprocessosecazes

detrabalho,masumdosmembrossempreconcentramaioresforçonarevisãodaqualidade

dessesprocessos.

EstácorretooqueseafirmaSOMENTEem

a)I,IeII.

b)I,IIeV.

c)I,IIeIV.

d)I,II,IVeV.

e)I,I,IVeV.

Aprimeirafraseestáincorreta,poisoidealéqueashabilidadesecompetências

sejamcomplementares.Oconceitodehabilidadesrandômicasédotrabalhoem

grupo.AsfrasesIIeIIIestãocorretas.

Jáaquartafraseestáerrada.Oerrodessafraseéa“coordenaçãoeintegraçãodo

trabalhodosmembrosdaequipepelaaltadireçãodaorganização”.Naverdade,a

coordenaçãoeintegraçãodevemserfeitaspelosmembrosdaequipe.

Entretanto,aquintafraseestácorreta.Eu,particularmente,nãoconcordocom

o“sempre”,masabancarealmentedeclarouafrasecomocorreta!Portanto,o

gabaritoéaalternativaB.

QuestõesComentadas

2.(Cespe/TRT/RN/Analista/2010)Aexistênciadasequipesdetrabalhoreeteanecessidadedo

homemdesomaresforçosparaalcançarobjetivosque,isoladamente,nãoseriamalcançados

ouoseriamdeformamaistrabalhosa.

Perfeito.Formamosequipesparaque,unidos,possamosalcançarobjetivosque

nãoseriamatingidosdemodoisolado.Ogabaritoéquestãocorreta.

3.(FCC/Infraero/Administrador/2009)Comrelaçãoàcapacidadedetrabalharemequipe,elemento

essencialnoprocessodegestãoporcompetências,écorretoafirmar:

a)oelementocentraldotrabalhoemequipeéoplanejamento,acapacidadedeseanteciparatodasas

possibilidades.Overdadeirolíderdeequipeéaquelequecontrolatodasasvariáveisinternas,evitando

osconflitos,eexternas,eliminandoasincertezas;

b)otrabalhoemequipesófuncionaquandoolídercompreendeasnecessidadesdecadamembroe

procurasatisfazê-las.Issoevitainsatisfações,conitosinternoseosurgimentodecompetiçãoem

tornodaliderança;

c)otrabalhoemequipeexigeelevadadisciplina,sensodehierarquiaedisposiçãodesacrifíciodos

membrosemrelaçãoàsdecisõesdolíder;

d)notrabalhoemequipe,ashabilidadesdecadaumsãocomplementares,osconitossãoresolvidos

pelodiálogoealiderançasearmapelaconançadosdemaisnasuacompetênciaparaconduzira

equipe;

e)umaequipedetrabalhoecazbaseia-senaconançadecadaumenasuacompetênciaindividual.

Quandocadaumfazoqueconsideramaisadequado,otrabalhotendeaseharmonizardeforma

espontânea,semanecessidadedeumaliderançaforte.

Aprimeiraalternativaestáincorreta,poisoelementocentraldotrabalhoem

equipenãoéacapacidadedeseanteciparatodasaspossibilidades(tarefa

impossível!),nemdecontrolartodasasvariáveis.

AopçãoBtambémestáincorreta,poisnotrabalhoemequipeosmembros

governamasimesmosedevemterhabilidadeparaajudaraosoutrosmembrosno

desenvolvimentodeseupotencial.

Dessaforma,aalternativaCtambémestáincorreta.Nãoexisteapreocupação

comahierarquianotrabalhoemequipe.AalternativaDestácorretaeéonosso

gabarito.JáaalternativaEestáerrada,poisaconançadevesermútuaenão

individual.OgabaritoéaalternativaD.

4.(FGV/Sefaz-RJ/Auditor/2011)Umaequipebem-sucedidarequerhabilidadesespeciaisquedevem

serinter-relacionadas,mutuamentereforçadaseinterdependentes.Aarticulaçãodessas

habilidadespodeajudarumaequipeadenirmelhoroqueeladeveecomofazer.Clocke

Goldsmithpropõemhabilidadesqueosmembrosdeumaequipedevemdesenvolver,

corretamentelistadasnasalternativasaseguir,ÀEXCEÇÃODEUMA.Assinale-a.

a)Habilidadedeautogerenciamento:aequipedeve,emconjunto,ultrapassarobstáculospormeioda

construçãodeumsensodepropriedade,responsabilidade,compromissoeeciênciadecada

membro,encorajandoatotalparticipaçãoeautocríticaparamelhorarincessantementeascondições

detrabalho.

b)Habilidadedecomunicação:aequipedevetrabalharcolaborativamenteparacomunicarabertae

honestamente,ouvirativamenteparaobtersinergia.

c)Habilidadedeliderança:aequipedevecriaroportunidadesparaquecadaparticipantesirvacomolíder.

Paratanto,cadamembrodeveaprenderaorganizar,colaborar,planejar,facilitar,relacionareservir

comocoachementor.

d)Habilidadederesponsabilidade:cadamembrodaequipeéresponsávelsomentepeloseutrabalho.A

responsabilidadenãoécompartilhadacomosdemais.

e)Habilidadedeapoioàdiversidade:quantomaisdiversicadaaequipetantomaiorsuacapacidadede

responderanovosproblemaseapresentarnovassoluções.Novasideiasproporcionamdiferentes

opiniõesqueenriquecemotrabalhodaequipe.Estereótiposepreconceitosdevemsereliminados.

Nessaquestão,aFGVjogouuma“cascadebanana”paraoscandidatos,poisa

letraDsugereque,atravésdahabilidadederesponsabilidade,cadamembroé

responsávelapenaspeloseutrabalho!

Comovimos,éexatamenteocontrário.Dentrodeumaequipe,a

responsabilidadedevesercompartilhada.Nãoadiantafazeroseutrabalhobemse

seucolegaestáfalhando,poistodaaequipeserá“penalizada”.Assim,ogabaritoé

aletraD.

QuestõesPropostas

5.(Cespe/Detran/Analista/2009)Umadiferençaimportanteentreosconceitosdegrupoeequipe

detrabalhodizrespeitoaemquemrecaiaresponsabilidadepelosresultadosdotrabalho.No

casodasequipesdetrabalho,aresponsabilidadepeloresultadonalécompartilhada,nocaso

dosgrupos,éindividual.

6.(FCC/TCE/GO/Desenv.Org./2009)Considereasarmativasaseguir,relacionadasàmontagemde

equipes.

I.Nãopodehaverharmoniaemequipesmontadasemtornodeobjetivosemetasde

desempenhopreviamenteestabelecidaspelaorganização.Paraisso,énecessárioqueas

normaseasmetascomunssejamdefinidaspeloprópriogrupo.

II.Asequipessãomontadas,principalmente,paramelhoraraecáciaorganizacionalenãopara

aproximarpessoasnemaprimoraroclimadaorganização.

III.Asequipesdevemsealinharàsmacroestratégiasdaorganizaçãoenãoconstruirsuaprópria

visãodamissão,dosvaloresedasmetas.

IV.Cadaequipedevesermontadacombaseemsuamissãodentrodeumescopomaisamplo,

universal,deformaanãolimitarsuasaçõesnemacriatividadenecessáriaparainovar.

V.Norecrutamento,éfundamentaldeixarclaroaoscandidatosoquesedesejaalcançar,quais

sãoasrecompensaspossíveis,mas,semacentuarosreaisdesafioseriscos.

EstácorretooqueseafirmaAPENASem:

a)IIeV;

b)I,IVeV;

c)IeI;

d)I,IIeIV;

e)I,IeII.

7.(Cespe/Abin/Ocial/2010)Asequipestrabalhamcompropósitoclaroecomsensode

responsabilidadeemrelaçãoaobjetivosmensuráveisdedesempenho,cujoprincipalindicador

éasatisfaçãodoclienteouusuário.

8.(Cespe/TRE/ES/Analista/2010)Nafasedeformaçãodegruposeequipes,écomumaexistênciade

conitosentreosseusmembros,amdeseestabeleceremospapéisepadrõesde

comportamento.

9.(FGV/Sefaz-RJ/Auditor/2011)Abasefundamentaldotrabalhodecadaexecutivoestánaequipe.

Elaconstituiasuaunidadedeação,asuaferramentadetrabalho.Comela,oexecutivoalcança

metaseproduzresultados.Paratanto,eleprecisasabercomoescolhersuaequipe,como

desenharotrabalhoparaaplicarascompetênciasdela,comotreinareprepararaequipepara

aumentarsuaexcelência,comoliderareimpulsionaraequipe,comomotivá-la,comoavaliaro

seudesempenhoparamelhorá-locadavezmaisecomorecompensá-laparareforçare

reconhecerseuvalor.Essaéasuapraia.Trabalharcomaequipepassaaseraatividadeprincipal

doexecutivocomogestordepessoas.Maslidarcomequipesexigecuidadosespeciais.Háuma

variedadedetiposdeequipes,conformedescritonasalternativasaseguir,ÀEXCEÇÃODE

UMA.Assinale-a.

a)Equipesdemelhoriadeprocessos:éumgrupodepessoascomousemexperiência,vindasdeforada

entidade.Osmembrossãogeralmentepessoascomperfilmaisconservador.

b)Equipesfuncionaiscruzadas:sãocompostasdepessoasvindasdediversasáreasdaempresa

(marketing,produtos,nanças,engenharia)esãoformadasparaalcançarumobjetivoespecícopor

meiodeummixdecompetências.Quasesempresãodesignadas.

c)Equipesdeprojetos:sãoformadasespecialmenteparadesenharumnovoprodutoouserviço.Os

participantessãodesignadosnabasedesuahabilidadeparacontribuirparaosucesso.Ogrupo

geralmentedebandaapóscompletadaatarefa.

d)Equipesautodirigidas:sãocompostasdepessoasaltamentetreinadasparadesempenharumconjunto

detarefasinterdependentesdentrodeumaunidadenaturaldetrabalho.Osmembrosusamo

consensonatomadadedecisãoparadesempenharotrabalho,resolverproblemasoulidarcom

clientesinternosouexternos.

e)Equipesdeforça-tarefa:umaforça-tarefaédesignadapararesolverimediatamenteumproblema.O

grupocaresponsávelporumplanodelongoprazopararesoluçãodoproblemaquepodeincluira

implementaçãodasoluçãoproposta.

10.(Cespe/TJ/PA/Analista/2006)Julgueositensaseguir,acercadotrabalhoemequipe.

I.Umgrupotransforma-seemequipequandoosmembrospassamaprestaratençãoasua

formadetrabalhareprocuramresolverosproblemasqueafetamoseufuncionamento.

II.Opapeldacomunicaçãonãoétãoimportantequantoashabilidadessociaisdecolaboraçãoe

deapoioparaaeficáciadotrabalhoemequipe.

III.Umadasmaneirasdelidarcomasdiculdadesderelaçõesnasequipeséestimularacriação

deconfiançaentreosmembros.

IV.Ofuncionamentonãoharmoniosodaequipepermitequeseusmembrospossam

concentrar-seemmelhorarosprocessosdetrabalho,apesardasdisputasinternas.

Aquantidadedeitenscertoséiguala

a)1

b)2

c)3

d)4

Gabaritos

1.B

6.D

2.C

7.C

3.D

8.E

4.D

9.A

5.C

10.B

Bibliografia

Chiavenato,Idalberto.AdministraçãoGeralePública.2aed.RiodeJaneiro:

Elsevier,2008.

______.Administraçãonosnovostempos.2aed.RiodeJaneiro:Elsevier,2010.

Robbins,StephenP.OrganizationalBehavior.11aed.UpperSaddleRiver:Pearson-

Prentice-Hall,2004.

Capítulo18

GestãodePessoas

Todasasorganizaçõesnecessitamdepessoasparaexistir.Aprenderalidarcom

elassemprefoiumadasatividadesimportantesparaumgestor.Entretanto,naEra

doConhecimentoesseprocessocouaindamaisestratégicoparaqueuma

empresatenhasucessoesedestaque.

Issoocorre,poisatualmenteaspessoasdemandammaisdeseusempregosdo

queantigamente.Alémdisso,asatividadespedemcadavezmenostrabalho

“braçal”emaisintelectoeumacapacidadedeinovação.Atualmente,émuitomais

necessárioeimportantetercriatividadeeconhecimentodoqueforçafísica.

Dessamaneira,umnovotipodeprossionalestásendodemandadopelo

mercado,maiscapacitadoecomoutrasnecessidadesedesejos.Portanto,as

pessoasacabampreferindotrabalharemempresasquemostrammaiscuidadocom

seusempregadosequelhesfornecemmaisoportunidadesecontrolesobresuas

vidas.1

Assim,aspessoashojeemdiasãomuitomaisexigentesdoqueantigamentecom

relaçãoaocontextodotrabalho!Somentesuprirsuasnecessidadesmateriaisnãoé

maissuficiente.

Aorganizaçãomodernadeveprocurarsuprirtambémsuasdemandas

psicológicas,suanecessidadeporumtrabalhodesaadoregraticante,porum

climaorganizacionalacolhedor.2Eissonãoépossívelcomumantigo

departamentodeRecursosHumanospreocupadoapenascomtarefasburocráticas.

EstamosvendoatransformaçãodaáreadeRecursosHumanossairdeuma

posiçãoreativaepreocupadacomaspectosdecontroleepapelada(o

departamentodepessoal),passandoporumsetordeRHmaispreocupadocoma

motivação,comoclimaorganizacionaleosaspectosdeliderança(gestãode

pessoas)atéchegaraumsetorderecursoshumanospreocupadocomo

atingimentodosobjetivosestratégicosdaorganização(gestãoestratégicade

pessoas).3

Figura18.1–Evoluçãodaáreadegestãodepessoas

Chiavenatodefineoprocessodegestãodepessoascomoumconjuntointegrado

deprocessosdinâmicoseinterativos.4Assimsendo,refere-seàspolíticase

atividadesquesãoutilizadasparageriraspessoasnocontextodaorganização.

Dentreosobjetivosdagestãodepessoas,oautorditacomoprincipais:5

ØPossibilitaràorganizaçãoatingirseusobjetivosesuamissãoevisão

estratégica.

ØAtrairemanterpessoastreinadasemotivadas.

ØAumentaracompetitividadedaorganização.

ØAumentarasatisfaçãodaspessoasnaempresa.

ØAumentaraqualidadedevida.

ØDesenvolverumambientedemudanças.

ØDesenvolverumambienteético.

Atualmente,osórgãosdegestãodepessoasestãobuscandoatuarmaiscomo

consultoriasinternas,ouseja,demodoaassessorarosgerentesdelinha(queestão

diretamenteligadosàexecuçãodosprocessosprincipaisdaorganização).

Dessaforma,atendênciaéqueosprofissionaisdeRecursosHumanosdeixemde

estaremumórgão“separado”ouespecialista(comoumaGerênciadeRecursos

Humanos),normalmenteenvolvidosomentecomquestõesburocráticas,para

poderauxiliarosdiversossetoresdaorganizaçãoemsuamissãoestratégica.

PodemosdizerentãoqueoórgãodeRHestápassandoafuncionarcomouma

funçãodestaff,ouseja,desuporte(aocontráriodeantes,comoumagerênciade

linha).

Assimsendo,seumgerentedeoperações(delinha)precisatreinarsuaequipe,

porexemplo,pediráaajudadaequipedeRHparaqueelespossamassessorá-lo

nesseprocessodetreinamento.Resumindo,atendênciaéadescentralizaçãodas

funçõesdegestãodepessoas.

18.1.PanoramadaÁreadeRHnoSetorPúblico

Infelizmente,grandepartedasorganizaçõespúblicasaindaseenquadraemuma

visãoantiquadadagestãodepessoas.Assim,temosumaáreadeRHvoltada

principalmenteparacumprirnormaseleisrelativasaosquadrosdepessoal.Seria

oestágiodedepartamentodepessoal,comojávimos.

DeacordocomPiresetal.,6osórgãosdeRHdasorganizaçõesgovernamentais

perderamaindamaisexibilidadeapósaConstituiçãoFederalde1988.Essacarta

magnainstituiuoRegimeJurídicoÚnico(RJU)paratodososservidorespúblicos,

igualandootratamentodaAdministraçãoIndiretaaodaAdministraçãoDireta.

Boapartedotrabalhodessesórgãospúblicosdepessoal,deacordocom

Schikmann,7selimitaaocontroledafolhadepagamento,aomonitoramentodos

processosdeaposentadoriaeaproposiçãodeleiseans.Alémdisso,essesórgãos

acabamdesenvolvendoaçõesreativaseemergenciaisdetreinamentoe

capacitação.Alémdisso,nãoefetuamumplanejamentodasnecessidadestantode

númerodepessoasnecessárias,quantodospersecompetênciasnecessáriospara

aorganização.

Odesempenhodosservidoresedasorganizaçõesnãoémedido,nemtomado

emconsideraçãoparaaremuneraçãoouprogressãonacarreira.Comisso,nãohá

umamaiorpreocupaçãocomacapacitaçãoporpartedosempregados,nemuma

motivaçãoparaestesproduzammais.

Pensebem,sesuaremuneraçãonãodependedeseuesforço,nemserá

consideradaparaumapromoção,vocêacabaránãocandomuitomotivadopara

trabalharmais,nãoéverdade?

Alémdisso,comonãoexisteumaavaliaçãodedesempenhoefetiva(poisestaé

feitaéapenas“proforma”,ouseja,paraconstar),osservidoresnãorecebemesse

feedbackouretornodoseudesempenho,tãonecessárioparaqueestesaibaonde

precisaevoluir.

Comonãoexisteesselink(ligação)entreaáreadegestãodepessoaseo

planejamentoestratégicodainstituição,nãoexistetambémumapreocupaçãocom

ascompetênciasnecessáriasparaqueestasatinjamseusobjetivosestratégicos.

AgestãoestratégicadepessoasdemandaentãoqueosistemadeRHentenda

perfeitamentecomoaorganizaçãofunciona,quaissãoseusprincipaisprocessose

seusobjetivosemetas.

Comisso,essaáreapodeplanejarumapolíticadeRecursosHumanosqueleve

emconsideraçãoessesplanejamentosemetasparaocurto,médioelongoprazos.

DeacordocomSchikmann,8osprincipaismecanismoseinstrumentosdagestão

estratégicadepessoassão:

ØPlanejamentoderecursoshumanos.

ØGestãodecompetências.

ØCapacitaçãocontinuadacombaseemcompetências.

ØAvaliaçãodedesempenhoedecompetências.

Vamosvercomoessetemajáfoicobrado?

1.(Cespe/Abin/Ocial/2010)Aadministraçãodepessoal,umadasfasesdoprocessodeevoluçãoda

gestãodepessoas,tinhapornalidadeconferirqualidadeàrelaçãoentrecapitaletrabalhoe

operacionalizarosserviçosderecursoshumanos.

Estaquestãoestáerrada,poisoestágiodeAdministraçãodePessoal(oudo

“departamentodepessoal”)nãobuscavaaqualidadedarelaçãoentrecapitale

trabalho.Estavamaispreocupadacomaspectosformaisdessarelação.Ogabaritoé

questãoincorreta.

Continuando,parapodermosatingirosobjetivosdaorganizaçãoedaáreade

gestãodepessoas,necessitamosatuaremdiversosprocessosenecessidades–tanto

daorganizaçãoquantodosfuncionários.Aseguirpodemosverosprincipais

processosdegestãodepessoas:

Figura18.2–Processosdegestãodepessoas

Vamosvermaisdetalhadamenteessesprocessos?

18.2.Recrutamentoeseleçãodepessoas

Umdosprocessosmaisimportantes,oprocessodeagregarpessoasécomposto

pelasatividadesderecrutamentoeseleção.Dessaforma,necessitamosatrair

aquelesquetêmoperlqueaorganizaçãoprecisaparacadacargo,domodomais

rápidoemenosdispendiosopossível.

18.2.18.Recrutamento

Aspessoasnãonascemassociadasaumaorganização.Assimsendo,existea

necessidadedeque“algo”asatraiaparaumaempresaemparticular.As

organizaçõesprecisamcomunicarparaopúblicoexternoqueexisteuma

necessidadedepessoaleatrairpessoasquetenhamoperfildesejadoparaavaga.

Essaatividadeinclui,naturalmente,acomunicaçãoeaatraçãodaspessoas.

Dessamaneira,quantomaispessoascomumperldesejadosejamatraídasparao

processoseletivo,melhordesempenhoteveorecrutamento.

DeacordocomMarras,9sãotrêsosprincipaisfatoresqueafetamas

necessidadesderecrutamentodeumaorganização:oturnover(rotatividadede

funcionários),oaumentodequadroplanejadoeoaumentodequadro

circunstancial(comoacontrataçãodevendedorestemporáriosnoNatal,por

exemplo).

Dentrodorecrutamento,temosduaspossibilidadesdefoco:orecrutamento

internoeorecrutamentoexterno.Orecrutamentointernoocorrequandoa

empresabuscaatrairpessoasquejátrabalhamnaempresa.Portanto,aspessoas

sãotransferidasoupromovidasparacargosmaiselevados.

Jáorecrutamentoexternoocorrequandoaempresabuscaatrairpessoasde

foradela.Ouseja,buscaatrairpessoasnomercadodetrabalhoemgeral.Cada

tipoderecrutamentotemsuasvantagensedesvantagens.Vamosvernosquadrosa

seguirasprincipais:

Vantagensdo

DesvantagensdoRecrutamentoInterno

RecrutamentoInterno

Motivaosfuncionários.

Nãorequersocializaçãodos“novos”

funcionários.

Mantémostatusquo.

Custamenoseémaisrápida.

Aempresanãorecebenovasideiasepontosdevista.

Seleçãoficamaisfácil,poisaspessoasjásão

conhecidas.

Aculturaorganizacionalpodeficarextremamente

Incentivaafidelidade.

conservadora.

Funcionamelhoremumasituaçãode

estabilidade.

Figura18.3–Recrutamentointerno

Dessaforma,orecrutamentointernopossibilitauma“noçãodecarreira”aos

funcionáriosdaorganização.Issoocorre,poisestesentendemqueaorganização

“reservará”oscargosmaisaltosaosfuncionáriosjáexistentes.Assimsendo,esses

funcionáriossesentirãomaisincentivadosamantersuatrajetóriaprossional

dentrodaorganização.

Outroaspectointeressanteéamaiorfacilidadeparaselecionar.Comotodosos

“candidatos”játrabalhamnaempresa,carámuitomaisfácilselecioná-los,poisa

organizaçãojátemum“histórico”desuasatividadesedesempenhos.

Alémdisso,camaisbaratoerápido,poisnãoénecessárioutilizarnenhum

serviçoderecrutamentoexterno.Muitasvezes,apenasumanúncionomuralda

empresaounaintranetjáinformaaospossíveiscandidatosqueasvagasestão

disponíveis.

Entretanto,seaempresanãotrazpessoasdeforaacabacorrendooriscode

carmenosdinâmicaediversicada,poisaspessoasnãorecebemaquele“gás

novo”queexistequandoumapessoadefora(comoutrasexperiênciaseoutro

mododepensar)entranaorganização.

Orecrutamentoexternoaindapossibilitaaentradadepessoascomuma“visão

demundo”eperspectivasnovas,oquepodetornaraculturaorganizacionalda

instituiçãomaisadaptativae

exívelaosnovosdesaos.Entretanto,esse

recrutamentolevamaistempoedemandainvestimentosnasocializaçãodosnovos

funcionários.

Portanto,sedizqueorecrutamentointernofuncionamelhoremempresasque

funcionamemumambienteestável.Jáorecrutamentoexternotambémtemsuas

vantagensedesvantagens,comopodemosveraseguir:

Vantagensdo

DesvantagensdoRecrutamentoExterno

RecrutamentoExterno

Desestimulaosatuaisfuncionários.

Possibilitaaentradadenovasideiase

Diminuiafidelidadedosfuncionários.

modosdepensar.

Custamaiscaro.

Aumentaocapitalhumanoeintelectual.

Requerumsistemadesocializaçãodestesnovosintegrantes.

Renovaaculturaorganizacional.

Necessitadetécnicasmaisapuradasdeseleção,poisnãose

conhecebemaspessoas.

Figura18.4–Recrutamentoexterno

Dessaforma,amaioriadasempresasoptaporbalancearorecrutamentoexterno

comointerno,poispossibilitaàorganizaçãoreceberumuxodenovasideiase

pessoassemdesmotivarosatuaisfuncionáriosemdemasia.

18.2.2.SeleçãodePessoas

Apósotrabalhoderecrutaraspessoasparaoscargosvagosnaempresa,é

necessárioselecioná-las.Dessamaneira,aseleçãoéoprocessodeescolhero

melhorcandidatoparaumdeterminadocargoouempresa.10

Paraqueaseleçãosejabemfeita,éimportanteconhecerascaracterísticasdo

cargo,como:otipodeatividadesenvolvidas,onívelderesponsabilidadeeoperl

desejadodoocupante.

Umprocessodeseleçãoéconsideradoecientequandoconsegue“peneirar”da

melhorformaumagamagrandedecandidatos.Assimsendo,umadasmaneirasde

secalcularissoéatravésdocoeficientedeseleção.

Eleécalculadoatravésdadivisãodonúmerodepessoasadmitidasemrelação

aonúmerodepessoasquepassarampeloprocessodeseleção.Então,quanto

menoroíndice,maiorterásidoaeficiênciadoprocesso.

Nãosóascaracterísticastécnicasdaspessoas,mastambémosaspectos

comportamentaisepsicológicosdoscandidatosdevemseranalisados.Paraisso,

existemdiversastécnicasenvolvidas.Aseguirpodemosverasprincipais:

Figura18.5–Técnicasdeseleção

Ostestesdeconhecimentossãoutilizadosquandoaorganizaçãonecessita

conheceraprociênciadeumcandidatoemalgumtemaouassunto.Dessaforma,

sãoutilizadasprovasorais,provasescritasouderealizaçãoparaaferironívelde

domíniodoindivíduonaquelamatéria.Éométodoprimordialmenteutilizadono

setorpúblico,bemcomoemuniversidades.11

Asprovasderealizaçãosão,comoopróprionomediz,atividadesemqueo

candidatoépostopararealizarumatarefasemelhanteouigualàqueteráde

realizarnocargo.Assimsendo,seaempresanecessitademotoristasdecaminhão,

fazumtestededireção,porexemplo.

Jáasprovasescritassãoasmaisutilizadasnasorganizações.Umadesuas

vantagenséqueseprestaaanalisarumnúmerograndedecandidatos.Dessa

forma,seencaixambememcertasempresasquerecebemmilharesdecandidatos

àssuasvagas.

Enquantoostestesdeconhecimentobuscammediracapacidadeatualdos

candidatos,ostestespsicológicosprocuramaferiropotencialfuturo,ouseja,

suaaptidão.Portanto,aaptidãoéumapredisposiçãonaturalqueumapessoatem

pararealizarcertaatividadeoutarefa.Éoquechamamospopularmentede“dom”.

Assimsendo,ostestespsicológicosbuscamavaliarquaissãoasaptidõesdo

candidato,deformaapreverseucomportamentonocontextodotrabalho.No

casodeprofissionaiscompoucaexperiência,essetesteébastanteindicador.

Jáastécnicasdesimulaçãosão,essencialmente,dinâmicasdegrupo,emqueos

candidatossãoagrupadosparaqueinterajamunscomosoutrosedemonstrem

suasreações.

DeacordocomChiavenato,12aprincipaltécnicadesimulaçãoéo

psicodrama.Essastécnicassãoutilizadascomocomplementodasoutrastécnicas,

esãomaisindicadasaoscargosquenecessitamdeumagrandehabilidade

interpessoal,comoasdiretorias,gerências,áreasdeatendimentoaopúblicoetc.

Outratécnicadeseleçãosãoostestesdepersonalidade.Essestestesbuscam

avaliardiversostraçosdenidospelocaráter(adquiridos)epelotemperamento

(inatos).13Entreosaspectosprincipaisquesãoanalisadosnessestestestemos:

equilíbrioemocional,frustração,agressividade,motivação,ansiedadeetc.

Comoostestesdepersonalidadesãomaiscaros,somentesãoutilizadosem

casosespeciaisouemcargosdaaltadireção.14

Finalmente,temosasentrevistas,quesãoastécnicasmaisutilizadasnomeio

organizacional,asquaispodemterdiversasfunções,desdeumatriageminicialde

candidatosatéaentrevistadedesligamentodaempresa.

Normalmente,umaentrevistapermitequeaequipederecrutamentoeseleção

tenhaumcontatodiretocomocandidato,facilitandoassimaeliminaçãode

diversasdúvidaseacomparaçãodecandidatosentresi.

Comoenvolveotrabalhohumano,temcomoproblemaprincipala

subjetividade,poisaavaliaçãonecessariamentedependerádopontodevistado

entrevistadoredesuavisãodemundo.

Damesmaforma,acomunicaçãoentreoentrevistadoeentrevistadortemos

problemascomunsatodacomunicaçãohumana,comoruídoseproblemasde

percepção.Assimsendo,oentrevistadordeveserbem-treinadoeaentrevistadeve

serplanejadaanteriormente,deformaaavaliarocandidatodamelhorforma

possível.

18.2.3.RecrutamentoeSeleçãonosÓrgãosPúblicos

Comonosetorpúblicooplanejamentodasnecessidadesdepessoaléalgoraro,

normalmenteascontrataçõesocorremquandoexisteumquadrodecarência

extremadoórgãoouquandoogestorpúblicoquecomandaosetortempoder

políticoequerreforçaroquadrode“seu”órgão.

Assim,deacordocomCostin,somenteemtemposrecentespassaramaexistir

concursosanuaisparaalgunsórgãospúblicos.Deacordocomaautora:15

SórecentementealgunsestadoseoGovernoFederalcomeçaramaprepararconcursosanuaisa

partirdeumdiagnósticodecarênciasprossionaisconcretas.Normalmenteoqueocorreéuma

misturadeclientelismo,comindicaçãodepessoasparaosaindanumerososcargosdeconança

combaseemcompromissospolíticoseumatendimentodedemandasemergenciais.

Aimportânciadeconcursosfrequenteséconstantementeintroduzir“sangue

novo”nosquadrosdepessoal.Dessaforma,osórgãosestãosemprerecebendo

pessoascomnovasexperiênciaseconhecimentoseque,aprincípio,almejam

ingressarnainstituição.

Emrelaçãoaesseúltimoponto,existeoutrapreocupação.Apráticadese

efetuarconcursospúblicoscomumprazodeduraçãomáximoeclassicarum

grandenúmerodecandidatosexcedentes(oprazomáximoédedoisanos,

renováveispormaisdoisanos),apesarde“darmenostrabalho”eexibilidade

(poisaadministraçãotemsempreum“estoque”decandidatospara“cobrir”

necessidadesnãoprevistas),temramificaçõesnegativas.

Essapráticaresultaemingressantesmenoscapacitados(emmédia)enãocria

umaculturaorganizacionalquevalorizeagestãodoconhecimento(pois

concursosfrequentes“forçam”acriaçãodeumprocessodetreinamentoformale

decapacitaçãoconstantedenovosservidores).

Entretanto,muitosórgãostêmdiculdadedelançarconcursosporaspectos

políticos.Assim,quandoconseguemlançarumconcurso,preferemdeixá-locom

umgrandeprazodevalidade,demodoatercomoatenderdemandasfuturas.

Enquantooprazodoconcursoestáválido,aAdministraçãoPúblicanãopode

convocarcandidatosdeconcursosmaisrecentes.Assim,muitosórgãosmais

“maduros”emsuagestãodepessoastêmbuscadorealizarconcursoscomumprazo

maiscurtodevalidade.

Aocontráriodainiciativaprivada,quepoderestringiroacessoaoprocesso

seletivo,aAdministraçãoPúblicadeveobedeceraoprincípiodaisonomia.

Assim,umaempresaprivadapode,porexemplo,decidirquesócontratará

quemtivermaisdedoisanosdeexperiência,ouqueseformouemuma

universidadepública.OEstadonãopodeagirassim.

OEstadosópoderestringiroacessoquandoexistiralgumaleiquelimiteo

acessoacertasatividades(comonocasodosadvogados,médicosetc.)ouquando

existirummotivorazoávelparaqueessarestriçãosejafeita(comoéocasode

policiais,quedevemfazertestesdeaptidãofísica,porexemplo).

Alémdisso,aadministraçãopode“direcionar”ostestesdeconhecimentospara

queoconcursoacaberesultandoemcandidatosquetenhamosconhecimentos

desejados.Assim,ostemasinseridosnasprovasdevemteraderênciaaos

conhecimentosefetivamenteutilizadospeloservidornocontextodoseutrabalho.

Outradiculdadedosprocessosseletivospúblicos(concursospúblicos)éque

elestêmcomobaseostestesdeconhecimento,comprovasescritasedemúltipla

escolha.

Essestestesavaliam,obviamente,osconhecimentosdoscandidatos,masnão

captammuitasdashabilidadesnecessáriasouasatitudes(comooequilíbrio

emocional,ainiciativa,aambiçãoetc.).

Comisso,muitosórgãosestãobuscandoinseriroutrasetapasnoprocesso

seletivoqueconsigam“mapear”ashabilidadeseatitudes,como:cursosde

formaçãoetestespsicológicos(comoospsicotécnicos).

Vamosvercomoissojáfoicobradoemprovas?

2.(Cespe/Ceturb-ES/Administrador/2010)Oinstrumentodeseleçãoconhecidocomoteste

escritoéutilizadopelaadministraçãopúblicaparapreservaraisonomianacontrataçãode

servidores.

Ostestesdeconhecimentosouhabilidadessãoutilizadosemtodososconcursos

públicos,eservemparamediracapacidadeatualdocandidato.

Dentreessestestesouprovas,osescritos(existemtambémosoraiseospráticos)

sãoosmaisusados,eservemparamanteraisonomia(tratamentoigualitário)na

contrataçãodosservidores.Ogabaritoéquestãocorreta.

18.3.Cargos,CarreiraseRemuneração

Dentrodoprocessodeaplicarpessoasemumaorganizaçãoencontramosas

atividadesdedesenhoorganizacional,análiseedescriçãodoscargoseaavaliação

dodesempenhodosfuncionários.

Essasatividadesserelacionamcomadeniçãodequaisserãoastarefasdas

pessoasnotrabalho,edaorientaçãoeavaliaçãodeseudesempenho.

Aorientaçãoocorreapósoprocessoderecrutamentoeseleção.Aspessoas

chegamàempresaenecessitamdeumasocialização,ouseja,serintegradasno

contextodaorganizaçãoatravésdeumprocessodeaclimatação.

Asocialização,portanto,éoprocessopeloqualaempresaintegraonovo

membroàsuaculturaeaoseucontextooperacional,deformaquepossase

comportardeacordocomoesperado.16Portanto,onovofuncionáriopassaa

conhecermelhorasnormasevaloresdaempresa,bemcomoasuacultura

organizacional.

Outroaspectoimportantedesseprocessoéadescrição,análiseedesenhodos

cargos.Umcargoéumsomatóriodeatividadesquedevemserexecutadaspor

umapessoa,equetemcertaposiçãoformaldentrodoorganogramada

organização.

Dessaforma,odesenhodeumcargoserelacionacomadeniçãodequais

serãoasatividadesdesempenhadasporseuocupante,qualseráseuníveldepoder

equemserãoseusinterlocutores.17Ouseja,basicamenteéaestruturaçãodo

cargoemsi.Assimsendo,serelacionacomosmétodosqueserãoutilizadospara

executarotrabalhoeasrelaçõesdesubordinaçãoesupervisão.

Portanto,aodesenharumcargodevemosdistinguirastarefas,asatribuiçõesea

funçãoquepassarãoacomporessecargo.

Quandoumcargojáexiste,podemsernecessáriassuadescriçãoeanálise.Isso

podeocorrerquandoaprópriaorganizaçãojánãoentendemuitobemexatamente

emqueumcargoconsisteouquemdeveráocupá-lo.Passaaideiadequeocargoé

uma“caixa-preta”.

Decertaforma,adescriçãodeumcargonadamaisédoquearelaçãodas

atividadesdoseuocupante:oqueelefaz,porquefazaquilo,comofazetc.Éum

retratosimplificadodasresponsabilidadeseatividadesdesempenhadasnocargo.

Jáaanálisedocargosepreocupamaisdoperldesejadodeseuocupante,ou

seja,ascapacidades,habilidadeseconhecimentosnecessáriosdeumocupantepara

desempenharbemumcargoqualquer.

Portanto,adescriçãoeaanálisedoscargossãopassosfundamentaispara

diversasáreasdegestãodepessoas,comooprocessoderecrutamentoeseleção,

poisirãomostrarquaisserãoospersindicadosparacadacargovagona

organização.Serãoimportantestambémparaoprocessodetreinamentoe

desenvolvimentodepessoas,poisdestacarãoasáreasquenecessitamde

treinamento.

Nosetorpúblico,adescriçãodecargosacabalimitandoaatuaçãodos

funcionários,poiscriacertas“reservas”decargosparaalgumascarreiras,sem

preocupaçãocomumavisãomaissistêmicaporpartedosempregados.18

Assim,essadescriçãodoscargosébastantedetalhadanasleisqueregemas

determinadascarreiras.Comisso,muitasvezesnãoháumamaiorexibilidadena

administraçãodepessoalentreosórgãosefunções,consequentementearotação

defunçõesquasenãoéutilizada.

DesdeaConstituiçãoFederalde1988,nãoexistemaisatransposiçãodecargos.

Comisso,oprossionalquequeiradesempenharfunçõesquenãoestejam

previstasdentrodadescriçãodecargosdesuacarreiradeveprestaroutro

concursopúblico.

Assim,umanalistadaReceitaFederalpodeserextremamentecapacitado,mas

nãopoderáexecutarcertasatribuiçõesquesão“exclusivas”deauditoresda

ReceitaFederal.Essalimitaçãofoidecorrentedeóbviosabusosqueessa

transposiçãopermitia,masacabou“engessando”agestãodepessoas.

Asoluçãoparamuitosórgãospúblicosfoipassaradescreverseuscargosde

formamaisamplaegenérica,epassou-seacriaraschamadascarreirashorizontais,

quepodemterexercícioemqualquerórgãoousecretaria,comoocasodos

gestoresfederais(especialistasempolíticaspúblicasegestãogovernamental).19

18.4.AvaliaçãodoDesempenho

Aavaliaçãododesempenhoéumprocessosistemáticodeanálisedo

desempenhodeumindivíduoemfunçãodocontextodotrabalho.Dessamaneira,

serveparajulgarovalor,asexcelênciaseascapacidadesdecadaempregado,esua

contribuiçãoparaqueaorganizaçãoatinjaseusobjetivos.20

Portanto,aavaliaçãodedesempenhoajudaaorganizaçãoavaliarseseus

funcionáriosestãoconseguindo“entregar”resultados.A“entrega”,deacordocom

Dutra,21éacapacidadedeoempregadotrazerresultadoseatingirosobjetivos

desejadospelaorganização.Assim,oempregadoqueentregaestágerandovalor

paraaorganização.

Aavaliaçãodeve,portanto,subsidiarasdecisõesdeaumentodesalários,

promoções,transferênciase,eventualmente,demissõesdeempregados.Outro

benefícioéforneceraosfuncionáriosumanoçãodecomoseutrabalhoestásendo

“visto”pelagerência,demodoaqueelespossamcorrigirseuserrosereceberum

aconselhamento.

Issoocorrepoisémuitodifícilqueconsigamosmelhorarnossodesempenho

semsabercomoestamosnossaindo,nãoéverdade?Assim,aavaliaçãoauxiliao

processodeautodesenvolvimentodosempregados.22

Alémdisso,osresultadosdaavaliaçãosupremoprocessodetreinamentoe

desenvolvimento,poisindicamquaissãoasáreas,habilidadesecapacidadesque

necessitamserdesenvolvidasemcadafuncionárioeemcadasetordaorganização.

Aavaliaçãopodeserfeitapordiversos“atores”dentrodeumaorganização.A

seguirveremososprincipais:

ØAutoavaliação–nessemétodo,éoprópriofuncionárioquemseavalia.

Dessamaneira,eledeveterumasériedecritériosparaevitarumaexcessiva

subjetividade.Decertaforma,semprenosavaliamos,nãoémesmo?Orisco

dessaformadeavaliaçãoéapessoanãoquerer“encarar”suasdeciênciase

“mascarar”suasdificuldades.

ØGerente–esseéométodomaiscomumdentrodasorganizações.Ogerente,

normalmenteassessoradopelaequipedoRH,utilizacertosparâmetrospara

avaliarsuaequipedetrabalho,ouseja,seussubordinados.

ØEquipedetrabalho–aavaliaçãoéfeitapelosprópriosmembrosdaequipe.

Aconsciênciadequeaavaliaçãoserápositivaéfundamentalparaqueseja

vistacomoumaspectobenécoparafuncionamentodaequipe.Osmembros

maisexperientesnormalmenteservemdesuporteaosdemaisnessa

atividade.23

ØAvaliação360º–nessetipodeavaliação,ofuncionáriorecebeumaavaliação

detodasasáreasepessoascomquemtemcontatonocontextoda

organização.Dessaforma,oschefes,oscolegasdetrabalho,osclientes

internoseexternos,fornecedoresesubordinadosavaliam,cadaumdesua

forma,otrabalhodofuncionário.Assimsendo,aavaliaçãocamuitomais

completa,poisofuncionárioéanalisadoemrelaçãoatodasasdemandasque

enfrentadentrodaempresa.

ØComissãodeavaliação–éummétodoemqueéformadaumacomissãode

avaliaçãoparaavaliarosfuncionários.Dessemodo,podeserformadapor

pessoasdeumoumaissetoreseserpermanenteoutemporária.Éummétodo

caroequetomabastantetempo,masquetemcomobenefícioumamaior

padronizaçãodoscritériosdeavaliação.

ØAvaliaçãodebaixoparacima–aocontráriodométodoemqueogerente

avaliaseussubordinados,nessecasosãoossubordinadosqueavaliamoseu

superior.Dessamaneira,ogerenteéavaliadodeacordocomseuestilode

liderança,seuaspectomotivacionalehabilidadedecomunicação.Oobjetivo

évocalizarasdiculdadesqueosfuncionáriosestãotendocomsuachea,de

modoaqueessacheapossafornecerosmeioserecursosparaqueaequipe

consigaatingirosresultadosdesejados.

Agoraquejávimosquempodefazeraavaliação,vamosveraseguirquaissãoas

principaistécnicasutilizadasparaseavaliarodesempenho.

18.4.1.MétododasEscalasGráficas

Essemétodoébemsimples,eimaginoquemuitosdenósjáoutilizamosem

algummomentodenossasvidas.Desenhamosumquadrocomlinhasecolunas.

Assim,naslinhasdescrevemososaspectosquequeremosavaliar(porexemplo,

assiduidade,pontualidade,capacidadedeatingirosobjetivos,facilidadede

comunicaçãoetc.)enascolunasmarcamoscinco“notas”,quepodemserde1a5,

oudeEatéA.

Dessaforma,vamos“marcando”asnotasdosfuncionáriosparacada“quesito”

quedevemosavaliar.Asvantagensprincipaisdessatécnicasão:éfácildeconstruir

esimplesdeutilizar.Alémdisso,ofuncionárioentendeométodo.Portanto,é

maissimplesacomunicaçãodeseusresultados.

Entretanto,éummétodosubjetivoesujeitoaoefeitodegeneralização(ou

“efeitodehalo”).Oefeitodehaloéatendênciadoavaliadordeseconcentrarem

apenasumaspectodoavaliado,semconsiderarsuasoutrascaracterísticascom

imparcialidade.24Portanto,seeleconsideraumfuncionáriomuitopontual,pode

nãoatentarparaoutrascaracterísticas,como:produtividade,capacidadede

liderança,motivaçãoetc.

Essemétodo,alémdisso,tambéméumpoucorestritonosaspectosqueavalia.

Outrofatornegativoéofatodeserfechadoàparticipaçãodoavaliadono

processo.

18.4.2.ListasdeVerificação

Esseéummétodosimplicadodasescalasgrácas.Seufuncionamentoé

bastantesimilar,sendoutilizadoparaavaliaçõesmaisfrequentes.Dessemodo,

acabafuncionandocomoumachapadronizada,emqueogerentemarcaas

principaiscaracterísticasdosfuncionários.Tambéméconhecidapeloseutermo

eminglês:checklist.

18.4.3.MétododaEscolhaForçada

Esse

método

tentareduzirasubjetividade,ageneralizaçãoea

superficialidademostradasnométododasescalasgrácas.Dessaforma,são

escritasfrasesdescritivasdocomportamentopossíveldeumfuncionáriodentro

docontextodotrabalho(exemplo:épontual,temespíritodeequipe,nãorecebe

bemcríticasetc.)paraqueoavaliadorconsiga“visualizar”melhorosaspectosque

deveobservaremcadaindivíduo.

Assimsendo,emcadabloco,existirãoalgumasfrasespossíveisparaqueo

avaliadoranaliseeescolha(daíonome:escolhaforçada).Portanto,oavaliadorvai

marcaremcadaaspectoumaouduascaracterísticaspossíveisdecadaavaliado,da

formaquemelhorrepresenteseudesempenho.25

Comoprincipalvantagemdessemétodo,temosareduçãodasubjetividadeeda

generalização.Entretanto,éummétododedifícilconstrução,difícilcomunicação

ededifícilvisualizaçãodoresultadoglobal.

18.4.4.MétododosIncidentesCríticos

Esseéummétodobastantesimplesedefácilconstrução.Paracadacargosão

descritasalgumascaracterísticascríticas,ouseja,quelevamaumdesempenho

excelenteounegativo.Portanto,seestamosavaliandoumvendedor,porexemplo,

teríamoscomoaspectocríticopositivoafacilidadedecomunicação.

Jáumaspectonegativo,nessecaso,poderiaserafaltadeequilíbrioemocional

ouadiculdadedeaceitararejeição.Assim,teríamosumalistadeaspectos

críticospositivosenegativos.Oavaliadorutilizariaessalistaparaavaliaro

funcionáriodeacordocomessesaspectos.

Comovantagensdessemétodo,temosafacilidadedeconstruçãoedeaplicação.

Entretanto,elenãoproporcionaumamaneiradeavaliarocomportamento

normaldapessoa(somenteoscríticos),portantoacabasendoumpoucorestritoe

tendencioso.

18.4.5.MétododaPesquisadeCampo

Esseéummétodomaiscompletodeavaliação,eevidenciaafunçãodestaffda

equipedeRHdaempresa(quefuncionacomoumaconsultoriainterna,ajudando

osgerentesnoprocessodeavaliação),enquantoogerentetemaresponsabilidade

delinhadeavaliarosseusfuncionários.

Dessaforma,aequipedeRHentrevistaogerentenoseuambientedetrabalho

(daíonomedepesquisadecampo)eemconjuntocomeleavaliaodesempenho

dosfuncionários.26DeacordocomChiavenato,27ométodoabrangeasseguintesfases:entrevistadeavaliaçãoinicial,entrevistadeanálisecomplementar,

planejamentodasprovidênciaseacompanhamentoposteriordosresultados.

Comovantagensdessemétodo,temosaênfasenamelhoriadodesempenhoe

oplanejamentodasaçõesparaofuturo,atreladasaodesempenhopresente.

Dessaforma,existeumamaiorprofundidadenaavaliaçãododesempenho.

Entretanto,éummétodomuitomaiscaro,poisnecessitadoapoiode

especialistasdeRH,alémdesermaislento.Alémdisso,nãoexisteuma

participaçãodofuncionárionaavaliaçãoenoplanejamentodasaçõesparaseu

desenvolvimentofuturo.

Aseguirpodemosverumesquemacomosprincipaismétodosdeavaliaçãodo

desempenho:

Figura18.6–Métodosdeavaliação

Vamosvercomoissojáfoicobradoemprovas?

3.(Cespe/Embasa/Administração/2010)Nométododeescalagráficaparaavaliarodesempenhode

umempregadodeumaempresa,oavaliadoranalisaaspectoscomopontualidade,criatividade

eatingimentoderesultados,atribuindoparacadaaspectograusde1a5.

Ométododaescalagrácaéométododeavaliaçãomaisutilizadoemais

simplesdeserexecutado,porémpermitepoucasalternativasaoavaliadorefacilita

estereótiposegeneralizaçõessobreosavaliados.

Nessemétodo,oavaliadoratribuiuma“nota”ouconceitopadronizado(pode

sertambém:péssimo,ruim,regular,bom,excelente)paracadaaspectodo

desempenhodofuncionário.Ogabaritoéquestãocorreta.

18.5.ProcessodeRecompensarPessoas

Esseprocessoabordaasmaneiraspelasquaisumaorganizaçãopodeincentivare

motivarumfuncionário.Entreosinstrumentos,temosaremuneração,as

recompensaseosbenefíciosqueumaempresapodeforneceraumempregado,de

modoabuscarsatisfazersuasnecessidades.

Oprocessoderecompensarincluitodosostiposdeincentivosegraticações

queosindivíduosrecebememtrocadeseuesforçoedesempenhoparaquea

organizaçãoatinjaseusobjetivos.28

Obviamente,quandopensamosemtrabalharparaalgumaempresatemosa

remuneraçãocomoumdosprincipaispontos,nãoéverdade?Então,as

organizaçõesnecessitamadministraressasexpectativasdeacordocomosdesaos

quetemqueenfrentar.

DeacordocomChiavenato,29aremuneraçãototalconsisteem:remuneração

básica(queincluiumsaláriomensalouporhora),osincentivossalariais(bônus

eparticipaçãonoslucrosetc.)eosbenefícios(como:segurodevida,ticket

refeiçãoetc.).

Assim,arecompensapodesernanceiraenãonanceira.Jáasnanceiras

podemserdiretaseindiretas.Umarecompensananceiradiretaserelaciona

comosalário,osprêmioseascomissões.

Asrecompensasnanceirasindiretas,porsuavez,serelacionamcom

cláusulasdeacordocoletivas,bemcomolegislaçõestrabalhistasqueestipulam

certosbenefícios,alémdosplanosdebenefíciosdaorganização.

Entreesses,temosodécimoterceirosalário,ashorasextras,asfériasetc.Jáas

recompensasnãonanceirasserelacionamcomoreconhecimento,aqualidade

devidanaorganização,asoportunidadesdedesenvolvimentoetc.

Figura18.7–Recompensasorganizacionais

Osalárioéimportanteparaofuncionário,poisdeneseupadrãodevida,e

paraaorganização,poisimpactaoseucusto.30

18.6.ProcessodeDesenvolverPessoas

18.6.1.TreinamentoeDesenvolvimento

Aspessoasnãopodemmaisdeixardebuscarseuaprimoramentoconstante.O

ambientedenegóciosmudaconstantemente,easorganizaçõesdevemestarcada

vezmaisdinâmicasecompetitivasparaquepossamsobreviver.

Assimsendo,asorganizaçõesnecessitamtreinar

edesenvolverseus

funcionáriosparaqueestesconsigamajudá-lasaatingirseusobjetivos

estratégicos.

Aprimeiradiferençaquedevemosentenderentreotreinamentoeo

desenvolvimentoéoespaçotemporal.Ouseja,otreinamentoévoltadoparaas

tarefaseatividadesatuaisdofuncionário.Dessamaneira,éfocadonopresente.

Jáodesenvolvimentoéfocadonofuturo.Assim,sepreocupacomas

habilidadesecapacidadesqueserãoexigidasdosfuncionáriosfuturamente.

Entretanto,tantootreinamentoquantoodesenvolvimentosãoprocessosde

aprendizagem.31

Nonívelorganizacional,temosaaprendizagemorganizacional,queéo

desenvolvimentodeumaculturavoltadaparaoconstantemelhoramentodas

pessoas,demodoasempreinovareencararnovosdesafios.

Otreinamentoéumprocessocíclico,queécompostodequatroetapas:32

ØProgramação–nessemomentoaequipedevediagnosticarasnecessidades

detreinamento,alémdeescolherqualseráotreinamentonecessário,quando

deveráserefetuado,porquemeparaquem,entreoutrosfatores.Dessa

forma,éoplanejamentodoprocessodetreinamento.

ØOrganização–nessaetapaosrecursosnecessários(desdeinstrutores,salas

deaula,materiaisetc.)devemserregistrados,deformaqueaorganização

possareservarosrecursosdisponíveiseplanejaraaquisiçãodosqueainda

nãotiver.

ØImplantação–éaexecuçãodotreinamentoemsi.Ouseja,nessaetapao

treinamentoéefetuado.

Avaliação–nessemomentoserãoavaliadososresultadosdotreinamento,isto

é,seosobjetivosforamounãoatingidos.Deverãosermedidososefeitosdo

treinamentonocomportamentodosfuncionáriosnotrabalho.

Umaorganizaçãopodeterumaposturareativaouprospectiva(ouproativa)

nadeniçãodesuasnecessidadesdetreinamentoedesenvolvimento.Seaempresa

esperaqueocorraumafalhaouumadeciêncianosconhecimentosehabilidades

dosfuncionáriosparaagir,temumaposturareativa.

Jáseaorganizaçãobuscaseanteciparaosdesaosfuturos,estáadotandouma

posturaprospectiva,poisquandoofuturochegarjádeveráestarpreparadapara

asnovasdemandas.

18.6.1.1.MétodosdeTreinamento

Existemdiversasmaneirasdeseefetuarumtreinamento.Otreinamentopode

visaroaumentodeinformaçõesedehabilidades.Ostreinamentosvoltadosparaa

disseminaçãodeinformaçõessão:asleituraseainstruçãoprogramada.33

Aleituraéummétododemãoúnica,emqueuminstrutorpassaasinformações

aumaturmadealunos.Dessaforma,épassadaumagrandequantidadede

informações.Entretanto,éuma“viademãoúnica”,poisacabagerandouma

posturapassivanostreinandos.

Jáométododainstruçãoprogramadaéumatécnicamaismoderna,pois

permitesuautilizaçãoporcomputadores.Assim,podeserutilizadaemqualquer

lugaredemaneirapadronizada.Dessemodo,asinformaçõessãointercaladaspor

perguntas,quevãoindicandoparaotreinandoseeleestáabsorvendoounãoo

conteúdo.

Essemétodotambéméconhecidocomotécnicadeautoinstrução.Existem

doissistemasdeinstruçãoprogramada.DeacordocomMello:34

OprimeiroéochamadoSistemaLinearouExtrínseco,quepartedoprincípiodequeoprocesso

deaprendizagemdostreinandosoualunoséinibidoporerros.Destaforma,essesistemautiliza

umesquemaquetemsucessivoselementosdaslições,utilizandoinformaçõespequenase

repetitivas.Jáosegundomodelo,oSistemaRamicadoouIntrínseco,temporcaracterística

liçõesmaioresemaiscomplexas,demandandomaisesforçodotreinandoparaassimilação.O

materialdessetreinamentoéconcebidopartindodoprincípiodequeostreinandosestarão

prontosparaaplicarosensinamentosemalgummomentodoprograma,acarretandodessaforma

arápidapercepçãodostreinandosquantoaosresultadosoriundosdoseudesempenho.

Dessamaneira,oprimeirosistemasebaseiananoçãodequeoserrosinibemo

aprendizadodaspessoas.Asquestõesdevemserutilizadasdemodofrequente,para

queoalunoaindaestejacomoconteúdo“quente”nacabeça.

Alémdessesmétodos,existemosmétodosvoltadosparaoaumentodas

habilidadesdosfuncionários:otreinamentonocargo(onthejobtraining)eas

técnicasdeclasse.

Otreinamentonocargo,comoopróprionomediz,ocorrenoambientede

trabalho.Podeserfeitoatravésdofornecimentodeinformaçõeseavaliaçõesno

contextodotrabalho,bemcomoarotaçãodecargos(emqueapessoapassaa

efetuarnovastarefas,aprendendonovosconhecimentosehabilidades),alémde

projetosespeciais.

Astécnicasdeclassepodemenvolverdinâmicasdegrupo,jogosdenegóciose

psicodramas,demodoaencorajarainteraçãoentreaspessoaseatrocade

experiências.Éummétododemãodupla,aocontráriodaleitura.35

Vamosvercomoessetópicojáfoicobrado?

4.(Cespe/Basa/Administração/2010)Ocritériodeescolhadosparticipantesdeprogramasde

treinamento,desenvolvimentoeeducaçãodeveserbaseadonointeressedemonstradopelo

empregado.

Naverdadeaescolhadeveserfeitatendoemvistaprincipalmenteas

necessidadesdaorganização,enãosomenteointeressedofuncionário.Como

mapeamentodascompetênciasnecessáriasparaaempresa,oprocessode

treinamentoedesenvolvimentotentaráfecharessesgaps(oulacunas).Ogabaritoé

questãoerrada.

18.7.ProcessodeManterPessoas

Atualmente,aspessoassãomaisexigentesquantoaoseuambientedetrabalho.

Sóaremuneraçãojánãoconseguesatisfazerumapessoaemantê-lanaempresa.

Dessaforma,aorganizaçãodevecuidarparaqueexistaumambientequenão

“afugente”seusbonsfuncionários.

Entreosaspectosimportantesdesseprocessoestão:aculturaorganizacional,o

climaorganizacional,aqualidadedevida,entreoutros.Portanto,oprocessode

manutençãodaspessoasnaorganizaçãonecessitadeumasériedecuidados.

Umestilodegerênciamaisrígidoepadronizadopode“expulsar”funcionários

maiscriativosecapazes.Portanto,ogestordeveanalisarascaracterísticasda

empresa,bemcomooperldesejadodosfuncionários,demodoaadaptaroestilo

gerencialaotipodemodeloorganizacionalquesejaomaisindicadoparaquea

organizaçãotenhasucesso.

Outroaspectoimportantesãoasrelaçõescomossindicatoseosprogramasde

qualidadedevidaesegurançanaorganização.Umarelaçãomenosconituosa

comossindicatossempreéinteressante,poiscriaumcanalparaqueas

divergênciasentrepatrõesefuncionáriospossamservocalizadas,evitandoqueos

conflitoscheguemaumpontoderuptura.36

Aqualidadedevidaeasegurançasãooutrofatorimportante,poisgeramum

ambientefísicoepsicológicofavorávelaotrabalhoeaobomdesempenho,pois

“libera”ofuncionáriodeseusproblemasmaisbásicos,demodoquepossafocar

seupotencialnasatividadesdaorganização.

Umacreche,porexemplo,podefazertodaadiferençanodiaadiadeuma

funcionáriaquetenhaumbebêpequeno.Assim,elairátrabalhardespreocupadae

terámaisenergiasfísicasementaisparaencararosdesafiosprofissionais.

18.7.1.QualidadedeVida

Asorganizaçõesnãoconseguemchegaralugaralgumsempessoascompetentes

emotivadas.Eparaatrairereteressaspessoasénecessáriaumapreocupaçãocom

suaqualidadedevidanoambientedetrabalho.

Decertaforma,aqualidadedevidanotrabalho–QVT–éumconceitoque

indicaumamaiordemandadaspessoasporseubem-estaresatisfação,bemcomo

umacompreensãodasempresasdequeessaqualidadedevidaimpactana

produtividadeenaqualidadedeseusprodutoseserviços.

DeacordocomDavis,37

oconceitodeQVTrefere-seàpreocupaçãocomobem-estargeraleasaúdedostrabalhadoresno

desempenhodesuastarefas.

Dessamaneira,aqualidadenotrabalhoacabainuenciandonaprodutividadee

naqualidadedaprópriainstituição.Asinstituiçõesjáentenderamquepara

atendermelhoraosseususuárioseclientesnecessitamdarum“carinho”especial

aosseuscolaboradores.

ParaqueaorganizaçãocrieummodelodeQVTqueatendaaosseusobjetivos,

necessitapreocupar-secomosfatoresintrínsecos(internosdecadaindivíduoe

relacionadoscomoconteúdodocargo)bemcomoosfatoresextrínsecos

(externosouligadosaocontextodocargo).

DeacordocomChiavenato,38aQVTenvolveumasériedecomponentes:

ØAsatisfaçãocomotrabalhoexecutado.

ØAspossibilidadesdefuturonaorganização.

ØOreconhecimentopelosresultadosalcançados.

ØOsaláriopercebido.

ØOsbenefíciosauferidos.

ØOambientepsicológicoefísicodotrabalho.

ØAliberdadeeresponsabilidadedetomardecisões.

ØAspossibilidadesdeparticipar.

Assim,umainstituiçãoquedesejemelhoraroníveldesatisfaçãodos

funcionáriosnotrabalhodevesepreocuparemfornecerosaspectosintrínsecos

(exemplo,possibilidadesdemelhoriaprossional),bemcomoosaspectos

extrínsecos(exemplo,saláriocompatível),demodoaatrairemanterseus

melhoresfuncionários.

18.7.2.Equilíbrioorganizacional

ATeoriadoEquilíbrioOrganizacionaltentaexplicarcomoocorre,ounão,a

cooperaçãodosindivíduosemumaorganização.Deacordocomessateoria,as

pessoasrecebemincentivosdaorganização(dinheiro,benefíciosetc.)emtrocadas

suascontribuições(trabalho,dedicação,comprometimentoetc.).

Ateoriaapontaentãoqueaparticipaçãodeumapessoaocorresemprequeela

acreditarqueteráuma“recompensa”adequadaàssuascontribuições.Esses

participantespodemserdesdeempregadosaacionistas,fornecedores,clientesetc.

DeacordocomChiavenato,39

Adecisãodeparticiparéessencialnateoriadoequilíbrioorganizacional.Oequilíbrioorganizacional

reeteoêxitodaorganizaçãoemremunerarseusparticipantes(comdinheiroousatisfaçõesnão

materiais)emotivá-losacontinuaremfazendopartedaorganização,garantindocomissosua

sobrevivência.

Portanto,oequilíbrioorganizacionaléatingidoquandoascontribuições

oferecidaspelosparticipantessãoequivalentesàsrecompensasoferecidaspela

empresa.Esseequilíbriogeraumavontadeconstantedeparticipaçãoeasseguraa

continuidadedaempresa.

Semisso,osempregadosbuscamoutrosempregadores,osclientespassama

comprarprodutoseserviçosdeoutrasempresas,osfornecedorespassamanão

maisvenderparaaorganização,entreoutrosproblemas.

Osprincípiosbásicosdessateoriasão:40

ØUmaorganizaçãoéumsistemadecomportamentossociaisinter-

relacionadosdenumerosaspessoas,quesãoosparticipantesdaorganização.;

ØOsparticipantesrecebemrecompensasemtrocadascontribuiçõesquefazem

paraaorganização.

ØOsparticipantesmanterãosuasparticipaçõesnaorganizaçãosemprequeos

incentivosquelhessãooferecidosforemiguaisousuperioresdoqueas

contribuiçõesquelhessãoexigidos.

ØAscontribuiçõesoferecidaspelosparticipantessãoutilizadaspela

organizaçãoparaqueasrecompensassejaminteressantesepossíveis.

ØAorganizaçãocontinuaráexistindosomenteenquantoascontribuições

foremsucientesparaproporcionarincentivosemqualidadebastantepara

induziremosparticipantesàprestaçãodecontribuições.

Naturalmente,essesincentivosvariammuito.Podemser:bonssalários,boas

condiçõesdetrabalhoetc.Mastambémpodemserbonsdividendosaos

investidores,contratosdelongoprazoparaosfornecedores,produtosde

qualidadeparaosclientesetc.

Deveexistirentãoumequilíbrioentreascontribuiçõesdosparticipantesem

relaçãoaosincentivosrecebidos,paraqueessasparticipaçõescontinuem

ocorrendoeaorganizaçãocontinueaatuar.

Vamosveragoracomoissojáfoicobrado?

5.(Cespe/MPE-PI/Técnico/2012)Oequilíbrioorganizacionalpodeseralcançadomedianteatrocade

contribuiçõeseincentivosnarelaçãoentreaspessoaseasempresas.Nessatroca,aspessoas

colaboramparafacilitaroalcancedosobjetivosorganizacionais,easempresasfornecempara

essescolaboradoresosincentivosqueproporcionamarealizaçãodeseusobjetivospessoais.

Perfeito.EssaquestãonostrouxeumaboadeniçãodaTeoriadoEquilíbrio

Organizacional.Deacordocomessateoria,aparticipaçãodaspessoasna

organizaçãosomenteocorrequandoexisteumequilíbrioentreascontribuições

oferecidaseasrecompensasrecebidas.Ogabaritoéquestãocorreta.

18.8.ProcessodeMonitorarPessoas

Paraquepossamosentendereaprimorarotrabalhodaspessoas,necessitamos

sabercomoasatividadesestãosendoexecutadasequaissãoosresultados.Para

isso,asorganizaçõesutilizamumasériedesistemasdeinformaçãoebancosde

dados.

Assimsendo,monitoraréacompanharaspessoas,demodoafornecerdadose

informaçõessobrequaissãosuascaracterísticas,comoseutrabalhoestásendo

efetuadoequaissãoosresultadosgerados.Entretanto,essemonitoramentopode

serfeitosobduasperspectivas:abaseadanateoriaX(climadedesconançano

funcionário)eabaseadanateoriaY(climadeconfiançaeliberdade).

AabordagembaseadanateoriaXenfocaocontrolerígidodenormasdenidas

pelaorganização,compenalidadesparaosfuncionáriosquedescumpramas

exigências.Essesistemaécentralizadopelaaltacúpulaebuscaforçaros

funcionáriosaobedeceremàsregras“àforça”.

JáaabordagembaseadanateoriaYenfocaaautoavaliação,dentrodeumaideia

deexibilidadeeautonomiadofuncionárioparadenirosmétodospelosquais

deveatingirosresultadosdesejados.

Comoessemodelosebaseianacrençadequeaspessoassãoambiciosase

desejamalcançardesaosetomariniciativa,deixaoprocessodemonitoramento

funcionardemododescentralizado,deformaquecadagerênciadenaseus

métodosdecontroleemonitoramentodosdados.

Entreosdadoseinformaçõesenvolvidosnesseprocessoestão:osdadosdaforça

detrabalho(comoosdadospessoais,formaçãoacadêmica,habilidadese

conhecimentosetc.),omontantedessaforçadetrabalhoesuaevolução,entre

outros.

Assim,oobjetivoéqueexistaumsistemadeinformaçõesderecursoshumanos

emqueasinformaçõessejamcapturadaseorganizadasdeformaapossibilitaraos

gestoresatomadadedecisãonessaárea.

18.8.1.SistemasdeInformaçõesGerenciais

Asempresasprecisamsaberdiversasinformaçõessobreseusfuncionários,dados

diversoscomo:idade,capacitação,experiência,resultadospassados,faltas,salário,

dentreoutros,sãocoletadosedevemservirparaatomadadedecisão.

OsistemaqueproporcionaissoéoSIG–SistemadeInformaçõesGerenciais.

Atravésdele,aorganizaçãocoletaosdadosdequenecessitaeauxiliaos

prossionaisdeRecursosHumanosacompreendermelhorseusfuncionáriose

desenvolveraçõesmaisadequadasparatreiná-los,motivá-loseaumentarsua

produtividade.

Antigamente,essesistemacavacentralizadonaáreadeRecursosHumanose

consistiaemchasparacadafuncionário.Atualmente,asnovastecnologiasde

informaçãofacilitarammuitotantooprocessodecaptaçãoquantoodeacessoàs

informações.

DeacordocomDrucker,41

SistemadeInformaçãodeRHéumsistemautilizadoparacoletar,registrar,armazenar,analisare

recuperardadosarespeitodosrecursoshumanosdaorganização.Amaioriadossistemasde

informaçãodeRHécomputadorizada.

Basicamente,umSistemadeInformaçõesGerenciaispossibilitaareduçãode

custosedetemponaanálisedasinformações,bemcomopossibilitamaior

qualidadenatomadadedecisõesgerenciais.

Naturalmente,osgerentesdaáreadeRHpodemtomarmelhoresdecisões

quandorecebemmaisinformaçõesdosfuncionárioseemumtempomenor.Os

relatóriosgerenciaisproporcionadospeloSIGfacilitamemmuitootrabalhoda

áreadeRH.

Entreos“produtos”deumsistemadeinformaçõesgerenciais,temos:relatórios

legais(deacordocomalegislaçãotrabalhista),informaçõessobrefaltaseférias

dosfuncionários,folhadepagamento,bancodetalentosetc.

QuestõesComentadas

6.(FCC/TRT/PA/Analistaadm.//2010)Aavaliação360grausbaseia-senousodefontesmúltiplas

parafornecerfeedbacksobreascompetênciasdeumapessoa.Odesempenhoéavaliadode

maneiracircular,portodosqueinteragemcomoavaliado,inclusiveaautoavaliaçãoefetuada

pelopróprioempregado,emnúmeroidealde:

a)dozepessoas,produzindo-sedessaformadistintasinformações,enriquecendosubstancialmenteo

processoavaliativoparavislumbrarasdiversasdemandasqueoavaliadorecebedoseuambientede

trabalho;

b)quatropessoas,produzindo-sedessaformadistintasinformações,enriquecendosubstancialmenteo

processoavaliativoparavislumbrarasdiversasdemandasqueoavaliadorecebedoseuambientede

trabalho;

c)trêspessoas,produzindo-sedessaformadistintasinformações,enriquecendosubstancialmenteo

processoavaliativoparavislumbrarasdiversasdemandasqueoavaliadorecebedoseuambientede

trabalho;

d)dezpessoas,produzindo-sedessaformadistintasinformações,enriquecendosubstancialmenteo

processoavaliativoparavislumbrarasdiversasdemandasqueoavaliadorecebedoseuambientede

trabalho;

e)seispessoas,produzindo-sedessaformadistintasinformações,enriquecendosubstancialmenteo

processoavaliativoparavislumbrarasdiversasdemandasqueoavaliadorecebedoseuambientede

trabalho.

Essaquestãofoialvodeinúmerasreclamaçõesnaépocadaprova.AFCCquis

“inventar”umnúmeroidealdepessoasquedeveriamparticipardoprocessode

avaliação360º.Entretanto,nãoexistenaliteraturadosprincipaisautoresdessa

áreaumnúmeroespecíficoconsideradooideal.

AFCCtinhaconsideradoonúmerodeseispessoas.Assimsendo,ogabarito

provisóriotinhasidoaletraE.Felizmente,posteriormenteabancaanulouessa

questão.

7.(FCC/Defensoria/SP/Administrador/2010)Comrelaçãoàseleçãodepessoascomouma

responsabilidadedelinhaeumafunçãodestaff,considereasseguintesafirmativas.

I.Aescolhanaldocandidatocabeaochefeougerentedoórgãorequisitante,maso

responsávelpelaindicaçãodosmelhorescandidatoséoórgãoderecursoshumanos.

II.Atendênciaatualéparaacentralizaçãoeaconcentraçãodeserviçosderecrutamentoe

seleção,admissão,integração,treinamentoedesenvolvimento,administraçãodesaláriose

remuneraçãonasáreasdeARH.

III.AáreadeARHestásetransformandonumtipodeconsultoriainternaparapreparare

orientarosgerentesparaanovarealidade.

IV.Quandonãoexisteumadistinçãoclaraentrelinhaestaffnasorganizações,tendema

ocorrerconflitossobrequemtemautoridadedetomarasdecisõessobrepessoas.

V.Peloprincípiodaresponsabilidadedelinhaefunçãodestaffdeve-sedescentralizaragestão

daspessoasnoníveldasgerênciasdelinha,deumlado,enquantodeoutrosemantéma

funçãodeassessoriaeconsultoriainternaatravésdoórgãodeRH.

EstácorretooqueseafirmaAPENASem

a)IeIV.

b)I,IIeV.

c)I,II,IVeV.

d)II,IVeV.

e)I,II,IVeV.

Deacordocomaresponsabilidadedelinhaeafunçãodestaff,quemtomaa

decisãonalrelacionadaaocandidatoquedeveounãosercontratadoéogerente

delinha.Entretanto,oprocessoderecrutamentoeseleçãoéconduzidopela

equipedeRH.

AequipedeRHapresentaalgunscandidatosnonaldoprocessoaogerentede

linha,paraqueelefaçaaescolhafinal.Portanto,aprimeirafraseestácorreta.

Entretanto,asegundafraseestáerrada,poisatendênciaéexatamenteo

contrário–descentralização.Jáaterceirafraseestácorreta,poisaáreadeRHestá

realmentesetornandoumaconsultoriainterna.

Aquartafrasereetealgunsdosproblemasqueocorremquandoas

responsabilidadesefunçõesdosgerentesdelinhaedosórgãosdeRHnãoestão

claras.Portanto,estácorreta.Domesmomodo,aquintafraseestáperfeita.O

gabaritoémesmoaletraC.

8.(FMP/TCE-RS/Auditor/2011)Oconceitodegestãoestratégicaserefereaumtipodegestãoque

sepreocupacomosobjetivosemetasdaorganizaçãoecomodesempenhoeasformasde

atuaçãomaisadequadasparaconcretizá-los.Osprincipaismecanismoseinstrumentosda

gestãoestratégicadepessoassão,EXCETO:

a)planejamentoderecursoshumanos;

b)gestãodecompetências;

c)capacitaçãocontinuadacombaseemcompetências;

d)gestãosocial;

e)avaliaçãodedesempenhoedecompetências.

EssaquestãosebaseianadeniçãodeSchikmannquecitamos,eaúnica

alternativaquenãoserelacionacomosinstrumentoscitadospelaautoraéa

gestãosocial.Assim,aletraDéonossogabarito.

9.(Cespe/FUB/Administrador/2009)Recrutamentoéumaatividadepermanentequeseintensica

nasocasiõesemqueexistemvagasnaempresaerecomenda-sequesejaefetuadoemuma

unidadecentralizada.

Orecrutamentonãodeixadeserumaatividadederelações-públicasexternas,e

devesercontínuo,mesmoquandonãoexistemvagasabertas(hojeémuitocomum

vermosumaáreanositedasempresasnaqualoscandidatosaempregopodem

cadastrarseuscurrículosparaumapossívelvagafutura,porexemplo).

Geralmente,quemdeveexecutaroprocessoderecrutamentoéodepartamento

deRecursosHumanosdaorganização.Portanto,ogabaritoéquestãocorreta.

10.(FCC/TRT/PA/Analistaadm./2010)Agestãodepessoaséumconjuntointegradodeprocessos

dinâmicoseinterativos.Osseisprocessosbásicosdegestãodepessoassãoosprocessosde

agregar;aplicar;recompensar;desenvolver;manteremonitorarpessoas.Oprocessode

agregarpessoaséutilizadopara:

a)incluirnovaspessoasnaempresa.Podemserdenominadosprocessosdeprovisãoousuprimentode

pessoal;

b)acompanharecontrolarasatividadesdasnovaspessoasqueingressamnacorporaçãoevericarseus

resultados;

c)capacitareincrementaraadaptaçãodasnovaspessoasàsatividadesquedeverãoexercer.Incluemseu

treinamentoedesenvolvimento,gestãodoconhecimentoedascompetências;

d)incentivaresatisfazerasnecessidadesindividuaismaiselevadasdecadanovomembrodaempresa,

pormeiodoqualsegarantearetençãodetalentos;

e)criarcondiçõespsicológicassatisfatóriasparaasatividadesdaspessoas.Incluempesquisasdeclimae

programasdequalidadedevida.

Oprocessodeagregarpessoasserelacionacomaatraçãodenovaspessoaspara

aorganização.Dessaforma,aletraAestácorretaeéonossogabarito.Aprovisão

eosuprimentodepessoalseenquadramnainclusãodenovaspessoasemuma

organização,nãoéverdade?

AopçãoBserefereaoprocessodemonitorarpessoas.JáaletraCserefereao

processodedesenvolverpessoas.Incentivaresatisfazerasnecessidadesmais

elevadasserelacionamaoprocessoderecompensarpessoas.Portanto,aletraD

tambémestáerrada.

Finalmente,aletraEserefereaoprocessodemanterpessoas.Ogabaritoé

mesmoaletraA.

QuestõesPropostas

(Cespe/TCU/ACE/Gestãodepessoas/2008)Umaorganizaçãopúblicarealizouumconcursovisando

preenchervagasemalgunscargostécnicos.Paratanto,elaborouumperlcomasprincipais

característicasqueosocupantesdoscargosemquestãodeveriampossuir.Acercadessa

situação,julgueositensaseguir.

11.Operldecadaumdoscargosdeveserdenidocomfoconaexperiênciadostécnicosdaárea

degestãodepessoasdessaorganizaçãopública.

12.Porsetratardecargosdenaturezatécnica,adeniçãodospersdesejadosparaosfuturos

ocupantespoderáserfeitasemarealizaçãodeestudosoupesquisassobreascompetências

necessáriasparaseudesempenho.

13.Ascompetênciasquedevemfundamentaradeniçãodospersdosocupantesdoscargos

devemseraderentesàscompetênciasestratégicasdaorganização.

14.(Cespe/Min.Esporte/Administrador/2008)Apesardalegislaçãoespecícasobrecontrataçãode

pessoalnoserviçopúblico,aáreaderecrutamentoeseleçãodoMEpoderádirecionaroperl

desejadodosnovosservidores,pormeiodosconteúdoscobradosetiposdeavaliaçãorealizada

nosconcursospúblicos.

15.(FCC/MP/SE/Administrador/2009)Aspolíticasquedenemosprocedimentosderecrutamento,

seleção,integraçãoeambientaçãosãopolíticasde:

a)avaliaçãodedesempenho;

b)relaçõestrabalhistas;

c)remuneração;

d)provisão;

e)cargos.

16.(Cespe/Hemobras/Administrador/2008)Seumaorganizaçãoquetenhaumacultura

organizacionaladequadaasuasestratégiascompetitivasbuscamotivarseuquadrodepessoal

enecessitarecrutarpessoasparadeterminadopostodetrabalho,entãoorecrutamento

externoéomaisadequadoparaatenderaosseusobjetivos.

17.(FCC/TRT/PA/Analistaadm./2010)Aecáciadoprocessoseletivoseencontranofatode

conseguirtrazertalentosparaaempresa,tornando-acadadiamelhorcomasnovasaquisições

e/oumovimentaçõesinternas.Noentanto,algumasorganizaçõespreferemutilizaro

quocientedeseleçãoparamediraecáciadosprocessosseletivos.Àmedidaqueoquociente

deseleçãodiminui:

a)obtêm-sedadossobreasatisfaçãodoscandidatosquantoaoprocessoseletivoaplicado;

b)diminuiaeficiênciadoprocesso;

c)estabiliza-seaeficiênciadoprocesso;

d)aumentaaeficiênciadoprocesso;

e)mantém-seaeficáciadoprocesso.

18.(Cespe/Hemobras/Administrador/2008)Astécnicasdeseleçãodepessoasvisamidenticar,

comrapidezeconabilidade,ocandidatomaisadequadoaocargoaserocupado.Nesse

sentido,ostestespsicométricossãoosmaisadequadosparaavaliarconhecimentose

habilidadesdoscandidatos.

19.(FCC/TRT/PA/Analistaadm./2010)Entreosprincipaistestesdeseleçãoestãoostestesde

conhecimento,quenormalmentesão:

a)escritoseavaliamconhecimentosespecícosouhabilidades,comocapacidadedearticulaçãodo

pensamentoedoraciocínio;

b)recomendadosparavericaratitudes,comoasrelacionadasaoscargosdenaturezaoperacionaloude

produção;

c)utilizadosparaavaliaropotencialintelectual,comportamentosespecícosecertostraçosde

personalidadedoscandidatos;

d)osúnicosqueindicamdeformaclaraeprecisaapresençadeumpadrãodecomportamentoque

garantiráosucessoounãodocandidatonaexecuçãodotrabalhoquepretenderealizarnaempresa;

e)indicadosparaidentificaraspectospermanenteseconsistentesdocomportamentodeumapessoa.

20.(FCC/MP/SE/Administrador/2009)Noprocessoderecrutamentointerno,apromoçãoéumtipo

demovimentação:

a)vertical;

b)horizontal;

c)verticalehorizontal;

d)horizontaldireta;

e)indireta.

21.(FMP/TCE-RS/Auditor/2011)Noquetangeaorecrutamentoeàseleçãodepessoasnoserviço

público,pode-seafirmarque,EXCETO:

a)aAdministraçãoPúblicapodeoptarpelaincorporaçãodemaisumafaseemseusconcursos–como,

porexemplo,arealizaçãodeumcursodeformação–,comointuitodeviabilizaraidenticaçãodas

competênciasinterpessoais,estratégicasegerenciaiscujaobservaçãonãoseriapossívelpelaaplicação

deinstrumentos,comoprovasescritasecomprovaçãodetitulação;

b)oprazodevalidadedoconcursopúblicoserádeatédoisanos,prorrogávelumavez,porigualperíodo;

c)duranteoprazoimprorrogávelprevistonoeditaldeconvocação,oaprovadoemconcursopúblico

seráconvocadocomprioridadesobrenovosconcursadosparaassumircargoouemprego,nacarreira;

d)enquanto,nainiciativaprivada,orecrutamentopodeserdirigidoparaumaparcelaespecícada

população,privilegiandoinclusiveasdiferençasentreaspessoas,nosetorpúblicoorecrutamentoé

regidopeloprincípiodaisonomia;

e)oconcursopúbliconãoéexigívelparaaspessoasjurídicasdaAdministraçãoIndiretaqueexploram

atividadeeconômica.

22.(Cespe/Embasa/Administração/2010)Paraevitaroefeitohalonaavaliaçãodedesempenho,

deve-seadotarométododeescalagráfica.

23.(Cespe/Embasa/Administração/2010)Ométododeavaliaçãodedesempenhodenominado

escolhaforçadaproporcionaaoavaliadorgrandeclarezaarespeitodecomoestãosendo

mensuradososseussubordinados.

24.(Cespe/DFTRANS/Administrador/2008)Comoformadeevitardistorções,aavaliaçãode

desempenhodosmembrosdeumaequipedeveserrealizadasempreporfuncionáriosquenão

integramaequipe.

25.(Cespe/SGA/AC/Administrador/2008)Aavaliaçãodedesempenhoapresentaseusresultadosa

longoprazo,tendoosgerentescomoprincipaisbeneficiários.

26.(FMP/TCE-RS/Auditor/2011)Hádiversosmétodosparaavaliarodesempenhohumanonas

organizações.Umdessesmétodosestáidenticadofortementecomambientesdemocráticos

eparticipativosequesepreocupamtantocomoscenáriosinternosàorganizaçãoquantocom

osexternos.Trata-sedeummodeloemqueoavaliadoéfocadoporpraticamentetodosos

elementosquetenhamcontatocomele:subordinados,superiores,paresetc.Caracteriza-se

pelocompartilhamentodefeedbackscomquemexecutouotrabalhoporpartedaspessoas

diretamenteafetadaspelosresultadosproduzidos.Ométododeavaliaçãodedesempenho

mencionadoéo:

a)deEscalasGráficas;

b)de360Graus;

c)deAvaliaçãoporResultados;

d)deEscolhaForçada;

e)deIncidentesCríticos.

27.(FMP/TCE-RS/Auditor/2011)Emumprocessodeavaliação,oefeitodehaloacontecequando:

a)oavaliadorlevaemcontaapenasosfatosacontecidosnumcurtoespaçodetempoanteriorao

processoavaliativo;

b)oavaliadorvalorizademasiadamenteasqualidadesquepoderãorequeridasparaodesempenhoem

outrasfunções;

c)oavaliadorsedeixalevarporvaloresouatitudesqueoavaliadopossuiforadoambientedotrabalhoe

quepodeminuenciaroavaliadortantodeformapositivaquantonegativanahoraderegistraroseu

desempenho;

d)oavaliadorsedeixalevarporalgumacaracterísticadoavaliado,queomarcoudeformatãosignificativa

queoimpededeinterpretarasdemaiscaracterísticascomneutralidadeeclareza;

e)oavaliadorconfunderesultadosconcretamenterealizadoscomcaracterísticaspotenciaisque

identificamoavaliado.

28.(FMP/TCE-RS/Auditor/2011)Emmuitasáreasdaorganização,oscargospoucodescrevemoque

aspessoasefetivamenteestãofazendo,mastãosomenteoqueseesperaquefaçam.Em

funçãodisso,pode-searmarqueoqueaspessoasagregamdevalorparaaorganização

independentementedesuasfunçõesecargosqueocupamemtermosformaisé

denominado(a):

a)entrega;

b)padrãodecomplexidade;

c)espaçoocupacional;

d)competência;

e)habilidade.

29.(FMP/TCE-RS/Auditor/2011)Aavaliaçãodedesempenhoéumsistemaformaldegerenciamento

queprovêaavaliaçãodaqualidadedodesempenhoindividuale/ouinstitucionalemuma

organização.Assim,elapodevisarapenasaoindivíduooutambémàsequipes,àsáreaseà

organização.Noníveldoindivíduo,aavaliaçãodedesempenhopermite:

a)maioralinhamentodasunidadesdaorganizaçãocomsuasmetaseobjetivosestratégicos;

b)odesenvolvimentodeumavisãosistêmicaporpartedosindivíduosemrelaçãoàorganização;

c)obtersubsídiosparaaprogressãonacarreira,combaseemcompetênciasedesempenho,entre

outrosbenefícios;

d)odesenvolvimentodoespíritodeequipe;

e)apercepçãodainterdependênciaentreáreasepessoas.

30.(FCC/Infraero/Administrador/2009)Comrelaçãoaodesenhodecargoscomoumaestratégia

essencialnaáreaderecursoshumanos,considere:

I.Nodesenhodeumcargodeve-sedistinguirastarefas,asatribuiçõeseafunçãoqueo

compõe.

II.Odesenhodecargoséumprocessoqueconsisteemenumerarastarefasouatribuiçõesque

ocompõemequeotornamdistintodetodososoutrosexistentesnaorganização.

III.Odesenhodecargosdeveincluiradeniçãodoconteúdo,dosmétodoseprocessosde

trabalho,assimcomoasrelaçõesdesubordinaçãoesupervisãoassociadasaocargo.

IV.Odesenhodeumcargoéaúltimaetapadeumprocessoqueinclui,emprimeirolugar,a

descriçãodocargoe,emsegundolugar,aanálisedoimpactodessenovocargono

organogramadaorganização.

V.Nodesenhodeumcargodeve-selevaremconta,emprimeirolugar,aremuneração

desejada,depoisaqualicaçãoexigidae,porúltimo,asuaposiçãonoorganogramada

organização.

EstácorretooqueseafirmaAPENASem

a)IeII.

b)IIeIV.

c)IeII.

d)I,IeIV.

e)I,IeII.

31.(FCC/TRT/PA/Analistaadm./2010)Ométododeavaliaçãodedesempenhoqueconsisteem

quatroetapas:entrevistainicialparaavaliarodesempenhoeosfatoresqueafetarampositiva

ounegativamenteoresultadonal;entrevistadeanálisecomplementarparaobternovas

informaçõespertinentes;planejamentodasprovidênciasnecessáriaseacompanhamento

posteriordosresultados;édenominado:

a)escalasgráficas;

b)pesquisadecampo;

c)listasdeverificação;

d)escolhaforçada;

e)métododosincidentescríticos.

32.(FGV/Senado/Administrador/2008)Aavaliaçãodedesempenhovem,aolongodotempo,

recebendoumnúmeroexpressivodecontribuiçõesdeautoreseprossionais,certosdequea

avaliaçãoconvencionalperdeespaçoparanovosinsumostécnicos.Umbomexemploéa

avaliaçãode360º,queérelativamenterecentee,numcertosentido,revolucionouaáreade

recursoshumanos.Assinaleaalternativaquemelhorexpressaessaformadiferenciadade

avaliarpessoasnaorganização.

a)Aavaliaçãoé360ºporenvolverapessoanosmaisvariadosaspectosdaestruturadesuapersonalidade;

aavaliaçãoéfeitapelosuperiorimediatoepelossubordinados,enãoenvolveasopiniõesdepessoas

deoutrasorganizaçõesquelidamdealgumaformacomapessoaemavaliação.

b)Essaavaliaçãovisaaumaanálisecircular,ouseja,apenasaspessoasqueestãopróximasaoavaliado

podemresponderaosquestionamentosconvencionais.Nessecaso,aavaliaçãoébasicamentea

respeitodasatitudesecomportamento,masnãoháimpedimentosaumaavaliaçãosobrea

competênciatécnica.Nãoéumaavaliaçãoquedenaodestinodapessoa,massimalgoaser

consideradopelosuperiorimediato.

c)Partedaideiadequequememiteosfeedbackssãopessoassituadasemdiferentesposiçõesaoredor

doreceptorequefazempartedasuarededecontatos:superiorimediato,subordinadoseaté

mesmopessoasdeoutrasorganizaçõesquelidam,dealgumaforma,comapessoaemavaliação,

como,porexemplo,fornecedores.

d)Partedaideiadequequememiteosfeedbackssãopessoassituadasemdiferentesposiçõesaoredor

doreceptor,masosuperiorimediatorecolheaavaliaçãorealizadapelaspessoaspróximaseas

consideraounãocomosuporteàsuaprópriaavaliaçãoque,essasim,teráimportâncianavidafutura

doavaliado.

e)360ºéapenasetãosomenteumrótuloparaavaliaçãodedesempenhodocorpofuncional.As

questõesgiramemtornodapessoacomopessoaedapessoacomomembrodaestruturasocialda

organização,ouseja,nãoéumaavaliaçãocomquesitosapenasdaatividaderelativaaotrabalho,mas,

essencialmente,comquesitossobreapersonalidadedapessoa,seusdesejospessoaisprossionais,

educacionaiseassimpordiante.

33.(FCC/MP/RS/Administrador/2008)Atécnicadeavaliaçãodedesempenho360grausconsiste:

a)numaavaliaçãododesempenhodetodososfuncionáriosdeummesmonívelhierárquicodentrode

umaorganização,realizadoporauditoriaexternaquegarantaocaráterobjetivoeneutrodoprocesso;

b)numacomparaçãoentreapercepçãodosparesnotrabalho,dosuperiorimediato,dossubordinados

diretos,clientes,fornecedores,eapercepçãoqueoocupantedocargotemdesimesmo,noexercício

dasatividades;

c)numacomparaçãoentreosdesempenhosefetivosemtermosdeprodutividade,detodosos

funcionáriosdeummesmosetor,tendocomoobjetivoaimplementaçãodeumprocessode

downsizing;

d)emumasequênciadeavaliaçõesdossuperiorespelossubordinados,visandoaconstruçãode

indicadoresdeprodutividadeesatisfaçãocomasrelaçõesinterpessoaisnumdeterminadosetorda

organização.

e)emumprocessodebrainstormingenvolvendotodososfuncionáriosdeumaorganização,visando

subsidiaroplanejamentoestratégicosituacional.

34.(Cespe/Iema/ES–Administrador/2007)Oobjetivodapolíticadedesenvolvimentoderecursos

humanoséapromoçãodetreinamentospadronizados,comconteúdoidênticoparatodosos

membrosdaorganização.

35.(Cespe/Antaq/Anal.adm./2009)Dentrodeumaorganização,aaprendizagemdaspráticas

relevantesparaoexercíciodesuasfunçõesporumtrabalhadorsóocorreseestefor

submetidoasituaçõesformaisdeinstruçãooutreinamento.

36.(FCC/TRT/PA/Analistaadm./2010)Oprocessodetreinamentoécompostodequatroetapas:

a)diagnósticodenecessidades;definiçãoderecursos;aplicaçãoeacompanhamento;

b)programação;organizaçãodosrecursosdisponíveis;implantaçãoourealizaçãodotreinamentoemsi

eavaliaçãodosresultados;

c)elaboraçãodoconteúdo;escolhaderecursos;convocaçãoeaplicação;

d)comunicaçãoaosparticipantes;definiçãodasdinâmicas;aplicaçãoeavaliaçãodereação;

e)levantamentodenecessidades;análisedasnecessidades;deniçãodeindicadoresdeaprendizageme

implantaçãodaprogramaçãodetreinamento.

37.(Cespe/Embasa/Administração/2010)Aoconstatarqueexistepoucaversatilidadedos

colaboradoresnodesempenhodasatividadesdeumaequipe,ogestorderecursoshumanos

temumindicadoraprioridanecessidadedecapacitação.

38.(FCC/TRT/PA/Analistaadm./2010)Natécnicadeinstruçãoprogramadapode-seutilizardois

sistemas:olinearouextrínsecoeoramicadoouintrínseco.Osistemalinearparteda

premissadeque:

a)oserrosinibemoaprendizadodoaluno;

b)oserrossãoessenciaisparaaaprendizagemdoaluno;

c)errosnãodevemocorrer;

d)asquestõesdoprogramadevemsersobaformademúltiplaescolha;

e)asrespostasdevemserdissertativas.

39.(FCC/Metrô/Administração/2008)Olevantamentodasnecessidadesdetreinamentoe

desenvolvimentodetrabalhadoresdiagnosticaascarênciascognitivaseinexperiências

relativasaotrabalhoemdiferentescenários.Ocenárioquerepresentasituaçõesemquea

necessidadejáestápresente,ocasionandoproblemasreais,édenominado:

a)teórico;

b)prospectivo;

c)reativo;

d)metodológico;

e)motivacional.

40.(Cespe/Iema/ES–Administrador/2007)Asatividadesdetreinamentoedesenvolvimento

devemseraplicadasapenasquandoforemapresentadasdeciênciasnaavaliaçãode

desempenhodofuncionário.

41.(Cespe/AGU/Administrador/2010)Ogerenteconsegueoequilíbrioorganizacional,noque

tangeàgestãodepessoas,quandoexisteproporcionalidadeentreosbenefíciosofertados

pelaorganizaçãoeoscustospessoaisdesembolsados.

Gabaritos

1.E

15.D

29.C

2.C

16.E

30.C

3.C

17.D

31.B

4.E

18.E

32.C

5.C

19.A

33.B

6.X

20.A

34.E

7.C

21.E

35.E

8.D

22.E

36.B

9.C

23.E

37.E

10.A

24.E

38.A

11.E

25.E

39.C

12.E

26.B

40.E

13.C

27.D

41.C

14.C

28.A

Bibliografia

Chiavenato,Idalberto.Gestãodepessoas:eonovopapeldosrecursoshumanosnas

organizações.2aed.RiodeJaneiro:Elsevier,2004.

______.Introduçãoàteoriageraldaadministração.8aed.RiodeJaneiro:Elsevier,

2011.

Costin,Claudia.AdministraçãoPública.RiodeJaneiro:Elsevier,2010.

Ivancevich,JohnM.HumanResourceManagement.7aed.Boston:Irvin/McGraw-

Hill,1998.

Lima,CarlosAlbertoNogueirade.AdministraçãoPúblicaparaconcursos.Riode

Janeiro:Elsevier,2005.

Marras,JeanPierre.Administraçãoderecursoshumanos:dooperacionalao

estratégico.14aed.SãoPaulo:Saraiva,2011.

Mello,LucianaHeringerFreitasde.Umaavaliaçãodoimpactodotreinamentona

produtividadedotrabalho.RiodeJaneiro:FGV,2009.

Pires,AlexandreKalil,etal.Gestãoporcompetênciasemorganizaçõesdegoverno.

Brasília:Enap,2005.

Schikmann,Rosane.Gestãoestratégicadepessoas:basesparaaconcepçãodo

cursodeespecializaçãoemgestãodepessoasnoserviçopúblico.In:Gestãode

pessoas:basesteóricaseexperiênciasnosetorpúblico,MarizauraReisdeSouza

Camões,MariaJuliaPantojaeSandroTrescastroBergue,9-29.Brasília:Enap,

2010.

VilasBoas,AnaAlice,eRuiOtávioBernardesdeAndrade.Gestãoestratégicade

pessoas.1aed.RiodeJaneiro:Elsevier,2009.

Capítulo19

GerênciadeProjetos

DeacordocomoGuiaPMBOK1(ProjectManagementBodyofKnowledge,ou

GuiadoConjuntodeConhecimentosemGerenciamentodeProjetos),umprojeto

é:

umesforçotemporárioempreendidoparacriarumproduto,serviçoouresultadoexclusivo.

JáCaupin2defineumprojetocomo:

umprocessoúnico,consistindoemumgrupodeatividadescoordenadasecontroladascom

datasparainícioetérmino,empreendidoparaalcancedeumobjetivoconformerequisitos

específicos,incluindolimitaçõesdetempo,custoerecursos.

Outradeniçãodagestãodeprojetosseriaadeumaassociaçãode

planejamento,organização,direçãoecontrolederecursosparaatingirobjetivos

decurtoemédioprazos.

Portanto,umprojetoéumesforçoque,pordenição,nãotemumaideiade

continuidadeoutempoindeterminado.Éalgoqueteráumtempodeterminado

deduração.Ouseja,sealguémcomeçaumtrabalhoquenãotemdataparaacabar,

nãopoderiachamarissodeumprojeto.3

Entretanto,issonãoquerdizerqueumprojetotenhadeserdecurtaduração.

Umprojetopodeterumprazolongodeoperação.Existemestimativasdeque

algumaspirâmidesdoEgito,umdosprojetosmaisantigosqueconhecemos,

levaramcercade50anosparaficaremprontas.4

Outropontoimportanteéasuaexclusividade.Umprojeto,pordenição,

criaumproduto,serviçoouresultadoexclusivo.5Entretanto,issonãoquerdizer

quenãopossamexistirelementosrepetitivos.Vamosverumcasoprático?

Imagineumaconstrutoradeprédios.ElaconstróiprédiosresidenciaisnoRio

deJaneiro.Osedifíciosresidenciaisconstruídospodemutilizarmateriais

semelhantes,amesmaequipeeconceitosiguais,masterãosemprealgumas

característicasdiferentesunsdosoutros,nãoéverdade?

Mesmoquesejasomentealocalizaçãodiferente,algosempretornaráaquele

projetodistintodoanterior.Porisso,falamosqueumadascaracterísticasdos

projetoséseremexclusivos.

AgoravamosexplicarmelhoroqueéPMBOK.Essasiglarepresentaumguia

dasmelhorespráticasdaáreadegerenciamentodeprojetos.Abordadiversos

conhecimentosnecessáriosparaumaboagestãodeprojetoseéutilizadopor

gestoresdessaárea.Atualmente,estánasuaquartaedição(publicadaem2008),

masaindaencontramosquestõesdeconcursosqueabordamsuaterceiraedição.

OPMBOKépublicadopeloProjectManagementInstitute–PMI,queéuma

organizaçãodedicadaàpromoçãodousodetécnicasedasmelhorespráticasem

gerenciamentodeprojetosemdiversosramos.

Vamosvercomoessetemajáfoicobrado?

1.(FCC/TRT24a/MS/Téc.adm./2011)SegundoSchafferProchonw,projetoéumempreen-dimento

planejadoqueconsisteemumconjuntodeatividadesinter-relacionadasecoordenadas,sendo

umadesuascaracterísticasa:

a)exclusividade;

b)imperatividade;

c)integralidade;

d)continuidade;

e)generalidade.

Umadasprincipaiscaracterísticasdeumprojetoéaexclusividade.Ogabaritoé

aletraA.Asalternativasrestantesnãoseenquadramnascaracterísticasdos

projetos.

Figura19.1

Ogerenciamentodeprojetos,deacordocomoGuiaPMBOK,6é“aaplicação

deconhecimentos,habilidades,ferramentasetécnicasàsatividadesdoprojetoa

fimdeatenderaseusrequisitos”.

19.1.DiferençasentreProjetoseOperações

Umtemaquemuitasvezesépedidopelasbancaséadiferençaentreumprojeto

eumaoperação.Aprincipaldiferençaéaseguinte:umprojetoétemporário,

enquantoumaoperaçãoéumtrabalhocontinuo.

Ouseja,umprojetotemdataparaacabar.Alémdisso,produzumprodutoou

serviçoexclusivo,único.Jáumaoperaçãoentrega,basicamente,sempreomesmo

conjuntode“produtos”.

Asoperaçõessãoumafunçãoorganizacionalquerealizaaexecuçãocontínuadeatividadesque

produzemomesmoprodutooufornecemumserviçorepetitivo.Exemplosincluem:

operaçõesdeprodução,defabricaçãoedecontabilidade.

Dessamaneira,umprojetonãogeraumesforçocontínuo.Maspode,sim,

auxiliaraempresanoalcancedediversosobjetivosestratégicos,domesmomodo

queasoperações.

19.2.StakeholdersouPartesInteressadas

Outroconceitomuitoimportanteparaagestãodeprojetosestáligadoaos

diversos“públicos”afetadosporumprojeto.Umbomgerenciamentodeprojetos

deveidenticarquaissãoaspessoaseorganizaçõesquepodemserafetadas

(negativamenteoupositivamente),entenderquaissãosuasnecessidadese

demandasegerenciarasexpectativasdessaspessoasegrupos.

DeacordocomoPMBOK,7aspartesinteressadaspodemserdefinidascomo:

Pessoaseorganizações,comoclientes,patrocinadores,organizaçõesexecutoraseopúblico,que

estejamativamenteenvolvidasnoprojetooucujosinteressespossamserafetadosdeforma

positivaounegativapelaexecuçãooutérminodoprojeto.Elaspodemtambémexercerinuência

sobreoprojetoesuasentregas.

Essesstakeholderspodemserdesdeospatrocinadoresdoprojetocomotambém

comunidadeslocaisquepodemserafetadasporalgumaobradoprojeto,por

exemplo.

Comoessaspartesinteressadaspodeminuenciarnoandamentodoprojeto,

deveexistirumgerenciamentodessasinuênciasparaqueoprojetosejabem-

sucedido.

Naturalmente,ainuênciadessaspartesinteressadasvariamuito.Um

patrocinador,normalmente,teráuma“voz”maiornadeniçãodosobjetivosde

umprojetodoqueosindicatodostrabalhadores,porexemplo.

Alémdisso,aspartesinteressadasnãosãosempreasmesmasenemse

preocupamcomosmesmosaspectosdoprojeto–umsindicatopodeestar

preocupadocomascondiçõesdetrabalho,enquantoaassociaçãodemoradores

podereclamardapoluiçãooudeinterrupçõesnotrânsitodeveículos,por

exemplo.

Aidenticaçãodessesstakeholdersémuitoimportante,pois,senãosabemos

quaissãoelesequaissãosuasdemandaseobjetivos,podemosterdealteraro

projetonomeio,enfrentandoatrasos,aumentosdecustosetc.

Estamosvendoasituaçãonasobrasdeestádioseequipamentosparaeventos

comoCopadoMundoeOlimpíadas.Comomuitasvezesosconstrutoresnão

levavamemconsideraçãoasdemandasdoComitêOlímpicoInternacionaloudas

FederaçõesInternacionais,tiveramdefazerdiversasalteraçõesnosprojetos,

ocasionandomaiorescustosedesperdíciodetempo.

OPMBOKapontacomoosprincipaisstakeholders(oupartesinteressadas):8

ØClientes/usuários.

ØPatrocinador.

ØGerentesdeportfólios/comitêdeanálisedeportfólios.

ØGerentesdeprogramas.

ØEscritóriodeprojetos.

ØGerentesdeprojetos.

ØEquipedoprojeto.

ØGerentesfuncionais.

ØGerenciamentodeoperações.

ØFornecedores/parceiroscomerciais.

Ainuênciadaspartesinteressadaspodeserpositivaounegativaaoprojeto.A

funçãodeumgerentedeprojetosébalancearosdiversosinteressesenvolvidose

buscarumacomunicaçãoabertacomtodososstakeholders,demodoqueoprojeto

tenhasucesso.

19.3.GruposdeProcessosdosProjetos

Ogerenciamentodeprojetosenvolveautilizaçãodediversosconhecimentos,

técnicasehabilidadesnecessáriasaoatingimentodosobjetivosdoprojetoe,assim,

alcançaroseusucesso.Essesconhecimentosserãoutilizadosemdiversos

processosrelacionadoscomagestãodoprojeto.

Essesprocessossãosomatóriosdeatividadesetarefasinter-relacionadasque

sãoexecutadasembuscadeumobjetivodenido.Alémdisso,dizemosqueos

processosnogerenciamentodeprojetossãoiterativospornatureza,ouseja,são

executadosrepetidasvezeseemdiversasfasesdoprojeto.

Assim,paraquepossamosgerenciarosprojetos,precisamosconhecere

gerenciardeformaecazosseusprocessosprincipais.DeacordocomoPMBOK,

oscincoprincipaisgruposdeprocessossão:9

ØIniciação–realizadosparadenirumnovoprojetoouumanovafasedeum

projetoexistenteatravésdaobtençãodeautorizaçãoparainiciaroprojeto

ouafase.

ØPlanejamento–realizadosparadeniroescopodoprojeto,renaros

objetivosedesenvolverocursodeaçãonecessárioamdealcançaros

objetivosparaosquaisoprojetofoicriado.

ØExecução–realizadosparaexecutarotrabalhodenidonoplanode

gerenciamentodoprojetodemodoasatisfazerassuasespecificações.

ØMonitoramentoecontrole–necessáriosparaacompanhar,revisareregular

oprogressoeodesempenhodoprojeto,identicartodasasáreasnasquais

serãonecessáriasmudançasnoplanoeiniciarasmudançascorrespondentes.

ØEncerramento–executadosparanalizartodasasatividadesdetodosos

gruposdeprocessos,visandoencerrarformalmenteoprojetooufase.

Figura19.2–Grupodeprocessos

Cabelembrarqueessesgruposdeprocessosnãosãofasesdoprojeto!Além

disso,osgruposdeprocessosnãosãosequenciais.Muitasvezes,essesprocessos

ocorremsimultaneamente,principalmenteosdeexecuçãoecontrole.10

Aseguirvocêspodemvercomoosgruposdeprocessosocorremintercalados

comoutrosprocessosduranteociclodevidadeumafaseouprojeto.

Figura19.3–Gruposdeprocessosdeacordocomopassardotempo

(Fonte:PMI)

Principalmentequandoestamosnosreferindoaosprojetosgrandese

complexos,osgestorespodem“quebrá-los”emfasesousubprojetosdistintos.

Alémdisso,lembrem-sedequetodososgruposdeprocessoscitadospelo

PMBOKdevemserrepetidosacadafasedoprojeto.

Vamosverquaissãoosaspectosprincipaisdecadagrupo?Aprimeirafaseéa

deiniciação.Normalmente,umprojetoocorredeacordocomumademanda,

desejoouoportunidadequeseapresentaaumaorganização.

Nessaetapa,aorganizaçãoiráanalisarseoprojetorealmenteépromissor,sea

organizaçãoteráosrecursosnecessáriosparaexecutá-lo,seoscustossão

adequadoseosprazosfactíveis.

Nessegrupodeniremoseautorizaremosoprojetooufasedoprojeto.Entreos

processosdeiniciaçãoestãoincluídos:11

ØDesenvolverotermodeaberturadoprojeto–documentoqueautorizao

iníciodeumprojeto;

ØIdenticaraspartesinteressadas(stakeholders)–estasserelacionamcom

qualquerpessoaouentidadequeteminteresse(ouéimpactada,sejadeforma

positivaounegativa)peloprojeto.12Assim,incluem,porexemplo,

fornecedores,clientes,forçadetrabalho,comunidadesvizinhas,ONGs,

autoridadesepolíticosetc.

Vamosvercomoessetemajáfoicobrado?

2.(FCC/TRF1aRegião–Anal.adm./2011)Nafasedeiniciaçãodeumprojeto,antesdetudo,deve-se:

a)escolheraspessoascertasparaaimplantaçãoeavaliaçãodoprojeto;

b)decidirseumprojetodeveseriniciado,entreváriospossíveis;

c)definirasatividadesnecessáriasparadesenvolvimentodoprodutoaserentregue;

d)detalharoescopoeosrequisitosbásicosdoprojeto;

e)elaborardetalhadamenteasinformaçõessobreoprojeto.

Nafasedainiciação,oprojetoaindaestásendoavaliado,inclusivenasua

viabilidadeounão!Muitasvezesexistemdiversasoportunidadesentreosprojetos

disponíveis.

Dessaforma,cadaprojetoseráavaliado,eseráescolhidoaquelequetrouxera

maiorexpectativaderetorno.Portanto,aalternativacorretaéaletraB.

Continuando,opróximoprocessoéodePlanejamento.Nessemomento,o

gestordeprojetoscomeçaadetalharocronogramadeatividades,os“produtos”

quedevemserentregues(umapraça,umprédioetc.),aspessoasquedevemser

contratadasoualocadasnoprojeto,osrequisitosdequalidadequedevemser

atendidosetc.

DeacordocomoPMBOK,grupodeprocessosdeplanejamentoinclui:

ØDesenvolveroplanodegerenciamentodoprojeto–documentaçãodasações

necessáriasparadefinir,coordenareintegrartodososplanosauxiliares.

ØColetarosrequisitos–documentarasnecessidadesdaspartesinteressadas.

ØDefiniroescopo–descriçãodetalhadadoprojetoedoproduto.

ØCriaraestruturaanalíticadoprojeto(EAP)–consisteemsubdividiras

“entregas”emcomponentesmenoresparaquepossasermaisfacilmente

gerenciável.

ØDefinirasatividades.

ØSequenciarasatividades.

ØEstimarosrecursosdasatividades.

ØEstimaraduraçãodasatividades.

ØDesenvolverocronograma.

ØEstimaroscustos.

ØDeterminaroorçamento.

ØPlanejaraqualidade.

ØDesenvolveroplanoderecursoshumanos.

ØPlanejarascomunicações–denirasnecessidadesdeinformaçãodecada

parte.

ØPlanejarogerenciamentoderiscos.

ØIdentificarosriscos.

ØRealizaraanálisequalitativadosriscos–avaliaçãodaprobabilidadeda

ocorrênciaeimpacto.

ØRealizaraanálisequantitativaderiscos–analisarnumericamenteoefeito

dosriscosnoprojeto.

ØPlanejarrespostasariscos–desenvolvimentodeopçõeseações.

ØPlanejaraquisições–documentarasdecisõesdecompradoprojeto.

Comosprocessosdeiniciaçãoeplanejamentocumpridos,oprojetoentranos

processosdeexecução.Nessemomento,todososrecursossãoempregadospara

queosobjetivossejamalcançadoscomoplanejado.

DeacordocomVargas,13qualquererronosprocessosanteriorescaevidente

nessafase.Boapartedosesforçoshumanos,dotempoedosgastosdoprojetoé

despendidanessesprocessos.

Ogrupodeprocessosdeexecuçãoinclui:

ØOrientaregerenciaraexecuçãodoprojeto.

ØRealizaragarantiadaqualidade.

ØMobilizaraequipedoprojeto.

ØDesenvolveraequipedoprojeto.

ØGerenciaraequipedoprojeto.

ØDistribuirinformações.

ØGerenciarasexpectativasdaspartesinteressadas.

ØRealizaraquisições.

Aomesmotempoemqueocorreaexecução,passaaocorrerocontrole.Senão

controlamosemonitoramos,nãotemoscomosaberseestamosatingindoos

resultadosnecessários,nãoémesmo?

Oobjetivoéqueosprocessosdecontrolepossamoferecerinformaçõessobreo

andamentodaexecução,demodoqueosgestoresdoprojetoconsigamentender

ondeestãoerrandoeoquepodesermelhorado.

Dessemodo,comparamososobjetivosplanejadoscomosresultadosatingidos.

Comisso,sãopropostasmedidascorretivassemprequenecessário.

Ogrupodeprocessosdemonitoramentoecontroleinclui:

ØMonitorarecontrolarotrabalhodoprojeto.

ØRealizarocontroleintegradodemudanças–processodeavaliaçãodetodas

assolicitaçõesdemudanças.

ØVerificaroescopo.

ØControlaroescopo.

ØControlarocronograma.

ØControlaroscustos.

ØRealizarocontroledaqualidade.

ØReportarodesempenho.

ØMonitorarecontrolarosriscos.

ØAdministrarasaquisições.

Finalmente,todoprojetodevechegaraom.Nessemomentoosresultadossão

avaliados,osdocumentosrelativosaoprojetosãoclassicadosearquivados,e

busca-seaprenderomáximocomasexperiênciasvividasduranteoprojeto.

Nessegrupo,ocorreaentregadoprodutoouserviçoaocliente,e,

naturalmente,asatividadesdoprojeto(oudealgumafase)sãoencerradas.O

grupodeprocessosdeencerramentoinclui:

ØEncerraroprojetoouafase.

ØEncerrarasaquisições.

19.4.ÁreasdoConhecimentonoGerenciamentodeProjetos

DeacordocomoPMBOK,existemnoveáreasdoconhecimentonagestãode

projetos:integração,escopo,tempo,custos,qualidade,recursoshumanos,

comunicações,riscoseaquisições.

Aintegraçãorefere-seaosaspectosdeunicação,consolidação,articulaçãoe

açõesintegradorasquesãonecessáriasparaqueumprojetotenhasucesso.14

Esseprocessogeralmenteenvolveasescolhaseconcessõessobreobjetivos

conitanteseaadministraçãodasinterdependênciasentreasáreasenvolvidasno

projeto.

Decertaforma,oobjetivoégarantirquetodasasáreasestejamintegradaseque

asnecessidadesdosenvolvidossejamatendidas.15

Ogerenciamentodoescopodevegarantirqueoprojetoiráconcluir

exatamenteotrabalhorequerido.Seoprojetopedeaconstruçãode100kmde

estradas,nãodevemosconstruir120km,nãoémesmo?

Cabeaquidiferenciaroescopodoproduto(queserefereàscaracterísticasdo

produtoemsi)doescopodoprojeto(queenvolveaadministraçãodesua

execução,ouseja,ogerenciamentodoprojeto).

Portanto,tantooescopodoprodutoquantooescopodoprojetoestão

incluídosnogerenciamentodoescopo.Quantomaisdetalhadooescopo,mais

complexocagerenciaroescopo.Portanto,devemoscuidarparaqueesseescopo

nãoestejadetalhadoemdemasia.

Nadeniçãodoescopoéqueocorreoprocessodedivisãodasentregasdo

projetoem“partes”oucomponentesmenores,demodoafacilitaro

gerenciamento.

EssadivisãogeraoquechamamosdeEstruturaAnalíticadoProjeto–EAP(ou

WorkBreakdownStructure–WBS).Adivisãodasentregaséfeitadeforma

hierárquicaedeveserfeitademodoanãogerar“peças”pequenasdemais,pois

podeocasionarumaperdadeesforçodegerenciamento,emvezdefacilitaro

trabalho.

DeacordocomoPMBOK,16aEstruturaAnalíticadoProjetoé

Umadecomposiçãohierárquicaorientadaàentregadotrabalhoaserexecutadopelaequipedo

projetoparaatingirosobjetivosdoprojetoecriarasentregasnecessárias.Elaorganizaedeneo

escopototaldoprojeto.

Ogerenciamentodoescopoconsisteemcincoprocessosprincipais:ainiciação,

oplanejamento,adefinição,averificaçãoeocontroledasmudanças.

Vamosvercomoessetemajáfoicobrado?

3.(FCC/ALESP/SP/Gestãoprojetos/2010)Aáreadegerenciamentodeprojetosquegarantequeo

projetoincluitodoesomenteotrabalhorequerido,paraquesejacompletadocomsucesso,éa

de:

a)integração;

b)tempo;

c)custo;

d)escopo;

e)comunicações.

Comovimos,aáreadeconhecimentosdoprojetoquegarantequeoprojetoirá

executarexatamenteoqueésolicitadoéaáreadoescopo.Questãotranquilaessa,

nãoémesmo?OgabaritoéaletraD.

Seguindocomnossamatéria,ogerenciamentodotempobuscagarantirqueo

projetosejaconcluídonotempoplanejado.Nesseprocesso,estãoenvolvidosa

deniçãoeosequenciamentodasatividadeseadeniçãoeomonitoramentode

umcronograma.

Ocontroledosprazoséumgrandefocodeconitosdentrodeumprojeto.

Nesseprocessodegestãodotempo,devemossequenciarasatividadesealocaro

temponecessárioparacadatarefa.

Comocronogramaconcluído,devemoscontinuamentemonitoraro

cumprimentodosprazosparaqueoprojetonãoatrase.

Ogerenciamentodecustosdoprojetobuscagarantirqueoprojetoseja

concluídodentrodoorçamentoprevisto.Ouseja,queodinheiroreservadoe

disponívelparaoprojetosejasuficienteparaqueoprojetosejaconcluído.

Umcontroleefetivodoscustosdoprojetoéfundamentalparaqueesteseja

viávelequeaorganizaçãoconsigasobrevivernanceiramente.Seumprojeto

“estourar”oscustosprevistos,podeserqueaempresatenhaprejuízocomo

projeto,oupior–nãoconsigaterminá-lo.

DeacordocomVargas,17asmaiorescausasdefalhasnagestãodoscustossão:

•Interpretaçãoerradadotrabalhoaserrealizado.

•Omissãonadefiniçãodoescopo.

•Cronogramadefinidocompobrezaouexcessivamenteotimista.

•Fracassonaavaliaçãoenaquantificaçãodosriscos.

•Estruturaanalíticadoprojetomaldefinida.

•Parâmetrosdequalidademal-estabelecidos.

•Fracassonaestimativadoscustosindiretoseadministrativosdoprojeto.

Dentrodesseprocesso,teremosoplanejamentoeadeniçãodosrecursos

necessários,adefiniçãodoorçamentoeocontroledecustos.

Paraqueoprojetoentregueoprodutodentrodasespecicaçõessolicitadasno

projeto,existeogerenciamentodaqualidade.Ouseja,deveexistirapreocupação

ementregaro“produto”deacordocomasnecessidadeseexpectativasdocliente.

Apesardemuitasvezesnospreocuparmoscomumaqualidadeinsuciente(que

serárejeitadapeloclientedoprojeto),umaqualidadeexcessivatambémpodeser

problemática,poislevaráoprojetoatercustosquetalveznãoestejamcobertosno

planejamentoenãosejamvistoscomorelevantespelocliente.

Cabeaquiressaltarquequemdeneosrequisitosdequalidadeésempreo

cliente,enãoalgumdepartamentodaempresa.Dentrodesseprocesso,estão

relacionados:oplanejamento,omonitoramentoeocontroledaqualidade.

Nogerenciamentodosrecursoshumanos,busca-segarantiramelhor

utilizaçãopossíveldaspessoasquetrabalhamnoprojeto.Ouseja,autilizaçãodas

pessoascertas,naquantidadecertaenoslugarescertos.

Asrelaçõeshumanassãofundamentaisnosucessodequalquerprojeto,poissem

pessoasqualicadasemotivadasparaotrabalhoserámuitodifícilatingiros

objetivos.

Principalmentenoscasosdeprojetosquenecessitemdeprossionaisdealta

capacitação,acontrataçãoforadotempocertodeprossionaispodeaumentaro

custoe“estourar”osprazosdosprojetos.

Dentrodesseprocesso,encontramosoplanejamento,otreinamentoeo

desenvolvimentodasequipesdetrabalho.

Outroaspectonecessárioparaqueumprojetotenhasucessoéqueaspessoas

tenhamacessoàsinformaçõesnecessáriasaoseutrabalhoepossamtrocarideias.

Ogerenciamentodacomunicaçãobuscaassegurarisso.Comomuitosoutros

processosdependemdeumacomunicaçãoecienteeecaz,devemostermuita

atençãocomacomunicação.

Dentrodesseprocessoestãoenvolvidos:oplanejamentodainformação,a

distribuiçãodamesmaeosrelatóriosdedesempenho.

Alémdisso,énecessárioidenticaremonitorarosriscosenvolvidosnoprojeto.

Ogerenciamentodosriscosbuscaentãoavaliaraimportânciaeaprobabilidade

deocorrênciadecadafatorderiscoparaqueestesnãoatrapalhemoandamento

doprojeto.

Quantomaisentendemosquaissãoosriscospercebidos,melhorpoderemosnos

prepararparasituaçõesadversas.Entreasferramentasutilizadas,temos:aanálise

quantitativaequalitativadosriscos.

Emumprojeto,normalmentesãonecessáriascomprasdemateriaisedeserviços

externos.Dessaforma,ogerenciamentodeaquisiçõesbuscaráfazertodoo

planejamentodositensnecessáriosaoprojeto,aseleçãodosmelhores

fornecedores,ofechamentodasnegociaçõeseeventuaiscontratos,alémdo

gerenciamentodessescontratos.

Vejanográficotodasasáreasdoconhecimento:

Figura19.4–Áreasdoconhecimento

Vamosvercomoessetemajáfoicobrado?

4.(Esaf/ATRFB/Analista/2009)Deumaformageral,pode-searmarqueosprojetospossuemas

seguintescaracterísticas,exceto:

a)têmobjetivosdefinidos;

b)sãonãorepetitivos,ouseja:inexistemdoisoumaisprojetosiguais;

c)eliminamaincertezaquantoaosresultados;

d)ocasionammudançasnaorganização;

e)têmumperíododetempolimitado,cominícioefimbem-definidos.

Essaquestãoestábemtranquila,nãoépessoal?Todasasopçõesestãocorretas,

menosumaque

caaparentelogo“decara”.Nenhumprojetoeliminaa

incerteza(quemderaissofosseverdade,nãoémesmo?).

Qualquergerentedeprojetodevelidarcomasincertezasdoambientee

gerenciarosriscosinerentesaoprojeto.Ogabaritoé,assim,aletraC.

19.5.OGerentedeProjetos

Ogerentedeprojetoséoprossionalresponsávelpelosucessoouofracassode

umprojeto.DeacordocomMereditheMantelJr.,18

ogerentedeprojetosdeveintegrartodososaspectosdeumprojeto,assegurarquetodosos

conhecimentoserecursosestarãodisponíveisquandoeondenecessários,e,acimadetudo,

assegurarqueosresultadosdesejadossejamproduzidosnotemponecessárioedeformaeciente

enocustoadequado.

Ogerentedeve,portanto,analisartodososrecursosquejátemdisponíveiseos

quedeveadquirir.Outrapreocupaçãoé“mapear”ascondiçõesambientais

internaseexternasquepodemajudarouatrapalharseutrabalho.

Dessamaneira,umfatorexternocomoomercadodetrabalhoaquecidopode

dicultarascontrataçõesdepessoasnecessárias.Jáoclimapolíticoouum

ambienteeconômicofavorável(comoaCopadoMundonoBrasil)podemauxiliar

naconcessãodealvarásounaobtençãodefinanciamentos,porexemplo.

Essegerentenormalmentetrabalhaemumaestruturamatricial(emqueos

funcionáriosestãoligadosaogerentefuncional–demarketing,porexemplo--e

aoprojeto)ouatreladaaumgerentedeportfólio.

Umportfólioéumconjuntodeprojetosqueestãoacargodeuma

organização.19Assim,enquantoumgerentedeprojetossepreocupacomo

funcionamentodo“seu”processo,ogerentedeportfóliodevedecidirqual

projetodeveserpriorizadoouadiado,porexemplo,dentrodosrecursosescassos

daorganização.

Figura19.5–Gerênciadeportfólio

Cadaprojetotemumretornoespecíco(nanceironocasodeempresas

privadas,socialoupolíticonasorganizaçõespúblicas),riscoseoutrosfatores

importantesaseremavaliados.20Combasenessesfatores,ogerentedeportfólio

deveescolherquaisserãoosprojetosprioritários.

Outrasituaçãoéquandoaorganizaçãopossuiumescritóriodeprojetos(ou

ProjectManagementOffice–PMO),umcentrodeinformaçõesecontrolesobreos

projetos.21

DeacordocomVargas,osprincipaistiposdePMOsão:

ØProjetoAutônomo–Escritóriodeprojetoseparadodasoperaçõesda

empresa,destinadosaogerenciamentodeumprojetoouprograma

especíco,emquearesponsabilidadequantoaosucessooufracassoédo

PMO.

ØProjectSupportOffice–Escritóriodeprojetodeesferadepartamental

destinadoaoapoioadiversosprojetossimultâneos,fornecendosuporte,

ferramentaseserviçosdeplanejamento,controledeprazos,custos,

qualidade,entreoutros.

ØEntrepriseProjectSupportOffice–Escritóriodeprojetosdeesfera

corporativa,atuandonogerenciamentoestratégicodetodososprojetosda

organização.Suasprincipaisfunçõessãooplanejamentoestratégicodos

projetos,ogerenciamentodosprojetoscorporativoseinterdepartamentais,a

gestãodoconhecimentoempresarialemprojetos,alémderepresentara

interfaceentreosenvolvidosnoprojeto.

Assim,essaunidadedaempresa(PMO)tantopodeassessorareprestar

consultoriaaosgerentesdeprojetosquantopodeassumiratribuiçõesde

gerenciamentocentralizadodosprojetoseresponsabilidadepeloseusucesso.22

19.6.Portfólios,Programas,ProjetoseSubprojetos

Comovimos,umportfólioéumconjuntodeprogramasouprojetosquesão

agrupadosembuscadoatingimentodosobjetivosestratégicosdaorganização.De

acordocomoPMBOK,23

Umportfóliorefere-seaumconjuntodeprojetosouprogramaseoutrostrabalhos,agrupados

parafacilitarogerenciamentoecazdessetrabalhoamdeatingirosobjetivosdenegócios

estratégicos.

Osprojetosouprogramasinseridosdentrodeumportfóliopodemnãoser

necessariamenteinterdependentesoudiretamenterelacionados.Jáumprograma

écompostoporumgrupodeprojetosquesãorelacionadosequesãogerenciados

demodoqueaorganizaçãoobtenhabenefíciosquenãoalcançariademodo

individualizado.

Figura19.6–Divisãodeprojetos

Jáumsubprojetoéumadivisãodeumprojetomaior.Normalmente,essa

divisãoocorreparafacilitarogerenciamentodeprojetoscomplexos.

Vamosvercomoissojáfoicobrado?

5.(Cespe/Cespe/Correios/–Analista/–2011)Agerênciadeprojetoslevaemconsideraçãoapenas

osfatoresambientaisdaempresaquerestringemasopçõesdegerenciamentoetêm

inuêncianegativanoresultadodoprojeto.Essesfatoressãoconsideradoscomoentradapara

osprocessosdeplanejamento.

EssaquestãofoiretiradadeumtrechodoPMBOK24quediz:

Osfatoresambientaisdaempresareferem-setantoaosfatoresambientaisinternosquanto

externosquecercamouinuenciamosucessodeumprojeto.Osfatoresambientaisdaempresa

podemaumentarourestringirasopçõesdegerenciamentodeprojetosepodemteruma

influênciapositivaounegativanoresultado.Elessãoconsideradoscomoentradasnamaioriados

processosdeplanejamento.

Dessamaneira,nãosãosóosfatoresnegativosquedevemserlevadosem

consideraçãopelosgerentesdeprojeto.Assim,ogabaritoéquestãoerrada.

19.7.CiclosdeVidadosProjetos

Ociclodevidadeumprojetoconsistenasdiversasfasesporqueumprojeto

passa.Essasfasesnormalmentesãosequenciais,mastambémpodemse

sobrepor.Ouseja,nãoénecessárioquetenhamosnalizadoumafasepara

iniciarmosafaseseguinte.

Masatransiçãodeumafaseparaoutraserásemprecaracterizadaporuma

“entrega”,ouseja,algum“produto”sendoentregue.Comessas“entregas”tendo

sidoaprovadas,podemosiniciarapróximafase(nomodosequencial).

DeacordocomoPMBOK,25ociclodevidadeumprojetopodeserdescrito

assim:

ØIníciodoprojeto.

ØOrganizaçãoepreparação.

ØExecuçãodotrabalhodoprojeto.

ØEncerramentodoprojeto.

Então,oprojetoseiniciaquandoalgumanecessidade(ouproblema,ou

oportunidade)éidenticada.Emseguida,ociclopassapelodesenvolvimentoda

soluçãoparaessasnecessidadeseavançanaimplementaçãodessasolução,atéa

finalizaçãodoprojeto.26

19.8.CiclodeVidadeumProjetoxCiclodeVidadeumProduto

Algumasbancasgostamdeconfundirocandidatomisturandooconceitode

ciclodeprojetocomodeciclodeproduto.Ociclodevidadeumprojetoaponta

asfasesquedevemserexecutadasdesdeoiníciodeumprojetoatéoseu

encerramento.

Jáociclodevidadeumprodutoédeterminadopeloperíodoentreseu

lançamentonomercadoesuaretiradadecomercialização.Umprodutopodeter

umciclodevidaindenido(comooantigoFusca,umprodutoquecoudécadas

emcomercialização).DeacordocomoPMBOK,27

Ociclodevidadoprodutoconsisteemfasesdoproduto,geralmentesequenciaisenão

sobrepostas,determinadaspelanecessidadedeproduçãoecontroledaorganização.Aúltimafase

dociclodevidadeumprodutoéaretiradadecirculaçãodoproduto.Geralmenteociclode

vidadeumprojetoestácontidoemumoumaisciclosdevidadoproduto.Énecessárioter

cuidadoparadistinguirociclodevidadoprojetodociclodevidadoproduto.

Ouseja,ociclodevidadeumprodutopodeconterdiversosciclosdeprojetos.

OFusca,porexemplo,passoupordiversas“remodelagens”,ecadaumadessas

envolveuprojetosdiferentes.

19.9.CustoseRiscosnoCiclodeProjetos

Quantomaisavançamosemumafase,maioressãooscustosacumuladosemum

projeto.Issoébastantesimplesdeentender,nãoémesmo?Sevocêanalisaro

projetodeumacasa,porexemplo,quantomaisavançarmosnaobra,maioresserão

osgastosacumuladosenvolvidos.

Entretanto,ocontrárioocorrecomosriscos.Naturalmente,quantomais

avançadaestáafase,menoressetornamosriscosdeinsucesso.Assim,quantomais

“perto”deumobjetivoestamos,maisseguroscamossobreosucessoda

empreitada,poisexistemmenosvariáveisquepodemdarerrado.

Figura19.7–Ciclodevidadoprojeto–riscos

Alémdisso,autilizaçãodaspessoasnormalmentecomeçadeumnívelbaixoe

vaievoluindocomopassardotempo.Ouseja,começamososprojetoscom

poucosprossionaisevamosaumentandoessenúmeroatéatingirumplatôem

algummomento.

Entretanto,apóscertotempo,autilizaçãodepessoascaibruscamenteatéo

encerramentodoprojeto,emquetodasaspessoasserãodesmobilizadas.

Omesmoocorrecomosgastosporunidadedetempo,quecomeçam

normalmentemaisbaixos,sãoaumentadosduranteodesenvolvimentodoprojeto

ecaembastantenofinal.

Figura19.8–Ciclodevidadoprojeto–pessoas

Outrofatorimportanteéacapacidadeeocustodesefazerumaalteraçãono

projeto.Emseuinício,ocustoébaixoeapossibilidadealtadesepoderalterar

algumaspectonoprojeto.

Assimsendo,antesdecomeçarumaobraérazoavelmentefácilalteraroprojeto

eincluiralgumambiente,porexemplo.Jácomaobrano“meiodocaminho”

teremosdequebrarparedesedesperdiçarmaterial,tempoedinheiro,nãoé

mesmo?

Ouseja,comopassardotempo,cacadavezmaiscaroemaisdifícil

fazermosumaalteraçãonoprojeto.

Vamosvercomoessetemapodesercobrado?

6.(Esaf/STN/Desenv.institucional/2008)SegundooGuiaPMBOK–3aedição,asdescriçõesdociclo

devidadoprojetopodemsermuitogenéricasoumuitodetalhadas.Amaioriadosciclosde

vidadeumprojetocompartilhacaracterísticascomuns,sendoumadelas:

a)asfasessãosempresequenciaisedenidasporalgumformuláriodetransferênciadeinformações

técnicasoudeentregadecomponentestécnicos;

b)osníveisdecustosedepessoalsãoaltosnoinício,atingemovalormínimoduranteasfases

intermediáriaseaumentamrapidamentequandooprojetoéfinalizado;

c)oníveldeincertezasémaisaltoe,portanto,oriscodenãoatingirosobjetivosémaiornoiníciodo

projeto;

d)acertezadetérminogeralmentesetornacadavezmenoràmedidaqueoprojetocontinua;

e)acapacidadedaspartesinteressadasdeinuenciaremnascaracterísticasnaisdoprodutodoprojeto

eocustonaldoprojetoémaisbaixanoinícioetorna-secadavezmaisaltaconformeoprojeto

continua.

AletraAestáerrada,poisasfasesnemsempresãosequenciais,podendo

tambémsersobrepostas.NaletraB,oqueocorreéexatamenteocontráriodoque

abancadescreveu.

AletraCestáperfeitaeéonossogabarito.JáaletraDcontradizaletraCeestá

equivocada.Finalmente,aletraEnãoestácorreta.Quantomaisoprojetoavança,

maisdifícilecarocaparasefazeralgumaalteração.Assim,ogabaritoémesmoa

letraC.

19.10.RestriçõesaosProjetos

Osprojetosprecisamserexecutadosdeacordocomcertasvariáveisou

restrições.Essetemaéimportanteparavocêsqueestãosepreparandoparauma

provadeconcursos,poistivemosumaalteraçãoconsiderávelentreoPMBOKda

3aediçãoeoPMBOKda4aedição.Deacordocoma3aedição,asprincipais

restriçõesseriam:

ØTempo–Oprazoquetemosparaterminarumprojeto,ouumafasedele.

ØCusto–Quantotemosdeorçamentoparaconseguirterminaroprojeto.

ØEscopo–É,basicamente,oquedeveserfeito.

Essastrêsrestriçõessãochamadasde“triânguloderestrições”.Essenome

refleteofatodequenãoconseguimosalterarumavariávelsemalterarasoutras.

Ouseja,sequisermosacelerarotérminodoprojeto,teremosdegastarmais,

nãoéverdade?Ouentãoteremosdediminuiroescopodoprojeto(construiruma

estradamenor,umprédiomaissimplesetc.).

Dessamaneira,aomudarmosumavariável,issoafetaráasoutrasvariáveis.Veja

umarepresentaçãodo“triângulodasrestrições”:

Figura19.9–Triângulodasrestrições–PMBOK3aedição

JánoPMBOK4aedição,asrestriçõesforamampliadas.Dentrodasatuais

restriçõesaosprojetos,teríamos:oescopo,aqualidade,ocronograma(tempo),o

orçamento(custo),osrecursoseorisco.

Bom,jáimaginooquevocêestápensandoagora!Mas,mestre,oquemarcona

horadaprova?Bem,anãoserqueabancaapontequalediçãoutilizoucomobase

paraapergunta,devemosconsiderarasduasversõescorretas.

Muitasquestõesaindautilizamo“triânguloderestrições”eforamconsideradas

corretaspelabanca.Assim,nãocaiamno“erro”deconsiderarqueessetriângulo

está“errado”porqueexisteumanovaversão,ok?Pelomenosessetemsidoo

comportamentodasbancasatéomomento.

Figura19.10–RestriçõesdoPMBOK4aedição

Continuando,quandonecessitamosalteraralgumadessasrestriçõesou

variáveis,temosduastécnicasprincipais:oparalelismo(fasttracking)ea

compressão(crashing).28

Imaginequevocêpreciseacelerarumprojeto.Atravésdacompressão,você

poderiaaumentarotamanhodaequipe(contratandomaispessoas),aumentaro

númerodeturnosdetrabalho(pôropessoaltrabalhandodemadrugada)ou

aumentaronúmerodehorasextrasdaequipeatual.Vocêprovavelmentereduzirá

otempoparaotérminodoprojeto,mastambémelevaráoscustos.

Outratécnicapossíveléadoparalelismo.DeacordocomMendes,29secoma

compressãotemosumatrocadetempoporcusto,noparalelismotemosumatroca

detempoporrisco.

Imaginequevocêqueiraterminaroprojetodasuacasa.Oidealseriasóentrar

comopintorapósaconclusãodopiso(issoéumahipótese..nãoébemminha

área!),porexemplo.Entretanto,vocêestácompressa!

Atravésdoparalelismo,vocêpoderiacolocartantoopintorquantoo

instaladordopisoparatrabalharemaomesmotempo.Dessamaneira,existiráo

riscodeopintordeixarpingartintanopiso,nãoémesmo?Assim,estamos

correndomais“riscos”paraterminaroprojetomaisrápido.

Vamosvercomoessetemapodesercobrado?

7.(Esaf/CVM/Analista/2010)SegundooPMBOK,oGrupodeProcessodoGerenciamentode

Projetosengloba:

a)Planejamento.Programação.Execução.EspecificaçãoeMonitoramento.Encerramento;

b)Iniciação.Execução.Monitoramento.Reengenharia.Relatório;

c)Iniciação.Planejamento.Execução.MonitoramentoeControle.Encerramento;

d)Iniciação.Especificação.Planejamento.ControledeUsuários.Realimentação;

e)Concepção.Interação.Planejamento.Execução.Monitoramento.

Questãobem“decoreba”daEsaf.DeacordocomoPMBOK,oscincogruposde

processosnagestãodeprojetossão:iniciação,planejamento,execução,

monitoramentoecontroleeencerramento.Dessemodo,ogabaritoémesmoa

letraC.

QuestõesExtras

8.(Esaf/ANA/Analistaadm./2009)Asáreasquecompõemoconjuntodeconhecimentossobre

gerenciamentodeprojetossãonove.Paraessasnoveáreas,oPMBOKpropõeoagrupamento

deprocessosemfunçãodasuanatureza.Entreasopçõesaseguir,selecioneaquelaque

enunciacorretamenteosgruposdeprocessosdegerenciamentodeprojetos.

a)Iniciação,Planejamento,Execução,MonitoramentoeEncerramento.

b)Escopo,Tempo,Custo,RH,QualidadeeRiscos.

c)Escopo,Planejamento,Execução,MonitoramentoeImplantação.

d)Contrato,Escopo,Custo,Cronograma,PlanejamentoeMonitoramento.

e)TermodeAbertura,Iniciação,Contrato,GerenciamentodoProjeto,MonitoramentoeEncerramento.

Osprincipaisgruposdeprocessos,deacordocomoPMBOK,são:iniciação,

planejamento,monitoramentoecontroleeoencerramento.Dessamaneira,a

alternativacorretaéaletraA.

OutrostermosutilizadospelaEsafpara“embaralhar”acabeçadoscandidatos,

como:custos,RH,escopoetc.sãoalgumasdasáreasdoconhecimento,enão

gruposdeprocessos.

9.(Cespe/TRE-BA/Técnico/2010)Ogerenciamentodeprojetostentaadquirircontrolesobre

algumasvariáveisconsideradasfundamentaisparatodoprojeto.Essasvariáveis—tempo,

custoerecursos—sãotambémconhecidascomotriângulodagerênciadeprojeto.Seumlado

dessetriânguloé(variável)alterado,osoutrossofremimpacto.

Atríplicerestrição,outriânguloderestriçõesengloba:ocusto,otempo

(prazo)eoescopo(enãoosrecursos).Assim,seaprojetotiverdeserantecipado,

custarámaiscaro,porexemplo.Comoabancaalterouescopoporrecursos,o

gabaritoéquestãoincorreta.

10.(Cespe/Inca/Gestãopública/2010)Aodescobrirumerronoplanodeimplementaçãodoprojeto,

quevaiprovocarumimpactonadatadapróximaentregadoprojeto,aprimeiraatitudedo

gerentedeverásersolicitarumamudançanadataeencaminharparaaprovaçãodo

patrocinador.

Issonãofaznenhumsentido,nãoémesmo?Ouseja,seexistirumproblemana

execuçãodoprojeto,suaprimeiraideiaseria“empurrar”adataparaafrente.Isso

muitasvezesnemépossível(imagineumgestorpúblicoquedigaqueas

Olimpíadasde2016terãodeacontecerem2017,poisoestádiodevôleinãocará

pronto..).

Ogestordoprojetoterádetentaroutrasestratégias,comoocrashingouo

paralelismo.Ogabaritoéquestãoerrada.

11.(Esaf/ANA/Analistaadm./2009)Considerandoadeniçãodeescritóriodeprojetossegundoo

PMBOK,analiseasalternativasqueseseguemeselecioneaopçãoquecorrespondeao

resultadodesuaanálise:

()OescritóriodeprojetosouPMOéumaentidadeorganizacionalàqualdevemseratribuídas

responsabilidadesrelacionadasàelaboraçãodecontratos,masnãoaogerenciamentode

projetos.

()OescritóriodeprojetosouPMOpodeterresponsabilidadesquevariamdesdeo

fornecimentodefunçõesdesuporteaogerenciamentodeprojetosatéogerenciamento

diretodeumprojeto.

()UmescritóriodeprojetosouPMOéoórgãoresponsávelpelaemissãodoscerticadosPMP

naorganização,emespecialaautorizaçãodedocumentosderequisitosedemais

especificaçõesdeescopoeesforçodistinto.

a)C,C,C

b)C,C,E

c)C,E,E

d)E,E,E

e)E,C,E

Aprimeirafraseestáerrada,poisoPMOpode,sim,estarenvolvido

diretamentenogerenciamentodeprojetos.Jásegundafraseestácorreta,poiso

PMOtantopodeserum“suporte”comoassumirdiretamenteasresponsabilidades

degerenciamento.

Aterceirafraseéabsurda.AcerticaçãoPMP(ProjectManagementProfessional,

ouprossionaldegerênciadeprojetos)éconcedidapeloProjectManagement

Institute–PMI(queéumainstituiçãointernacional).Assim,nãoéemitidapelo

PMO,queéumórgãodaempresa.OgabaritoéaletraE.

QuestõesPropostas

12.(FCC/TRF4aRegiãoAnal.adm./2010)Ogerenciamentodoescopodeumprojetorefere-seà

definiçãodetodasasatividadesdoprojeto.Analise:

I.Ogerenciamentodoescopodoprojetocompreendetantooescopodoprodutoquantoo

escopodoprojeto.

II.Oescopodoprodutorefere-seàscaracterísticasdoprodutoouserviçoeoescopodo

projetoenvolveaadministraçãodasuaexecução.

III.Oconteúdodoprocessodeiniciaçãodoescopodoprojetoéabaseparaafaseseguinte,

denominadaverificaçãodoescopo.

IV.Adescriçãodetalhadadosrequisitosdoprodutodoprojetoédelineadanafasedecontrole

demudançasdoescopodoprojeto.

V.Duranteoprocessodeiniciaçãodoprojetodeve-seconsideraroplanoestratégicoda

empresa.

ÉcorretooqueconstaAPENASem

a)I,IIeIV.

b)I,IeV.

c)I,IVeV.

d)IIeV.

e)IeIV.

13.(FCC/TRF4aRegião/Anal.adm./2010)Oconjuntodefasescoletivasatravessadaspeloprojetoé

denominado:

a)análisedepassagem;

b)ciclodevidadoprojeto;

c)pontodeencerramento;

d)passagemdeestágio;

e)saídadefase.

14.(Esaf/ANA/Analistaadm./2009)Entreasarmativasqueseseguemsobreciclodevidado

projeto,selecioneopçãoincorreta.

a)Osgerentespodemdividirosprojetosemfasesparafacilitarocontrolegerencial.

b)Oconjuntodasfasesdeumprojetocorrespondeaociclodevidadoprojeto.

c)Ociclodevidadoprojetodefineasfasesqueconectamoiníciodeumprojetoaoseufim.

d)Acapacidadedeaspartesinteressadasinuenciaremoprodutodoprojetoémaiornasfasesiniciais

dociclodevidadoprojeto.

e)Oníveldecustosedepessoalémaiornaprimeiraenaúltimafasedociclodevidadequalquerprojeto.

15.(Cespe/Correios/–Analista/–2011)Ociclodevidadeumprodutoinicia-sequandoosciclosde

vidadosprojetosaelerelacionadossãoencerrados.

16.(FCC/Infraero/Administrador/2009)Emrelaçãoaciclodevidadeumprojeto,écorretoafirmar:

a)Ocustodasmudançaseacapacidadedeinuênciadaspartesinteressadasaumentamcomotempo

doprojeto.

b)Oencerramentodeumafaseautorizaoinícioformaldafaseseguinte,procedimentoquecontribui

parareduzirotempodeduraçãodoprojeto.

c)Emgeral,arevisãodotrabalhorealizadoocorreapósaconclusãodafasenaleserveparadenira

aceitaçãodosprodutosdetodasasfasesquecompõemoprojeto.

d)Ociclodevidadeumprojetodeneasfasesqueconectamoiníciodeumprojetoaoseunaleé

compostoporumafaseinicialeoutrafinal,alémdeumaoumaisfasesintermediárias.

e)Osníveisdecustosedepessoalsãoelevadosnoinícioecaemgradativamenteconformeoprojeto

avançaparaafasefinal.

17.(FCC/TCE/GO/Desenv.org./2009)Umatécnicaadequadaparaareduçãodocronograma,sem

reduçãodoescopodoprojeto,é:

a)odetalhamentodocronograma;

b)aparcelizaçãodasetapas;

c)acompressãodoprojeto;

d)aelaboraçãoprogressiva;

e)oretrabalhodasfases.

18.(Cespe/–TRE/BA/Técnico/2010)Ogerenciamentodeprojetostentaadquirircontrolesobre

algumasvariáveisconsideradasfundamentaisparatodoprojeto.Essasvariáveis—tempo,

custoerecursos—sãotambémconhecidascomotriângulodagerênciadeprojeto.Seumlado

dessetriânguloé(variável)alterado,osoutrossofremimpacto.

19.(Cespe/TCDF/ACE/2012)Nagestãodeprojetos,oprincipalprodutodaetapadeplanejamentoé

otermodeaberturadoprojeto−documentoquecontemplaaestruturaanalíticadoprojeto.

20.(Cesgranrio/Petrobras/Técnico/2012)Nagerênciadeumprojeto,ogerentedevelidarcomum

conjuntoderestriçõesemtrêsáreas,quesão,principalmente,asde:

a)Escopo,TempoeCusto;

b)Custo,ControleeEquipe;

c)Estratégia,TáticaeOperacional;

d)Planejamento,ExecuçãoeVerificação;

e)Qualidade,RiscoeRetorno.

21.(Cesgranrio/Petrobras/Técnico/2012)ApreparaçãodeumaequipebrasileiraparaasOlimpíadas

noBrasilem2016:

a)conguraumprojeto,poisenvolveuminvestimentosignicativodetempoedinheiro,buscandoum

retornosobreoinvestimento;

b)conguraumprojeto,poisopaísestáinvestindocontinuamenteemumaumentodonúmerode

medalhasconquistadas;

c)conguraumprojeto,poiséumesforçotemporário,comumresultadoúnico,umavezqueessa

equipenãoseráigualàdeoutraOlimpíada;

d)nãoconguraumprojeto,poishámuitaspessoascominteressesdiversosenvolvidos(comerciais,

esportivos,turísticos);

e)nãoconguraumprojeto,poisépossívelque,depoisdasOlimpíadas,asequipessejam

desmobilizadaseosestádiossejamutilizadosparaoutrosfins.

22.(Cespe/Detran-DF/Analista/2009)Projetosdealtaqualidadeentregamoproduto,oserviçoouo

resultadosolicitadodentrodoescopo,noprazoedentrodoorçamento.Ocorrendoalguma

alteraçãoemumdessesfatores,osdemaispodemalterar.

23.(Cespe/Antaq–Analista/2009)DeacordocomoPMBok,oentendimentodoambientedeum

projetopoderequererconhecimentodenaturezapolítica,legal,social,físicae(ou)ecológica.

24.(Cespe/TRE-ES/Analista/2011)Umprojetopodeserdenidocomooesforçotemporário

empreendidoparasecriarumproduto,serviçoouresultado,eoseutérminosomenteé

alcançadoquandoseatingemosobjetivos.

25.(Cespe/TCE-RN/Analista/2009)NoPMBOK,osstakeholderssãodenidoscomoindivíduosou

organizaçõesenvolvidosnoprojeto,ouqueserãoafetadospositivamenteounegativamente

peloresultadonaldeumprojeto.Essesindivíduoseorganizaçõesdevemseridenticados,

masnãosãoelementos-chaveemumprojeto.

26.(Cespe/Anatel/Analistaadm./2009)Osprojetosnãoprecisamnecessariamenteteruma

duração,poishácasosemquenãoépossíveldenirotempodeexecução,comonocasoda

elaboraçãodeprodutosintelectuais.

27.(Cespe/Seapa-DF/Administrador/2009)Ogerenciamentodeprojetosésegmentadoemcinco

estágiosougruposdeprocessos:iniciação,planejamento,execução,controleeencerramento.

28.(Cespe/Seapa-DF/Administrador/2009)Otriângulodagerênciadeprojetosrepresentaastrês

principaisvariáveisaseremconsideradasnogerenciamentodeprojetos:tempo,custoe

recursoshumanos.

29.(Esaf/STN/Desenv.institucional/2005)Assinaleaopçãoverdadeira.OPMBOKguidedescreveas

principaisáreasdegerenciamentodoprojetoeasdenecomosubconjuntosdeprocessos.As

áreasdegerenciamentodocusto,dotempoedaqualidadeenvolvemprocessosque

asseguram,respectivamente:

a)queoprojetosejaconcluídodeacordocomoorçamentoprevisto;queoprojetosejaconcluídono

prazoprevistoequeoprodutodoprojetoestejaemconformidadecomoesperado;

b)queoprojetosejaconcluídodeacordocomoorçamentoprevisto,queasinformaçõesdoprojeto

estejamemconformidadecomoesperadoequeoprodutodoprojetoestejaemconformidadecom

oesperado;

c)queasinformaçõesdoprojetoestejamemconformidadecomoesperado,queoprojetoseja

concluídodeacordocomoorçamentoprevistoequeoprojetosejaconcluídonoprazoprevisto;

d)queoprodutodoprojetoestejaemconformidadecomoesperado,queasinformaçõesdoprojeto

estejamemconformidadecomoesperadoequeoprojetosejaconcluídonoprazoprevisto;

e)queoprojetosejaconcluídodeacordocomoorçamentoprevisto,queoprojetosejaconcluídono

prazoprevistoequeasinformaçõesdoprojetoestejamemconformidadecomoesperado.

30.(FCC/TCE/GO/Desenv.org./2009)Considereasseguintesalternativasemrelaçãoao

gerenciamentodeprojetos.

I.Agerênciadeprojetoséadisciplinademanterosriscosdefracassoemumníveltãobaixo

quantonecessárioduranteociclodevidadoprojeto.Oriscodefracassoaumentadeacordo

comapresençadeincertezadurantetodososestágiosdoprojeto.

II.Geralmenteosprojetosdevemserexecutadoseentreguessobocondicionamentode

variáveis,principalmente:tempo,custoeescopo.

III.Agerênciadeprojetoséfrequentementeresponsabilidadedetodososindivíduos

envolvidosnoprojeto.Idealmente,todosdevemparticipardiretamentenasatividadesque

produzemoresultadofinal.

IV.Astrêsvariáveisquecompõemo“triângulodagerênciadeprojeto”nãopodemser

alteradassemimpactarumasàsoutras.

V.Oescopodoprojetosãoasexigênciasespecicadasparaoresultadoesperado,incluindoo

quesepretendeeoquenãosepretenderealizar.

EstácorretooqueseafirmaAPENASem

a)I,IeII.

b)I,I,IVeV.

c)I,II,IVeV.

d)IeV.

e)II,IVeV.

31.(Esaf/CVM/Analista/2010)Comferramentasdegerenciamentodeprojetos,ogerentede

projetopodedefiniruma:

a)equipededefiniçãodotrabalho(workdefinitionteam);

b)estruturadeapoioaousuário(usersupportstructure);

c)estruturadedivisãodotrabalho(workbreakdownstructure);

d)estruturadedivisãodeobjetos(objectbreakdownstructure);

e)metodologiadesegmentaçãodosprogramas(programsegmentationmethodology).

32.(Esaf/ANA/Analistaadm./2009)Analiseasseguintesarmaçõesarespeitodegerênciade

projetos.

I.Operaçãoéumempreendimentotemporáriodestinadoàcriaçãodeumprodutoouserviço.

II.Osprocessossãoiterativos,pornatureza.

III.Oescopodoprojeto,prazoecustossãorestriçõestípicascomasquaisogerentede

projetostemdelidar.

Assinaleaopçãocorreta.

a)ApenasasafirmaçõesIeIsãoverdadeiras.

b)ApenasasafirmaçõesIeIIsãoverdadeiras.

c)ApenasasafirmaçõesIeIIsãoverdadeiras.

d)AsafirmaçõesI,IeIIsãoverdadeiras.

e)Nenhumadasafirmaçõeséverdadeira.

33.(Esaf/ANA/Analistaadm./2009)Entreasarmativasqueseseguemsobreciclodevidado

projeto,selecioneopçãoincorreta.

a)Osgerentespodemdividirosprojetosemfasesparafacilitarocontrolegerencial.

b)Oconjuntodasfasesdeumprojetocorrespondeaociclodevidadoprojeto.

c)Ociclodevidadoprojetodefineasfasesqueconectamoiníciodeumprojetoaoseufim.

d)Acapacidadedeaspartesinteressadasinuenciaremoprodutodoprojetoémaiornasfasesiniciais

dociclodevidadoprojeto.

e)Oníveldecustosedepessoalémaiornaprimeiraenaúltimafasedociclodevidadequalquerprojeto.

34.(FCC/Infraero/Arquiteto/2009)Àassociaçãocomoplanejamento,organização,direçãoe

controlederecursosorganizacionaisparaobjetivosdecurtoemédioprazos,visandoà

complementaçãodeobjetivosespecícosdentrodeumperíododeterminado,aplicando-se

conhecimentos,habilidadesetécnicasnaelaboraçãodeatividadesrelacionadas,denomina-se:

a)planoplurianual;

b)planodemetas;

c)gerenciamentodeprojetos;

d)gerenciamentoderisco;

e)fluxodecaixaedecapitais.

35.(Esaf/CVM/Analista/2010)SegundooPMBOK,sãostakeholdersdeumprojeto:

a)GerentedoProjeto.PatrocinadordoProjeto.Cliente,Diretoria.GerenteExecutivo.Gerentesde

Departamento.Fornecedores.Distribuidores.

b)GerentedoProjeto.AnalistadoProjeto.Cliente.ProgramadeTreinamento.Assessoria.Gerentesde

Conta.FornecedoresConcorrentes.Mantenedores.

c)PooldeProgramação.ControladordoProjeto.Servidores.Diretoria.GerenteExecutivo.Gerentesde

Operações.EntidadesExternas.Webmasters.

d)GerentedoProjeto.PatrocinadordoProjeto.Cliente.Coachers.SecretariaExecutiva.Gerentesde

Treinamento.Fornecedores.EmpresasConcorrentes.

e)GerentedoProjeto.Estratégias.Logística.Diretoria.EscritóriodeProjetos.GerentesdeContingência.

Fornecedores.Distribuidores.

36.(FCC/TRT24a/MS/Analadm./2011)Osindivíduosquepodeminuenciardemaneirapositivaou

negativaemumprojetosãoos:

a)stakeholders;

b)stakeholers;

c)players;

d)backhloders;

e)throwers.

37.(FCC/TRT6aRegião/Analista/2012)Comrelaçãoaociclodevidadoproduto,segundooPMBOK,

considere:

I.Geralmenteociclodevidadeumprojetoestácontidoemumoumaisciclosdevidado

produto.Ociclodevidadoprodutoconsisteemfasesdoproduto,geralmentesequenciaise

nãosobrepostas,determinadaspelanecessidadedeproduçãoecontroledaorganização,

sendoqueaúltimafasedociclodevidadeumprodutoéaentregaparaoclientefinal.

II.Váriosaspectosdociclodevidadoprodutoprestam-seàexecuçãocomoprojetos,por

exemplo,arealizaçãodeumestudodeviabilidade,aconduçãodepesquisasdemercado,a

execuçãodecampanhaspublicitárias,ainstalaçãodeumproduto,realizaçãodedinâmicas

degrupoedeavaliaçõesdeprodutosemmercadosdeteste.Emcadaumdessesexemplos,

ociclodevidadoprojetodiferedociclodevidadoproduto.

III.Quandoasaídadoprojetoestárelacionadaaumproduto,existemmuitasrelações

possíveis.Porexemplo,odesenvolvimentodeumnovoprodutopodeserumprojetoemsi.

Aomesmotempo,umprodutoexistentepodesebeneficiardeumprojetoparaoacréscimo

denovasfunçõesoucaracterísticas.

Estácorretooqueseafirmaem

a)IeII,apenas.

b)I,IeII.

c)IeII,apenas.

d)I,apenas.

e)II,apenas.

38.(FCC/TJ-RJ/Analista/2012)OProjectManagementBodyofKnowledge(PMBoK)éumconjuntode

práticasemgerênciadeprojetosquedivideociclodevidadoprojetoemcincogruposde

processos,entreosquais:

a)MonitoramentoeControle,ExecuçãoeCrítica;

b)Iniciação,CríticaeExecução;

c)Planejamento,OtimizaçãoeGarantiadaQualidadedoProcesso;

d)Otimização,PlanejamentoeEncerramento;

e)Iniciação,PlanejamentoeMonitoramentoeControle.

39.(FCC/TRE-CE–Técnico/2012)SegundoadeniçãodoPMBOKsobreGerenciamentode

Portfólios,éINCORRETOoqueconstaem:

a)umportfóliorefere-seaumconjuntodeprojetosouprogramaseoutrostrabalhos;

b)osucessoémedidopelaqualidadedosprojetoseprodutosegraudesatisfaçãodocliente;

c)serefereaogerenciamentocentralizadodeumoumaisportfólios;

d)seconcentraemgarantirqueosprojetoseprogramassejamanalisadosamdepriorizaraalocação

derecursos;

e)osprojetosouprogramasdoportfóliopodemnãosernecessariamenteinterdependentesou

diretamenterelacionados.

40.(FCC/Infraero/Analista/2011)OTermodeAberturadoProjetoéasaídadoprocesso

“Desenvolverotermodeaberturadoprojeto”,queestávinculadonoPMBOKaumaáreade

conhecimentodenominada:

a)Iniciação;

b)Integração;

c)Planejamento;

d)Aquisições;

e)Escopo.

41.(FCC/Infraero/Analista/2011)NoPMBOK4aedição,ogrupodeprocessosExecuçãocontém

exclusivamenteosprocessos:

a)Definirasatividades;Identificaraspartesinteressadas;Controlarocronograma;

b)Coletarosrequisitos;Gerenciaraequipedoprojeto;Distribuirinformações;

c)Desenvolveraequipedeprojeto;Conduzirasaquisições;Realizaragarantiadaqualidade;

d)Administrarasaquisições;Definiroescopo;Realizarocontroleintegradodemudanças;

e)Orientaregerenciaraexecuçãodoprojeto;Denirasatividades;Realizaraanálisequantitativados

riscos.

Gabaritos

1.A

15.E

29.A

2.B

16.C

30.B

3.D

17.C

31.C

4.C

18.E

32.C

5.E

19.E

33.E

6.C

20.A

34.C

7.C

21.C

35.A

8.A

22.C

36.A

9.E

23.C

37.A

10.E

24.E

38.E

11.E

25.E

39.B

12.B

26.E

40.B

13.B

27.C

41.C

14.E

28.E

Bibliografia

Gido,Jack,eJamesP.Clements.SuccessfulProjectManagement.3aed.Mason:

ThomsonSouth-Western,2006.

Heerkens,GaryR.ProjectManagement.NovaYork:McGraw-Hill,2002.

Mendes,JoãoRicardoBarroca,AndréBittencourtdoValle,eMarcantonioFabra.

Gerenciamentodeprojetos.1aed.RiodeJaneiro:FGV,2009.

Meredith,JackR.,eSamuelJ.MantelJr.ProjectManagement:amanagerial

approach.5aed.Hoboken:JohnWiley&Sons,2003.

Umguiadoconhecimentoemgerenciamentodeprojetos(GuiaPMBOK).4aed.

NewtonSquare:ProjectManagementInstitute,s.d.

Valeriano,Dalton.Modernogerenciamentodeprojetos.SãoPaulo:Prentice-Hall,

2005.

Vargas,RicardoViana.Gerenciamentodeprojetos:estabelecendodiferenciais

competitivos.5aed.RiodeJaneiro:Brasport,2003.

Capítulo20

AdministraçãodeMateriais

Existemdiversostiposderecursosemumaorganização,comorecursos

nanceiros,recursoshumanos,recursostecnológicosetc.Quandofalamosde

recursosmateriais,estamosnosreferindoaoselementosfísicosquesãoutilizados

pelaempresanasuaatuação.

Assimsendo,agestãodemateriaisenglobaoplanejamento,aexecuçãoeo

controledetodasasatividadesquepossibilitamoadequadosuprimentoparaa

organizaçãodessesrecursosmateriaisnecessáriosaoseufuncionamento.De

acordocomViana,aadministraçãodemateriaisé,1

oplanejamento,acoordenação,adireçãoeocontroledetodasasatividadesligadasàaquisiçãode

materiaisparaaformaçãodeestoques,desdeomomentodesuaconcepçãoatéseuconsumofinal.

AAdministraçãodeMateriaiséfundamentalemqualquerentidadepúblicae

privada,poiséatravésdelaqueaorganizaçãoadquireosinsumosdeque

necessitaráparaproduzirumprodutooufornecerumserviço.

Dessaforma,aARMéumconjuntodeprocessosdentrodeumaorganização

quedevemassegurarqueestaconsigaadquirirosdiversosmateriaisnecessários

paraseufuncionamento.

Nocasodosetorpúblico,essafunçãoadquireaindaumaimportânciamaior,

poisosvalorescompradossãomuitoaltos.Assim,umaboaadministraçãoé

importanteparaevitardesviosderecursosepossibilitaramaximizaçãodos

recursosdacoletividade.

Essesdesviospodemocorrertantonoprocessodecompra(com

superfaturamentos,porexemplo)comonodearmazenamento(subtraçãodos

itensemestoque,comoorouboderemédiosemhospitais).

Alémdisso,oEstadotambémpodeatuarcomprandooschamados“estoques

reguladores”,comoocorrianoscasosemquecompravacafédosprodutorespara

“amortecer”aflutuaçãodepreços.

Portanto,comoaAdministraçãoPúblicaéumdosmaiorescompradoresna

economianacionaleéumclientegrandeparadiversasempresas,elaacaba

recebendopressõespolíticasecorporativas.Dessaforma,essaáreaéumfocode

corrupçãopotencialedeveserbemcontrolada.

AAdministraçãodeMateriaispodeserdivididanasseguintesáreasdeatuação:2

ØGestãodeestoques.

ØGestãodecompras.

ØAlmoxarifado.

ØPlanejamentoecontroledaprodução.

ØImportação(quandoforocaso).

ØTransportesedistribuição.

Vamosveragoraasprincipaisáreasdaadministraçãodemateriais?

20.1.AdministraçãodeEstoques

Agestãodeestoquesédefundamentalimportâncianasorganizações,poisafeta

diversasáreas.Seutrabalhoéligadoàáreadeprodução,devendasedasfinanças.

Estoqueé,nadefiniçãodeViana,3conceituadocomo:

Materiais,mercadoriasouprodutosacumuladosparautilizaçãoposterior,demodoapermitiro

atendimentoregulardasnecessidadesdosusuáriosparaacontinuidadedaempresa,sendoo

estoquegerado,consequentemente,pelaimpossibilidadedeprever-seademandacomexatidão.

Anecessidadedoestoqueentãoexisteporquenãotemoscomo“adivinhar”

comosecomportaráoconsumodosmateriais,queéderivadodademandapelos

produtoseserviçosdaorganização.

Alémdisso,nemsempreconseguimosreporositensdequenecessitamosemum

prazocurtoeconáveldetempo.Dessemodo,precisamosteralgum

“lubricante”quepossibiliteàorganizaçãocontinuaroperandoatéqueconsigaos

materiaisdequenecessita.4

Oproblemaquedeveserequacionadoporessaáreaéoseguinte:quando

mantemosumnívelaltodeestoques,demandamosuminvestimentoaltoem

instalações,dinheiroparadoemmateriaisetc.Ouseja,essesestoquescustam

caro.Comisso,quantomaisestoquesaempresativerdemanter,maiscaroserá

paraela.

Comosesabequeagestãodeestoquesimpactanosrecursosnanceiros

disponíveisdaempresa,osetornanceirobuscasempreoestoquemínimo

possíveldemodoa“liberar”aomáximooseucaixa.

Jáossetoresdeproduçãoedevendasdesejamumestoquemaior,paraquea

possibilidadedequeestefaltesejaminimizada.Imaginequevocêtenhaa

responsabilidadede,porexemplo,manteralinhadeproduçãofuncionando.A

últimacoisaquevocêdesejaéqueotrabalhopareporquefaltouumparafusono

estoque,nãoémesmo?

Damesmaforma,umvendedorcarábravoseconseguirconvencerumcliente

acomprarumprodutodaempresa,masdescobrirqueoprodutonãoestámais

disponívelnoestoque!Assim,existesempreumconitodeinteressesentreos

diversossetoresdaempresacomagestãodeestoquesquedeveserbalanceado.

Osmateriaissãoclassificadoscomo:

ØMatérias-primas–são,basicamente,osinsumosqueserãoutilizadosna

fabricaçãodosprodutosdaorganização.Dessamaneira,seaempresafabrica

móveis,elatemcomomatéria-primaamadeira,porexemplo.

ØProdutosemprocesso–sãoaquelesquejáestãosendoutilizadosna

fabricaçãodosprodutosdaempresa.

ØProdutosacabados–referem-seaosprodutosquejáestãoprontosparaser

entreguesaosconsumidoresfinais.

ØMateriaisauxiliaresedemanutenção–sãoosmateriaisquenãosão

utilizadosnaproduçãodeprodutosdaempresa,massãonecessáriosaodiaa

diadaorganização,comomateriaisdeescritório,porexemplo.

20.2.PrevisãoparaosEstoques

Aprevisãodoconsumodosmateriaisquesãoestocadoséumtrabalho

fundamental,poisirábalizartodoogerenciamentodosestoques.Dessamaneira,

deveserfeitoumplanejamentodequandoserãonecessáriososmateriais,emque

montante,dequemodoetc.

Existemdiversastécnicasparasepreveroconsumo:

ØProjeção–partedalógicadequeatendênciadoconsumopassadoirá

continuar.Dessamaneira,seoconsumodoitemtemvariado10%aoano,

amplia-seaquantidadeemmais10%.

ØExplicação–utilizandodetécnicasderegressãoedecorrelaçãodefatores

tenta-seestimaroconsumoagregandooutrosfatoresquesãoconhecidos

(comoavariaçãodoPIB,porexemplo).

ØPredileção–temumcarátermaisqualitativo(subjetivo),poissebaseiana

opiniãodefuncionáriosexperientesouconhecedoresdomercado,quedão

suaopiniãosobreatendênciadoconsumo.

Dentrodasprincipaistécnicasconsideradasquantitativas(poisdependemde

valoresquepodemserquantificados),temosasseguintes:

20.2.1.MétododoÚltimoPeríodo

Ométododoúltimoperíodorefere-seexatamenteaseutilizaroúltimo

consumodoperíodoanterior.Ouseja,seforamconsumidasmilunidadesdeum

determinadoinsumoemumdeterminadomês,compram-semilunidadesno

próximomês.

20.2.2.MétododaMédiaMóvel

Essemétodoémuitocobradoemconcursos.Basicamente,éamédiaaritmética

(sim,aquelafácildaescola)considerandoosúltimosmesesqueabancasolicitar.

Vejaumexemplo:

Figura20.1

Imaginequeabancatepediuparacalcularamédiamóveldosúltimostrês

mesesdeconsumo.Basicamente,vocêterádeselecionarosúltimostrêsconsumos

(mesesdeoutubro,setembroeagosto)edividirportrês(númerodemeses).

Ficariaassim:

Oproblemadamédiamóveléqueelaconsideraosmesesmaisantigoscomotão

importantesquantoosmesesmaisrecentes.Alémdisso,essamédiaédistorcida

porvaloresextremos,casovenhamaocorrer.

20.2.3.MétododaMédiaMóvelPonderada

Nessemétodo,faremosquaseomesmoquezemosnamédiamóvel,mascoma

utilizaçãodepesos.Dessaforma,normalmente,asorganizaçõespreferemdarum

pesomaioraosdadosmaisrecentes.

Assim,vamosvoltaraocasoanterior?Nessasituação,daremosumpesotrês

paraoúltimomês(outubro),pesodoisparaopenúltimomês(setembro)epeso

umparaoantepenúltimomês(agosto).

Figura20.2

Alémdisso,devemosdividiressesnúmerosporseis(somatóriodonúmerode

mesesvezesopesoquedemosacadaumdeles).Vejacomoficarianossafórmula:

Seaunidadedomaterialforindivisível,teremosde“arredondar”paracima,de

formaqueoresultadoseria1318unidades.Tranquilo,nãoémesmo?

20.3.CustosdosEstoques

Existemdiversoscustosassociadosaosestoques.Alémdoóbviocustonanceiro

decomprá-los,temososcustosdearmazenamento,depedidoedefaltade

estoque.Vamosverosprincipaiscustos.

20.3.1.CustosdeArmazenamento

Paraquepossamosmanterestoquesemboacondiçãodeutilização,devemoster

umaestruturaadequadaparacadatipodematerial,demodoquenãosepercam.

Alémdisso,devemosterpessoasemateriaisparafazeressaarmazenagem.

Somadosaesses,temososcustosdealuguel,desegurosetc.

Essescustossãomuitasvezesxos.Ouseja,mesmoqueconsigamosdiminuir

bastanteoníveldosestoques,nãoconseguimosdiminuirmuitooscustosde

armazenamento.

Outroscustosassociadosaoarmazenamentosão:juros,obsolescência,

depreciação,deterioraçãoetc.

20.3.2.CustosdePedido

Ocustodepedidoéassociadoaosmateriaisnecessáriosparaqueumpedido

sejaefetuado,comopapéis,custohomem/horadaspessoasenvolvidasno

processo,custodostelefonemasetc.

Decertaforma,quantomaiorvalordopedido,menorseráocustodepedido

proporcionalmenteaovalordacompra.Assim,busca-seequilibrarovaloraser

pedido,demodoqueocustodopedidonãosejaexcessivo.

20.3.3.CustosdeFaltadeEstoque

Ocustodafaltadeumestoquemuitasvezesnãopodesermedido,poisenvolve

diversasvariáveisqualitativas(subjetivas).

Imagineumhospitalquedeixefaltarummedicamento.Devidoaessafalta,uma

pessoanãoconseguesersalva.Esseprejuízoéincalculável,nãoémesmo?Uma

vidahumananãopodeserquantificada.

Alémdisso,areputaçãodaorganizaçãoseriacertamenteabalada.Dessa

maneira,apesardesermuitasvezesdedifícilmensuração,existesempreumcusto

casooestoqueacabe.

20.4.NíveisdeEstoque

Oplanejamentodonívelcorretodosestoquesdependedediversasvariáveis,

desdeotempoqueofornecedorlevaparaentregaromaterialatéatendênciade

consumo,omontantedoestoqueatual,entreoutros.

Nográcoaseguir(chamadode“dentedeserra”),podemosverumasituação

hipotéticadagestãodeummaterial.Oestoqueinicialdoitem(milunidades)vai

sendoutilizadopelaempresaatéatingiropontozero.

Figura20.3

Assimqueessepontoéatingido,aempresarecebeumanovaremessado

fornecedor,restaurandooestoqueaoseumontanteinicial,quandooprocessose

reinicia.

Claroque,navidareal,ográconãoseria“redondinho”assim,poisoconsumo

nãoé,normalmente,constante,enãotemoscomotercertezadequeofornecedor

nosentregaráaremessaoupedidonoexatomomentoemqueiremosnecessitar.

Dessemodo,énecessárioum“seguro”paralimitaroureduziraschancesdeque

essasvariáveisafetemaproduçãodaorganização.Portanto,reservamoscerto

estoquedesegurança,ouestoquemínimo.

Figura20.4

Nasituação,criaríamosumestoquede200unidadesparaesseestoquemínimo,

queaprincípionãoseriautilizadopelaorganização.

Afunçãodesseestoquemínimoéservirde“colchão”desegurançaparaocaso

dealgumproblemanaentrega,porexemplo.Assim,teríamosumareservaparaser

utilizadanessassituações.

Entretanto,parasabermosqualdeveseromontantedoestoquemínimodeum

item,temosdesaberqualéotempoderessuprimento(oudereposição)dele.

Essetempoconsistenotempoqueaempresalevaparaemitiropedidoparao

seufornecedor,maisotempoemqueessefornecedorlevaráparafabricaro

materialeentregá-lo(transporteedistribuição)paraaempresa.

Outrainformaçãonecessáriaéadoconsumomédiodoitem.Essedadoé

facilmentecalculável,poiséamédiaaritméticadosúltimosperíodos.Dessa

maneira,teremosumaideiadavelocidadeemqueessesmateriaissãoconsumidos.

Emseguida,podemoscalcularopontodepedido.Opontodopedidoéo

momentoemquedevemosemitirumpedido,demodoqueoestoquenão“entre”

dentrodoníveldesegurança.

Essecálculoéfeitoatravésdaseguintefórmula:

PP=(CMédio×TR)+EstoqueSeg.

Vamosverumcasoprático?Imaginequeabancatrouxeainformaçãodequeo

consumomédiomensaldaempresaéde200unidades.Otempoderessuprimento

éde90dias(3meses)eoestoquedesegurançaéde50unidades.Nossopontode

pedidoseria:

PP=(200×3)+50=650

Dessemodo,quandooestoquedaorganizaçãoindicar650unidades,estáno

momentodeserfeitoumnovopedido.Tranquilo,nãoémesmo?

Outroindicadormuitocobradoéodogirodeestoque,oqualdáumanoção

dequantasvezesaoanooestoquefoireposto.Assim,quantomaiorforessedado,

maiorfoiaeciênciananceira,poisvendemosmaisvezescomomesmo

investimento.

Vejaumcasoprático.Aempresaconsumiu1200unidadesduranteoano,eo

estoquemédiofoide220unidades.Assim,ogirodeestoquefoi:

Ouseja,oestoque“girou”cercade5,45vezesduranteoano.

Vamosvercomoessetemajáfoicobrado?

1.(Esaf/Susep/Adm.nanceira/2010)Combasenosseguintesdadossobreoconsumodeum

materialqualquer,assinaleaopçãoqueindica,corretamente,opontodepedido(P)ea

quantidade(Q)aseradquiridaemcadapedido:

–consumomensal:50unidades.

–tempodereposição:1,5mês.

–estoquemínimo:2,0mesesdeconsumo.

a)P=175//Q=75

b)P=100//Q=50

c)P=150//Q=75

d)P=175//Q=50

e)P=150//Q=50

Opontodopedidoéomomentoemquedevemosemitirumpedido,demodo

queoestoquenão“entre”dentrodoníveldesegurança.Essecálculoéfeito

atravésdaseguintefórmula:

PP=(CMédio×TR)+EstoqueSeg.

Assim,nocasocitadonaquestão,teríamososeguintecálculodopontode

pedido:

PP=(50×1,5)+(2×50)=(75)+(100)=175

Jáaquantidadedopedidotemestecálculo:

QP=(Consumo×TR)

Assim,comosdadosdaquestão,teríamososeguintecálculo:

QP=(50×1,5)=75

Dessemodo,opontodopedidoseria175unidades,eaquantidadecomprada

seriade75unidades.Ogabaritoseria,portanto,aletraA.

20.5.ClassificaçãoABC

AclassicaçãoABCutilizaofamosoprincípiodePareto,quedizquepoucos

itenscausamgrandepartedosefeitos.Esseprincípioémuitoimportantena

gestãodeestoques,poisindicaositensdemateriaisquedevemserpriorizados.

Deacordocomessemétododeclassificação,temos:

ØClasseA–sãoositensdemaiorrelevância,quegeramosmaioresimpactos

naorganização.Essesitensdevemseracompanhadoscomumaatenção

especialpelaempresa.Normalmente,nãoultrapassamos20%dototaldos

itens.

ØClasseB–sãocompostosporitensdeimportânciaintermediária,ouseja,

nãosãotãoimportantesquantoosdaclasseA,massãomenos

“insignificantes”dosqueositensdaclasseC.

ØClasseC–sãocompostosdeitensque,apesardograndenúmero,não

causamumimpactorelevantenaorganização.Assim,devemser

acompanhadoscommenosatençãodoqueosoutrositens.Mascuidado:

essesitenscontinuamtendodeseracompanhadosecontrolados!

Imagineoestoquedeumaconcessionáriadeautomóveis.Temos,nesseestoque,

desdecarrosnovosnalojaatéparafusos.Claroqueosautomóveisrepresentamum

grandeimpactonaoperaçãodaempresa.

Cadacarrovendidoimpactamuitomaisdoquetodososparafusosvendidosno

anointeiro.Assim,essesitensdevemseracompanhadosdeperto.Jáosparafusos

podemsercontroladosporamostragem,porexemplo.

Normalmente,umacurvaABCtemestetipodecomportamento:

Figura20.5–CurvaABC

Assim,poucositenscompõemboapartedovalordeumestoque,porexemplo,

emuitositenssãopoucorelevantes.

20.6.AvaliaçãodeEstoques

Muitasvezes,necessitamosavaliarquantosrecursostemosinvestidosem

estoquesnaorganização,sejaporqueprecisamoscumprirumapolítica

determinadadeestoqueouparaalgumprocessodetomadadedecisão.5

Aavaliaçãoéfeita,naturalmente,pelovalordositenspresentesnoestoque.Esta

podeserfeitadetrêsmaneirasdiferentes:pelocustomédio,pelométodo

PEPS(ouFIFO–rstin,rstout)(primeiroaentrar,primeiroasair)epelo

métodoUEPS(ouLIFO–lastin,rstout)(últimoaentrar,primeiroasair).A

seguir,podemosvercadaumemmaisdetalhes:

Figura20.6–Métodosdeavaliaçãodeestoques

Vamosvercomoessetemapodesercobrado?

2.(FCC/MPE-SE/Analista/2009)noprocessodeavaliaçãodeestoque,quandoasaídadoestoqueé

feitapelopreçodoúltimoloteaentrarnoalmoxarifadoométododeavaliaçãoutilizado

denomina-se:

a)custoajustado;

b)UEPSouLIFO;

c)PEPSouFIFO;

d)customédio;

e)custodereposição.

Quandoestamosavaliandoestoqueseutilizamoscomobaseopreçodosúltimos

itensaseremcomprados,estamosutilizandoométodoUEPS(últimoaentrar

primeiroasair).

Portanto,ogabaritoémesmoaletraB.

20.7.Just-in-time

Osistemajust-in-timefoiumarevoluçãonagestãodemateriais,poistrouxe

umanovavisãoparaoprocessoprodutivo.Osistematradicional,de“empurrar”a

produçãoparaafrente,ouseja,euproduzoedepoisvejocomofareiparavender

paraocliente,deixoudeservistocomointeressantenestesnovostemposde

globalizaçãoemercadoscadavezmaiscompetitivos.

Deacordocomojust-in-time,oprocessoprodutivoseinverte.Sóéproduzidoo

quejátemdemandacerta.Dessamaneira,éconhecidocomoumsistemaque

“puxa”aprodução.

Paraqueessesistemafuncione,sãonecessáriasumaparceriaeumatrocade

informações,antesimpensáveisentreosfornecedoreseclientesdentrodacadeia

deprodução.Acompetiçãonorelacionamentoentrecompradorevendedordeve

darlugaràcolaboração.

Assim,imaginequevocêtrabalheemumamontadoradeautomóveis.Vocênão

querumestoquegrandedentrodesuafábrica,poisissoacarretacustos(comojá

vimosantes).

Entretanto,paraquesejapossívelaoseufornecedorlheentregartodososdias

ositensdequevocêprecisará,énecessárioqueeletenhatantogarantiadeque

vocêcomprarádelequantoconhecimentodequantositensserãonecessários.Ele

deveconhecer,afundo,asnecessidadesdoseuprocessoprodutivo.

Ouseja,nessesistema,oseufornecedordeveserencaradocomoparceiroenão

comoadversário.Assim,eleganharánovolumeenagarantiadetervocêcomo

cliente,evocêteráumaconabilidademaiornofornecimentoquelhe

possibilitaráterpoucosestoquesdesegurança.

Dessamaneira,essemétodoacarretaumadiminuiçãotremendadosestoques,

poisaempresarecebeconstantementeosmateriaisdequenecessitará,muitas

vezesdiariamente.

Vamosvercomoessetópicojáfoicobradoemprovas?

3.(Cesgranrio/Termoaçu/Administradorjr./2008)Qualéumadasprincipaiscaracterísticasda

programaçãojust-in-timedesuprimentos?

a)Relaçõesprivilegiadascompoucosfornecedores.

b)Garantiadeprodutosaosconsumidorescomgrandesestoques.

c)Aumentodotempodesuprimentoentreofornecedoreaempresa.

d)Aumentodotempoedoscustosassociados,quepermitealcançarboassoluções.

e)Usodainformaçãoeficienteparagerarestoqueseatenderaosconsumidores.

Nojust-in-time,aempresanecessitadeumaquantidademenordefornecedores.

Apesardisso,devemserfornecedoresmaiséiseconáveis.Dessaforma,aletraA

estácorreta.AletraBestáerrada,porqueoobjetivoéreduzirestoques,não

aumentá-los.

NaletraC,oerroestánofatodequeotempoderessuprimentodeveser

reduzido,enãoaumentado.OmesmoerroocorrecomaletraD.Oaumentodos

custosedotempoénegativo,nãoumavantagem.

Finalmente,aletraEestáincorreta,poisumainformaçãoecientedeveser

utilizadaparareduzirosestoques,nãoaumentá-los.Assim,ogabaritodaquestãoé

mesmoaletraA.

20.8.AdministraçãodeCompras

Afunçãodecomprasémuitoimportanteparaosucessodeumaorganização.

Elaéaresponsávelporadquirirosbenseserviçosnecessáriosparaaoperaçãoda

instituiçãoporumcustoadequado,buscandoaqualidadecertaeoprazomais

curtopossíveldeentrega.

Muitasvezes,umaestratégiacorretadecomprasseráumdiferencial

competitivoquefarácomqueaempresaconsigaganharparticipaçãodemercado,

venderseusprodutosporumpreçomaisbaixodoqueseusconcorrenteseteruma

lucratividademaior.

Imagineumaempresatendodepararsualinhademontagemporfaltadeum

produtoessencial.Quantoprejuízoissocausaria?Omesmopoderíamosdizerde

umamáquinaquequebreporcausadeummaterialdebaixaqualidadecomprado.

Umaboagestãodascompraséfundamentalparaqueaorganizaçãoatinjaseus

objetivosestratégicos.

Atualmente,oprocessodecomprasdeveserfeitoporcompradores

especializados,queutilizemtécnicasmodernasdecomprasequeestejamem

contatoconstantecomasdiversasáreasdaorganização.

Assim,essesprossionaisdevemconhecerosfornecedoresdecadaproduto,as

característicasdessesprodutosaseremcomprados,asalternativasviáveis,os

temposdeentrega,entreosdiversosaspectosquepodemafetaracompetitividade

daempresa.

DeacordocomMonteAltoetal.6

Ocompradoratualdeixoudeserum“tirador”depedidosparaexercerospapéisdepesquisador,

consultoreanalistadevalor(custoxbenefício)queadquireprodutoscomfunçõesdequalidade

(utilidadefuncional),desempenhooperacional(produtividade),facilidadesdeoperação

(minimizaçãodecustos)ecustosdemanutenção(reposiçãodepeças),procurandoagregarserviços

quelevemaoaumentodalucratividade.

Alémdisso,oscompradoresdevemdesenvolveremanterumarelação

construtivacomseusfornecedores.Aindaexistempessoasquetêmumavisão

dosfornecedorescomosefossemseusinimigos.Achamque,paraquesuaempresa

possaganhar,devem“espremer”aomáximoosfornecedores.Seriaumavisão

“ganha-perde”dorelacionamentocomprador-vendedor.

Entretanto,essetipoderelacionamentonãoéinteressanteparaaempresano

longoprazo.Umfornecedorquesempre“perca”sairádomercado,nãoémesmo?

Paraumrelacionamentoconstrutivo,devemosteremmentequearelaçãoentreo

compradoreofornecedorpodegerarbenefíciosparaasduaspartes.Essaéavisão

“ganha-ganha”.

Osprincipaisobjetivosdafunçãocomprassão:7

ØGarantirumfluxocontínuodemateriais,serviçoseinformaçõesqueatenda

àsnecessidadesgeraisdaempresademodoareduzircustosnacadeiade

suprimentosenosistemalogístico.

ØAdquirirmateriaisdeformaeconômica,compatíveiscomaqualidade

requeridaedeacordocomasuafinalidadeouaplicação.

ØIncentivarecolaborarnapadronizaçãoesimplicaçãodemateriaise

equipamentos.

ØConsideraraslimitaçõesdaempresaeacapacidadedearmazenamento.

ØMinimizarocustodoprocessoaquisitivo.

ØPesquisaromercadofornecedorembuscadefontesdefornecimentonovase

alternativas.

ØManterasrelaçõescomosfornecedoreseníveldecooperação,lealdadee

respeito.

ØObteramáximaintegraçãoecooperaçãodasoutrasáreasdaempresa

(técnica,nanceira,jurídica,produção,operação,manutenção,logística)e,

particularmente,daáreadegestãodeestoques.

Alémdisso,afunçãodecomprasdevemanterumaposturaéticaemseus

processosdetrabalho.Essaéumaáreamuitovisadanaempresa,poislidacomum

montantederecursosimenso.Ogestordaáreadeve,portanto,focarseusesforços

emtertransparênciaeumapráticaéticaemsuasnegociações.

Asprincipaisatividadesdeumsetordecomprassãoasseguintes:8

Figura20.7–Principaisatividadesdosetordecompras.

(AdaptadodeFenili,2011)

20.9.EstratégiasdoSetordeCompras

Existemalgumasestratégiasparaafunçãodecomprasquesãomuito

recorrentesemconcursos.Vamosveralgumas?

20.9.1.VerticalizaçãoouHorizontalização

Umaestratégiadeverticalizaçãoacontecequandoumaorganizaçãobusca

produzirosprodutosdequenecessitadentrodaprópriaempresa.Assim,ela

tentarácomprarforaomínimopossível.Semprequepuder,tentaráproduzir

internamente.

Essaestratégiageravantagensedesvantagens.Comovantagem,podemoscitara

independênciaqueaempresaganhado“mercado”.Aorganizaçãodominaráa

tecnologiadeproduçãoepoderádeterminarquandoequantosprodutosserão

produzidos.Elaterá,portanto,umdomíniomaiorsobreasuacadeiaprodutiva.

Entretanto,existemdiversasdesvantagensnessaestratégia.Paraproduziros

bensinternamente,torna-senecessáriaumagrandeestrutura,commáquinase

empregadosquepodemmuitasvezesficarociosos.

Issodemandatambémumgrandeinvestimento,quemuitasvezespodeser

necessárioemáreasmaisprioritárias.9Comisso,essaestratégianãotemsido

adotadaatualmente.Asempresastêmbuscadoahorizontalização!

Ahorizontalizaçãoéumaestratégiaquebuscaexatamenteocontrário–

compraromáximodeprodutosnomercado.Comessaestratégia,asempresas

buscammontarparceriascomosprincipaisfornecedoresparaqueestes

entreguemprodutosdequalidade,compreçoscompetitivosenaqualidade

desejada.

Naturalmente,essasestratégiastambémgeramvantagensedesvantagens.As

principaisvantagenssão:anecessidadedeinvestimentosmenoresemmaquinárioe

instalações,amanutençãodofocodaempresanosseusprocessosessenciaisea

flexibilidadeparaaproveitarnovastecnologiaseprodutosdosparceiros.

Infelizmente,aestratégiatambémapresentaumamaiordependênciados

fornecedoreseumdomíniomenordastecnologiasdeprodução.Apesardesses

problemas,essaéaestratégiaquetemsidoadotadapelamaioriadasempresase

governosportodoomundo.

20.9.2.CentralizaçãoouDescentralização

Ascompraspodemserorganizadasdediferentesmaneiras.Nessaclassicação,

estamosvendo“quem”éoresponsávelpelascomprasnaorganização.Assim,o

processodecomprasécentralizadoquandotodasascomprasdaempresa

ficamacargodeumúnicosetor.

Quandoaresponsabilidadepelascomprasestá“pulverizada”pormuitos

órgãosdaempresa,dizemosqueexisteumadescentralizaçãodoprocessode

compras.MonteAltoetal,apresentamtrêstiposdeorganização:10

ØCentralizaçãototal,quesignicaquetodasascomprassãoexecutadaspela

unidadecentraldecompras.

ØDescentralizaçãototal,quesignicaquetodasasunidadesquecompõemo

sistemaempresarialtêmautonomiapararealizarascomprasrelativasàssuas

necessidades.

ØOrganizaçãomista,queéumacombinaçãodosdoismodelosanteriores.

Naturalmente,ambasasestratégiasapresentamvantagens.Umaestrutura

centralizadadámaiorcontroledoprocessodecomprasparaacúpuladaempresa

etambémmaiorpoderdebarganhaparaacentraldecompras(porqueoslotes

compradosserãomaiores).

Jáumaestruturadescentralizadacostumaserapreferidadeempresasque

estãodistribuídasporgrandesáreasgeográcas.Essaestratégiadámaior

agilidadeaoprocessodecomprasepossibilitaumaexibilidadeparacadaáreade

modoagerirsuascomprasdeacordocomasnecessidadeslocais.Aseguir

podemosverasprincipaisvantagensdecadasituação:

Figura20.8–Vantagensdacentralizaçãoedadescentralização.

(Baseadoem:FrancischinieGurgel,2004)

Vamosvercomoessetemajáfoicobrado?

4.(Cespe/Antaq/Técnico/2009)Umavantagemdeseadotaracentralizaçãodoprocessode

compraséaobtençãodemaiorcontroledemateriaisemestoque.

Perfeito.Quandoaempresacentralizaoprocessodecomprasemumúnico

setor–comoemumacentraldecompras–,elaganhaumcontrolemaiordetodo

osistema,poistodasascomprassãofeitaspelomesmosetor.Issopossibilita

tambémumganhodeescalaeummaiorpoderdebarganhadoscompradores.O

gabaritoémesmoquestãocorreta.

20.10.ComprasnoSetorPúblico–Licitações

Osgestorespúblicossãoimpedidosdecomprarbensouserviçosdequalquer

fornecedorquelhesparecermaisconveniente.Aexibilidadedequedispõea

iniciativaprivadaparacomprardofornecedorquelheconviernãoexistena

gestãopública.

Comoosbenscompradosouosserviçosprestadossãodevaloresmuitoaltos,o

setorpúblicodeveseguirprincípiosquenorteiamaatuaçãoadministrativa,

visandogarantircondiçõesiguaisaqualquerum.

Logo,antesdeaAdministraçãorealizarumacompraoufecharumcontrato,ela

deveráprocedercomumprocessopúblicodenominadolicitação.

Cadaentefederativoadministrasuasprópriascomprasecontratos,masasleise

normasqueregemaslicitaçõeseascontrataçõesnosetorpúblicosãodeorigem

privativadaUnião.

AConstituiçãoFederalde1988,aLeino8666/93eaLeino10.520/02têmos

principaisdispositivoslegaisqueregulamentamaslicitações.Deacordocoma

CF/1988,aslicitações(comotodososatosdaAdministraçãoPública)devem

seguirosprincípiosconstitucionaisde:

ØLegalidade.

ØImpessoalidade.

ØMoralidade.

ØPublicidade.

ØEficiência.

JáaLeino8.666/93,noseuart.3o,citaaindaosprincípiosdaprobidade

administrativa,vinculaçãoaoinstrumentoconvocatórioejulgamentoobjetivo.

EstãosubmetidasaessesnormativostantoaAdministraçãoDiretaquantoa

AdministraçãoIndiretadaUnião,dosestados,doDistritoFederaledos

municípios.

Comonormageral,alicitaçãoéobrigatória.Sóexistemessasexceções

permitidasporlei:nainexigibilidadeounadispensadelicitação.

AslicitaçõesbuscamgarantiramelhorpropostapossívelparaaAdministração

Públicaeassegurarumtratamentoisonômicoaosconcorrentes(sem

favorecimentos).Entretanto,cabeaquilembrarqueamelhorpropostanãoserá

necessariamenteademelhorpreço!11

DeacordocomHelyLopesMeirelles,12alicitaçãoé:

oprocedimentoadministrativomedianteoqualaAdministraçãoPúblicaselecionaapropostamais

vantajosaparaocontratodeseuinteresse.

Deacordocomoart.45daLeino8666/93,existemquatrotiposdelicitação:

MenorPreço–éamaiscomumnosetorpúblicoelevaemconsideraçãoapenas

opreçodosconcorrentes,desdequecumpramascondiçõesestabelecidasno

edital.

MelhorTécnica–essetipoéutilizadoexclusivamenteparaserviçosdenatureza

predominantementeintelectual,emespecialnaelaboraçãodeprojetos,cálculos,

scalização,supervisãoegerenciamentoedeengenhariaconsultivaemgerale,em

particular,paraaelaboraçãodeestudostécnicospreliminareseprojetosbásicose

executivos.

MelhorTécnicaePreço–funcionadomesmomodoqueotipoanterior,mas

nessecasoaspropostassãoavaliadasdeacordocomumamédiaponderadadas

valorizaçõestécnicasedepreço,deacordocomospesosdeterminadosno

instrumentoconvocatório.

MaiorLance–serelacionacomassituaçõesemqueoEstadoestávendendo

bensouefetuandoconcessõespúblicas.

Figura20.9–Tiposdelicitação

20.11.ModalidadesdeLicitação

Alémdisso,aLeino8666/93descrevediversasmodalidadesdelicitação,de

acordocomomontante(valor)aserlicitado.Existemmodalidadesquenãoestão

especicadasnaLeino8666/93,comoopregão,quefoiinstituídonaLeino

10.520/2002.Vamosaelas?

20.11.1.Concorrência

Éamodalidadedelicitaçãoentrequaisquerinteressadosque,nafaseinicialde

habilitaçãopreliminar,comprovempossuirosrequisitosmínimosdequalicação

exigidosnoeditalparaexecuçãodeseuobjeto.13

Éamodalidadequemaistomatempoparasecompletar,poisafasede

habilitaçãodosconcorrenteséprévia.Assim,adocumentaçãodosdiversos

competidoresdeveseranalisadaantesdeseobservaremasrespectivaspropostas.

Caberessaltarquequalquercomprapodeserfeitapormeiodaconcorrência,

mesmoasdevaloresbaixos(quepoderiamserenquadradasnasmodalidadesde

convite,porexemplo),masosgestorestêmohábitodesomenteautilizarquando

demaisalternativasnãosãopermitidas,poisessaéamaisdemorada.

Aconcorrêncianãotemlimitesdevalores(podeserfeitaparaqualquervalor).

Entretanto,essamodalidadedeveseraplicadanosseguintescasos:

ØObraseserviçosdeengenhariaestimadosemmaisdeR$1.500.000,00.

ØCompraseserviçosestimadosemmaisdeR$650.000,00.

20.11.2.TomadadePreços

Éamodalidadedelicitaçãoentreinteressadosdevidamentecadastradosouque

atenderematodasascondiçõesexigidasparacadastramentoatéoterceirodia

anterioràdatadorecebimentodaspropostas,observadaanecessária

qualificação.14

Nessamodalidade,asempresasjácadastradasouqueconsigamsecadastrar

atétrêsdiasantesdadatadorecebimentodaspropostaspodemconcorrer,

desdequesejamqualificadasparatal.

Essamodalidadeéadequadaparaoscasosde:

ØObraseserviçosdeengenhariaestimadosemmaisdeR$150.000,00emenos

deR$1.500.000,00.

ØCompraseserviçosestimadosemmaisdeR$80.000,00emenosde

R$650.000,00.

20.11.3.Convite

Éamodalidadedelicitaçãoentreinteressadosdoramopertinenteaoseu

objeto,cadastradosounão,escolhidoseconvidadosemnúmeromínimode3

(três)pelaunidadeadministrativa,aqualaxará,emlocalapropriado,cópiado

instrumentoconvocatórioeoestenderáaosdemaiscadastradosna

correspondenteespecialidadequemanifestaremseuinteressecom

antecedênciadeaté24(vinteequatro)horasdaapresentaçãodaspropostas.15

Essaéumamodalidademenos“burocrática”epodeserutilizadaquandoos

objetosaseremcompradossãodepequenovalor.Nessasituação,aAdministração

deveconvidarnomínimotrêsempresasinteressadasemparticipar.

Essasempresasconvidadaspodemestarcadastradasounão,desdequeoperem

noramorelativoaosprodutose/ouserviçosaseremadquiridos.Paraquea

licitaçãosejaválida,devemexistirpelomenostrêspropostasválidas.

Essamodalidadeéadequadaparaoscasosde:

ØObraseserviçosdeengenhariaestimadosemmenosdeR$150.000,00.

ØCompraseserviçosestimadosemmenosdeR$80.000,00.

20.11.4.Pregão

Modalidadenova,introduzidapelaLeino10.520/02,queseprestaàslicitações

debenseserviçoscomuns,ouseja,quepossamserdenidosdemodosimples,de

acordocomespecificaçõesdemercado(compradepapelA4,porexemplo).

Aaquisiçãodebenseserviçoscomuns,nopregão,podeserdequalquervalor,

pois,diferentementedaconcorrência,oqueseprezanopregãoéaqualidadee

nãoaquantidade.

Agrandevantagemdessamodalidade,quelogofoiaceitapelosdiversosórgãos

públicos,éasuarapidez.Nessamodalidade,ahabilitaçãodosconcorrentessóé

feitaapósaapresentaçãodaspropostas.

Assim,emvezdeseavaliaremashabilitaçõesdediversasempresas(eosdiversos

recursoseimpugnaçõesqueesseprocessogera),aAdministraçãoPública

somenteanalisaadocumentaçãodequemapresentouapropostavencedora.

Seestanãoforaceita,passa-separaasegundacolocadaeassimpordiante.

Dessamaneira,poupam-semuitotempoeesforço.Essamodalidadepodeser

aplicadaparaqualquervalor,masocritériodejulgamentosópodeserode

melhorpreço.

Valelembrarqueopregãopodeserusadoemtodasasesferasadministrativase

nãosónoâmbitofederal.

Opregãopodeocorreremduasmodalidades:presencialoueletrônico.O

Decretono5.450/05regulamentouopregãoeletrônicoe,noseuart.4o,obrigouo

seuusonaesferafederal.Edispôsno§1oqueousodamodalidadeconvencional

(oupresencial),noâmbitofederal,sóépermitidoemcasosdecomprovada

inviabilidadeejustificadospelaautoridadecompetente.

Posteriormente,oDecretono5.504/05impôsamodalidadepregão,com

preferênciapelopregãoeletrônico,pelaAdministraçãoPúblicadosestados,

DistritoFederaledosmunicípios,alémdasentidadesprivadas,quandoestas

adquirirembensouserviçoscomunsrepassadosvoluntariamentepelaUnião.

Adiferençaentreessasmodalidadesdepregãoéquenapresencial,nodia,hora

elocaldesignados,realiza-seumasessãopúblicacomapresençadosinteressados

oudeseusrepresentantes,queserãoavaliadoseclassicadospelocritériode

menorpreço,enquantonoeletrônicotodosospassossãorealizadosadistância,

comapoiodainternetetambémemsessãopública.

Paranalizarmosopregão,devemoslevaraseguinteinformação:nopregão,é

vedadaaexigênciadegarantia,deaquisiçãodoeditalcomocondiçãoparase

participardalicitaçãoedopagamentodetaxaseemolumentos,salvoos

referentesafornecimentodoedital,quenãoserãosuperioresaocustodesua

reproduçãográfica.

20.11.5.Concurso

Éamodalidadedelicitaçãoentrequaisquerinteressadosparaescolhade

trabalhotécnico,cientícoouartístico,medianteainstituiçãodeprêmiosou

remuneraçãoaosvencedores,conformecritériosconstantesdeeditalpublicadona

imprensaoficialcomantecedênciamínimade45(quarentaecinco)dias.16

20.11.6.Leilão

Éamodalidadedelicitaçãoentrequaisquerinteressadosparaavendadebens

móveisinservíveisparaaadministraçãooudeprodutoslegalmenteapreendidos

oupenhorados,ouparaaalienaçãodebensimóveis,aquemofereceromaior

lance,igualousuperioraovalordaavaliação.17

20.11.7.Consulta

EssamodalidadedelicitaçãonãoestálistadanaLeino8.666/93.Elafoicriada

pelaLeino9.472/97,queinstituiuaAgênciaNacionaldeTelecomunicações,

Anatel.

Aconsultarecaiapenassobreasagênciasreguladorasfederaisparaaaquisição

debenseacontrataçãodeserviçospormeiodeprocedimentospróprios,sendo

proibidaparafirmarcontratosdeobraseserviçosdeengenharia.

Aseguirpodemosvertodasasmodalidadesdelicitação:

Figura20.10–Modalidadesdelicitação

Vamosvercomoessetemajáfoicobrado?

5.(Cespe/Aneel/Técnico/2010)Convite,leilão,concursoecompradiretasãomodalidadesde

licitaçõespúblicas.

Olhaa”pegadinha”naárea!Acompradiretanofornecedornãoéuma

modalidadedelicitaçãopública.Asmodalidadesdelicitaçãosão:opregão,a

concorrência,atomadadepreços,oconvite,oleilãoetc.Ogabaritoéquestão

errada.

20.12.Almoxarifado

Umalmoxarifado,armazémoudepósitosãooslocaisemqueosmateriaisda

empresasãoestocados.Dessamaneira,aarmazenageméligadaaorecebimento,à

classicação,àmovimentaçãoeaoacondicionamentodessesmateriais.Deacordo

comFranklin:18

armazenageméaguardatemporáriadeprodutosestocadosparaposteriordistribuição.

Umagestãoecientedoalmoxarifado,portanto,devegerarumamovimentação

maisrápidadosmateriais,evitando-seperdasporextraviosequedase

aproveitando-semelhordasmáquinasedaspessoasenvolvidasnessetrabalho.

DeacordocomViana,aeciênciadeumalmoxarifadodepende

fundamentalmente:19

ØDareduçãodasdistânciasinternaspercorridaspelacargaedoconsequente

aumentodasviagensdeidaevolta.

ØDoaumentodotamanhomédiodasunidadesarmazenadas.

ØDamelhorutilizaçãodesuacapacidadevolumétrica.

Assim,umdosaspectosmaisimportantesaseremobservadosnagestãodas

operaçõesdealmoxarifadoéolayout(arranjofísico)dasinstalações,ouseja,a

disposiçãodasmáquinas,pessoasemateriaisdeumaorganização.20

Dessamaneira,olayoutrelaciona-secomomodoemqueascoisasestão

“arrumadas”naempresa.Algunsaspectosaseremobservadossão:seoscorredores

deescoamentosãolargosobastante,seosmateriaisestãoguardadosdeformaa

seremfacilmenteencontrados,seasinstalaçõesestãoadequadasaotipode

materialguardadoetc.

DeacordocomMoura,osobjetivosdeumlayouteficientedevemser:

ØAssegurarautilizaçãomáximadoespaço.

ØPropiciaramaiseficientemovimentaçãodemateriais.

ØPropiciaraestocagemmaiseconômica,comrelaçãoàsdespesasde

equipamento,espaçoemãodeobra.

ØPropiciarexibilidademáximaparasatisfazerasnecessidadesdemudança,

deestocagemedemovimentação.

Comumlayouteciente,amovimentaçãodosmateriaiséemmuitofacilitada.

Ballou21acreditaqueessaéumaáreacrítica,poisnaatividadedemanuseio

(movimentação)demateriais,porserrepetidainúmerasvezes,pequenasineciências

emalgumasdasviagenspodemsignicargrandesperdasfeitassistematicamentepor

umlongoperíodo.

Paraessamovimentaçãodemateriais,sãoutilizadosdiversosmétodose

instrumentos.Bowersox&Closs22classicamessesmétodoscomo:mecanizados

(queincluemasempilhadeiras,ostratoresetc.),ossemiautomatizados(que

automatizamcertasatividadesdemanuseio),osdemanuseioautomatizado(que

prescindemdapresençahumana)eosbaseadoseminformação(quesão

totalmentecomandadosporcomputadores).

20.13.IdentificaçãodoMaterial

Aclassicaçãoeaidenticaçãodosmateriaissãopeçasfundamentaisnas

operaçõesdealmoxarifado,poispermitemumcontroleecientedoestoquee

facilitamumarápidaretiradadosmateriaisnecessários.

Antigamente,identicavam-seosmateriaisatravésdesinaisecores.

Atualmente,esseprocessoéfeitodeformamuitomaiseciente,pormeiodos

códigosdebarraeleitorasóticas.

Atravésdasinformaçõescontidasnoscódigosdebarra,aorganizaçãopode

melhormanejarseusestoques.Alémdoacréscimodeinformação,esseprocesso

evitadiversoserroshumanosdeleituraeinterpretaçãoedemanuseiodos

materiais.

20.14.Picking

Opickingnadamaisédoqueotrabalhoderecolheromaterialnoalmoxarifado

semprequesolicitadoporumclienteinternoouexterno.

Deacordocomotamanhodaempresa,oconsumoearotatividadedecada

item,opickingdeveserplanejadocalculando-seonúmerodepessoasencarregadas

defazeressetrabalho,oprocessodeseparaçãodospedidos,onúmerodecada

itemseparadoporcoletaetc.23

DeacordocomLima,24existemtrêstiposdepicking:

ØPickingdiscreto:cadapedidoéiniciadoecompletoporapenasum

operador,sendoqueapenasumprodutoécoletadoporvez.Apresentaum

baixoíndicedeerro,poiséoperacionalmentebastantesimples.Poroutro

lado,apresentaumabaixaprodutividade,decorrentedotempoexcessivo

gastocomodeslocamentodooperador.

ØPickingporzona:aáreadearmazenagemédivididaemzonas,cadazona

armazenadeterminadosprodutosecadaoperadoréresponsávelporuma

determinadazona.Nesseprocesso,cadaoperadorseparaositensdopedido

queestãoarmazenadossobresua“responsabilidade”eoscolocanumaárea

comumdeconsolidação,paraqueopedidopossaser“montado”e

completado.Issosignicaquemaisdeumoperadorpodeestarenvolvidona

separaçãodosprodutosdeumúnicopedido.Amaiorvantagemdesse

métodoéquereduzodeslocamentodosoperadores,cadaumoperaapenas

numazona.Suamaiordiculdadeébalancearacargadetrabalhoentreas

zonas,poisexistemdiferençasnosgirosdosprodutosenodesempenhodos

equipamentos.Aszonasmaisequipadasnecessariamenteterãomaior

produtividade.

ØPickingporlote:ospedidossãoacumulados,juntam-seasquantidadestotais

decadaproduto,e,então,ooperadorvaiatéaáreadeestocagempara

coletarasomadospedidos.Essemétodopermiteumaumentona

produtividadedodeslocamentodooperador,desdequehajabaixavariedade

ebaixovolumedeitensemcadapedido.Poroutrolado,aumentam-seas

chancesdeerrosnaseparaçãoeordenaçãodospedidosdevidoàsuamaior

complexidade.

20.15.Inventários

Oinventárioocorrequando,poralgumarazão,temosdecontarositensdeum

almoxarifado.Sãonecessáriosparaquesepossatercertezadequeossistemas

(entreelesocontábil)estãorefletindoarealidadenoestoquefísico.

Assim,sãochecadosositenssicamente,umaum.Oprocedimentode

inventárionormalmenteéfeitoporduasequipesdepessoas.Aprimeirafaza

primeiracontagemeasegundarevisaosresultados,deformaaexistirumduplo

controle.

DeacordocomaInstruçãoNormativano205/88,25

Inventáriofísicoéoinstrumentodecontroleparaavericaçãodossaldosdeestoquesnos

almoxarifadosedepósitos,edosequipamentosemateriaispermanentes,emusonoórgãoou

entidade.

NaAdministraçãoPública,oinventárioéobrigatório.Tantoosbensmóveis

quantoosbensimóveisdevemserinventariados.Deacordocomalegislação,26o

levantamentodebensmóveiseimóveisteráporbaseoinventárioanalíticodecada

unidadeadministrativaeoselementosdaescrituraçãosintéticadacontabilidade.

Osgestorespúblicosdevemcontrolarosbensemateriaispúblicosemsuaárea

deresponsabilidade,poispoderãoresponderemcasodeperdaoudanoaosbens.

DeacordocomDias,27oinventáriotemosseguintesprincípios:

ØPrincípiodainstantaneidade:determinaomomento(diaehora)para

realizarolevantamentodoinventário.

ØPrincípiodaoportunidade:estabelecequeotempodeexecuçãodo

trabalhodelevantamentodeveseromínimopossível.Aexatidãodosdados

estáintimamenteligadaàproximidadedarealizaçãodotrabalho.

ØPrincípiodaespecificação:deneaformapelaqualoselementosdevemser

classificados,individualizadoseagrupadosdentrodamesmaespécie.

ØPrincípiodahomogeneidade:elegeumdenominadorcomumpara

expressarumaideiavalorativadetodososelementos,emgeralamoeda

corrente.

ØIntegridade:determinaque,umavezxadososlimitesdoinventário,todos

oselementospatrimoniaiscompreendidosdeverãoserobjetodo

levantamento.

ØPrincípiodauniformidade:determinaoestabelecimentodasmesmas

normas,estruturaçãoecritériosgeraisparaaconfecçãodetodosos

inventários,anoapósano,deformaapermitircomparaçõesentreeles.

20.16.FasesdoInventário

DeacordocomSantosetal.,28asfasesdoinventáriosãoasseguintes:

ØLevantamento–englobaacoletadedadossobreopatrimônio,podendoser

denominadolevantamentofísico(feitodiretamentenaidenticaçãoena

contagemdoscomponentesmateriais)oucontábil(feitoatravésdos

elementosregistradosnoslivrosenasfichasdeescrituração).

ØArrolamento–éoregistrodascaracterísticasedasquantidadeslevantadas

anteriormente.Podeserfeitodeformaresumida(sintética)oudemaneira

individual(analítica).

ØAvaliação–nessafaseatribuímosumaunidadedevaloraobem,deacordo

comseucusto.Semessafase,sófaríamosocontrolefísicodosbens,sem

poderavaliaroimpactoeconômico.

20.17.TiposdeInventário

Ostiposmaisconhecidosdeinventáriosãoosseguintes:

20.17.1.InventárioAnual

Ocorrenonaldoano(ounaldoanoscal,háalgunspaísesemqueelesnão

coincidem).Comosãocontadostodosositensdeumavez,esseprocessoé

bastantelongo.

20.17.2.InventáriosRotativos

Existemitensdeumestoquequedevemserchecadosmaisdeumavezporano.

Assim,muitasvezessãochecadosacadacompraouvendadematerial.

Itensdealtovalor,comojoiaseautomóveis,porexemplo,seriamchecados

rotineiramente.JáitenscomoresmasdepapelA4seriaminspecionadassomente

noinventárioanual.

Figura20.11–Tiposdeinventários

20.18.AvaliaçãodosBensnoInventário

Osbenspodemseravaliadosemuminventáriodeacordocomvárias

classificações.Diasapontaasseguintesclassificaçõesdosbens:29

ØReversível:éobemmóvelouimóvelqueintegraoacervopatrimonialdo

órgãoequeéindispensávelàprestaçãodoserviçopúblico.

ØAlienável:éobemmóvelouimóvelinservívelaoentepúblico.

ØInservível:éobemconsideradoocioso,cujarecuperaçãoéantieconômica

ouimpossível,nãosendo,portanto,maisviávelsuautilizaçãoemqualquer

atividaderelacionadaaoserviçoprestado.

ØAlienávelocioso:éobemqueseencontraemperfeitascondiçõesdeuso,

masnãoestásendoutilizado.

ØAlienávelderecuperaçãoantieconômico:éobemqueapresentadesgaste

prematuro

ouobsolescênciaepossuirendimentoprecário,cuja

recuperaçãoseriaonerosa,salvoaqueledevalorhistóricoquepoderáser

aproveitadopeloentepúblico.

ØIrrecuperável:éobemquenãopodemaisserutilizadoparaomaquese

destinadevidoàperdadesuascaracterísticas.

Vamosvercomoessetópicojáfoicobrado?

6.(FMP/TCE-RS/Auditor/2011)Opickingdiscreto,diferentedopickingporzona,estáassociadoa:

()separaçãodeprodutosdeumafamíliadeprodutos.

()separaçãodeprodutosrealizadaporumoperadorisoladamente.

()separaçãodeprodutosaseremauditadospelasorganizações,especialmenteaspúblicas.

a)FVF

b)VFV

c)VVF

d)VFF

e)FFV

Aprimeirafraseseidenticacomopickingporzona,enãocomopicking

discreto.Jáasegundaestáperfeita.

Aterceirafraseestáincorreta,poisopickingnãoestárelacionadoaositensque

serãoobjetodeauditorias,masatodasasretiradasdemateriaisdeum

almoxarifado.Dessaforma,ogabaritodaquestãoéaletraA.

QuestõesComentadas

7.(Cespe/MPU/Anal.adm./2010)Arotatividadedeumestoqueédeterminadapelonúmerode

vezesqueositensarmazenadossãorenovadosemdeterminadoperíododetempo.

Perfeito.Ogiroourotatividadedeestoqueécalculadodividindo-seoconsumo

médioanualpeloestoquemédiodesseperíodo.Quantomaioressenúmero,mais

eficienteaorganizaçãofoiaogeriressesestoques.Ogabaritoéquestãocorreta.

8.(FMP/TCE-RS/Auditor/2011)Oaumentodavariabilidadedoleadtimederessuprimentogeraa

necessidadedaseguintedecisão:

a)aumentaroestoquedesegurança;

b)reduçãodoestoquedesegurança;

c)aumentaraquantidadecomprada;

d)reduçãodaquantidadecomprada;

e)nenhumadelas.

Avariabilidadedoleadtimeéavariabilidadedoprazodeentrega(umavezleva

trêsdias,daoutralevatrêsmesesetc.)outempoderessuprimento.Quandoo

fornecedornãoéconável,oestoquedesegurançadaempresadeveser

aumentado.

Entretanto,nemsempreaquantidadecompradadeveaumentar(apesardeser

bastantecomum).Issoocorre,poisaquantidadedemandadapelosclientespode

diminuir.Ouseja,mesmoaumentandoamargemdesegurança,aquantidade

compradapodecarinalteradaoudiminuirtambém,ok?Ogabaritoémesmoa

letraA.

9.(FCC/MP/SE/Administrador/2009)Oconsumomédioanualdeumdeterminadoiteméde6.000

unidades.Oseuestoquemédiomensaléde600unidades.Oíndicederotatividadedesseitem

éiguala

a)5.

b)10.

c)15.

d)20.

e)30.

Oíndicederotatividadenadamaisédoqueogirodeestoques.Comojávimos,

afórmulapararesolveressaquestãoéaseguinte:

Deacordocomosdadosfornecidospelabanca,aequaçãoficariaassim:

Portanto,ogirodeestoqueéde10vezes,tornandoaletraBcorreta.

10.(Cespe/AGU/Administrador/2010)Osinventáriosrotativossãoefetuadosnonaldecada

exercíciofiscaldaempresaeincluematotalidadedositensdeestoquedeumasóvez.

Osinventáriosquesãoefetuadosumavezporanosãoosinventáriosanuais,não

osrotativos.Estessãoefetuadossemprequenecessário.Ogabaritoéquestão

errada.

11.(Cespe/Fundac/Administrador/2008)Aarmazenagemdemateriaisperigososdeveserfeitano

almoxarifadocomum,semqualquertipodeproteçãoespecial.

Esseiteméfácil,nãoépessoal?Claroquemateriaisperigososdevemser

acondicionadosdeformaespecial,demodoaprotegerasinstalaçõeseaspessoas

emcontatocomessesitens.Ogabaritoéquestãoincorreta.

12.(Cespe/AGU/Administrador/2010)Considerandoquecertaempresautilizeosistemadeponto

depedidoparaatingiramáximaeciênciadasreposiçõesdeseuestoque;queoconsumo

diáriodedeterminadoitemnessaempresasejade120unidades;queoperíodode

reabastecimentodoitemsejadedoisdiasequeoestoquedesegurançadoitemcorresponda

aoconsumodeumdia,écorretoarmarqueopontodepedidodoitememquestãoéde360

unidades.

Bom,vamosparaumaquestãoquepedeocálculodopontodepedido.De

acordocomabanca,temosumconsumodiáriode120unidadeseumtempode

ressuprimentodedoisdias.Alémdisso,abancaarmaqueoestoquemínimoéde

umdia,ouseja,120unidades.

Comisso,vamosutilizaraseguintefórmula:

PP=(CMédio×TR)+EstoqueSeg.

Substituindoosvalores:

PP=(120×2)+120=360

Assim,opontodepedidoémesmode360unidades,comoaassertivadoCespe

informou.Portanto,ogabaritoéquestãocorreta.

13.(Cespe/Aneel/Técnico/2010)Opregãoconstituimodalidadedelicitaçãoparaaquisiçãode

serviçoscomuns,qualquerquesejaovalorestimadodacontratação.

Perfeito.Opregãoéumamodalidadedelicitaçãoquevisacomprasdeprodutos

eserviçoscomuns,semlimitesdevalorespossíveisdeseremcontratados.O

gabaritoéquestãocorreta.

14.(Cespe/TRT/RN/Analista/2010)Évedadaacombinaçãodasmodalidadesdelicitaçãoprevistas

emlei,mas,noscasosemquecouberconvite,aadministraçãopoderáutilizaratomadade

preçose,emqualquercaso,aconcorrência.

Exato.Aproibiçãolegalexisteparaoscasosemqueovalorsejamaiordoquea

modalidadepermite(utilizartomadadepreçosquandoovalorindicaa

concorrência,porexemplo).Entretanto,ocontráriopode,sim,serfeito.A

concorrência,porexemplo,semprepodeserutilizada,mesmoquandoovaloré

baixo.Ogabaritoéquestãocorreta.

QuestõesPropostas

15.(Cespe/MPU/Anal.adm./2010)Métodosdeprevisãodeestoque,embasadosemmédiamóvel,

alémdeapresentaremformulaçãoexcessivamentecomplexa,constituemprocedimentoque

priorizaosdadosmaisrecentesemdetrimentodosmaisantigos.

16.(Cespe/Abin/OcialTécnico/2010)Oestoquedesegurançaécriadoapartirdaantecipaçãode

umademandafuturaconhecida.

17.(FMP/CmStaBárbaraD’Oeste/Logística/2010)Qualarotatividadedeumitemqueapresenta

umconsumoanualde12864unidadeseoestoquemédioiguala50%doconsumomédio

mensaldoperíodo?

a)24vezes/ano.

b)12vezes/ano.

c)18vezes/ano.

d)6vezes/ano.

e)20vezes/ano.

18.(FCC/MP/RS/Administrador/2008)Considera-seumagestãodemateriaisbem-sucedidaaquela

queconsegueestabelecerumequilíbrioentre:

a)acessoacréditoequalidadedeserviço;

b)taxadelucroesperadaeníveldeestoque;

c)capacidadedeendividamentoedemandaefetiva;

d)necessidadedefinanciamentoeníveldeoferta;

e)disponibilidadedecapitaldegiroeníveldeserviço.

19.(Cespe/MEC/Analista/2009)Quandoadotaumaadministraçãodemateriaisemestoqueque

privilegiaasaídadosmateriaisquederamentradamaisrecentemente,oencarregadode

materialutilizaométodoUEPS.

20.(Cespe/Anac/Analista/2009)Sistemasdeproduçãoembasadosnométodojust-in-timesão

intensivosemutilizaçãodeespaçofísicoparaestocagemdematéria-primaoudemercadorias

aseremvendidaspelaorganização.

21.(Cesgranrio/CasadaMoeda/Analistagestão/2009)OsprincípiosdoJust-in-time(JIT)produziram

mudançasradicaisemrelaçãoàpráticatradicionaldagestãodaproduçãoutilizadaatéas

décadasde1960/70.EntreascaracterísticasefatoresadotadosnoJIT,podem-secitar:

I.produçãoantecipada,consideradacomodesperdício;

II.melhoreficiênciadamãodeobra,dedicandoumtrabalhadorpormáquina;

III.sistemaempurradodecontroledaprodução;

IV.utilizaçãodelotesdeproduçãoreduzidos.

Estãocorretosositens

a)IeI,apenas.

b)IeIV,apenas.

c)IeII,apenas.

d)I,IIeIV,apenas.

e)I,I,IIeIV.

22.(Esaf/Susep/Adm.nanceira/2010)Considerandoaboatécnicadagestãodemateriais,é

incorretoafirmar:

a)opontodopedidodevecoincidircomomomentoemqueoestoqueaserressupridoatingeonível

desegurança;

b)aoutilizarmosaCurvaABCsobreototaldositensaseremestocados,osmateriaisquerepresentem

20%daquantidadee80%dovalordevemserclassificadoscomo‘A’;

c)emumsistemadeestoque,amovimentaçãodemateriaispodeserrepresentadaporumgráco

conhecidopor‘dentedeserra’;

d)LoteEconômicoéoresultadodeumprocedimentomatemáticopormeiodoqualaorganização

adquire,pelocustomaisbaixo,omaterialnecessárioàssuasatividades;

e)oaltogirodosestoquescontribuiparaareduçãodovalordocapitaldegiro.

23.(Esaf/Susep/Adm.nanceira/2010)NoâmbitodaAdministraçãoPública,acercadoinventário

debenspatrimoniais,éincorretoafirmar:

a)inventárioéadiscriminaçãoorganizadaeanalíticadetodososbensevaloresdeumpatrimônio,em

umdeterminadomomento,visandoatenderumafinalidadeespecífica;

b)oinventáriodebensmóveiséfacultativo.Odebensimóveiséobrigatório;

c)oprincípiodaintegridadedeterminaque,umavezxadososlimitesdoinventário,todosos

elementospatrimoniaiscompreendidosdevemserobjetodolevantamento;

d)oinventárioédivididoemtrêsfases:levantamento,arrolamentoeavaliação;

e)classifica-secomoinservívelobemociosocujarecuperaçãosejaantieconômicaouimpossível.

24.(Esaf/TJ-CE/Auxiliar/2002)Assinaleaopçãoquenãodescreveumatarefadeumsetorde

controledeestoque:

a)retirardoestoqueosprodutosobsoletosedanificados;

b)confeccionarasembalagensparaguardadosprodutos;

c)solicitaraosetordecomprasaaquisiçãodoprodutoemfalta;

d)determinarquandooestoquedeveserreabastecido;

e)fornecerinformaçõessobreaposiçãodoestoque.

25.(Esaf/TJ-CE/Auxiliar/2002)Nasaídadematerialdeumalmoxarifado,umdosprocedimentos

quedeveserusadoé:

a)fazeroregistrodomaterialqueestásendoretirado;

b)providenciaraemissãodarequisiçãodecompra;

c)buscarinformaçõesarespeitodosolicitante;

d)pedirautorizaçãoaodiretordaorganizaçãoparaatenderàrequisição;

e)receberumaviadanotadedevolução.

26.(Cespe/Abin/OcialTécnico/2010)Duranteoprocessodeinventário,éimprescindívelquese

procedaaumasegundacontagemdositensparaaconferênciadaprimeiracontagem,ainda

queporamostragem.

27.(FGV/Senado/Técnico/2008)OtermoVerticalizaçãoéutilizadonaadministraçãodemateriaise

significa:

a)aposiçãoqueaáreademateriaistemnahierarquiadaorganização;

b)apossibilidadequeaempresatem,juntoaterceirosefornecedores,dealterarasespecicaçõesdos

materiaisdequenecessitaparaproduçãoouconsumointerno;

c)apossibilidadequeaempresatem,juntoaterceirosefornecedores,deidenticaropadrãode

qualidadedosmateriaisdequenecessitaparaproduçãoouconsumointernoedeexigirumpadrãode

qualidadesuperior;

d)aestratégiaqueprevêqueaempresaproduziráinternamentetudooquepuderou,aomenos,

tentaráproduzir,parausonosprodutosfinais;

e)aestratégiaqueprevêacompradomaiornúmerodeitensdequenecessitaparaoprodutofinaloude

materialdeusoouconsumodeummesmofornecedor.

28.(Vunesp/Cetesb/Analista/2009)Adeniçãodeumaestratégiacorretadecompraspodedarà

empresaumagrandevantagemcompetitiva.Se,porumlado,eladecidirpelaverticalização,

ouseja,produzirinternamente,ganhaindependência,masperdeexibilidade.Seadecisãofor

pelahorizontalização,ouseja,comprardeterceiros,tem-seaseguintevantagem:

a)aumentodaestruturadaempresa;

b)maioreslucros;

c)reduçãodecustos;

d)maiorautonomia;

e)maiorexposição.

29.(Cespe/TSE/Analista/2007)Comoaumentodacompetitividadeentreasempresas,a

negociaçãodecomprasmodernaécaracterizadapeladisputaacirradaentrecompradore

vendedor,comavitóriadeapenasumdeles.

30.(Cespe/Cehap-PB/Administrador/2009)Aqualicaçãodosfornecedoresnãosetornarelevante

paraaanálise,vistoqueomenorpreçosempreéoquemelhoratenderáosinteressesda

empresacompradora.

31.(Cespe/Cehap-PB/Administrador/2009)Atualmente,afunçãodecompradornãoémaisade

anotadordepedidos,massimadeumconhecedordasmercadoriasedosfornecedoreseum

bomnegociador.

32.(FMP/TCE-RS/Auditor/2011)Ascomprasdeentidadespúblicasdevemobservar,porforçadalei,

algunsprincípios.Sãoeles:

I.impessoalidade

II.moralidade

III.publicidade

IV.pontualidade

Corretas:

a)sóIeI

b)sóIeII

c)sóI,IeII

d)sóIeIV

e)todas

33.(FCC/MP-SED/Cargo2/2009)Emrelaçãoàsetapasdalicitação,écorretoafirmarquea:

a)inversãodasfasesdehabilitaçãoejulgamentodaspropostaséadmitidaapenasnamodalidade

convite;

b)fasedehabilitaçãodevesempreprecederadejulgamentodaspropostas,independentementeda

modalidadelicitatória;

c)inversãodefasesdehabilitaçãoedejulgamentodaspropostaséadmitida,acritériodacomissãode

licitação,independentementedamodalidadelicitatória;

d)fasedeclassificaçãodaspropostasprecedeadehabilitação,namodalidadepregão;

e)fasedejulgamentodaspropostasdevesempreprecederadehabilitação,nasmodalidadespregãoe

concorrênciapública.

34.(FMP/TCE-RS/Auditor/2011)Sobrealicitação,assinaleaassertivacorreta.

a)Opregãoémodalidadedelicitação.

b)Ovencedordalicitaçãopossuidireitopúblicosubjetivoàcelebraçãodocontrato.

c)Adeclaraçãodeinegibilidadedelicitaçãoéatoadministrativodiscricionário.

d)Considera-sedesertaalicitaçãoquandonenhumlicitanteforconsideradohabilitado.

e)Aanulaçãodoprocedimentodelicitaçãogeraodeverdeindenizaraoslicitantesquenãoderamcausa

anulidade.

35.(FCC/TRF/Técnico/2006)OlimiteparalicitaçõesdeatéR$1.500.000,00,paraobraseserviçosde

engenharia,refere-sea:

a)pregão;

b)leilão;

c)convite;

d)concorrência;

e)tomadadepreços.

36.(Cespe/TCU/Técnico/2007)OestudodaslicitaçõesdeveterporbaseaLeino8.666/1993,aqual

estabelece,minuciosamente,asnormassobrelicitaçõesecontratosdaadministraçãopública.

37.(FMP/TCE-RS/Auditor/2011)Determinadoprefeitomunicipal,deummunicípiodoestadodoRio

GrandedoSul,noexercíciodoseupodernormativo,comanalidadederegulamentaraLeino

8.666/1993,editoudecretomunicipalestabelecendoqueashipótesesdelicitaçãodispensável

einexigibilidadedelicitação,apartirdoanode2012,seriamdecididaspelaComissão

PermanentedeLicitações,apenascombasenopoderdiscricionário.Leiacomatençãoas

seguintesassertivas:

I.Oprefeitomunicipalnãopoderiaeditarodecreto,poisaLeino8666/93nãoseaplicaaos

municípios.

II.Emrelaçãoàlicitaçãodispensável,efetivamente,emalgumashipótesesdoart.24daLeino

8.666/1993,hácertograudediscricionariedadeadministrativa.

III.NoquadronormativodaLeino8.666/1993,umadashipótesesdelicitaçãodispensável

verica-sequandonãoacudireminteressadosàlicitaçãoanterioreesta,justicadamente,

nãopuderserrepetidasemprejuízoparaaadministração,mantidas,nessecaso,todasas

condiçõespreestabelecidas.

IV.Noexercíciodopoderadministrativodeeditardecretosmunicipaisnãoincidemos

princípiosqueregulamaatividadeadministrativa,comoosprevistosnoart.37,caput,da

ConstituiçãoFederal.

V.Ainexigibilidadedelicitação,institutoexpressamenteprevistonoart.25daLeino8.666/93,

deveserutilizada,preferencialmente,paraacontrataçãodeserviçosdepublicidadee

divulgação.

Assinaleaúnicaalternativacorreta.

a)AsassertivasIeIIestãocorretas.

b)AsassertivasIeIIestãocorretas.

c)AsassertivasIVeVestãocorretas.

d)SomenteaassertivaIVestácorreta.

e)SomenteaassertivaVestácorreta.

38.(UFBA/UFBA/Assistenteadm./2006)Alicitaçãodestina-seagarantiraobservânciadoPrincípio

ConstitucionaldaIsonomia.

Gabaritos

1.A

14.C

27.D

2.B

15.E

28.C

3.A

16.E

29.E

4.C

17.A

30.E

5.E

18.E

31.C

6.A

19.C

32.C

7.C

20.E

33.D

8.A

21.B

34.A

9.B

22.A

35.E

10.E

23.B

36.C

11.E

24.B

37.A

12.C

25.A

38.C

13.C

26.C

Bibliografia

Ballou,RonaldH.BusinessLogisticsManagement:Planning,organizingand

controllingthesupplychain.4aed.UpperSaddleRiver:Prentice-Hall,1998.

Barros,MonicaCoutinhode.Warehousemanagementsystem(WMS):conceitos

teóricoseimplementaçãoemumcentrodedistribuição.DissertaçãodeMestrado

–DptodeEngenhariaIndustrial.RiodeJaneiro:PUC-Rio,2005.

Costin,Claudia.AdministraçãoPública.RiodeJaneiro:Elsevier,2010.

Dias,AlexandraFurtadodaSilva.GestãoPatrimonialdaAdministraçãoPública

Estadual.Florianópolis:GovernodoEstadodeSantaCatarina,Novembro2006.

Dias,MarcoAurélioP.AdministraçãodeMateriais.6aed.SãoPaulo:Atlas,2009.

FariaJunior,Cyonil,eSandroHenriqueBernardes.LicitaçõeseContratos.2aed.

RiodeJaneiro:Elsevier,2010.

Fenili,Renato.Administraçãoderecursosmateriaisepatrimoniaisparaconcursos.

RiodeJaneiro:Método,2011.

Francischini,PaulinoG.,eFlorianodoAmaralGurgel.Administraçãodemateriais

edopatrimônio.SãoPaulo:PioneiraThomsonLearning,2004.

MonteAlto,ClélioFeres,AntônioMendesPinheiro&PauloCaetanoAlves.

Técnicasdecompras.RiodeJaneiro:FGV,2009.

PresidênciadaRepública.InstruçãoNormativano205,de08deAbrilde1988.

1988.

Santos,EvanirAguiardos,LaoneSimonettideOliveira,MirtaMelo,Paulo

Caregnatto,eRogerMacieldeOliveira.InventárioPúblico.CursodePós-

GraduaçãoemAuditoriaePerícia.PortoAlegre:FaculdadesPorto-Alegrense,

junho2008.

Teixeira,Carolina.Administraçãoderecursosmateriaisparaconcursos:teoriae

exercíciosdoCespecomentados.SãoPaulo:Método,2010.

Viana,JoãoJosé.AdministraçãodeMateriais:umenfoqueprático.SãoPaulo:Atlas,

2010.

Notas

Capítulo01

1Chiavenato,HistóriadaAdministração:entendendoaadministraçãoesuapoderosainuêncianomundomoderno,2009.

2AndradeeAmboni,2011.

3Chiavenato,IntroduçãoàTeoriaGeraldaAdministração,2011.

4CertoeCerto,2006.

5SobralePeci,2008.

6Daft,2005.

7Chiavenato,IntroduçãoàTeoriaGeraldaAdministração,2011.

8SchemerhornJr.,2008.

9SobralePeci,2008.

10Chiavenato,IntroduçãoàTeoriaGeraldaAdministração,2011.

11AndradeeAmboni,2011.

12Chiavenato,IntroduçãoàTeoriaGeraldaAdministração,2011.

13CertoeCerto,2006.

14AndradeeAmboni,2011.

15Fayol,1955apudChiavenato,HistóriadaAdministração:entendendoaadministraçãoesuapoderosainuêncianomundomoderno,2009.

16Fayol,1955apudChiavenato,HistóriadaAdministração:entendendoaadministraçãoesuapoderosainuêncianomundomoderno,2009.

17CertoeCerto,2006.

18AndradeeAmboni,2011.

19CertoeCerto,2006.

20Daft,2005.

21Chiavenato,HistóriadaAdministração:entendendoaadministraçãoesuapoderosainuênciano

mundomoderno,2009.

22SobralePeci,2008.

23RobbinseCoulter,1998.

24SobralePeci,2008.

25AndradeeAmboni,2011.

26AndradeeAmboni,2011.

27EtzioneapudLima,2005.

28Lima,2005.

29Chiavenato,IntroduçãoàTeoriaGeraldaAdministração,2011.

30Presthus,1965apudChiavenato,IntroduçãoàTeoriaGeraldaAdministração,2011.

31WhiteJr.,1966apudChiavenato,IntroduçãoàTeoriaGeraldaAdministração,2011.

32GouldnerapudChiavenato,IntroduçãoàTeoriaGeraldaAdministração,2011.

33Chiavenato,HistóriadaAdministração:entendendoaadministraçãoesuapoderosainuêncianomundomoderno,2009.

34Bertalanffy,1975apudChiavenato,HistóriadaAdministração:entendendoaadministraçãoesuapoderosainfluêncianomundomoderno,2009.

35Chiavenato,IntroduçãoàTeoriaGeraldaAdministração,2011.

36SobralePeci,2008.

37MottaeVasconcelos,2004.

38MottaeVasconcelos,2004.

39Chiavenato,AdministraçãoGeralePública,2008.

Capítulo02

1Schein,1993.

2Robbins,OrganizationalBehavior,2004.

3SobralePeci,2008.

4Rocha,etal.2010.

5Robbins,OrganizationalBehavior,2004.

6Robbins,OrganizationalBehavior,2004.

7RobbinseCoulter,Administração,1998.

8Disponívelem:<www.fundacaohantipoff.mg.gov.br/pdf/pesquisaclima/o_que_e.pdf>

9Disponívelem:<www.fundacaohantipoff.mg.gov.br/pdf/pesquisaclima/o_que_e.pdf>Capítulo03

1PearceeRobinson,2004apudSobralePeci,2008.

2SobralePeci,2008.

3Oliveira,2007.

4Chiavenato,AdministraçãoGeralePública,2008.

5Oliveira,2007.

6SobralePeci,2008.

7SchemerhornJr.,2008.

8Chiavenato,Administraçãonosnovostempos,2010.

9SobralePeci,2008.

10KaplaneNorton,2000.

11MatoseChiavenato,1999apudBarbosaeBrondani,2004.

12Chiavenato,2010.

13Oliveira,2007.

14VasconcelosePagnoncelli,2001apudLobato,etal.2009.

15Lobato,etal.2009.

16SchemerhornJr.,2008.

17Mintzberg,AhlstrandeLampel,2005.

18Andrews,1982apudMintzberg,AhlstrandeLampel,2005.

19Porter,1980apudMintzberg,AhlstrandeLampel,2005.

20Mintzberg,AhlstrandeLampel,2005.

21Schwartz,1996.

22Oliveira,2007.

23Porter,WhatisStrategy?,1996.

24ChandlerapudOliveira,2007.

25Porter,HowCompetitiveForcesShapeStrategy1979.

26Porter,EstratégiaCompetitiva:técnicasparaanálisedeindústriasedaconcorrência,1986.

27Decourt,NeveseBaldner,2012.

28Lobato,etal.2009.

29Decourt,NeveseBaldner,2012.

30HamelePrahalad,1989.

31Oliveira,2007.

32Oliveira,2007.

33Oliveira,2007.

34Oliveira,2007.

35Oliveira,2007.

36PitassieMacedo-Soares,2002apudTauhataedeMacedoSoares,2004.

37Tavares,2002apudTauhataedeMacedoSoares,2004.

38Gulati,etal.2000apudTauhataedeMacedoSoares,2004.

39TauhataedeMacedoSoares,2004.

40Drucker,1954apudSobralePeci,2008

41SobralePeci,2008.

42Lobato,etal.2009.

43KaplaneNorton,1996.

44Paludo,2010.

45KaplaneNorton,1996.

46Chiavenato,2010.

47KaplaneNorton,1996.

48KaplaneNorton,1996.

49KaplaneNorton,1996.

50Herrero,2005.

51Herrero,2005.

52KaplaneNorton,2004apudHerrero,2005.

53KaplaneNortonapudHerrero,2005.

54Disponívelem:<http://www2.anac.gov.br/arquivos/ANAC%20Planejamento%20Estrategico.pdf>

55Chiavenato,2010.

56Oliveira,2007.

Capítulo04

1SchermerhornJr.,2008.

2SobralePeci,2008.

3Chiavenato,Administraçãonosnovostempos,2010.

4SchermerhornJr.,2008.

5Daft,2005.

6Chiavenato,AdministraçãoGeralePública,2008.

7SobralePeci,2008.

8Daft,2005.

9Chiavenato,IntroduçãoàTeoriaGeraldaAdministração,2011.

10SobralePeci,2008.

11Chiavenato,IntroduçãoàTeoriaGeraldaAdministração,2011.

12SobralePeci,2008.

13Chiavenato,IntroduçãoàTeoriaGeraldaAdministração,2011.

14SchermerhornJr.,2008.

15SobralePeci,2008.

16RobbinseCoulter,1998.

17Chiavenato,AdministraçãoGeralePública,2008.

18Daft,2005.

19Maximiniano,1995.

20SobralePeci,2008.

21MintzbergeQuinn,2001.

22MintzbergeQuinn,2001.

Capítulo05

1Daft,2005.

2RobbinseCoulter,Administração,1998.

3Bergamini,1990.

4Clegg,KornbergerePitsis,2008.

5Clegg,KornbergerePitsis,2008.

6Chiavenato,AdministraçãoGeralePública,2008.

7Chiavenato,Administraçãonosnovostempos,2010.

8Robbins,OrganizationalBehavior,2004.

9Robbins,OrganizationalBehavior,2004.

10RobbinseCoulter,Administração,1998.

11Robbins,OrganizationalBehavior,2004.

12Robbins,OrganizationalBehavior,2004.

13Locke,1968apudRobbins,OrganizationalBehavior,2004.

14Yassuda,LascaeNeri,2005.

15Chiavenato,GestãodePessoas:eonovopapeldosrecursoshumanosnasorganizações,2004.

16RobbinseCoulter,Administração,1998.

17RobbinseCoulter,Administração,1998.

18Chiavenato,GestãodePessoas:eonovopapeldosrecursoshumanosnasorganizações,2004.

19Kanter,1983apudChiavenato,GestãodePessoas:eonovopapeldosrecursoshumanosnasorganizações,2004.

Capítulo06

1Zaleznik,1992.

2Daft,2005.

3Chiavenato,AdministraçãoGeralePública,2008.

4Daft,2005.

5Zaleznik,1992.

6Chiavenato,Administraçãonosnovostempos,2010.

7SobralePeci,2008.

8Daft,2005.

9Daft,2005.

10RobbinseCoulter,Administração,1998.

11BlakeeMouton,1985apudClegg,KornbergerePitsis,2008.

12Clegg,KornbergerePitsis,2008.

13Chiavenato,IntroduçãoàTeoriaGeraldaAdministração,2011.

14SobralePeci,2008.

15CertoeCerto,2006.

16Clegg,KornbergerePitsis,2008.

17Clegg,KornbergerePitsis,2008.

18CongereKanengoapudRobbins,2002apudCavalcanti,etal.2009.

19Robbins,OrganizationalBehavior,2004.

20Robbins,OrganizationalBehavior,2004.

21Cavalcanti,etal.2009.

22BenniseNanus,1988apudCavalcanti,etal.2009.

23AlbrechtapudBoyetteBoyett,1999apudCavalcanti,etal.2009.

24BenniseNanus,1988apudCavalcanti,etal.2009.

Capítulo07

1RobbinseCoulter,1998.

2Maximiniano,1995.

3SobralePeci,2008.

4Daft,2005.

5SobralePeci,2008.

6SobralePeci,2008.

7Chiavenato,2008.

8RobbinseCoulter,1998.

9Rua,s.d.

10Rua,s.d.

11TribunaldeContasdaUnião,2009.

12Palvarini,2010.

13RummlereBracheapudSouza,etal.2009.

14Souza,etal.2009.

15Palvarini,2010.

16Bennett,1976;Wholey,1979apudPalvarini,2010.

17MinistériodoPlanejamento,2009.

18TribunaldeContasdaUnião,2009.

19TribunaldeContasdaUnião,2009.

Capítulo08

1Gulick,1937apudGrahamJr.eHays,1994.

2FayolapudChiavenato,2011.

3Mintzberg,1979apudGrahamJr.eHays,1994.

Capítulo09

1Chiavenato,2010.

2Berlo,2003apudMarchiori,2008.

3SchemerhornJr.,2008.

4Macêdo,etal.2007.

5Marchiori,2008.

6Robbins,2004.

7SchermerhornJr.,2008.

8Daft,2005.

9Macêdo,etal.2007.

10Macêdo,etal.2007.

11Daft,2005.

12Daft,2005.

13Macêdo,etal.2007.

14Robbins,2004.

15Macêdo,etal.2007.

Capítulo10

1CertoeCerto,2006.

2SchermerhornJr.,2008.

3CertoeCerto,2006.

4SchermerhornJr.,2008.

5CertoeCerto,2006.

6SchermerhornJr.,2008.

7Lemos,1999.

8SchermerhornJr.,2008.

9CertoeCerto,2006.

10SchermerhornJr.,2008.

11SchermerhornJr.,2008.

12Kotter&Schlessinger,1979apudSchermerhornJr.,2008.

13LewinapudSchermerhornJr.,2008.

Capítulo11

1Aronson,WilsoneAkert,2004.

2Freud,1930apudAronson,WilsoneAkert,2004.

3Daft,2005.

4HarveyapudDaft,2005.

5SchermerhornJr.,2008.

6Daft,2005.

7Daft,2005.

8Chiavenato,AdministraçãoGeralePública,2008.

9Chiavenato,AdministraçãoGeralePública,2008.

10Chiavenato,AdministraçãoGeralePública,2008.

11Chiavenato,GestãodePessoas:eonovopapeldosrecursoshumanosnasorganizações,2004.

12RobbinseCoulter,1998.

13Daft,2005.

Capítulo12

1Davenport,1992.

2Gonçalves,Processo,queprocesso?,2000.

3Harrington,1991apudGonçalves,Processo,queprocesso?,2000.

4Davenport,1992.

5Nunes,2010.

6HammereStanton,1999apudGonçalves,AsEmpresassãograndescoleçõesdeprocessos,2000.

7Gonçalves,Processo,queprocesso?,2000.

8Gonçalves,1997apudGonçalves,AsEmpresassãograndescoleçõesdeprocessos,2000.

9Martins,HumbertoFalcãoeMarini,2010.

10Gonçalves,Processo,queprocesso?,2000.

11Martin,1996apudGonçalves,Processo,queprocesso?,2000.

12Gonçalves,Processo,queprocesso?,2000.

13ABNT,2000apudValls,2004.

14ABNT,2000.

15Zacharias,2001apudValls,2004.

16Valls,2004.

17MaranhãoeMacieira,2010.

18MaranhãoeMacieira,2010.

19MaranhãoeMacieira,2010.

20HammereChampy,1994.

21HammereChampy,1994.

22HammereChampy,1994.

23HammereChampy,1994.

24HammereChampy,1994.

25HammereChampy,1994.

26J.L.Gonçalves,1994.

Capítulo13

1Tuchman,1980apudJunior,etal.2008.

2Edwards,1968apudJunior,etal.2008.

3Gilmore,1974apudJunior,etal.2008.

4Garvin,1988.

5Garvin,1988.

6Dale,1999.

7Garvin,1988.

8Junior,etal.2008.

9Dale,1999.

10ISO8402apudDale,1999.

11Crosby,1990.

12Junior,etal.2008.

13SobralePeci,2008.

14Daft,2005.

15Coltro,1996.

16Junior,etal.2008.

17Junior,etal.2008.

18Junior,etal.2008.

19Deming,1990apudJunior,etal.2008.

20Junior,etal.,2008.

21Mello,2011.

22Mello,2011.

23Mello,2011.

24CarvalhoePaladini,2012.

25CarvalhoePaladini,2012.

26Mello,2011.

27Lima,2005.

28Lima,2005.

29Lima,2005.

30Fonte:<http://www.fnq.org.br/site/376/default.aspx>

31Fonte:<http://www.fnq.org.br/site/376/default.aspx>

Capítulo14

1Santos,2001.

2DavenportePrusak,1998apudCarbone,etal.2009.

3Santos,2001.

4NonakaeTakeuchiapudSantos,2001.

5NonakaeTakeuchi,1997apudAlves,OliveiraeQuandt,s.d.

6Quandt,2001apudAlves,OliveiraeQuandt,s.d.

7Drucker,1993apudCoelho,2004.

8Castells,1999apudCoelho,2004.

9Terra,2001apudCoelho,2004.

10Polanyi,1983apudCarbone,etal.2009.

11PolanyiapudNonaka,1994.

12Fernandes,2009.

13ArgyriseSchon,1978apudAntonelloeGodoy,2010.

14Motta,2009.

15Motta,2009.

16MarsickeWatkins,2001apudFernandes,2009.

17Malcom,HodkinsoneColley,2003apudFernandes,2009.

18Moresi,2001.

19Nonaka,1994.

20Coelho,2004.

21Quandt,2001apudAlves,OliveiraeQuandts.d.

Capítulo15

1Maximiniano,1995.

2RobbinseCoulter,1998.

3Chiavenato,AdministraçãoGeralePública,2008.

4Chiavenato,AdministraçãoGeralePública,2008.

5Daft,2005.

6Daft,2005.

7Chiavenato,IntroduçãoàTeoriaGeraldaAdministração,2011.

8Daft,2005.

9Maximiniano,1995.

10Maximiniano,1995.

11Junior,etal.2008.

12Lewin,1951apudMaximiniano,1995.

13Junior,etal.2008.

14SobralePeci,2008.

Capítulo16

1Taylor,1982apudDutra,2010.

2LeBoterf,1994apudDutra,2010.

3Carbone,etal.2009.

4BundcheneSilva,2005.

5PrahaladeHamelapudBrandãoeBahry,Gestãoporcompetências:métodosetécnicasparamapeamentodecompetências,2005.

6BrandãoeGuimarães,Gestãodecompetênciasegestãodedesempenho:tecnologiasdistintasou

instrumentosdeummesmoconstruto,2001.

7BundcheneSilva,2005.

8Mills,etal.2002apudDutra,2010.

9Carbone,etal.2009.

10Carbone,etal.2009.

11BrandãoeBahry,Gestãoporcompetências:métodosetécnicasparamapeamentodecompetências,2005.

12BrandãoeBahry,Gestãoporcompetências:métodosetécnicasparamapeamentodecompetências,2005.

13Brandão,Zimmer,etal.2008.

14BrandãoeBahry,Gestãoporcompetências:métodosetécnicasparamapeamentodecompetências,2005.

15BrandãoeGuimarães,Gestãodecompetênciasegestãodedesempenho:tecnologiasdistintasouinstrumentosdeummesmoconstruto,2001.

16SobralePeci,2008.

17RobbinseCoulter,1998.

Capítulo17

1Robbins,2004.

2Chiavenato,Administraçãonosnovostempos,2010.

3ClockeGoldsmithapudChiavenato,AdministraçãoGeralePública,2008.

4Robbins,2004.

Capítulo18

1Ivancevich,1998.

2Marras,2011.

3Schikmann,2010.

4Chiavenato,Gestãodepessoas:eonovopapeldosrecursoshumanosnasorganizações,2004.

5Chiavenato,Gestãodepessoas:eonovopapeldosrecursoshumanosnasorganizações,2004.

6Pires,etal.2005.

7Schikmann,2010.

8Schikmann,2010.

9Marras,2011.

10VilasBoaseAndrade,2009.

11VilasBoaseAndrade,2009.

12Chiavenato,Gestãodepessoas:eonovopapeldosrecursoshumanosnasorganizações,2004.

13VilasBoaseAndrade,2009.

14Chiavenato,Gestãodepessoas:eonovopapeldosrecursoshumanosnasorganizações,2004.

15Costin,2010.

16Lima,2005.

17Chiavenato,Gestãodepessoas:eonovopapeldosrecursoshumanosnasorganizações,2004.

18Schikmann,2010.

19Costin,2010.

20VilasBoaseAndrade,2009.

21Dutra,2001apudSchikmann,2010.

22Marras,2011.

23VilasBoaseAndrade,2009.

24Marras,2011.

25Chiavenato,Gestãodepessoas:eonovopapeldosrecursoshumanosnasorganizações,2004.

26VilasBoaseAndrade,2009.

27Chiavenato,Gestãodepessoas:eonovopapeldosrecursoshumanosnasorganizações,2004.

28Ivancevich,1998.

29Chiavenato,Gestãodepessoas:eonovopapeldosrecursoshumanosnasorganizações,2004.

30Chiavenato,Gestãodepessoas:eonovopapeldosrecursoshumanosnasorganizações,2004.

31Chiavenato,Gestãodepessoas:eonovopapeldosrecursoshumanosnasorganizações,2004.

32VilasBoaseAndrade,2009.

33Chiavenato,Gestãodepessoas:eonovopapeldosrecursoshumanosnasorganizações,2004.

34Mello,2009.

25Chiavenato,Gestãodepessoas:eonovopapeldosrecursoshumanosnasorganizações,2004.

36Chiavenato,Gestãodepessoas:eonovopapeldosrecursoshumanosnasorganizações,2004.

37Davis,1966apudChiavenato,Gestãodepessoas:eonovopapeldosrecursoshumanosnasorganizações,2004.

38Chiavenato,Gestãodepessoas:eonovopapeldosrecursoshumanosnasorganizações,2004.

39Chiavenato,IntroduçãoàTeoriaGeraldaAdministração,2011.

40Chiavenato,IntroduçãoàTeoriaGeraldaAdministração,2011.

41Drucker,1992apudChiavenato,Gestãodepessoas:eonovopapeldosrecursoshumanosnasorganizações,2004.

Capítulo19

1Umguiadoconhecimentoemgerenciamentodeprojetos(GuiaPMBOK),s.d.

2Caupin,1999apudValeriano,2005.

3Mendes,ValleeFabra,2009.

4Fonte:<http://pt.wikipedia.org/wiki/Pirâmides_de_Gizé>

5Umguiadoconhecimentoemgerenciamentodeprojetos(GuiaPMBOK),s.d.

6Umguiadoconhecimentoemgerenciamentodeprojetos(GuiaPMBOK),s.d.

7Umguiadoconhecimentoemgerenciamentodeprojetos(GuiaPMBOK),s.d.

8Umguiadoconhecimentoemgerenciamentodeprojetos(GuiaPMBOK),s.d.

9Umguiadoconhecimentoemgerenciamentodeprojetos(GuiaPMBOK),s.d.

10Valeriano,2005.

11Umguiadoconhecimentoemgerenciamentodeprojetos(GuiaPMBOK),s.d.

12Heerkens,2002.

13Vargas,2003.

14Umguiadoconhecimentoemgerenciamentodeprojetos(GuiaPMBOK),s.d.

15Vargas,2003.

16Umguiadoconhecimentoemgerenciamentodeprojetos(GuiaPMBOK),s.d.

17Vargas,2003.

18MereditheMantelJr.,2003.

19Mendes,ValleeFabra,2009.

20Umguiadoconhecimentoemgerenciamentodeprojetos(GuiaPMBOK),s.d.

21Vargas,2003.

22Umguiadoconhecimentoemgerenciamentodeprojetos(GuiaPMBOK),s.d.

23Umguiadoconhecimentoemgerenciamentodeprojetos(GuiaPMBOK).s.d.

24Umguiadoconhecimentoemgerenciamentodeprojetos(GuiaPMBOK),s.d.

25Umguiadoconhecimentoemgerenciamentodeprojetos(GuiaPMBOK),s.d.

26GidoeClements,2006.

27Umguiadoconhecimentoemgerenciamentodeprojetos(GuiaPMBOK),s.d.

28Mendes,ValleeFabra,2009.

29Mendes,ValleeFabra,2009.

Capítulo20

1Viana,2010.

2Dias,2009.

3Viana,2000apudTeixeira,2010.

4Dias,2009

5Fenili,2011.

6MonteAlto,PinheiroeAlves,2009.

7MonteAlto,PinheiroeAlves,2009.

8Fenili,2011.

9Teixeira,2010.

10MonteAlto,PinheiroeAlves,2009.

11FariaJunioreBernardes,2010.

12Meirelles,1996apudCostin,2010.

13LeiFederalno8.666/93.

14LeiFederalno8.666/93.

15LeiFederalno8.666/93.

16LeiFederalno8.666/93.

17LeiFederalno8.666/93.

18Barros,2005.

19Viana,2010.

20Barros,2005.

21Ballou,1998.

22Bowersox&Closs,2001apudBarros,2005.

23Rodrigues,1999apudBarros,2005.

24Lima,2002apudBarros,2005.

25PresidênciadaRepública;1988.

26Santos,etal.2008.

27Dias,2006.

28Santos,etal.2008.

29Dias,2006.

DocumentOutlineCadastroFolhadeRostoCréditosDedicatóriasAgradecimentosOAutorPrefácioApresentaçãoSumárioCapítulo1–EvoluçãodasTeoriasAdministrativas

1.1.AAdministraçãoCientífica1.2.ATeoriaClássica1.3.TeoriadasRelaçõesHumanas1.4.Burocracia1.5.TeoriaEstruturalista1.6.TeoriadosSistemas1.7.TeoriaContingencial

Capítulo2–CulturaOrganizacional2.1.NíveisdaCultura2.2.CulturaForteeCulturaFraca2.3.ElementosdaCultura

2.3.1.ArtefatosVerbais2.3.2.ArtefatosComportamentais2.3.3.ArtefatosFísicos

2.4.CulturaMecanicistaeCulturaOrgânica2.5.CulturaDominanteeSubculturas2.6.DisfunçõesdaCultura2.7.ClimaOrganizacional

Capítulo3–PlanejamentoeEstratégia3.1.Planejamento3.2.NíveisdoPlanejamento3.3.MissãoeVisãoeNegócio3.4.DiagnósticoEstratégico–AnáliseSWOT3.5.Objetivos,MetasePlanos3.6.EscolasdoPlanejamento3.7.PlanejamentoporCenários3.8.AnáliseCompetitivaeEstratégiasGenéricas–Estratégia3.9.EstratégiacomoPosicionamento3.10.ModelodeCincoForçasdePorter3.11.EstratégiasGenéricasdePorter3.12.MatrizdeAnsoff3.13.EstratégiacomoMovimento

3.14.TiposdeEstratégia3.14.1.EstratégiadeSobrevivência3.14.2.EstratégiadeManutenção3.14.3.EstratégiadeCrescimento3.14.4.EstratégiadeDesenvolvimento

3.15.Redesealianças3.16.AdministraçãoporObjetivos3.17.ControleEstratégico3.18.BalancedScorecard3.19.MapaEstratégico

Capítulo4–EstruturaOrganizacional4.1.EstruturasFormaiseInformais4.2.EspecializaçãodoTrabalho4.3.EnriquecimentodoTrabalho4.4.CadeiadeComando4.5.AmplitudedeControle4.6.CentralizaçãoeDescentralização4.7.PosiçõesdeLinhaeStaff4.8.Departamentalização4.9.DepartamentalizaçãoFuncional

4.9.1.DepartamentalizaçãoporProduto4.9.2.DepartamentalizaçãoporCliente4.9.3.DepartamentalizaçãoTerritorialouGeográfica4.9.4.DepartamentalizaçãoporProcesso

4.10.DesenhoEstruturaldasOrganizações4.11.EstruturaLinear4.12.EstruturaFuncional4.13.EstruturaDivisional4.14.EstruturaMatricial4.15.EstruturaemRede4.16.FatoresqueInfluenciamaEstrutura

4.16.1.AEstruturaSegueaEstratégia4.16.2.AEstruturaSegueoAmbiente4.16.3.AEstruturaseAdaptaàTecnologia4.16.4.AEstruturaSegueoTamanho

4.17.EstruturasdeMintzbergCapítulo5–Motivação

5.1.TeoriasdeProcessoeTeoriasdeConteúdo5.1.1.TeoriadasNecessidadesdeMaslow5.1.2.TeoriaXeYdeMcGregor5.1.3.TeoriadosDoisFatoresdeHerzberg5.1.4.TeoriadoReforço5.1.5.TeoriaERC5.1.6.TeoriadaExpectânciadeVroom5.1.7.TeoriadasNecessidadesAdquiridas,deMcClelland5.1.8.TeoriadaEquidade5.1.9.TeoriadoEstabelecimentodeObjetivos

5.2.“Empoderamento”ouEmpowermentCapítulo6–Liderança

6.1.DiferençasentreLidereseGerentes6.2.AbordagensdeLiderança6.3.TeoriadosTraçosdeLiderança6.4.EstilosdeLiderança6.5.ContinuumdaLiderança6.6.GridGerencialdeBlakeeMouton6.7.LiderançasContingenciaiseSituacionais6.8.ModelodeFiedler6.9.TeoriaSituacionaldeHerseyeBlanchard–OciclodevidadaLiderança6.10.LiderançaTransacionalxTransformacional6.11.LiderançaCarismática6.12.LiderançaVisionária

Capítulo7–ControleeAvaliação7.1.MomentodoControle7.2.ConceitosdeEficiência,EficáciaeEfetividade7.3.UsodeControleseIndicadoresdeProdutividade7.4.IndicadoresdeDesempenho7.5.Indicadoresligadosàdimensãoresultado7.6.IndicadoresLigadosàDimensãoEsforço7.7.QualidadedosIndicadores

Capítulo8–Coordenação8.1.EstratégiasdeCoordenação8.2.AjusteMútuo8.3.SupervisãoDireta8.4.PadronizaçãodosProcessosdeTrabalho8.5.PadronizaçãodosResultados8.6.PadronizaçãodeHabilidades8.7.ConflitosnaCoordenação

Capítulo9–Comunicação9.1.ComunicaçãoEficienteeEfetiva9.2.CanaisdeComunicação9.3.ComunicaçãoVerbaleNãoVerbal9.4.ComunicaçãoOrganizacional9.5.BarreirasàComunicação

Capítulo10–GestãodasMudanças10.1.Criatividadeeinovaçãoorganizacional

10.1.1.Criatividade10.1.2.Inovação10.1.3.InovaçãoIncrementaleRadical

10.2.MudançaOrganizacional10.2.1.AgentesdeMudança10.2.2.ModelosdeMudança10.2.3.MudançadeCimaparaBaixo10.2.4.MudançadeBaixoparaCima10.2.5.MudançaIntegrada

10.2.6.ResistênciasàsMudanças10.2.7.MudançasPlanejadas

Capítulo11–GestãodeConflitos11.1.CausasdosConflitos11.2.EstilosdeGestãodeConflitos11.3.AbordagensnaResoluçãodeConflitos

11.3.1.AbordagemEstrutural11.3.2.AbordagemdeProcesso11.3.3.AbordagemMista

11.4.EfeitosdoConflitoCapítulo12–GerenciamentodeProcessos

12.1.CadeiadeValor12.2.TiposdeProcessos12.3.NíveisdeDetalhamentodosProcessos12.4.MapeamentodeProcessos12.5.TécnicasdeMapeamento,AnáliseeMelhoriadeProcessos12.6.ProjetodeMapeamentoeModelagemdeProcessos12.7.ProcessoseCertificaçãoISO9000:200012.8.NoçõesdeEstatísticaAplicadaaoControleeàMelhoriadeProcessos12.9.PopulaçãoeAmostra12.10.MédiaeVariabilidade12.11.MelhoriadosProcessos12.12.Reengenharia12.13.OqueaReengenhariaNãoÉ?

Capítulo13–GestãodaQualidade13.1.ErasdaGestãodaQualidade

13.1.1.EradaInspeção13.1.2.EradoControleEstatísticodaQualidade13.1.3.EradaGarantiadaQualidade13.1.4.EradaGestãoEstratégicadaQualidadeouGestãodaQualidadeTotal

13.2.PrincipaisAutoresou“Gurus”daQualidade13.2.1.Shewart13.2.2.CicloDemingouPDCA13.2.3.Deming13.2.4.Juran13.2.5.Feigenbaum13.2.6.Crosby13.2.7.Ishikawa

13.3.Ferramentasdegestãodaqualidade13.4.DiagramadeCausaeEfeitooudeIshikawa13.5.FolhadeVerificação13.6.Histograma13.7.GráficodePareto13.8.DiagramadeCorrelaçãoouDispersão13.9.Fluxograma13.10.GráficodeControle13.11.Programa5S

13.12.SeisSigma13.13.Kaizen13.14.ModelodaFundaçãoNacionaldaQualidade13.15.QualidadeeProdutividade

Capítulo14–GestãodoConhecimento14.1.ConceitosdeDados,InformaçãoeConhecimento14.2.GestãodoConhecimento14.3.ConhecimentoTácitoeExplícito14.4.AprendizagemOrganizacional14.5.EspiraldoConhecimentodeNonakaeTakeuchi

Capítulo15–ProcessoDecisório15.1.Estruturadasdecisõesempresariais

15.1.1.Tiposdedecisões15.1.1.1.DecisõesProgramadaseNãoprogramadas

15.2.RacionalidadeeRacionalidadeLimitada15.3.Certeza,RiscoeIncerteza15.4.ProcessoLineareSistêmico15.5.TécnicasdeAnáliseeSoluçãodeProblemas15.6.FerramentasdeDesenvolvimentodeAlternativas

15.6.1.BrainstormingeBrainwriting15.6.2.AnálisedoCampodeForças15.6.3.DiagramadeÁrvoredeDecisão

Capítulo16–GestãoporCompetências16.1.CompetênciasHumanaseOrganizacionais16.2.EtapasdaGestãoporCompetências16.3.HabilidadesAdministrativasdeKatz

Capítulo17–TrabalhoemequipeCapítulo18–GestãodePessoas

18.1.PanoramadaÁreadeRHnoSetorPúblico18.2.Recrutamentoeseleçãodepessoas

18.2.18.Recrutamento18.2.2.SeleçãodePessoas18.2.3.RecrutamentoeSeleçãonosÓrgãosPúblicos

18.3.Cargos,CarreiraseRemuneração18.4.AvaliaçãodoDesempenho

18.4.1.MétododasEscalasGráficas18.4.2.ListasdeVerificação18.4.3.MétododaEscolhaForçada18.4.4.MétododosIncidentesCríticos18.4.5.MétododaPesquisadeCampo

18.5.ProcessodeRecompensarPessoas18.6.ProcessodeDesenvolverPessoas

18.6.1.TreinamentoeDesenvolvimento18.6.1.1.MétodosdeTreinamento

18.7.ProcessodeManterPessoas18.7.1.QualidadedeVida18.7.2.Equilíbrioorganizacional

18.8.ProcessodeMonitorarPessoas18.8.1.SistemasdeInformaçõesGerenciais

Capítulo19–GerênciadeProjetos19.1.DiferençasentreProjetoseOperações19.2.StakeholdersouPartesInteressadas19.3.GruposdeProcessosdosProjetos19.4.ÁreasdoConhecimentonoGerenciamentodeProjetos19.5.OGerentedeProjetos19.6.Portfólios,Programas,ProjetoseSubprojetos19.7.CiclosdeVidadosProjetos19.8.CiclodeVidadeumProjetoxCiclodeVidadeumProduto19.9.CustoseRiscosnoCiclodeProjetos19.10.RestriçõesaosProjetos

Capítulo20–AdministraçãodeMateriais20.1.AdministraçãodeEstoques20.2.PrevisãoparaosEstoques

20.2.1.MétododoÚltimoPeríodo20.2.2.MétododaMédiaMóvel20.2.3.MétododaMédiaMóvelPonderada

20.3.CustosdosEstoques20.3.1.CustosdeArmazenamento20.3.2.CustosdePedido20.3.3.CustosdeFaltadeEstoque

20.4.NíveisdeEstoque20.5.ClassificaçãoABC20.6.AvaliaçãodeEstoques20.7.Just-in-time20.8.AdministraçãodeCompras20.9.EstratégiasdoSetordeCompras

20.9.1.VerticalizaçãoouHorizontalização20.9.2.CentralizaçãoouDescentralização

20.10.ComprasnoSetorPúblico–Licitações20.11.ModalidadesdeLicitação

20.11.1.Concorrência20.11.2.TomadadePreços20.11.3.Convite20.11.4.Pregão20.11.5.Concurso20.11.6.Leilão20.11.7.Consulta

20.12.Almoxarifado20.13.IdentificaçãodoMaterial20.14.Picking20.15.Inventários20.16.FasesdoInventário20.17.TiposdeInventário

20.17.1.InventárioAnual

20.17.2.InventáriosRotativos20.18.AvaliaçãodosBensnoInventário

Notas

top related